Download as pdf or txt
Download as pdf or txt
You are on page 1of 722

PharmacyPREP.

com Human anatomy

PHARMACY PREP
HUMAN ANATOMY
1. Etiology is concerned primarily with
A. Symptomology
B. Racial origin
C. Causation
D. Language facility
E. Disease

2. Abduction is:
A. A muscle, which moves a body part away from the midline of the body
B. A muscle, which moves a body part toward the midline of the body
C. A muscle, which moves a body part downward the body
D. A muscle, which moves a body part upward the body
E. None of the above
3. Adduction is
A. A muscle, which moves a body part away from the midline of the body.
B. A muscle, which moves a body part toward the midline of the body
C. A muscle, which moves a body part downward the body
D. A muscle, which moves a body part upward the body
E. None of the above
4. Movement of limb or body part closer to or towards the mid line of the body is
A. Abduction
B. Adduction
C. Extension
D. Flexion
E. Eversion
5. Planes and sections of the body. Separates the body into
equal right and left portions is referred as:
A. Sagittal plane
B. Mid-sagittal plane
C. Coronal plane
D. Parasagittal plane
E. Oblique

Copyright © 2000-2014 TIPS Inc. Unauthorized reproduction of this manual is prohibited. This manual is being used during
review sessions conducted by PharmacyPrep. 1-1
PharmacyPREP.com Human anatomy
6. The pelvis bones, each pelvic or hip joints (ball & socket. have?
A. Pubis, ileum, and ischium
B. Scapula, ileum and ischium
C. Humerus, ileum, and ischium
D. Ligament, humerous, scapula
7. Which of the following do not include the skull bones?
A. Facial bones
B. Ossicles
C. Cranial bones
D. Temporal
E. Humerus
8. Which of the following bones is considered a sesamoid bone: A. sternum
B. ethmoid
C. femur
D. patella
E. phalanx
9. "Articulations" refers to: A. bone formation B. A joint between two bones C. bone death D. structures on
bones where muscles attach E. the study of individual bones
10. What is perleche?
A. Dry lips
B. Cuts in lips
C. Cuts corner of mouth
D. White fluid secreting corner of the mouth
E. Space in lips
11. Which of the following is not part of the axial skeleton: A. femur
B. sternum
C. mandible
D. sacrum
12. Incomplete closure of the vertebral column results in: A. kyphosis
B. scoliosis
C. sinusitis

Copyright © 2000-2014 TIPS Inc. Unauthorized reproduction of this manual is prohibited. This manual is being used during
review sessions conducted by PharmacyPrep. 1-2
PharmacyPREP.com Human anatomy
D. spina bifida
E. None of the above
13. Which of the following joints also called ball and socket joints?
A. Hip joints
B. Knee joints
C. Shoulder joint
D. Wrist joint
14. What are the following muscle fibers help in muscle contraction?
A. Actin and myocin
B. Myosin and inhibin
C. Inhibin and actin
D. Actin, myocin and inhibin
15. All of the following have flexor muscles, except:
A-Shoulder
B-Elbow
C-Wrist
D-Tongue
E-Basal joints
16. Which of the following is NOT involve weight bearing exercise?
A. Walking
B. Jogging
C. Running
D. Swimming
E. Stair climbing
17. All are examples of weight bearing joints, except?
A. hip joints
B. Knee joints
C. Skull joints
D. Shoulder joints
E. Elbow joints
18. Which of the following bones is not part of the cranium: A. sphenoid
B. palatine
C. ethmoid
D. occipital

Copyright © 2000-2014 TIPS Inc. Unauthorized reproduction of this manual is prohibited. This manual is being used during
review sessions conducted by PharmacyPrep. 1-3
PharmacyPREP.com Human anatomy
19. Which of the following exercises is the least recommended in prevention of osteoporosis or to improve
bone density?
A. Jogging
B. horse riding
C. Rope jumping
D. Stair climbing
E. swimming
20. Popleital artery pain is indicator of?
A. deep vein thrombosis
B. intermittent claudication
C. Raynaud’s phenomenon
D. Coronary artery pain
E. Sciatica pain

21. What is NOT a function of skin?


A. Protection
B. Temperature regulation
C. Excretion
D. Absorption
E. Immunity

22. Melanin is a dark color light sensitive pigment that is found in skin, protects?
A. microorganism
B. UV radiation
C. Excretion of minerals
D. Thermal regulation
E. Regulation of blood flow in skin

Copyright © 2000-2014 TIPS Inc. Unauthorized reproduction of this manual is prohibited. This manual is being used during
review sessions conducted by PharmacyPrep. 1-4
PharmacyPREP.com Human anatomy
ANSWERS:
1. C
2. A
3. B
4. B
5. B
Tips: Sagittal plane separates the body into right and left portions
Midsagittal plane (Median plane. separates the body/body part into equal right/left portions
Parasagittal planeseparates the body into unequal right and left portions.
Coronal planeseparates the body/body-part into anterior and posterior portions.
Transverse planedivides the body/body-part into superior and inferior portions.
Oblique plane passes through the body/body-part at an angle
6. A
Tips: Hip is "pii" Pubis, ileum, and ischium
7. E
Tips: Skulls bones include: frontal, Parietal, Sphenoid, Temporal, Nasal, Maxilla, Occiptal, Zygomatic, and
Mandible. Skull have 25 bones, of which Cranial bones (8., Facial bones (14. and Ossicles (ear bones. 3
8. D
Tips: Sesamoid bones are embedded into tendons or soft bones of knee joints. The patella and febella are
sesamoid bones
9. B
Tips: a joint between two bones is referred as articulations.
10. C
11. A
12. D
Tips: Incomplete closure of the vertebral column results in spina bifida. Some drugs like phenytoin and
valproic acid use in pregnancy may cause spina bifida side effects in new born.
13. A
14. C
15. D
16. D
17. C
18. B
19. E
20. B
21. D
Tips: skin functions include protection, temperature regulations, excretion, and immunity, enable
movement and growth and endocrine functions like vitamin D synthesis.
22. B

Copyright © 2000-2014 TIPS Inc. Unauthorized reproduction of this manual is prohibited. This manual is being used during
review sessions conducted by PharmacyPrep. 1-5
PharmacyPrep.Com Gastrointestinal System

PHARMACY PREP
GASTROINTESTINAL SYSTEM
1. What are the parts of small intestines in sequence from stomach
to rectum?
A. Duodenum, jejunum and ileum
B. Jejunum, duodenum and ileum
C. Ileum, jejunum and duodenum
D. Ileum, duodenum and jejunum
E. Duodenum, ileum and jejunum
2. The lower part of gut contain the following micro organisms
A. 10 to 50% anaerobic
B. 90 to 99% anaerobic
C. 10 to 50% aerobic
D. 90 to 99% aerobic
E. Lower part of gut does not have any microbes
3. Where is the least pH in GI tract:
A. Esophagus
B. Stomach
C. Small intestine
D. Colon
E. Mouth
4. What is most basic (alkaline). in gastrointestinal tract?
A. Esophagus
B. Stomach
C. Small intestine
D. Colon
E. Mouth
5. Maltase breakdown maltose to?
A. Glucose +fructose
B. Glucose + glucose
C. Glucose + Starch
D. Glucose + lactose
E. Starch + sucrose
6. Celiac disease is caused by?
A. Deficiency of gluten
B. H. pylori infection
C. It is type of inflammatory bowel disease

Copyright © 2000-2014 TIPS Inc. Unauthorized reproduction of this manual is prohibited. This manual is being used during
review sessions conducted by PharmacyPrep. 2-1
PharmacyPrep.Com Gastrointestinal System
D. Deficiency of parietal cells
E. None of the above
7. Stethorrhea can result in to?
A. Fat absorption impaired
B. Indigestion of carbohydrate
C. Secretion of vitamin A, D, E and K
D. Excessive absorption of lipids
E. None of the above
8. Ulcerative colitis occurs at?
A. Colon
B. stomach
C. All GI track
D. small intestine
E. Duodenum
9. Fistula occurs in?
A. Irritable bowel disease
B. Ulcerative colitis
C. Crohn's disease
D. Gastritis
E. GERD
10. Excessive acid secretion causes?
A. Heart burn
B. Peptic ulcers
C. Ulcerative colitis
D. Zollinger Ellison syndrome
E. GIT cancers
11. What is incorrect about Crohn's and ulcerative colitis?
A. Methotrexate is used for Crohn's disease
B. Crohn's disease has symptoms of fever
C. Ulcerative colitis drug of choice is 5-aminosalicylic acid
D. Corticosteroids can be used for Crohn's and Ulcerative colitis
E. Crohn's disease is localized to colon
12. Irritable bowel syndrome (IBS) symptoms include: except
A. Diarrhea
B. Constipation
C. GI bleeding
D. bloating
E. nausea and vomiting

Copyright © 2000-2014 TIPS Inc. Unauthorized reproduction of this manual is prohibited. This manual is being used during
review sessions conducted by PharmacyPrep. 2-2
PharmacyPrep.Com Gastrointestinal System
13. Cystic fibrosis is mainly effects?
A. Disorder of pancreatic secretion
B. Disorder lung function
C. Disorder of bones
D. Disorder of electrolyte imbalance
E. disorder of bladder
14. Why are the most drugs absorbed from small intestine?
A. because drug first enters into small intestine
B. Because small intestine have large surface area
C. Because small intestine is not acidic
D. because small intestine is small
E. because small intestine have more blood vessels
15. Crohn's diseases occur?
A. Colon
B. stomach
C. Small intestine and colon
D. small intestine
E. Duodenum
16. Ulcerative colitis occurs at?
A. Colon
B. stomach
C. All GI tract
D. small intestine
E. Duodenum
17. Nursing mother cannot breast feed to her infant, which of the following type of product is most likely
recommended?
a) Cow milk based formula milk
b) Lactose free formula milk
c) Soya milk
d) Solid food
e) Soft foods

18. All of the following diseases/drugs cause the GI bleeding, except:


A. Gastroenteritis
B. Peptic ulcer
C. NSAIDs
D. GI cancer
E. Pancreatitis

Copyright © 2000-2014 TIPS Inc. Unauthorized reproduction of this manual is prohibited. This manual is being used during
review sessions conducted by PharmacyPrep. 2-3
PharmacyPrep.Com Gastrointestinal System
19. A regular customer of your pharmacy, is taking treatment of osteoarthritis. Recently got gastritis? What
is correct?
A. Patient is using acetaminophen
B. Patient is using corticosteroids
C. Patient is using NSAIDs
D. Patient is using methotrexate
E. Patient has heartburn
20. What food/diet induce HCl secretion?
A. Carbohydrate
B. Fat
C. Proteins
D. Vitamins
E. Minerals
21. All of the following are irritable bowel syndrome (IBS) symptoms, except?
A. Gastrointestinal bleeding
B. Constipation
C. Diarrhea
D. Abdominal cramps
E. Nausea and vomiting

22. What is first line treatment for GERD?


A. Aluminum antacids
B. Calcium antacids
C. Omeprazole
D. Ranitidine
23. Triple therapy include two antibiotics and one proton pump inhibitor is used to treat?
A. heart burn
B. Gastro Esophagus Reflux Disease
C. H. pylori ulcer
D. Zollinger Ellison syndrome
E. All of the above
24. What macrolide antibiotics is effective for H. pylori?
A. Amoxicillin
B. Omeprazole
C. Clarithromycin
D. Metronidazole
E. Bismuth subsalicylate
25. What is good time to take antacids?
A. Take at bedtime

Copyright © 2000-2014 TIPS Inc. Unauthorized reproduction of this manual is prohibited. This manual is being used during
review sessions conducted by PharmacyPrep. 2-4
PharmacyPrep.Com Gastrointestinal System
B. Take first thing in morning
C. Take few minutes of symptoms of heartburn
D. Take with or without food
E. Take with full glass of water
26. What is true of Sucralfate ?
A. Cytoprotective or mucosal protective agent
B. Do not take with antacids, sucralfate require acidic stomach.
C. Taken empty stomach
D. Used to treat gastric ulcers
E. All of the above
27. Mechanism of sucralfate?
A. act as barrier, this prevents acid contact to ulcers
B. act by neutralization
C. inhibits H2 receptors and prevents HCl secretion
D. Inhibit proton pump inhibitors
E. None of the above
28. A person with ASA allergy should avoid taking all except?
A. Bismuth subsalicylate
B. Ibuprofen
C. Naproxen
D. ASA
E. Acetaminophen
29. What is true about bismuth subsalicylate?
A. Indicated for prevention and treatment of diarrhea
B. Have antibacterial, stimulation of mucus.
C. May give dark tongue and stools
D. May be used in ASA allergy
E. A, B and C only
30. Bismuth subsalicylates darken stools due to?
A. Bismuth decrease bisulfates and have adsorption action.
B. It is side effect
C. Bismuth antidiarrheal drug
D. Prevents travellers diarrhea
E. None of the above
31. What is NOT symptoms of irritable bowel syndrome
A. Constipation
B. Bleeding from gut
C. Nausea and vomiting
D. abdominal cramps, and pain

Copyright © 2000-2014 TIPS Inc. Unauthorized reproduction of this manual is prohibited. This manual is being used during
review sessions conducted by PharmacyPrep. 2-5
PharmacyPrep.Com Gastrointestinal System
E. Significant weight loss
32. Anemia due to iron deficiency include, all except?
A. Peptic ulcer
B. Pregnancy
C. Menstruation
D. GERD
E. hemorrhage
33. A patient have osteoarthritis using medication for pain. Now patient have gastritis. What drugs patient
may be on?
A. Morphine
B. Codeine
C. NSAIDS
D. ASA 81 mg
E. Acetaminophen
34. Which of the following GI conditions is NOT associated with GI bleeding?
A. Gastritis
B. Gastroenteritis
C. Pancreatitis
D. GI cancer
E. Ulcerative colitis
35. Chronic alcoholism may cause?
A. Gastritis
B. Crohn's disease
C. Peptic ulcer
D. GI cancer
E. Ulcerative colitis
36. Fistula or granulomas or skip patches occurs in?
A. Irritable bowel disease
B. Ulcerative colitis
C. Crohn's disease
D. Gastritis
E. GERD
37. Polypeptides and caffeine stimulates secretion of?
A)parietal cells
B)vitamin B12
C)gastrin
D)chyme
E)amylase

Copyright © 2000-2014 TIPS Inc. Unauthorized reproduction of this manual is prohibited. This manual is being used during
review sessions conducted by PharmacyPrep. 2-6
PharmacyPrep.Com Gastrointestinal System
38) What gastrointestinal conditions are associated with GI bleeding symptoms? except
A) Gastrointestinal cancer
B) Pancreatitis
C) Gastritis
D) Crohn's disease
E) Ulcerative colitis
39) What defined as the protrusion of an organ or the fascia of an organ through the wall of the cavity that
normally contains it
A) hernia B) aneurysm C) Atherosclerosis D) sarcoma E) hepatotoxicity

Copyright © 2000-2014 TIPS Inc. Unauthorized reproduction of this manual is prohibited. This manual is being used during
review sessions conducted by PharmacyPrep. 2-7
PharmacyPrep.Com Gastrointestinal System
ANSWERS:
1. A
Tips: "DJI" = Duodenum, jejunum and ileum
2. B
Tips: The colon has the highest anaerobic bacteria, and it can be from 90 to 99%.
3. B
Tips: The stomach is the highest acidic and it means least pH.
4. D
Tips: colon is the most basic of GI track.
5. B
Tips: Maltase breakdown maltose to two glucose units.
6. E
Tips: Celiac disease is due to sensitivity to gluten, it is caused by malabsorption of gluten.
7. C
Tips: Stethorrhea can result into excessive secretion of fat soluble vitamins A,D,E and K. Drugs like orlistat
(Xenical). can cause stethorrhea
8. A
9. C
Tips. Fistula can be present in Crohn's disease
10. D
Tips: GERD is heart burn cause by reflux of acid into esophagus,
ulcers cause bacteria or drugs induced.
11. E
12. C
Tips: Irritable bowel syndrome (IBS. gives bloating, constipation, diarrhea, N&V symptoms. However
Inflammatory bowel disease (IBD. (Crohn's disease and UC. may give bleeding and diarrhea
13. B
Tips: Cystic fibrosis (CF. is the most common, fatal genetic disease affecting young Canadians. CF is a multi-
organ disease affecting primarily the lungs and the digestive system. In the lungs, CF causes severe
breathing problems. A build-up of thick mucus makes it difficult to clear

Copyright © 2000-2014 TIPS Inc. Unauthorized reproduction of this manual is prohibited. This manual is being used during
review sessions conducted by PharmacyPrep. 2-8
PharmacyPrep.Com Gastrointestinal System
bacteria and leads to cycles of infection and inflammation, which damage the delicate lung tissues. They
must follow a demanding daily routine of physical therapy to keep the lungs free of congestion and
infection. In the digestive tract, CF makes it extremely difficult to digest and absorb adequate nutrients
from food.
14. B
15. C
16. A
17. A
18. F
Tips: Pancreatitis can cause hemorrhagic bleeding, NSAID side effects have GI bleeding
19. C
Tips: osteoarthritis use NSAIDs to treat joint pain. however this can cause gastritis side effect.
20. C
21. A
Tips. Gastrointestinal bleeding is NOT Irritable bowel syndrome symptom.
22. C
23. C
24. C
25. C
26. E
27. A
28. E
Tips: person with ASA allergy must avoid all NSAIDS and COX-II inhibitors. Bismuth subsalicylate contains
salicylates and this can cross allergy.
29. E
30. A

Copyright © 2000-2014 TIPS Inc. Unauthorized reproduction of this manual is prohibited. This manual is being used during
review sessions conducted by PharmacyPrep. 2-9
PharmacyPrep.Com Gastrointestinal System
31. B
32. D
33. C
34. D
35. A
Tips: NSAIDs, smoking and chronic alcoholism cause gastritis. It causes inflammation of GI lining. The most
common cause of gastritis in Canada is due to chronic alcoholism.
36. C
37. C
38. B
39. A

Copyright © 2000-2014 TIPS Inc. Unauthorized reproduction of this manual is prohibited. This manual is being used during
review sessions conducted by PharmacyPrep. 2-10
PharmacyPrep.Com Nervous System

PHARMACY PREP
Nervous System
1) Which of the following is NOT a part of nerve cell?
A. Axon
B. Dendrites
C. Cell body
D. Myelin sheath
E. synapse cleft
2) Blood brain barrier is present at?
A) Endothelial cell lining in cerebrospinal tissues.
B) Carotid arteries
C) in brain
D) in blood circulations
E) in peripheral
Ans: A
Tips. Blood brain barrier is endothelial cell lining in cerebrospinal tissues.
3) What following statement is correct about sciatic nerve?
A. It pass through neck down to buttocks
B. It pass through buttock, down to foot
C. It pass through elbow to wrist
D. It pass through knee down to foot
E. It pass from lower spine to foot
Ans. B
Tips: Sciatic nerve is the branch of sacral plexus. It divides into tibial and common fibular nerves. Which
supply the muscles of the posterior thigh and all of the leg and foot.
4. Damage of auxiliary nerve thought to cause?
A. paralysis of fifth finger
B. impaired sensation
C. numbness of extremities
D. inability to bend elbow
E. inability to walk
5) Damage to radial nerve can cause?
A) inability to move neck
B) inability to move shoulders
C) inability to move wrist and fingers
D) inability to move toes
E) all of the above
Ans. C

Copyright © 2000-2014 TIPS Inc. Unauthorized reproduction of this manual is prohibited. This manual is being used during
review sessions conducted by Pharmacy Prep. 3-1
PharmacyPrep.Com Nervous System
Tips. Radial nerve a branch of brachial plexus. It supplies the triceps brachii, brachioradials, and muscle of
the extensors compartment of forearm. Radial nerve palsy caused prolong compression of radial nerve.
Which runs along the underside of bone in the upper arm. Some time called Saturday night palsy. Usually it
improves once the pressure is relieved.
6. A CNS disease where the myelin sheath of motor neurons is degenerating or being destroyed, which
interferes with neuronal impulses?
A. Parkinson's Disease
B. Multiple sclerosis
C. Alzheimer's dementia
D. Fibromyalgia
E. Neuralgia
7) Which of the following is NOT a cognitive function?
A) Executing functions
B) Disorientation
C) Hallucinations
D) confusion
E) Imagination

Ans. C
8) Betahistine is the drug of choice for Menere’s disease.
A. histamine agonist
B. histamine antagonist
C. first generation antihistamine
D. anticholinergic
E. cholinergic

Ans. A

Copyright © 2000-2014 TIPS Inc. Unauthorized reproduction of this manual is prohibited. This manual is being used during
review sessions conducted by Pharmacy Prep. 3-2
PharmacyPrep.com Cardiovascular System

PHARMACY PREP
CARDIOVASCULAR SYSTEM
1) Absolute refractory period (ARP) reflects the time during which no action potential can be initiated
regardless of how much inward current are supplies. The ARP begins and ends at.
A) Phase I to Phase 4
B) Phase 0 to phase 1
C) Phase 1 to phase 2
D) Phase 1 to phase 3
E) Phase 2 and 3 only
2)Angiotensin-converting enzyme catalyzes the conversion of Angiotensin I to Angiotensin II, primarily in?
A. Kidney B. Blood C. Lungs D. Liver E. Heart
3. What is the initial symptom of congestive heart failure?
A. dizziness B. blurred vision C. dyspnea D. chest pain E. headache
4. All of the following initial symptoms of stroke, except?
A. dizziness B. blurred vision C. dyspnea D. slurred speech E. headache
5. Which of the following statement is incorrect about vasopressin (antidiuretic hormone ADH)?
A. It is potent vasoconstrictor that increase TPR in arterioles
B. Increase water resorption by renal distal tubule and collecting ducts.
C. ADH released when atrial receptors detect the decrease in blood volume
D. All of the above
E. None of the above
6. Which of the vasoactive hormone causes arterial constriction?
A. Antihistamines
B. Bradykinin
C. Prostaglandin
D. Thromboxane A2
E. Prostacyclin
7. Myocardial contractility is best correlated with the intracellular concentration of
2+ + + - 2+
A. Ca B. Na C. K D. Cl E. Mg
8. Aldosterone is secreted by adrenal cortex. What is incorrect about aldosterone?
A. Increases sodium chloride (NaCl) reabsorption by the renal distal tubules thereby increase blood volume
and arterial pressure
B. It causes vasoconstriction of the arterioles, thus increase TPR and mean arterial pressure

Copyright © 2000-2014 TIPS Inc. Unauthorized reproduction of this manual is prohibited. This manual is being used during
review sessions conducted by PharmacyPrep. 4-1
PharmacyPrep.com Cardiovascular System
C. Angiotensin II stimulates the synthesis and secretion of aldosterone by the adrenal cortex
D. all of the above
E. none of the above
9) Repolarization process produces?
A) Excessive negative charge in cell
B) Sodium ion influx
C) Chloride efflux
D) Excessive positive charge inside cell
E) Excessive negative and positive charge inside cells
10) What is NOT associated circulatory system?
A) Thrombus
B) Aneurism
C) Stannosis or Stenosis
D) Embolism
E) Oxygen
11) Embolus is?
A) Floating blood clot
B) A blood clot at site of plaque
C) Fibrin clot
D) Bulging of arteries
E) Hemorrhage
12. The electrical activity occurred during depolarization and repolarization transmitted through electrodes
attached to the body and transformed by an electrocardiograph (ECG) in to series of waveforms. The
"QRS" wave indicates?
A) Atrial depolarization
B) Ventricular depolarization
C) Atrial repolarization
D) Ventricular repolarization
E) Plateau
13) Q-T prolongation (prolong interval) can cause?
A. Atrial fibrillation
B. Supraventricular fibrillation
C. Ventricular fibrillation or ventricular tachycardia
D. Dysarrhythmias
E. None of the above

Copyright © 2000-2014 TIPS Inc. Unauthorized reproduction of this manual is prohibited. This manual is being used during
review sessions conducted by PharmacyPrep. 4-2
PharmacyPrep.com Cardiovascular System
14. Which of the following conditions risk factor is NOT related to first degree relative (family history)?
A. premature cardiovascular disease
B. Diabetes
C. Coronary artery diseases
D. dyslipidemia
E. Hepatitis
15. Which of the following is the indicator of ventricular activation?
A) P wave
B) Q wave
C) Q-T wave
D) QRS wave
E) T wave
16. Which of the following medical conditions patient is recommended to self care by using of tight
stockings?
A. coronary artery disease
B. Heart failure
C. Stroke
D. Deep vein thrombosis
E. All of the above
17. Atrial fibrillations ECG or changes in electrode potential curve shows?
A) P wave
B) QRS wave
C) T wave
D) QT wave
E) U wave
18. Which one of the following ECG wave are associated with ventricular contraction?
A) P wave
B) QRS wave
C) T wave
D) QT interval prolongation
E) U wave

Copyright © 2000-2014 TIPS Inc. Unauthorized reproduction of this manual is prohibited. This manual is being used during
review sessions conducted by PharmacyPrep. 4-3
A 9. D ANSWERS:
1.
A 10. B 2.
A 11. C 3.
A 12. C 4.
C 13. E 5.
E 14. D 6.
D 15. A 7.
Tips:
Myocardial
cell
contraction
s depends
on the
intracellular
Ca2+.
Which
regulated
by calcium
ion entry
across the
cell
membrane
during the
plateau of
the action
potential.
D 16. B 8.
B 18. B 17.

www.Pharmacyprep.com Endocrine System

PHARMACY PREP.
ENDOCRINE SYSTEM
1. Megaloblastic anemia may be caused due to a result of:
I-Folic acid deficiency
II-Vitamin B12 deficiency
III-Iron deficiency
A. I only
B. III only
C. I and II only
D. II and III only
E. All are correct
2) The doctor is suspecting that his patient is suffering of hypothyroidism, which assay can be used to
confirm his diagnostic?
A. Increase Serum TSH
B. Decreased FT4 in urine
C. T4 increase
D. TSH in urine
E. TT3 counts
3. Follicle Stimulating Hormone and Luteinizing Hormones levels may be mainly increased during:
A. Pregnancy
B. Ovulation
C. Conception
D. Last trimester of pregnancy
E. Amenohrrea
4. Which is/are true about placenta:
I- The umbilical cord transport nutrients from mother to fetus passing through the placenta.
II- The umbilical cord is formed of two umbilical arteries and one umbilical vein
III-Placenta is an organ characteristic of true mammals during pregnancy
A. I only
B. III only
C. I and II only
D. II and III only
E. All are correct
5. Hypothyroidism maybe resulted due to:
I- Grave’s disease
II- Hashimoto thyroiditis
III- Treatment with anti-thyroid drugs

Copyright © 2000-2014 TIPS Inc. Unauthorized reproduction of this manual is prohibited. This manual is being used
during review sessions conducted by PharmacyPrep. 5-1
www.Pharmacyprep.com Endocrine System
A. I only
B. III only
C. I and II only
D. II and III only
E. All are correct
6. Characteristics of hypoparathyroidism may include:
I- It is characterized by decrease of calcium and increase of Phosphorous
II -It is a deficiency of thyroid gland
III- It is characterized by an increase of calcium and decrease of phosphorous
A. I only
B. III only
C. I and II only
D. II and III only
E. All are correct
7. All of the following diseases can be caused by oral contraceptives, EXCEPT
A. Epilepsy
B. Tromboembolism
C. Breast cancer
D. Uterine cancer
E. Myocardial infarction
8. Hyperaldosteronism, a hyper function of adrenal cortex may be characterized by:
I- Abnormality of electrolyte metabolism resulting in high secretion of aldosterone
II- Can be treated by aldosterone antagonist: Spirolactone
III- Hyperaldosteronism is characterized typically by hypokalemia
A. I only
B. III only
C. I and II only
D. II and III only
E. All are correct
9. Which is the main hormone secreted by the adrenal cortex and responsible for the regulation of
electrolytes and water balance?
A. FSH
B. Thiazides
C. ADH
D. Aldosterone
E. Vasopressin
10. Ketoacidoses are acidosis accompanied by the accumulation of ketone bodies in the body tissues
and fluids and may be a result of:
I- Starvation

Copyright © 2000-2014 TIPS Inc. Unauthorized reproduction of this manual is prohibited. This manual is being used
during review sessions conducted by PharmacyPrep. 5-2
www.Pharmacyprep.com Endocrine System
II- Juvenile type diabetes
III- Inadequate insulin treatment
A. I only
B. III only
C. I and II only
D. II and III only
E. All are correct
11. What does insulin pump mean?
A. Insulin with fast action
B. Insulin supply for long period of time
C. Insulin with fast absorption
D. Insulin rapidly excreted
E. Insulin rapidly metabolized
12. Hypofunction of adrenal cortex may cause:
A. Cushing disease
B. Gray’s disease
C. Addison’s disease
D. Reye’s disease
E. All are wrong
13) Cushing syndrome a disorder of adrenal cortex is mainly due to:
I) Hyper function of adrenal cortex
II) High secretion of cortisone
III) Hypofunction of adrenal cortex
A. I only
B. III only
C. I and II only
D. II and III only
E. All are correct
14. Correct mechanism of methimazole as antithyroid drug may include:
I- Inhibit iodine oxidation
II- Catalyze the incorporation of iodine into tyrosine
III- Inhibit the synthesis of thyroid hormones
A. I only
B. III only
C. I and II only
D. II and III only
E. All are correct
15. All of the following cells are normally presented in pancreas, EXCEPT:
A. β-cell

Copyright © 2000-2014 TIPS Inc. Unauthorized reproduction of this manual is prohibited. This manual is being used
during review sessions conducted by PharmacyPrep. 5-3
www.Pharmacyprep.com Endocrine System
B. α-cell
C. Sertoli cell
D. Delta cell
E. F-cell
16. Correct statement regarding pernicious anemia may include:
A. Due to dietary deficiency of vitamin B12
B. Prevented by oral administration of vitamin B12
C. Treated by parenteral administration of folic acid
D. Treated by parenteral administration of vitamin B12
E. Caused by dietary deficiency of iron
17. Iron deficiency may cause a type of anemia known as:
A. Pernicious anemia
B. Megaloblastic anemia
C. Pernicious and megaloblatic anemia
D. Microcystic anemia
E. All of the above
18. Side effects that may be expected to occur from the administration of oral contraceptives include
all the following, EXCEPT:
A. Fluid retention
B. Headaches
C. Hypertension
D. Constipation
E. Depression
19. The most appropriate insulin for diabetic patients with ketoacidoses may include:
A. Crystalline Zinc Insulin
B. Ultralent Insulin
C. Protamine Insulin
D. Extended Insulin Zinc Suspension
E. Isophane Insulin
20. Which is not secreted from the anterior pituitary
A. Prolactin
B. Vasopressin
C. Gonadotropin hormones
D LH
E. FSH
21. Example of oral antidiabetic agent most likely to cause lactic acidosis is:
A. Glyburine
B. Chlorpropamide

Copyright © 2000-2014 TIPS Inc. Unauthorized reproduction of this manual is prohibited. This manual is being used
during review sessions conducted by PharmacyPrep. 5-4
www.Pharmacyprep.com Endocrine System
C. Metformin
D. Repaglinide
E. Acarbose
22. All of the following substance may be classified as steroid hormone, EXCEPT:
A. Vitamin D3
B. Estrogen
C. Cortisone
D. Androgen
E. Stilbestrol
23. Medical condition related to hypothyroidism, the decrease of thyroid hormones might include:
A. Cushing syndrome
B. Addison’s syndrome
C. Grave’s disease
D. Hashimoto disease
E. Myasthenia Gravis
24. Examples of rate limit of insulin may include:
I- Capillary passage
II- Zinc crystalline insulin
III- Protein binding
A. I only
B. III only
C. I and II only
D. II and III only
E. All are correct
25. Wrong statement regarding FSH and LH may include:
A. They are hormones
B. They are secreted from hypothalamus
C. Secreted specifically from posterior pituitary gland
D. They increase their concentration during ovulation
E. FSH is the folliculum-stimulating hormone
26. False statement regarding insulin may include:
A. Fast acting insulins has an onset of action of 30 min
B. Protamine zinc insulin is long acting insulin
C. Regular and crystalline insulins are used in diabetic patients with ketoacidoses
D. Regular insulin is the only one that can be administrated intravenously
E. Long acting insulins has a maximum duration of action of 24 hours

Copyright © 2000-2014 TIPS Inc. Unauthorized reproduction of this manual is prohibited. This manual is being used
during review sessions conducted by PharmacyPrep. 5-5
www.Pharmacyprep.com Endocrine System
27. Correct statements regarding the action and examples of insulin may include:
I- Fast acting: Crystalline and acid regular
II- Intermediate acting: Zinc suspension, globulin zinc and isophane insulin
III- Long acting: Protamin zinc suspension, extended zinc suspension and ultralent.
A. I only
B. III only
C. I and II only
D. II and III only
E. All are correct
28. After an insulin dose, hypoglycemia may occurs due to:
A. Low carbohydrate diet
B. Increase in glucose levels
C. Increase in glycogen levels
D. High carbohydrate diet
E. All are correct
29. Insulin is a hormone secreted by which cells?
A. α-cells
B. β-cells
C. Glucagon
D. Amylase
E. Pituitary
30. Thyroid gland is extremely important due to its ability to secrete:
I- Thyroid hormones
II- Calcitonin
III- Calcium
A. I only
B. III only
C. I and II only
D. II and III only
E. All are correct
31. Correct statements regarding glucagon, a polypeptide hormone, may include:
I- Secreted by the α-cells of pancreas
II- It increase blood glucose levels
III- Stimulate hepatic gluconeogenesis and glycogenolysis
A. I only
B. III only
C. I and II only
D. II and III only
E. All are correct

Copyright © 2000-2014 TIPS Inc. Unauthorized reproduction of this manual is prohibited. This manual is being used
during review sessions conducted by PharmacyPrep. 5-6
www.Pharmacyprep.com Endocrine System
32. Examples of therapeutically used hormones and related drugs may include:
A. Adrenocorticosteroids
B. Antidiabetic agents
C. Thyroid hormones
D. Gonadal and pituitaria hormones
E. All are correct
33. Examples of gonadotropin hormone may include:
A. ACHT
B. FSH
C. TSH
D. TRH
E. Somatropin
34. False statement regarding VASOPRESSIN includes:
A. It is an antidiuretic hormone
B. It is used to treat neurogenic diabetes insipidus
C. It is used to treat postoperative abdomen
D. It acts on proximal renal epithelium promoting the absorption of water
E. It acts on distal renal epithelium promoting the absorption of water
35) Hormone which sustain the corpus luteum in pregnancy is:
A) Estrogen
B) Follicle Stimulating Hormone (FSH)
C) Gonadotropin hormones like hCG
D) Progesterone
E) LH and prolactin
36. Characteristics of diabetes insipidus may include:
A. Polyuria
B. Thirst
C. Decrease in antidiuretic hormones levels
D. Decrease in osmolarity
E. All are correct
37. Drug used in the treatment of diabetes insipidus may include:
A. Glyburine
B. Metformin
C. Oxytocin
D. DDAVP-Desmopressin/Vasopressin
E. Repaglinide

Copyright © 2000-2014 TIPS Inc. Unauthorized reproduction of this manual is prohibited. This manual is being used
during review sessions conducted by PharmacyPrep. 5-7
www.Pharmacyprep.com Endocrine System
38. Diabetes insipidus may happen due to drug induction. Drug that may induce the development of
diabetes insipidus include:
I- Lithium
II- Tricyclic antidepressant
III- Benzodiazepinics
A. I only
B. III only
C. I and II only
D. II and III only
E. All are correct
39. Hormone used to improve uterine contraction in labor and to control postpartum bleeding is:
A. Vasopressin
B. Oxytocin
C. Somatropin
D. Prolactin
E. Progesterone
40. Which of the following hormones can be classified as non-steroidal synthetic hormone?
A. Estradiol
B. Ethyl estradiol
C. Diethylstilbestrol
D. Progesterone
E. Estrogen
41. Examples of estrogens antagonists may include:
I- Medroxyprogesterone
II- Clomiphene
III- Tamoxifen
A. I only
B. III only
C. I and II only
D. II and III only
E. All are correct
42. Which of the following hormone has an aromatic ring in its molecular structure?
A. Estrogen
B. Progesterone
C. Testosterone
D. Cortisol
E. Ethyl androgen

Copyright © 2000-2014 TIPS Inc. Unauthorized reproduction of this manual is prohibited. This manual is being used
during review sessions conducted by PharmacyPrep. 5-8
www.Pharmacyprep.com Endocrine System
43. Adrenocorticosteroids hormones are naturally in the body and are synthesized by:
A. Hypothalamus
B. Pituitaria
C. Adrenal cortex
D. Adrenal gland
E. Pancreas
44. Which of the following is not a side effect associated with oral corticosteroids treatment?
A. Edema
B. Hypoglycemia
C. Increased susceptibility to infection
D. Weigh gain
E. Ulcers
45. Which of the following hormone may stimulate the release of TSH-Thyroid Stimulating
Hormones:
A. ADH B. LFH C. TRH D. LH E. Aldosterone
46. What DEIODINATION means?
A. The soma of thyrosyl residues with thyroglobulin proteolysis
B. The catalyzation of T4 to T3 formation
C. The production of iodine by the thyroid gland
D. The degradation of thyroid hormones that are eliminated by feces and urine.
E. The degradation of iodine
47. Hypothyroidism is due to hypofunction of thyroid gland. Drug(s) used in hypothyroidism
treatment may include:
I- Levothyroxin (T4)
II- Liiothyroxin (T3)
III-Propylthiouracil
A. I only
B. III only
C. I and II only
D. II and III only
E. All are correct
48. Hyperthyroidism is characterized by high levels of thyroid hormones. Example(s) of drugs used in
hyperthyroidism treatment may include:
I) Propanolol
II) Methimazole
III) Propylthyiuracil

Copyright © 2000-2014 TIPS Inc. Unauthorized reproduction of this manual is prohibited. This manual is being used
during review sessions conducted by PharmacyPrep. 5-9
www.Pharmacyprep.com Endocrine System
A. I only
B. III only
C. I and II only
D. II and III only
E. All are correct
49. Characteristic symptoms of hypothyroidism may include all of the following, EXCEPT:
A. Sensitivity to cold
B. Heat intolerance
C. Lethargy
D. Constipation
E. Weight gain
50. Autoimmune Insulin dependent diabetes mellitus is also known as:
I) Diabetes Type I
II) Diabetes insipidus
III) Diabetes Type II
A. I only
B. III only
C. I and II only
D. II and III only
E. All are correct
51. Example of antidiabetic agent that reduces gluconeogenesis and increase glucose utilization may
include:
A. Glyburine
B. Chlorpropamide
C. Repaglinide
D. Metformin
E. Acarbose
52. Glyburide is an antidiabetic agent characterized by which of the following mechanism of action?
A. Decrease the absorption of carbohydrates thus decreasing postprandial rise of glucose
B. Reduces glucogeneogenesis and increase glucose utilization
C. Increase peripheral insulin sensitivity
D. Stimulates the release of insulin
E. Reduce gluconeogenesis only
53. Glyburide and metformin, oral antidiabetic agents are often prescribed together in the treatment
of type II diabetes mellitus due to their:
A. Antagonist effect
B. Addition effect

Copyright © 2000-2014 TIPS Inc. Unauthorized reproduction of this manual is prohibited. This manual is being used
during review sessions conducted by PharmacyPrep. 5-10
www.Pharmacyprep.com Endocrine System
C. Synergistic effect
D. Tolerance
E. Competitive effect
54. Which of the following statements is/are true regarding regular insulin:
I) It is a suspension
II) It may be administrated either SC or IV
III) It is longer acting than lispro insulin
A. I only
B. III only
C. I and II only
D. II and III only
E. All are correct
55. Which of the following agents may have the same therapeutic indication as propyltiuracil does?
A. Danazol
B. Levothyroxine
C. Azathioprine
D. Omeprazole
E. Methimazole
56) A diabetic patient mistakenly taken double dose of insulin. Which of the following is NOT effect
of excessive insulin administration?
A) Sweating
B. palpitation
C. Confusion
D. Diarrhea
E. Fatigue
57. A regular customer of your pharmacy. Using insulin injections for treatment of type I diabetes.
His has active lifestyle and athletic and social. What is incorrect?
A. He should take insulin injection before heavy meals
B. He should take insulin injection before jogging for 5 km
C. He should take insulin before bedtime
D. He should take snack before jogging for 5 km
E. He should take meals with every insulin injection
58. If patient takes insulin before exercise like jogging 5 km. What symptoms pharmacist recommend
to monitor? except
A. sweating
B. confusion
C. palpitation
D. dizzy

Copyright © 2000-2014 TIPS Inc. Unauthorized reproduction of this manual is prohibited. This manual is being used
during review sessions conducted by PharmacyPrep. 5-11
www.Pharmacyprep.com Endocrine System
E. hypertension
59. Which of the following is NOT effect of excessive dose of thyroxin?
A. diarrhea
B. Tachycardia
C. sensitive to heat
D. oily skin
E. hypertension
60) A 49 yo women is using treatment of Levothyroxine (Synthroid) 75 mcg daily for hypothyroidism.
She complains palpitation, weight loss and sensitivity to heat. What is appropriate?
A) refer doctor to increase dose of Synthroid 100 mcg
B) refer to doctor to decrease dose of Synthroid
C) hypothyroid symptoms so should refer to doctor
D) She may have some new problem so refer to doctor
E) None
61) KCR the above patient serum TSH will be?
A) Less than 0.5 mU/L
B) More than 5 mU/L
C) No change in serum TSH
D) None of the above
62) MK is 30 year old female taking insulin for type 1 DM. Which of the following conditions decrease
requirement of insulin dose?
A) Pregnancy
B) Infection
C) Physical activities
D) Heavy meals
E) Acute illness
63) Which of the following stimulates secretion of calcitonin from thyroid gland?
A) hypocalcemia
B) hypercalcemia
C) Hypokalemia
D) Hyperkalemia
E) hypertension

Copyright © 2000-2014 TIPS Inc. Unauthorized reproduction of this manual is prohibited. This manual is being used
during review sessions conducted by PharmacyPrep. 5-12
www.Pharmacyprep.com Endocrine System
CORRECT ANSWERS
ENDOCRINOLOGY
1. C
Tips: Iron deficiency is called mycrocystic anemia while megaloblastic anemia is a deficiency of
cyanocobalamin and folic acid.
2. A
Tips: Serum TSH test has a greater sensitivity in the detection of thyroid disease than any other test.
3. B
Tips: FSH and LH are predominantly increased during ovulation.
4. E
Tips: Placenta is an organ characteristic of true mammals during pregnancy, joining mother and
offspring, providing endocrine secretion and selective exchange of soluble, but not particulate,
blood-borne substances through an apposition of uterine and tromphoblastic vascularized parts.
5. D
Tips: Grave’s disease is characterized by hyperthyroidism.
6. A
Tips: Hypoparathyroidism is a condition produced by greatly reduced function of the para-thyroids
possibly due to autoimmune disease or genetic factors, or by removal of those bodies. It leads to fall
in calcium levels followed by a rise in plasma phosphate level.
7. A
Tips: Epilepsy is a paroxysmal transient disturbance of brain function that may be manifested as
episodic impairment or loss of consciousness and it is not associated with contraceptives use.
8. E
Tips: Hyperaldosteronism is an abnormality of electrolyte metabolism caused by excessive secretion
of aldosterone. The over secretion of aldosterone by adrenal cortex is mainly characterized by
hypokalemia.
9. D
Tips: Aldosterone is the main mineral corticoid hormone secreted by the adrenal cortex. It is
principal activity is the regulation of electrolyte and water balance by promoting the renal retention
of sodium and the excretion of potassium.

Copyright © 2000-2014 TIPS Inc. Unauthorized reproduction of this manual is prohibited. This manual is being used
during review sessions conducted by PharmacyPrep. 5-13
www.Pharmacyprep.com Endocrine System
10. E
Tips: Ketoacidoses are acidosis accompanied by the accumulation of ketone bodies (ketosis) in the
body tissues and fluids. It can be caused by juvenile diabetes, diabetes acidosis, starvation and
inadequate treatment with insulin.
11. B
Tips: Insulin pump means insulin that provides insulin for a long period of time. It is normally
indicated for selected diabetes patients with widely fluctuating blood glucose levels, irregular or
inconsistent work schedule, lifestyles or meals or who achieve less than desired control using
frequent injection routines.
12. C
Tips: Hypo function of adrenal cortex is well known as Addison’s disease while the hyper function of
adrenal cortex is known as Cushing’s syndrome.
13. C
Tips: Cushing syndrome is characterized by an increase in cortisone levels resulting from a hyper
function of adrenal cortex.
14. E
Tips: Methimazole inhibit iodination of tyrosine groups and coupling of these groups to form thyroid
hormones, therefore it inhibits thyroid synthesis.
15. C
Tips: Sertoli cells are found in the tubules of the testes to which the spermatids become attached.
They provide support, protection and nutrition until the spermatids become transformed into
mature spermatozoa.
16. D
Tips: Pernicious anemia is characterized by deficiency of vitamin B12-cyanocobalamine and therefore
should be treated with intramuscularly injection of vitamin B12 because this vitamin is poorly
absorbed orally.
17. D
Tips: Pernicious anemia is a deficiency of vitamin B12 only while megaloblastic anemia is a deficiency
of both vitamin B12 and folic acid.
18. D
Tips: Neither constipation nor diarrhea are side effects of oral contraceptives.
19. A
Tips: The most appropriate insulin for diabetic patients with ketoacidoses are regular insulin and
insulin zinc suspension.

Copyright © 2000-2014 TIPS Inc. Unauthorized reproduction of this manual is prohibited. This manual is being used
during review sessions conducted by PharmacyPrep. 5-14
www.Pharmacyprep.com Endocrine System
20. B
Tips: Vasopressin is an anti-diuretic hormone secreted from the posterior pituitary.
21. C
Tips: Lactic acidosis is the main side effect of the oral hypoglycaemic agent biguanide metformin.
22. E
Tips: Stilbestrol is an anticancer drug used mainly used in the treatment of breast cancer.
23. D
Tips: Hypothyroidism is a hypo function of thyroid gland and responsible for the development of
Myxodiema or Gull’s disease.
24. C
Tips: Insulin does not bind to protein bounds.
25. C
Tips: FSH-Follicle Stimulating Hormone and LH-Luteinizing Hormone are pituitary gonadotropin
hormones secreted from the anterior pituitary.
26. E
Tips: Ultralent and Extend Insulin Zinc Suspension are examples of long acting insulin that may have
their effect up to 28 hours. With an onset of action of 2 to 4 hours.
27. E
Tips: Crystalline and acid regular are examples of fast acting insulin with an onset of action of half to
one hour. Insulin as zinc suspension, globulin zinc and isophane are classified as intermediate acting
insulin with an onset of action of 2 to 4 hours. Insulin with onset of action of 4 to 5 hours are
classified as long acting and include protamin zinc suspension, extended zinc suspension and
ultralent.
28. A
Tips:After an insulin dose, hypoglycemia may occurs due to low carbohydrate diet
29. B
Tips: Insulin is a protein hormone secreted by the beta cells of the pancreas. It is secreted in
response to elevated blood levels of glucose, amino acids, ketones, fat acids and promotes the
efficient storage and utilization of these molecules.
30. C
Tips: Calcium is not secreted by the thyroid hormones.

Copyright © 2000-2014 TIPS Inc. Unauthorized reproduction of this manual is prohibited. This manual is being used
during review sessions conducted by PharmacyPrep. 5-15
www.Pharmacyprep.com Endocrine System
31. E
Tips: Glucagon is a polypeptide hormone secreted by the alpha cells of pancreas in response to
hypoglycemia or to stimulation by the growth hormone of the anterior pituitary; it stimulates
glycogenolysis in the liver by inducing activation of liver phosphorylase.
32. E
Tips: Adrenocorticosteroids, antidiabetic agents, thyroid hormones, gonadal and pituitary hormones
are all therapeutically used hormones and related drugs.
33. E
Tips: FSH-Follicle Stimulant Hormone is a gonadotropin hormone excreted from the anterior
pituitary.
34. D
Tips: Vasopressin does not act on proximal but on the distal renal epithelium promoting the
absorption of water.
35. E
Tips: LH and Prolactin sustain the corpus luteum in pregnancy. FSH is increased only during
ovulation.
36. E
Tips: Diabetes insipidus results in a deficient quantity of antidiuretic hormones being released or
produced, and thus failure of tubular reabsorption of water in the kidney. As a result, a large amount
of urine of low specific gravity is excreted, followed by dehydration and great thirst.
37. D
Tips: Desmopressin and vasopressin an anti-diuretic hormone are the drugs of choice in the
treatment of diabetes insipidus.
38. A
Tips: One of the most known side effects in the long-term treatment with lithium is diabetes
insipidus.
39. B
Tips: Oxytocin is therapeutically used to improve uterine contraction in labour and to control
postpartum bleeding. It also acts in milk ejection stimulation.
40. C
Tips: Diethylstilbestrol is a non-steroidal synthetic estrogen used in oral contraceptives, mainly in
“morning after pills”.

Copyright © 2000-2014 TIPS Inc. Unauthorized reproduction of this manual is prohibited. This manual is being used
during review sessions conducted by PharmacyPrep. 5-16
www.Pharmacyprep.com Endocrine System
41. D
Tips: Medroxyprogesterone is a synthetic progestin used in contraceptives agents.
42. A
Tips: Estrogen is the only steroid hormone with aromatic ring in the ring A of its molecular structure.
43. C
Tips: Adrenal cortex is the responsible for the release of cortisone-adrenocorticosteroids.
44. B
Tips: Common side effects with Corticosteroids therapy may include: Edema, weight gain,
osteoporosis, GIT effects, hypertension, suppression of pituitary adrenal integrity and increased
susceptibility to infections.
45. C
Tips: TRH is the Thyroid Regulating Hormone, secreted from the anterior pituitary and responsible
for the stimulation of TSH’s release.
46- B
Tips: Deiodination is activation of thyroxine Tthe degradation of thyroid hormones that are
eliminated by feces and urine. 4 to active triiodothyronine T3 in peripheral and liver tissues.
Deiodination is
47. C
Tips: Propylthyiuracil is used in the treatment of hyperthyroidism.
48. E
Tips: Propylthyiuracil and Methimazole are main drugs used in the treatment of hyperthyroidism.
However propanolol is also used in the short-term preoperative management of thyrotoxic crises.
49. B
Tips: Heat intolerance is one of the major side effect of hyperthyroidism.
50. A
Tips: Type I diabetes, also called diabetes mellitus is considered insulin dependent. Diabetes
insipidus in mainly treated with anti-diuretic hormones and diabetes type II is non-insulin dependent
mainly treated with oral hypoglycaemic agents.
51. D

Copyright © 2000-2014 TIPS Inc. Unauthorized reproduction of this manual is prohibited. This manual is being used
during review sessions conducted by PharmacyPrep. 5-17
www.Pharmacyprep.com Endocrine System
Tips: Metformin is a biguanide agent used in hyperglycemia-treatment of diabetes type II by
reducing gluconeogenesis and increasing glucose utilization
52. D
Tips: Glyburide as antidiabetic agent is classified as an oral sulfonylurea and act by stimulating the
release of insulin.
53. C
Tips: Glyburide and metformin are often prescribed together in the treatment of type II diabetes
mellitus due to their synergistic effect. Glyburine will stimulate the release of insulin while
metformin reduce gluconeogenesis and increase glucose utilization
54. D
Tips: Regular insulin is not a suspension, it is a clear solution that is way can be administrated either
IV or SC. It is classified as short acting insulin because it acts for up to 7 hours. When compared with
lispro insulin, we find that lispro has a faster onset of action than regular insulin.
55. E
Tips: Propylthyiuracil and Methimazole are often prescribed together in the treatment of
hyperthyroidism.
56. D
Tips: excessive insulin can cause hypoglycemia symptoms such as sweating, palpitation, confusion,
dizzy, nervousness. However, diarrhea is NOT a symptom of hypoglycemia.
57. B
Tips: Physical activity and exercise decrease requirement of insulin. So avoid taking insulin before
physical activity or jogging. Insulin should always take with meals.
58. E
59. E
60. B
61. A
62. C
63.B
Tips. Hypercalcemia stimulate secretion of calcitonin hormone from thyroid gland.

Copyright © 2000-2014 TIPS Inc. Unauthorized reproduction of this manual is prohibited. This manual is being used
during review sessions conducted by PharmacyPrep. 5-18
www.Pharmacyprep.com Endocrine System
BIBLIOGRAPHIC REFERENCE
1- PHARMACY PREP – Evaluating Exam Review Guide - 2013/2014
2- CPS-COMPENDIUM OF PHARMACEUTICALS AND SPECIALITIES - Canadian Pharmacist Association –
2014 edition.
th
3- MEDICAL DICTIONARY – Dorland’s illustrated – 27 edition.
4- PHARMACY PREP – Evaluating Exam Review Guide - 2013/2014
5- THERAPUTIC CHOICES – Canadian Pharmacist Association -Third edition
th
6- USP DI – Drug Information for the Health Care Professional–15 edition – Volume I.

Copyright © 2000-2014 TIPS Inc. Unauthorized reproduction of this manual is prohibited. This manual is being used
during review sessions conducted by PharmacyPrep. 5-19

Pharmacyprep.com

PHARMACY PREP
RENAL DISORDERS
1. Factors that may increase the risk of renal failure may include:
A. Hemorrhage
B. Heart diseases
C. Oliguria
D. Pulmonary edema
E. All are correct
2. Reabsorption of drugs in kidney does NOT depend on:
A. Flow rate
B. Tonicity
C. Tubular secretion
D. pH
E. Metabolism
3. Characteristics of nephrotic syndrome may include all of the following, EXCEPT:
A. Proteinurea
B. Hypoalbuminemia
C. Edema
D. Hyperalbuminemia
E. Hyperlipidemia
4. Cholecystitis, normally characterized by edema and congestion to severe infection with gangrene
and perforation is an inflammation of:
I- Gallbladder
II- Liver
III- Heart
A. I only
B. III only
C. I and II only
D. II and III only
E. All are correct
5. All of the following statements about renal failure are correct EXCEPT:
A. Increased total body clearance
B. Increased drug half life
C. Decrease body clearance
D. Affect drugs that excrete via kidneys
E. Drugs metabolized via liver are the least to be affected

Copyright © 2000-2014 TIPS Inc. Unauthorized reproduction of this manual is prohibited. This manual is being used
during review sessions conducted by PharmacyPrep. 6-1
Pharmacyprep.com
6. Secretion of potassium ions by the distal tubule will be decreased by
A. Metabolic alkalosis
B. A high potassium diet
C. Hyperaldosteronism
D. Spironolactone administration
E. Thiazide administration

7. Azotemia means?
A. Increase in nitrogen waste in body
B. Increase in nitrogen waste in urine
C. Decrease in nitrogen waste in body
D. Decrease in nitrogen waste in urine
E. No change in nitrogen
8. Proteinuria is?
A. Proteins in urine
B. Proteins in blood
C. proteins in food
D. Proteins in liver
9. Pre-renal acute renal failure is due to?
A. low renal blood perfusion
B. High BP
C. CHF
D. creatinine clearance
E. Filtration
10. Intracellular alkalosis is caused by?
A. Spironolactone
B. Thiazides
C. Loop diuretics
D. ACE I
E. Mannitol
11. All of the following drug cause metabolic alkalosis, except?
A. Acetazolamide
B. Thiazides
C. hydrochlorothiazide
D. Loop diuretics
E. Furosemide
12. A man with history of severe diarrhea, the loss of
- -
HCO3 from GI tract causes a decrease in blood [HCO3 ],
and increase CO2 his values are thus this man has?

Copyright © 2000-2014 TIPS Inc. Unauthorized reproduction of this manual is prohibited. This manual is being used
during review sessions conducted by PharmacyPrep. 6-2
Pharmacyprep.com
pH = 7.25, pCO2 3= 24 mmHg, [HCO-] = 10 mEq/L
A. Metabolic alkalosis
B. Metabolic acidosis
C. Respiratory acidosis
D. Respiratory alkalosis
E. Normal acid base status
13. Prerenal ARF is characterized as?
A) Inadequate blood circulation (perfusion) to the kidneys
B) Excessive blood perfusion to the kidney
C) Excessive drug elimination
D. All of the above
14. A drug metabolized in livers and has renal elimination. If patient have renal failure. What is
correct?
A. Drug accumulates in blood, can cause toxicity
B. Drug eliminates from stools
C. Drug excretes in urine, and not metabolized
D. Drug have first pass metabolism
E. Drug has faster elimination and reabsorption
15. What is incorrect about potassium levels?
A. Chronic renal disease may cause hyperkalemia
B. Adrenal cancer may cause hypokalemia
C. ACE Inhibitors like captopril may cause hypokalemia
D. spironolactone may cause hyperkalemia
E. hydrochlorothiazide may cause hypokalemia
16. What is correct about renal failure?
I-Renal failure not only alters the renal elimination, but also the non-renal disposition of drugs that
are extensively metabolized by the liver.
II-Renal failure reduce the drug clearance
III- Chronic renal failure (CRF) has been shown to significantly reduce the non-renal clearance and
alter bioavailability of drugs predominantly metabolized by the liver and intestine
A. I only
B. III only
C. I and II only
D. II and III only
E. All are correct
17. The shock and airway edema of anaphylaxis may be best treated by administration of which of
the following agents?
A. Salbutamol

Copyright © 2000-2014 TIPS Inc. Unauthorized reproduction of this manual is prohibited. This manual is being used
during review sessions conducted by PharmacyPrep. 6-3
Pharmacyprep.com
B. Diphenhydramine
C. Adrenaline
D. Acetazolamine
E. Aminiphyline
18.Renal nephritis causes
I- Glomerular to be permeable
II-Leak the proteins to urine
III-The capillary membrane becomes impermeable
A. I only
B. III only
C. I and II only
D. II and III only
E. All of the above
19. Glomuronephritis symptoms?
I-Protein urea
II-Decrease in Na ions
III-Azotemia
A. I only
B. III only
C. I and II only
D. II and III only
E. All are correct
20.What is nephritic syndrome?
I-Damage of renal tubule
II-Excretion of protein in urine
III- Blood in gut
A. I only
B. III only
C. I and II only
D. II and III only
E. All are correct
21. A patient have renal disease. Which of the following medication use is the concern for the
pharmacist?
A. Opioids
B. Statins
C. ACE Inh.
D. NSAIDs
E. Codeine

Copyright © 2000-2014 TIPS Inc. Unauthorized reproduction of this manual is prohibited. This manual is being used
during review sessions conducted by PharmacyPrep. 6-4
Pharmacyprep.com
22. Acute pre-renal failure is caused by all, except?
A. sepsis
B. hypertension
C. heart problems
D. congestive heart failure
E. Gentamycin
23. A regular customer of your pharmacy, age 55 yo, medication profile include Ramipril 10 mg,
Valsartan 5 mg, and recently doctor added spironolactone. Patient diet include excessive intake of
bananas. All of the following are pharmacist concern, except?
A. Ramipril
B. Valsartan
C. Spironolactone
D. Age 55 yo
E. Bananas
24) Which of the following is the least preferable combination antihypertensive in a patient whose
K+ levels are 5.5 mEq/L?
A. ramipril/ Hydrochlorothiazide
B. Valsartan/Amlodipine
C. Perindropril/spironolactone
D. Losartan/Hydrochlorothiazide
E. Amlodipine/Atorvastatin
25) Characteristics of nephrotic symptoms, include all except?
A) Proteinuria
B) Hypoalbuminemia
C) edema
D) Hyperalbuminemia
E) hyperlipidemia
ANSWERS:

Copyright © 2000-2014 TIPS Inc. Unauthorized reproduction of this manual is prohibited. This manual is being used
during review sessions conducted by PharmacyPrep. 6-5
Pharmacyprep.com
1. E
2. C
Tips: tubular section occurs after reabsorption. Thus it may affect reabsorption.
3. D
Tips= nemia = in blood, nurea = in urine
4. A
Tips: Cholecystitis is an inflammation of gallbladder. Its acute form is normally characterized by
edema and congestion to severe infection with gangrene and perforation.
5. A
6. D
+
Tips: Spironolactone diuretics act at distal tubules and prevents K secretion, thus it causes hyperkalemia.
7. A
Tips: renal disease (glomerular filtration), decrease excretion of urea into urine, that leads to urea
retention in blood, it is referred as azotemia or uremia.
8. A
9. A
10. A
Tips: spironolactone gives intracellular alkalosis. Thiazides and loop diuretics gives metabolic
alkalosis. Acetazolamide and spironolactone gives metabolic acidosis.
11. A
Tips: drugs that gives metabolic alkalosis are thiazides, and loop diuretics. And metabolic acidosis is
CA inhibitors, potassium-sparing diuretics.
12. B
13. A
Tips: Prerenal ARF is characterized by inadequate blood circulation (perfusion) to the kidneys, which
leaves them unable to clean the blood properly. Many patients with prerenal ARF are critically ill and
experience shock (very low blood pressure). There often is poor perfusion within many organs, which
may lead to multiple organ failure.

Copyright © 2000-2014 TIPS Inc. Unauthorized reproduction of this manual is prohibited. This manual is being used
during review sessions conducted by PharmacyPrep. 6-6
Pharmacyprep.com
14. A
15. C
Tips: ACE Inhibitors cause hyperkalemia.
16. C
17. C
18. C
Tips: Renal nephritis = excessive secretion of proteins in urine.
19. A
20. C
21. D
Tips: Sulindac have least renal problems. However all NSAIDs can cause chronic renal disease and
should be avoided.
22. E
23. D
24. C
25. D

Copyright © 2000-2014 TIPS Inc. Unauthorized reproduction of this manual is prohibited. This manual is being used
during review sessions conducted by PharmacyPrep. 6-7
PharmacyPrep.Com Liver Function and Diseases

PHARMACY PREP
LIVER PHYSIOLOGY AND CHRONIC LIVER DISEASES
1. The signs of alcoholic liver cirrhosis
I-Accumulation of fluid in peritoneal cavity (ascites)
II-Microcytic anemia
III-Blood in gut
A. I only
B. III only
C. I and II only
D. II and III only
E. I, II, III
2. Which of the following is NOT a chronic hepatitis infection?
A. Hepatitis A
B. Hepatitis B
C. Hepatitis C
D. Hepatitis B and C
E. None of the above
3) Obstruction of bile secretion into duodenum from liver is referred as?
A. Cholestasis
B. Cholecystitis
C) Cholelithiasis
D. Hepatic encephalopathy
E. Ascites
4. Which of the following OTC drug most likely associated with hepatotoxicity?
A. Acetyl salicylic acid
B. Ibuprofen
C. Acetaminophen
D. Pepto-Bismol
E. All of the above
5. All of the following are the symptoms of liver toxicity, except?
A. dark urine
B. stomach pain
C. blood in stools
D. yellowing of skin
E. Yellowing of eye and mucus

Copyright © 2000-2014 TIPS Inc. Unauthorized reproduction of this manual is prohibited. This manual is being used
during review sessions conducted by PharmacyPrep. 7-1
PharmacyPrep.Com Liver Function and Diseases
6. Which of the following vaccine, least likely used in travelers
A. Hepatitis A
B. Hepatitis B
C. Dukoral
D. Gerdasil
E. None of the above
7. Acetaminophen hepatotoxicity can be reduced by:
A. Glucuronidation
B. Glutathione conjugation
C. Acetylation
D. Sulfate conjugation
E. Methylation
8. Which of the following non-toxic end product of glutathione conjugation?
A. Acetylcysteine
B. cysteine
C. Mercapturic acid
D. Glutathione conjugation
E. Glutathione only
9. If a patient take hepatitis B vaccine, this vaccine also protects?
A. Hepatitis A
B. Hepatitis C
C. Hepatitis D
D. Hepatitis A, B & C
E. Hepatitis A, B & D
10. Liver cirrhosis is a type of end stage chronic liver disease (liver cirrhosis)? What clinical laboratory
tests are elevated?
A) ALT
B) AST
C) Bilirubin
D) Albumin
E) Proteins
11. All of the following open in duodenum of small intestine, except?
A. Saliva
B. Bile
C. Pancreatic duct
D. Gastric secretions

Copyright © 2000-2014 TIPS Inc. Unauthorized reproduction of this manual is prohibited. This manual is being used
during review sessions conducted by PharmacyPrep. 7-2
PharmacyPrep.Com Liver Function and Diseases
12. Hepatitis A mainly caused by:
A. Sexual contact
B. Blood transfusion
C. Food and drink contamination
D. Traveling abroad
E. Drugs
13. A person is travelling to south east Asia, should take hepatitis vaccine?
A) Hepatitis A only
B) Hepatitis B only
C) Hepatitis A and B
D) Hepatitis A, B and C
E) Hepatitis B and C
14. Which of the following drugs metabolism decreased in chronic hepatic diseases?
A. Proton pump inhibitors
B. Warfarin
C. antacids
D. Lipid soluble drugs
E. Parenteral drugs
15. A patient is on treatment of ascites using diuretics. Which of the following drug use is the
pharmacist concern?
A. spironolactone
B. NSAIDs
C. acetaminophen
D. Ibuprofen
E. codeine
16. Lactulose is laxative is used to treat the following chronic liver disease?
A. constipation
B. ascites
C. encephalitis
D. liver cirrhosis
E. Hepatitis
17. Travelers should receive the following vaccine to prevent water and food contaminated
hepatitis?
A. Hepatitis A
B. Hepatitis B
C. Hepatitis A and B
D. Hepatitis C
E. Hepatitis D

Copyright © 2000-2014 TIPS Inc. Unauthorized reproduction of this manual is prohibited. This manual is being used
during review sessions conducted by PharmacyPrep. 7-3
PharmacyPrep.Com Liver Function and Diseases
18. What type hepatitis can be prevented by prophylaxis?
A. Hepatitis A
B. Hepatitis A and B
C. Hepatitis B and C
D. Hepatitis A, B, C
E) Hepatitis A, B, C, D and E
19. Hepatitis B is transmitted by all, except?
A) contact with blood and its products
B) sexual transmission
C) contact oral secretion
D) sharing needle
E) blood transfusion
20. Phase II metabolic reaction glucuronidation is effected by
Enterohepatic recirculation. Which of the following drugs are effected?
A) drugs that undergoes phase II metabolism
B) estrogen/progestins oral contraceptive pills
C) drugs with phase I metabolism
D) Drug with first pass metabolism
E) Lipid soluble drugs

Copyright © 2000-2014 TIPS Inc. Unauthorized reproduction of this manual is prohibited. This manual is being used
during review sessions conducted by PharmacyPrep. 7-4
PharmacyPrep.Com Liver Function and Diseases
ANSWERS:
1. E
2. A
3. A
4. C
5. C
6. D
Tips: Gerdasil vaccine is given for cervical cancer, from the age 13 to 26. this prevents cervical cancer
caused by papilloma virus
7. B
Tips: glutathione conjugation reduce hepatotoxicity.
8. C
Tips: Hepatitis vaccination is recommended for travelers. Dukoral is oral vaccine for traveler’s
diarrhea. Gerdasil is vaccine human papilloma virus (HPV), which is administered in teenagers to
prevent cervical cancer.
9. C
Tips: hepatitis B is a DNA type of virus. Whereas hepatitis D is RNA type of virus. However, hepatitis
D to grow require assistance of hepatitis B is DNA. Thus by preventing hepatitis B, also prevents
infection of hepatitis D.
10. B
Tips: Alcoholic liver disease can result into liver cirrhosis. AST to ALT ratio >2:1 strongly suggest
alcohol abuse.
11. A
12. C
13. C
14. B
Tips: warfarin mechanism of action takes place in liver
15. C
Tips: acetaminophen is hepatotoxic. It is concern in chronic liver diseases like ascites.

Copyright © 2000-2014 TIPS Inc. Unauthorized reproduction of this manual is prohibited. This manual is being used
during review sessions conducted by PharmacyPrep. 7-5
PharmacyPrep.Com Liver Function and Diseases
16. C
Tips: Lactulose traps excessive ammonia that is produced by chronic liver disease encephalitis.
17. A
18. B
19. C
Tips: oral secretion contact can cause hepatitis A.
20. A

Copyright © 2000-2014 TIPS Inc. Unauthorized reproduction of this manual is prohibited. This manual is being used
during review sessions conducted by PharmacyPrep. 7-6
PharmacyPrep.Com Respiratory System

PHARMACY PREP.
RESPIRATORY COMPLICATIONS
1) Emphysema is?
A) COPD
B) Fibrosis
C) Bronchitis
D) Airway disease
E) None of the above
2. Dyspnea means
A. Painful muscle spasms
B. Pain in the heart
C. Pain in extremities
D. Painful breathing
E. Painful menstruation

3. Which of the following vitamins is depleted in the blood by smoke?


A. Vitamin C
B. Vitamin B
C. Vitamin E
D. Vitamin D
E. Vitamin A
4. Condition characterized by a reversible form of airflow obstruction is
known as:
A. Aneurism
B. Emphysema
C. Embolism
D. Cirrhose
E. Jaundice
5. Advantages of nasal route over oral route absorption of drugs
may include:
I- Nasal route has a higher surface area than GI tract
II- Nasal route has more blood vessels
III- Nasal route has a higher onset of absorption
A. I only
B. III only
C. I and II only
D. II and III only
E. All are correct

Copyright © 2000-2014 TIPS Inc. Unauthorized reproduction of this manual is prohibited. This manual is being used
during review sessions conducted by PharmacyPrep. 8-1
PharmacyPrep.Com Respiratory System
6. Which is not a drug used for smoke cessation?
A. Nicotine polacrilex
B. Transdermal nicotine patches
C. Nicotine sublingual tablets
D. Bupropion
E. Buspirone
7. Example of leukotriene antagonist drug includes:
A. Salbutamol
B. Ipratropium
C. Zafirlukast
D. Montelukast
E. C and D are right
8. Which reaction can ozone cause in the lungs?
A. Oxidation
B. Hydration
C. Reduction
D. Complexation
E. Hydrolysis
9. Correct statements regarding glossitis include which of the following?
I- Inflammation of the tongue
II- Characterized by presence of pinkish-red central lesions on the dorsal surface of the tongue
III- It can be a chronic condition of the tongue seen in pernicious anemia.
A. I only
B. III only
C. I and II only
D. II and III only
E. All are correct
10- Intal-Sodium cromoglycate is characterized by which of the following statements?
I- Non-steroidal drug with anti-inflammatory proprieties
II- They can be used in prophylaxis of asthma and allergic rhinitis
III Used in asthma only
A. I only
B. III only
C. I and II only
D. II and III only
E. All are correct

Copyright © 2000-2014 TIPS Inc. Unauthorized reproduction of this manual is prohibited. This manual is being used
during review sessions conducted by PharmacyPrep. 8-2
PharmacyPrep.Com Respiratory System
11. Drug of choice in the treatment of chronic obstructive pulmonary disease may include:
A. Salbutamol
B. Ipratropium
C. Salmereol
D. Sodium cromoglycate
E. Fluticasone
12. Chronic progressive lower airway destruction causing reduced pulmonary inspiration and
expiratory capacity is known as:
A. Asthma
B. Bronchitis
C. GERD
D. COPD
E. CHF
13. Asthma is a condition of respiratory tract that may be aggravated by:
A. Allergens
B. Cold weather
C. Execise
D. Emotion stress
E. All are correct
14. Which of the following is considered as the best treatment of asthma?
A. β2-agonists
B. β-Blockers
C. α-agonists
D. Anticholinergic drugs
E. Cholinergic drugs
15. Which of the following agents or combinations may be appropriated to treat severe intermittent
asthma?
A. High doses of inhaled corticosteroid
B. Long acting β2-agonists
C. Short acting β2-agonists
D. Oral corticosteroids
E. Combination of all the above medication
16. Examples of drug(s) that may precipitate asthma include all of the following, EXCEPT:
A. Aspirin
B. β-Blockers
C. Ipratropium
D. NSAIDs
E. Cholinergic drugs

Copyright © 2000-2014 TIPS Inc. Unauthorized reproduction of this manual is prohibited. This manual is being used
during review sessions conducted by PharmacyPrep. 8-3
PharmacyPrep.Com Respiratory System
17. All of the following are pathophysiologic characteristics of asthma, EXCEPT:
A. Airway obstruction and inflammation
B. Destructive changes in the alveolar walls
C. Thickened smooth muscle of airway
D. Mucosal edema
E. Narrowed lumen of airway
18) The MOST well known characteristic symptom of asthma include:
A. Wheezing
B. Mucosal edema
C. Cough
D. Chest tightness
E. Tachycardia
19. Drug considered as the drug of choice in a specific asthma emergency treatment include:
A. Salmeterol
B. Adrenaline
C. Albuterol
D. Cortisone
E. Zafirlukst
20. Salbutamol is short acting β2-agonists that may be used in the treatment of which of the
following asthma type?
A. Mild asthma
B. Emergency asthma
C. Severe intermittent asthma
D. Moderate asthma
E. All kind of asthma
21. All are correct concerning the action of corticosteroids in asthma status, EXCEPT:
A. Suppress the inflammatory response
B. Decrease production of inflammatory mediators
C. Decrease airway responsiveness to inflammation
D. Relieve brochocontriction
E. Increase β-agonist receptors response
22. All are examples of corticosteroids inhalers used in asthma, EXCEPT:
A. Beclometasone
B. Sodium cromoglycate
C. Flunisolide
D. Fluticasone
E. Budesonide

Copyright © 2000-2014 TIPS Inc. Unauthorized reproduction of this manual is prohibited. This manual is being used
during review sessions conducted by PharmacyPrep. 8-4
PharmacyPrep.Com Respiratory System
23. All are examples of drugs used in the treatment of bronchoconstriction, EXCEPT:
A. β2-agonists
B. Anticholinergic
C. Methyl xanthenes
D. Xanthine oxidase inhibitors
E. Leukotriene modifiers
24. Best drug used in the prevention of exercise-induced asthma may include:
A. Long acting β2-agonists
B. Short acting β2-agonists
C. Anticholinergics
D. Leukotriene modifiers
E. Corticoids
25. All are β2-agonists used as brochodilators in the treatment of bronchospasm, EXCEPT:
A. Albuterol
B. Salmeterol
C. Terbutaline
D. Pindolol
E. Levalbuterol
26. Example of methyl xanthine used in the treatment of respiratory complications includes:
A. Theophyllin
B. Terbutaline
C. Ipratropium
D. Zafirlukast
E. Salmeterol
27. Agents considered as precursor of leukotriene include:
A. Tyrosine
B. Cyanide
C. Arachidonic acid
D. Prostaglandins
E. Histamine
28. Indication of leukotrienes modifiers in the treatment of asthma includes which of the following?
I- Acute treatment
II- Prophylaxis
III- Chronic treatment
A. I only
B. III only

Copyright © 2000-2014 TIPS Inc. Unauthorized reproduction of this manual is prohibited. This manual is being used
during review sessions conducted by PharmacyPrep. 8-5
PharmacyPrep.Com Respiratory System
C. I and II only
D. II and III only
E. All are correct
29. Drug known as the new relative cromolyn like drug includes:
A. Cromolyn sodium
B. Cromoglycanate sodium
C. Cromolyin disodium
D. Disodium cromoglycanate
E. Nedocromil sodium
30. Leukotriene antagonist is referred as steroid sparing agents, thus it is indicated to:
A. Reduce corticosteroid doses
B. To avoid increase of corticosteroid dose
C. Do not used together with corticosteroids
D. It increases toxicities of steroids
31. Emphysema is?
A. Asthma
B. COPD
C. Chronic bronchitis
D. Permanent enlargement of alveoli
E. None of the above
32. Montelukast a Leukotriene antagonist therapeutic use include?
I-Acute asthma exacerbations
II-Asthma maintenance
III-ASA induced asthma
A. I only
B. III only
C. I and II only
D. II and III only
E. All are correct
33. Patient has a relevant increase in his asthma symptoms due to environmental allergies. He is
currently having salbutamol and fluticasone inhalers. What would you advise as an adjunct
medication to treat his symptoms?
A. Oral corticosteroids
B. Higher dose of corticosteroid inhalers
C. Salmeterol
D. Sodium cromoglycate
E. Anticholinergic agents

Copyright © 2000-2014 TIPS Inc. Unauthorized reproduction of this manual is prohibited. This manual is being used
during review sessions conducted by PharmacyPrep. 8-6
PharmacyPrep.Com Respiratory System
34. Treatment of smoke withdrawal symptoms may include which of the following?
I- Nicorette
II- Nicoderm
III- Zyban
A. I only
B. III only
C. I and II only
D. II and III only
E. All are correct
35. What drugs are used to acute asthma?
A. Inhaled corticosteroids
B. Salbutamol
C. Salmeterol
D. Prednisone po
E. hydrocortisone iv
36. Smoking is risk factor of the following except:
A. COPD
B. CVD
C. Cancer
D. Dermatitis
37. Drugs use to treat asthma causes:
A. Bronchodilatation
B. Vasodilatation
C. Bronchoconstriction
D. Arterial dilatation
38. Factors that decrease the respiration except:
a. Mucosal edema
b. Increase brachial secretion
c. Increase ventilation
d. Bronchospasm
39. Defined as the gaseous exchange that occurs at alveoli capillary membrane:
a. Respiration
b. Ventilation
c. Inhalation
d. Expiration

Copyright © 2000-2014 TIPS Inc. Unauthorized reproduction of this manual is prohibited. This manual is being used
during review sessions conducted by PharmacyPrep. 8-7
PharmacyPrep.Com Respiratory System
40. Bronchodilators for asthma:
a. Are often used as oral meter-dose inhalers, diskus, turbuhaler
b. Are obsolete since the introduction of nedocromil
c. Include theophylline, B2 agonists, and corticosteroids
d. Are ineffective by inhaled route.

Copyright © 2000-2014 TIPS Inc. Unauthorized reproduction of this manual is prohibited. This manual is being used
during review sessions conducted by PharmacyPrep. 8-8
PharmacyPrep.Com Respiratory System
CORRECT ANSWERS
RESPIRATORY COMPLICATIONS
1. A
2. D
3. A
Tips: Smokers have more tendencies to vitamin C depletetion rather than other vitamins
4. B
Tips: Aneurysm is an abnormal dilatation of a blood vessel while embolism is an obstruction of
airway by a mass transported in the circulation. Cirrhoses and jaundice are liver related
complications.
5. D
Tips: Although nasal route has a smaller surface area than intestine drugs administrated by nasal
route will have faster onset of action due to large quantity of blood vessels in the nasal route.
6. E
Tips: Buspirone is an atypical anxiolitic agent used to treat anxiety disorders.
7. E
Tips: Zafirlukast and montelukast are classics leukotriene antagonists used in the oral treatment of
asthma
8. A
Tips: Ozone is a gas that may cause irritation and toxicity of pulmonary system. The main reaction of
ozone in the body is oxidation.
9. E
Tips: Glossitis is an inflammatory disease of tongue of unknown etiology characterized by multiple
annular areas of desquamation of the filiform papillae on the dorsal surface of the tongue. Usually
presenting pinkish-red central lesions, normally seen in pernicious anemia.
10. C
Tips: Sodium cromoglycanate is an anti-inflammatory, nonsteroid inhaler, mast cell stabilizer used in
asthma prophylaxis and as antiallergic agent.
11. B

Copyright © 2000-2014 TIPS Inc. Unauthorized reproduction of this manual is prohibited. This manual is being used
during review sessions conducted by PharmacyPrep. 8-9
PharmacyPrep.Com Respiratory System
Tips: Ipratropium is a cholinergic antagonist used as the drug of choice in the treatment of COPD-
Chronic Obstructive Pulmonary Disease by blocking the brochoconstriction caused by activation of
the parasympathetic nervous system.
12. D
Tips: COPD-Chronic Obstructive Pulmonary Disease
13. E
Tips: Asthma is a chronic inflammatory disorder of the airway that may be induced by allergens,
occupational exposure, exercise, emotion, weather, drugs and so on.
14. A
Tips: Inhalated short-acting β-agonists are the most effective drugs available for all the kinds of
asthma treatment.
15. E
Tips:Severe intermittent asthma treatment is best done by short and long-acting β-agonists plus
inhaled and oral corticosteroids.
16. C
Tips: Ipratropium is an anticholinergic drug that be used in adjunct treatment of asthma.
17. B
Tips: Destructive changes in the alveolar walls are characteristics of a respiratory complication called
“Emphysema”
18. A
Tips: Mainly symptoms of asthma are: Wheezing, shortness of breathing, chest tightness, cough,
tachycardia, tachypnea and pulses paradoxus.
19. C
Tips: Albuterol is the fastest acting β-agonist used in the treatment of bronchocostriction
emergencies-asthma.
20. E
Tips: Salbutamol is a fast acting β-agonist and considered the drug of choice in all kind of asthma.
21. D
Tips: The use of corticosteroids in asthma suppresses the inflammatory response and decreases the
airway hyperesponsiveness by decreasing the production of inflammatory mediators and mucus
production.
22. B

Copyright © 2000-2014 TIPS Inc. Unauthorized reproduction of this manual is prohibited. This manual is being used
during review sessions conducted by PharmacyPrep. 8-10
PharmacyPrep.Com Respiratory System
Tips: Sodium cromoglycanate is an anti-inflammatory, antiallergenic and mast cell stabilizer drug.
23. B
Tips: Xanthine oxidase inhibitor is mainly used in the treatment of gout.
24. B
Tips: Salbutamol, a fast acting β-agonist, is the most effective drug in the prevention of exercise-
induced asthma.
25. D
Tips: Pindolol is a β-blocker used mainly in the treatment of hypertension.
26. A
Tips: Ipratropium is an anticholinergic drug used in COPD as the drug of choice.
27. C
Tips: Cysteinyl leukotrienes are products of arachidonic acid metabolism. Leukotriene modifiers
drugs inhibit leukotriene synthesis by inhibiting 5-lipoxygenase that catalyzes the conversion of
arachidonic acid to leukotriene.
28. D
Tips: Leukotriene modifiers are approved for oral prophylaxis and chronic treatment of asthma.
29. E
Tips: Nedocromil sodium is the new relative cromolyn like drug acting in a very similar way as sodium
cromoglycanate does.
30. A
31. D
32. D
33. D
Tips: Sodium cromoglycanate would be the best choice because it is a safe anti-inflammatory inhaler
for children and very useful in treating environmental allergy and asthma prophylaxis.
34. C
Tips: Nicoret and nicoderm are nicotine replacement therapy for smoking cessation, therefore their
use avoid the smoke withdrawal effect.

Copyright © 2000-2014 TIPS Inc. Unauthorized reproduction of this manual is prohibited. This manual is being used
during review sessions conducted by PharmacyPrep. 8-11
PharmacyPrep.Com Respiratory System
35. B
36. Ans: D
Tips: Risk factors includes: COPD, Cancer, Ulcer, Stroke, and CVD
37. Ans: A
38. Ans: C
39. Ans: A
40. Ans: A
Tips: Nedocromil is not a bronchodilator.
BIBLIOGRAPHIC REFERENCE
1- COMPREHENSIVE PHARMACY REVIEW – Lippincott William & Wilkins – Fourth edition
2- CPS-COMPENDIUM OF PHARMACEUTICALS AND SPECIALITIES - Canadian Pharmacist Association –
2001 edition.
th
3- MEDICAL DICTIONARY – Dorland’s illustrated – 27 edition.
4- PHARMACY PREP – Lectures series & study guide for Evaluating Examination-TIPS - 2003/2004
5- THERAPEUTIC CHOICES – Canadian Pharmacist Association -Third edition
th
6- USP DI – Drug Information for the Health Care Professional–15 edition – Volume I.

Copyright © 2000-2014 TIPS Inc. Unauthorized reproduction of this manual is prohibited. This manual is being used
during review sessions conducted by PharmacyPrep. 8-12
www.pharmacyprep.com Urinary System

Pharmacy Prep

Urinary System
1. Which of the following is the pathophysiological defence mechanism of urinary tract infection?
A) urine acidic pH is intolerable by bacteria
B) Female gender
C) antibiotic use
D) Age
E) bacterial colonization
2. The drug of choice for the treatment of urinary incontinence is?
A. Diphenhydramine
B. Ipratropium
C. Tiotropium
D. Furosemide
E. oxybutynin
3. Which of the following is NOT a symptoms of benign prostatic hyperplasia?
A. Frequent urine
B. Nocturia
C. Irritation
D. Jet urination
E. All of the above
4. Which of the following is NOT a symptom of urinary tract infection?
A) increase in urinary frequency
B) burning sensations
C) Fever
D) bactiurea
E) Weight loss
5. What is the drug of choice against benign prostatic hyperplasia?
A. oxybutynin
B. Furosemide
C. Finasteride 5 mg (Proscar)
D. Finasteride 1 mg (Propesia)
E. Tamsulosin
6) Which of the following is stress incontinence?
A) relaxed pelvic floor
B) Urethral blockade

Copyright © 2000-2014 TIPS Inc. Unauthorized reproduction of this manual is prohibited. This manual is being used
during review sessions conducted by PharmacyPrep. 09-1
www.pharmacyprep.com Urinary System
C) inability void urine
D) Urgency of urine
E) Urinary retention
7. A 70 yo Parkinson's disease patient visiting in a new place like airport or offices. However, often
cause involuntary passage of urine. This can be categorized as?
A. involuntary incontinence
B. urge incontinence
C. Stress incontinence
D. overflow incontinence
E. None
8. Which of the following conditions have NO urinary burning sensation?
A. Urinary tract infections
B. Prostatitis
C. Benign prostatic hyperplasia
D. Urinary incontinence
E. Cystitis
9) Which of the following medical conditions cause difficulty to urination in stream?
A. Urinary tract infections
B. Prostatitis
C. Benign prostatic hyperplasia
D. Urinary incontinence
E. Cystitis
10) What type of urinary incontinence is caused by drugs that cause urinary retention side effects?
A) urge incontinence
B) stress incontinence
C) overflow incontinence
D) all of the above
11)Tamsulosin is used to treat benign prostatic hyperplasia symptoms. It blocks alpha1a receptors
and facilitates urine flow?
A) coronary arteries
B) bladder arteries
C) prostate arteries
D) cerebral arteries
E) all of the above

Copyright © 2000-2014 TIPS Inc. Unauthorized reproduction of this manual is prohibited. This manual is being used
during review sessions conducted by PharmacyPrep. 09-2
www.pharmacyprep.com Urinary System
12) Testosterone to dihydrotestosterone is catalyzed by 5-alpha reductase. The dihydrotesterone
cause prostate growth. What drugs inhibit 5-alpha reductase?
A) Alpha blockers
B) tamsulosin
C) terazosin
D) androgen agonist
E) androgen antagonist

Copyright © 2000-2014 TIPS Inc. Unauthorized reproduction of this manual is prohibited. This manual is being used
during review sessions conducted by PharmacyPrep. 09-3
www.pharmacyprep.com Urinary System
ANSWERS.
1. A
2. E
3. D
Tips: in BPH normal stream of urination is not possible.
4. E
5. C
6. A
7. B
8. D
9. C
10. C
Tips: avoid urinary retention drugs in overflow incontinence if possible.
11. Ans. B
12.E
Tips. Finasteride and dutasteride are androgen antagonist. act by inhibiting 5-alpha reductase.

Copyright © 2000-2014 TIPS Inc. Unauthorized reproduction of this manual is prohibited. This manual is being used
during review sessions conducted by PharmacyPrep. 09-4
www.PharmacyPrep.com Human anatomy

PHARMACY PREP
The Eye
1. What is the other name of blind spot?
A. Optic disc
B. Retina
C. Cornea
D. Optic nerve
E. None of the above
Ans: A
2) What is the rate-limiting step in ophthalmic drops?
A. Optic disc
B. Retina
C. Cornea
D. Optic nerve
E. None of the above
Ans:
3. At night dark vision (dimlight), what is the most sensitive for motion, photoreceptors in retina?
A) Cones
B) Rods
C) Cones and rods
D) Optic nerve
E) Cornea
Ans: B
4. What photoreceptors are active in color light?
A. Cones
B. Rods
C. Cones and rods
D. Optic nerve
E. Cornea
Ans: A
Tips: C for color and C for cones
5. What is myopia?
A. night vision
B. far sight
C. near sight
D. color vision
E. color blindness

Copyright © 2000-2014 TIPS Inc. Unauthorized reproduction of this manual is prohibited. This manual is being used
during review sessions conducted by PharmacyPrep. 10-1
www.PharmacyPrep.com Human anatomy
Ans: C
6. Drugs that cause pupil dilatation, except?
A.Morphine
B. Atropine
C. Benztropin
D. Ipratropium
E. Trihexphenidyl
Ans: A
Tips: all anticholinergic drugs cause pupil dilatation
7. Hyperopia is?
A. Far sight
B. Near sight
C. Glaucoma
D. None of the above
Ans: A
8. The sclera and cornea constitute this layer of the eye: a. retinal tunic b. vascular tunic c. fibrous
tunic d. nervous tunic
Ans: C
9. "Night blindness" is an early sign of: a. cataracts b. glaucoma c. vitamin A deficiency d. myopia
Ans: C
10. Sympathetic stimulation of the iris causes: a. astigmatism b. pupillary constriction c. pupillary
dilation d. glaucoma
Ans: C
11. Cone cells are photoreceptors that: a. respond to low light levels b. respond to colored light c.
are found in the optic disc d. are found in the vascular tunic
Ans: B

Copyright © 2000-2014 TIPS Inc. Unauthorized reproduction of this manual is prohibited. This manual is being used
during review sessions conducted by PharmacyPrep. 10-2
www.PharmacyPrep.com Human anatomy
12. The region where the lens focuses the image onto the retina is the: a. optic nerve b. fovea c.
pupil d. blind spot
Ans: B
13. The fluid that fills the posterior chamber of the eye is the: a. lacrimal fluid b. vitreous humor c.
aqueous humor d. jocular humor
Ans: B
14. The major light absorbing pigment in retinal photoreceptors is: a. rhodopsin b. melanin c.
glutamate d. chlorophyll
Ans: A
15. Photoreceptors release more neurotransmitters: a. in brighter light b. in darkness c. only when
stimulated by bipolar cells d. none of the above
Ans: B
16) What is incorrect about sympathetic blockers?
A) decrease glaucoma
B)decrease IOP by inhibiting formation of aqueous humor.
C) Decrease IOP by increase outflow of aqueous humor.
D) first line therapy for glaucoma
E) ophthalmic drops are available
Ans. C
17. What are the more sensitive photoreceptors in eye?
A. Cones
B. rods
C. rods and cones
D. cataract
E. pupil
Ans: B

Copyright © 2000-2014 TIPS Inc. Unauthorized reproduction of this manual is prohibited. This manual is being used
during review sessions conducted by PharmacyPrep. 10-3
www.PharmacyPrep.com Human anatomy
18. What is correct statement about photo receptors in eye?
A. Rods are sensitive to color light
B. Cones are active in dark
C. Rods are less in number than cones
D. Cones are less in number than rods
E. Cones and rods photo cells are equally to sensitive
Ans: D
19. Which of the following drugs can cause dry eye side effect?
A. Salbutamol
B. Timolol
C. Latonoprost
D. Epinephrine
E. Atropine
Ans: E
20. Which of the following can discolour lens?
A. Timolol
B. hydrochlorothiazide
C. Rifampin
D. Metronidazole
E. Atropine
Ans: C
21. Emergence medication in glaucoma?
A. pilocarpine
B. acetazolamide
C. atropine
D. Timolol
Ans: A
22. All of the following drug have effect on eye, except?
a) amiodarone
B) hydrochloroquine
C) rifampin
D) ethambutol
E) isotretinoin
23. JP is a 50 year old asthma patient and also receiving treatment for hypertension and using
metoprolol 50 mg daily. Recently diagnosed for glaucoma. Doctor considering to prescribe
latanoprost to treat glaucoma. Why doctor want to choose prostaglandin analog?
A) patient has asthma and also using metoprolol
B) patient has hypertension also using metoprolol

Copyright © 2000-2014 TIPS Inc. Unauthorized reproduction of this manual is prohibited. This manual is being used
during review sessions conducted by PharmacyPrep. 10-4
www.PharmacyPrep.com Human anatomy
C) Patient has asthma and hypertension
D) Patient is 50 year old, asthma and hypertension
E) Patient has open angle glaucoma

Copyright © 2000-2014 TIPS Inc. Unauthorized reproduction of this manual is prohibited. This manual is being used
during review sessions conducted by PharmacyPrep. 10-5
www.PharmacyPrep.com Human anatomy
ANSWERS:
1. A
2. C
3. B
4. A
Tips: C for color and C for cones
5. C
6. A
Tips: all anticholinergic drugs cause pupil dilatation
7. A
8. C
9. C
10. C
11. B
12. B
13. B
14. A
15. B
16. C
17. B
18. D
19. E
20. C
21. A
22. ans: C
23.

Copyright © 2000-2014 TIPS Inc. Unauthorized reproduction of this manual is prohibited. This manual is being used
during review sessions conducted by PharmacyPrep. 10-6
PharmacyPrep.Com Blood and Anemia

PHARMACY PREP
BLOOD AND ANEMIA
1. Microcytic anemia is characterized as?
A. Hypochromic anemia
B. Vitamin B12 deficient anemia
C. Folate deficient anemia
D. Sickle cell anemia
E. Decreased total iron binding capacity
2. Megalobastic anemia is characterized as?
A. Iron deficient anemia
B. Vitamin B12 and folic acid deficient anemia
C. Folate deficient anemia
D. Sickle cell anemia
E. Pernicious anemia
3. Pernicious anemia is characterized as?
A. Iron deficient anemia
B. Vitamin B12 deficiency only
C. Folate deficient anemia
D. Sickle cell anemia
E. Vitamin B12 and folic acid deficiency
4. Pernicious anemia is due to?
A. Iron deficient anemia
B. Deficiency of intrinsic factors
C. Folate acid synthesis inhibitors
D. Sickle cell anemia
E. Hemolytic anemia
5. Intrinsic factors are secreted from?
A. Esophagus
B. Stomach
C. Small intestine
D. Large intestine
E. Parietal cells
6. Pernicious anemia patient should NOT take vitamin B12 supplement by?
A. Oral
B. Subcutaneous injection
C. Intra muscular injection
D. Intravenous
E. None

Copyright © 2000-2014 TIPS Inc. Unauthorized reproduction of this manual is prohibited. This manual is being used
during review sessions conducted by PharmacyPrep. 11-1
PharmacyPrep.Com Blood and Anemia
7. A 75 years old elderly person have deficiency of cyanocobalamine, this can lead to:
I-Megalobastic anemia
II-Mycrocytic anemia
III-Intrinsic factor
A. I only
B. III only
C. I and II Only
D. II and III only
E. I, II, III
8. Megaloblastic anemia may be caused due to a result of:
I-Folic acid deficiency
II-Vitamin B12 deficiency
III-Iron deficiency
A. I only
B. III only
C. I and II Only
D. II and III only
E. I, II, III
Tips: Iron deficiency is called micro cystic anemia while megaloblastic anemia is a deficiency of
cyanocobalamin and folic acid.
9. In child bearing age megaloblastic anemia is common, and it is due to deficiency of?
A. Vitamin B12
B. Iron
C. Folic acid
D. Intrinsic factor
E. vitamin B12 and folic acid
10. Megaloblastic anemia is due to deficiency of?
A. Vitamin B12
B. Iron
C. Folic acid
D. Intrinsic factor
E. vitamin B12 and folic acid
11. A 75 yo frequent customer of your pharmacy has been diagnosed with megaloblastic anemia,
and it is due to deficiency of?
A. Vitamin B12
B. Iron
C. Folic acid
D. Intrinsic factor
E. genetic defect

Copyright © 2000-2014 TIPS Inc. Unauthorized reproduction of this manual is prohibited. This manual is being used
during review sessions conducted by PharmacyPrep. 11-2
PharmacyPrep.Com Blood and Anemia
11. A 35 yo frequent customer of your pharmacy has been diagnosed with megaloblastic anemia.
Currently she is on treatment of epilepsy and using phenytoin capsule. His megaloblastic anemia is
due to deficiency of?
A. Vitamin B12
B. Iron
C. Folic acid
D. Intrinsic factor
E. genetic defect
12. Filgrastim is:
A. Interleukin
B. Colony stimulating factor
C.Tumor necrosis factor
D. Oprelvekin
E. Erythropoietins
13. The MOST common cause for megaloblastic anemia is
A. Gastric bleeding
B. Lack of dietary iron
C. Lack of dietary folic acid
D. Lack of dietary pyridoxine
E. Lack of absorption of vitamin B12

14. Hematocrit is?


A. Circulating iron storage
B. Measures the total iron binding capacity
C. Measures the total free iron
D. Packed volume of RBCs
E. It is serum ferritin
Tips: The percentage of red blood cells to the blood volume is the Hct. (Packed cell volume).
15. Which of the following blood cells cannot be distinguished from basophils?
A. Neutrophils
B. White blood cells
C. Eosinophills
D. Red blood cells
E. Mast cells
16. Low mean cell volume (MCV) indicates?
A. Megaloblastic anemia
B. Microcytic anemia
C. Hemolytic anemia
D. Narmocytic anemia
E. Aplastic anemia

Copyright © 2000-2014 TIPS Inc. Unauthorized reproduction of this manual is prohibited. This manual is being used
during review sessions conducted by PharmacyPrep. 11-3
PharmacyPrep.Com Blood and Anemia
17. Pernicious anemia is
A. Due to deficiency intrinsic factor thus no oral vitamin B12 absorption
B. Due to deficiency of vitamin B12, administer oral vitamin B12
C. Due to deficiency of folic acid supplements, administer folic acid supplements
D. Due to deficiency of iron supplements, administer iron supplements
E. Due to deficiency of vitamin B12 and folic acid supplements
18) In microcytic anemia (iron deficient), all of the following decrease, except?
A) Mean cell volume (MCV)
B) Hematocrit (Hct)
C) Hemoglobin
D) Serum ferritin
E) Total iron binding capacity (TIBC)
Tips: In microcytic anemia total iron binding capacity increase. Hematocrit is actual volume of RBCs
in a unit volume of whole blood).
19. A customer of your pharmacy calls and express concern that his 2 yr old daughter swallowed 10
iron capsules this morning past 4 hours ago. What to do? So far no unusual symptoms.
A. wait and watch
B. ask to go emergency
C. contact poison control centre and provide phone number
D. Recommend to take excessive fluids
E. ask to see doctor
20. What is true about microcytic anemia?
A. It may cause due deficiency of iron during pregnancy
B. Deficiency of folic intake can cause microcytic anemia
C. Deficiency of folic acid can cause microcytic anemia
D. Deficiency of vitamin B12 can cause microcytic anemia
E. Deficiency of intrinsic factor
21. Laboratory tests for iron deficiency anemia EXCEPT:
A. Hemoglobin (HgB)
B. Hematocrit (Hct)
C. Erythrocyte Sedimentation Rate
D. Serum Ferritin
E. Iron binding capacity

Tips: Erythrocytes sedimentation rate (ESR) is not screening test for microcytic anemia. ESR is used
acute, chronic infections, malignancies, rheumatic factors, and seizures.
22. A patient needs something for anemia as he was diagnosed with it and did not take any
medication and feel fainting and has black stool. He should do all EXCEPT:

Copyright © 2000-2014 TIPS Inc. Unauthorized reproduction of this manual is prohibited. This manual is being used
during review sessions conducted by PharmacyPrep. 11-4
PharmacyPrep.Com Blood and Anemia
A. To see his doctor right away
B. Symptoms may due to GI bleeding
C. Ask if pink color stool as maybe lower GI bleeding
D. Ask about allergy before giving him something

23. Which of the following iron supplements have the highest elemental iron?
A. Ferrous sulphate
B. Ferrous gluconate
C. Ferrous fumarate
D. Combination of iron supplements
24. The most accurate to screening iron deficient anemia is serum ferritin. What is true about serum
ferritin?
A. Increase because serum ferritin is protein that stores iron
B. Increase because serum absorbs water in iron deficiency
C. Decrease because serum ferritin is protein that stores iron.
D. Decrease because serum ferritin increase protein and iron stores
E. No change in serum ferritin
Tips: Serum ferritin is protein that stores iron. In case of iron deficiency, tissue absorbs water and
dilutes iron stores thus decrease serum ferritin.
25. A 79 yo lady comes to pharmacy to buy some iron supplements. She currently using ibuprofen
400 mg qid for her joint pains. She feels like she is very week and fainting. She observed occult blood
in stool, however she is NOT sure? What is appropriate to do?
A. Recommend ferrous fumerate because high elemental iron
B. Recommend ferrous glucose because it is light on stomach
C. Refer her to doctor to determine cause of bleeding
D. Ask her to stop taking ibuprofen
E. Ask her to take low dose of ibuprofen
26. Pregnancy related anemia is due to?
A. deficiency of iron
B. decrease in blood volume of plasma
C. deficiency of vitamin B12
D. deficiency of folic acid
E. deficiency of folic acid and vitamin B12
27) Which of the following is cause of megaloblastic anemia?
A) Due to impaired synthesis of serum ferritin
B) Impaired DNA synthesis
C) Impaired red blood cells formation
D) Due genetic defects in formation of rbc
E) Due to deficiency G6PD enzyme

Copyright © 2000-2014 TIPS Inc. Unauthorized reproduction of this manual is prohibited. This manual is being used
during review sessions conducted by PharmacyPrep. 11-5
PharmacyPrep.Com Blood and Anemia
28) Which of the following is correct about ferrous gluconate 300 mg dose.
A) available by prescription
B) Pharmacist recommended
C) Over the counter or unscheduled
D) Not available in Canada
29) KP is a 30 year old women walks into your pharmacy. She wants to buy iron supplements
because feels tiered and fatigue. What is appropriate action?
A) recommend ferrous gluconate 300 mg daily
B) recommend ferrous sulfate because has the highest elemental iron.
C) Ask her if she had seen doctor for anemia symptoms
D) Ask her take more rest and diet
30) A patient complains constipation and currently using iron supplements. What is correct
recommendation?
I) Take with food
II) Take orange juice
III) Take plenty of water
A) I only
B) III only
C) I and II
D) II and III
E) I, II, III
31) Which of the following is the common deficiency in pregnancy?
A) Iron
B) Vitamin B12
C) Folic acid
D) Multivitamin
E) Dimenhydrinate
32) Death due to cyanide poisoning results from?
A) cyanide hemoglobin complex formation
B) cyanide combining with red blood cells
C) Cyanide inhibiting cytochrome oxidase
D) coronary vessel occlusion

Copyright © 2000-2014 TIPS Inc. Unauthorized reproduction of this manual is prohibited. This manual is being used
during review sessions conducted by PharmacyPrep. 11-6
PharmacyPrep.Com Blood and Anemia
ANSWERS:
1. A
2. B
3. B
4. B
5. E
6. A
7. A
8. C
Tips: Iron deficiency is called micro cystic anemia while megaloblastic anemia is a deficiency of
cyanocobalamin and folic acid.
9.
10.
11.
12. B
13. C
14. D
Tips: The percentage of red blood cells to the blood volume is the Hct. (Packed cell volume).
15. E
16. B
17. A
18. E
Tips: In microcytic anemia total iron binding capacity increase. Hematocrit is actual volume of RBCs
in a unit volume of whole blood)
19. C

Copyright © 2000-2014 TIPS Inc. Unauthorized reproduction of this manual is prohibited. This manual is being used
during review sessions conducted by PharmacyPrep. 11-7
PharmacyPrep.Com Blood and Anemia
20. A
21. C
22. C
23. C
24. C
Tips: Serum ferritin is protein that stores iron. In case of iron deficiency, tissue absorbs water and
dilutes iron stores thus decrease serum ferritin
25. C
26. D
27. Ans: B
Tips: Megaloblastic anemia arise because of impaired DNA synthesis caused by deficiency of vitamin
B12 (cobalamine) or folate or due to impaired DNA and RNA metabolism.
28.
29.
30.
31. A
32.

Copyright © 2000-2014 TIPS Inc. Unauthorized reproduction of this manual is prohibited. This manual is being used
during review sessions conducted by PharmacyPrep. 11-8
PharmacyPREP.COM Biochemistry

PHARMACY PREP
BIOCHEMISTRY
1. Glycine reaction is normally mediated by:
A. Acetyl Coenzyme A
B. UDP Glucoronyl tranferase
C. PAPS- Phosphoadenosine-5-Phosphosulfate
D. Sulfotranferase
E. GST-Glutathione S-Transferase
2. Abundant element in the interstitial fluid may include:
A. Potassium
B. Sodium
C. Magnesium
D. Calcium
E. Chlorine
3. Which is responsible for nitric oxide (NO) in vivo?
A. Arginin
B. Tryptophan
C. Guanine
D. Thiamine
E. Leucine
Ans: A
4. Enzyme that enables DNA fragments from different sources to be joined:
A. DNA polymerase
B. DNA gyrase
C. DNA ligase
D. RNA transferase
E. None of the above
5. Which of the followings are types of RNA?
I- RNAt - RNA transferase
II- RNAm – RNA mensager
III-RNAr – RNA ribossome
A. I only
B. III only
C. I and II only
D. II and III only
E. All are correct

Copyright © 2000-2014 TIPS Inc. Unauthorized reproduction of this manual is prohibited. This manual is being used
during review sessions conducted by PharmacyPrep. 12-1
PharmacyPREP.COM Biochemistry
6. Which vitamin is/are fat-soluble?
I- Vitamin E
II- Vitamin K
III- Vitamin B
A. I only
B. III only
C. I and II only
D. II and III only
E. All are correct
7. Which of the following vitamin is not water-soluble?
A. Vitamin B
B. Vitamin C
C. Vitamin D
D. Folic acid
E. Riboflavin
8. The most abundant metal ion in human body
A. Iron
B. Magnesium
C. Aluminum
D. Phosphorous
E. Potassium
9. Which of the following is considered the normal potassium serum in human body?
A. 2.0 to 3.5 mEq/L
B. 3.5 to 5.0 mEq/L
C. 5.0 to 7.5 mEq/L
D. 3.5 to 7.5 mEq/L
E. 3.0 to 6.0 mEq/L
10. Which of the following is considered normal sodium serum in human body?
A. 35 to 47 mEq/L
B. 147 to 150 mEq/L
C. 135 to 147mEq/L
D. 75 to 135 mEq/L
E. 125 to 145 mEq/L
11. Which of the following is considered as essential vitamin for breast-feed babies?
A. Vitamin A
B. Vitamin B
C. Vitamin C
D. Vitamin D
E. Vitamim E

Copyright © 2000-2014 TIPS Inc. Unauthorized reproduction of this manual is prohibited. This manual is being used
during review sessions conducted by PharmacyPrep. 12-2
PharmacyPREP.COM Biochemistry
12. Which of the following hormone acts on cell wall?
A. Insulin
B. Prolactin
C. Cortisol
D. Estrogen
E. Alanine
13. Which of the following vitamins has tetrahydrofolate coenzyme activity based on its pteridine
ring?
A. Thiamine
B. Vitamin D
C. Vitamin K
D. Folic acid
E. Pyridoxine
14. What is correct regarding the formation of proteins?
A. Formed by condensation of peptide bond
B. Geometric sequence of amino acids
C. The formation occurs by complexation of amino acids
D. Proteins are formed from carbohydrates
E. None of the above is right
15. Which is the strongest endogenous pain producer in human body?
A. Leukotriene
B. Enkephalin
C. Bradykinin
D. Cytokine
E. Angiotensin
16. Which of the following is considered the FIRST precursor of vitamin A formation?
A. Retinoic acid
B. Carotene
C. Cobalt
D. Calciferol
E. Pantothenic acid
17. Biological catalysts responsible for supporting almost all of the chemical
reactions that maintain human life process:
A. Nucleic acid
B. Amino acids
C. Carbohydrates
D. Oligopeptides
E. Enzymes

Copyright © 2000-2014 TIPS Inc. Unauthorized reproduction of this manual is prohibited. This manual is being used
during review sessions conducted by PharmacyPrep. 12-3
PharmacyPREP.COM Biochemistry
18. Which of the following statements is/are correct regarding enzymes?
I-Enzymes are proteins
II- Enzymes are catalyst because they are never altered during a reaction
III- Michaelis-Menten theory describe the enzymatic reactions
A. I only
B. III only
C. I and II only
D. II and III only
E. All are correct
19. Zwitter ion reacts with a substrate as:
A. Neutral ion
B. Proton acceptor ion
C. Proton donor ion
D. Anionic
E. Cationic
20. Correct statements regarding mutation may include:
I- Cell division
II- Change in genotype
III- Change in DNA replication altering the gene
A. I only
B. III only
C. I and II only
D. II and III only
E. All are correct
21. Which pyrimidine base is not present in DNA?
A. Thiamine
B. Adenine
C. Uracil
D. Cytosine
E. Guanine
22. The codon sequence Adenine-Guanine-Cytosine-Uracil is seen in
I- RNAs
II- DNAs
III- Both DNAs and RNAs
A. I only
B. III only
C. I and II only
D. II and III only
E. All are correct

Copyright © 2000-2014 TIPS Inc. Unauthorized reproduction of this manual is prohibited. This manual is being used
during review sessions conducted by PharmacyPrep. 12-4
PharmacyPREP.COM Biochemistry
23. Shaped network present in the cell between nucleus and cell wall:
A. Ribossomes
B. Endoplasmic reticulum
C. Mitocondria
D. Cell membrane
E. Nucleotide
24. The Michaelis-Menten equation-Enzymatic reactions will appears ZERO order when:
A. The substrate concentration is much smaller than Km
B. When Km is much smaller than the substrate concentration
C. When Vmax is much smaller than Km
D. When Vmax is much larger than Km
E. When Km approaches Vmax
25. Which vitamin can be classified as hormone?
A. Vitamin D
B. Vitamin D3
C. Vitamin E
D. Vitamin K
E. Vitamin B12
26. False statement regarding vitamin D3 metabolism include:
A. Vitamin C facilitates absorption of vitamin D3 in the liver
B. Hydroxylation of D3 is in kidney
C. Hydroxylation of is D3 in the liver
D. Parathyroid hormone activate metabolism of vitamin D3 in the kidney
E. Skin activates synthesis of vitamin D3 by sunlight
27. Hydrolysis of fixed oils may yields:
A. Saturated + Unsaturated acids
B. Glycerol + Fatty acids
C. Glucose + Fatty acids
D. Fatty acids + Triglycerides
E. None of the above
28. Example of compound that is phospholipid includes:
A. Glycogen
B. Leucine
C. Lecithin
D. Casein
E. Phenylalanine

Copyright © 2000-2014 TIPS Inc. Unauthorized reproduction of this manual is prohibited. This manual is being used
during review sessions conducted by PharmacyPrep. 12-5
PharmacyPREP.COM Biochemistry
29. In an enzymatic reaction, addition of competitive inhibitor leads to:
A. Increase the rate of reaction
B. Decrease the rate of reaction
C. Same rate of reaction
D. Decrease rate and increase substrate concentration
E. Decrease both rate and substrate concentration
30) Which of the following is the end product of ANAEROBIC glycolysis?
A) Pyruvic acid
B) CO2
C) CO2 + H2O
D) Lactic acid
E) Proteins
31. Which of the following is the end product of AEROBIC reactions?
A. Pyruvic acid
B. CO2
C. CO2 + H2O
D. Lactic acid
E. Proteins
32. Linolenic acid is type of essential fatty acid?
15129COOH1Alpha-Linolenic Acid (omega-3)
A. omega 3 B. omega 6 C. Omega 9 D. EPA E) DHA
33. Which of the following vitamins deficiencies can occur in patient using low protein diet?
A. Vitamin A
B. Vitamin B1
C. Vitamin B3
D. Vitamin B12
E. Vitamin C
34) All of the following can cause denaturation of proteins, except?
A) acids
B) heating
C) bases
D) mechanical mixing
E) Buffers

Copyright © 2000-2014 TIPS Inc. Unauthorized reproduction of this manual is prohibited. This manual is being used
during review sessions conducted by PharmacyPrep. 12-6
PharmacyPREP.COM Biochemistry
35) Gluconeogenesis is?
A) formation of glucose from glycogen
B) formation of glucose from proteins
C) formation of glucose from fats
D)formation of glucose from fats and proteins
E) formation of glucose from maltose
36) KP is a 35 year old women is using ferrous gluconate to treat anemia. What decreases absorption
of ferrous gluconate?
A) Food
B) Proton pump inhibitors
C) citrus juice
D) Apple juice
E) grapefruit juice
37) What is the end product of carbohydrates?
A) lactose
B) Sucrose
C) Maltose
D) Fructose
E) Glucose
38) What is the end product metabolism?
A) glucose
B) CO2+H2O
C) Fructose
D) Purine
E) Urea
39) What is primary protein structure?
A) linear sequence
B) Alpha helix
C) Beta pleated sheet
D) Protein consisting more than one protein chain
E) Three dimensional structure
40) What aminoacid is precursor for synthesis of –CH2- in tetrapyrrole structure of porphyrin ring in
hemoglobin.
A) alanin
B) Tyrosine
C) Glycine
D) Phenyl alanin
E) Acetyl CoA

Copyright © 2000-2014 TIPS Inc. Unauthorized reproduction of this manual is prohibited. This manual is being used
during review sessions conducted by PharmacyPrep. 12-7
PharmacyPREP.COM Biochemistry
41) Heme containing enzymes are formed from protoporphyrin. What are the examples of heme
containing enzymes or proteins?
a) hemoglobin
b) myoglobin
c) cytochromes
d) all of the above

Copyright © 2000-2014 TIPS Inc. Unauthorized reproduction of this manual is prohibited. This manual is being used
during review sessions conducted by PharmacyPrep. 12-8
PharmacyPREP.COM Biochemistry
ANSWERS:
1. A
Tips: Glycine reaction is a reaction mediated by acetyl Coenzyme A.
2. B
Tips: Interstitial fluid is situated between parts or in the interspaces of a tissue therefore outside the
cells. Sodium is the most abundant element outside the cells therefore in the interstitial fluid.
3. A
Tips: Arginine is an essential amino acid responsible for N.
4. C
Tips: DNA ligase is an enzyme that enables DNA fragments from different sources to be joined.
5. E
Tips: All, RNA transferase, RNA mensager and RNA ribosome are types of RNAs.
6. C
Tips: All vitamins B and vitamin C are water-soluble.
7. C
Tips: Vitamin A, D, E and K are fat-soluble vitamins.
8- A
Comments: Iron is the most abundant metal ion in human body.
9. B
Tips: Normal potassium serum in human body range between 3.5 to 5.0mEq/L.
10. C
Tips: Normal sodium serum in human body range between 135 to 147mEq/L
11. D
Tips: The most essential vitamin for breast-feed babies is vitamin D.
12. A
Tips: Insulin is a hormone that acts on cell wall
13. D
Tips: Folic acid is a vitamin B9 that has tetrahydrofolate coenzyme activity based on its pteridine ring.

Copyright © 2000-2014 TIPS Inc. Unauthorized reproduction of this manual is prohibited. This manual is being used
during review sessions conducted by PharmacyPrep. 12-9
PharmacyPREP.COM Biochemistry
14. A
Tips: Proteins are formed by a reaction between amino acids called condensation between peptid
bonds.
15. C
Tips: Bradykinin is the strongest endogenous analgesic-pain producer in human body.
16. B
Tips: Vitamin A is formed from retinoic acid that is formed from beta-carotene.
17. E
Tips: Enzymes are biological catalysts responsible for supporting almost all of the chemical reactions
that maintain human life process, and accelerate reactions by lowering the energy of the transition
state.
18. E
Tips: Enzymes are proteins or biological catalysts responsible for supporting almost all of the
chemical reactions. They are called catalyst because are never altered during a reaction. Michaelis-
Menten theory describes the enzymatic reactions.
19. A
Tips: Zwitter ion is a neutral ion with two charges, positive and negative, that reacts neutrally with a
substrate.
20. D
Tips: Mutation is a permanent transmissible change in the genetic material, usually in a single gene.
21. C
Tips: Uracil is only present in RNAs.
22. A
Tips: The codon sequence Adenine-Guanine-Cytosine-Uracil is seen in RNAs only. DNAs differ only in
the presence of Thiamine instead of uracil.
23. B
Tips: Endoplasmic reticulum is a shaped network present in the cell between nucleus and cell wall.
24. E
Tips: The Michaelis-Menten equation-Enzymatic reactions will appears ZERO order when Km
approaches Vmax and FIRST order when the substrate concentration is smaller than Km.

Copyright © 2000-2014 TIPS Inc. Unauthorized reproduction of this manual is prohibited. This manual is being used
during review sessions conducted by PharmacyPrep. 12-10
PharmacyPREP.COM Biochemistry
25. B
Tips: Vitamin D3 man be classified as hormone
26. A
Tips: Vitamin C does not interfere with absorption of vitamin D3 in the liver.
27. B
Tips: Glycerol and fatty acids are the end products of hydrolysis of fixed oils.
28. C
Tips: Lecithin is a phospholipid
29. C
Tips: Addition of competitive inhibitor in an enzymatic reaction does not alter the rate of reaction.
30. D
Tips: Pyruvic acid is the end product of anaerobic reactions
31. C
Tips: CO2 and H2O are the end products of aerobic reactions.
32. A
33. C
34. E
Tips: Buffers are safe to prepare proteins preps such as insulin's are prepared in buffers.
35. Ans. D
36. Ans: A
37.
38.
39. Ans. A
40. Ans. C
41.

Copyright © 2000-2014 TIPS Inc. Unauthorized reproduction of this manual is prohibited. This manual is being used
during review sessions conducted by PharmacyPrep. 12-11
PharmacyPREP.COM Biochemistry
Copyright © 2000-2014 TIPS Inc. Unauthorized reproduction of this manual is prohibited. This manual is being used
during review sessions conducted by PharmacyPrep. 12-12
www.Pharmacyprep.com Clinical Biochemistry

PHARMACY PREP
CLINICAL BIOCHEMISTRY
1. Thyroid Replacement therapy for hypothyroidism should be monitored by:
A. Free T4
B. Sensitive TSH assay
C. Free thyroxine assay
D. Total thyroxin
E. Serum TSH assay
2. Serum TSH is elevated in?
A. Hyperthyroidism
B. Hypothyroidism
C. Hyperparathyroidism
D. Hypoparathyroidism
E. Pheochromacytoma
3. Which of the following plasma proteins has the GREATEST ability to bind acidic drugs
A. Albumin
B. Fibrinogen
C. Hemoglobin
D. Alpha glycoproteins
E. Beta lipoprotein

4. Which of the following is not the correct match:


A. Hypoparathyroidism hypocalcimia
B. Hypoparathyroidism hyperphosphetemia
C. Hyperparathyroidism hypercalcemia
D. Hyperparathyroidism causes deficiency phosphates
E. Hyperparathyroidism causes deficiency of calcium
5. All of the following are tests are used in monitoring hypothyroidism EXCEPT;
A. Sensitive TSH assay
B. Thyrotropic (TSH) stimulation test
C. Free thyroxin index (FTI)
D. Total thyroxine (TT4)
E. Resin triiodothyronine uptake (RT3U )
6. Which of the following is NOT decreased in elderly?
A. Creatinin clearance
B. Serum creatinin
C. Blood urea nitrogen
D. Albumin

Copyright © 2000-2014 TIPS Inc. Unauthorized reproduction of this manual is prohibited. This manual is being used
during review sessions conducted by PharmacyPrep. 13-1
www.Pharmacyprep.com Clinical Biochemistry
E. Adipose tissue
7. Which of the following is NOT decreased in elderly?
A.fat content
B. lean body mass
C. kidney function
D. Glomerular filtration
E. Serum creatinine
8. What are the following lab test increase indicate myopathies?
A. Creatinin kinase (CK-MM)
B. Creatinin kinase (CK-BB)
C. Creatinin kinase (CK-MB)
D. ADH, AST, ALT & ALP
E. ALP & ALT
9. A patient using warfarin should be monitored for?
A. Creatinine kinase B. PT C. aPTT D-LFT E. RFT
10) Which of the following laboratory investigations help to determine, myocardial infarction?
A) Troponin-I and CK-MB
B) CK-MB, Troponin C and ECG
C) ECG, Troponin I, CK-MM
D) Troponin I, CK-MB, ECG
E) Troponin C, CK-MB, ECG
11. What are the following lab test increase indicate myocardial infarction?
A. Creatinine kinase B. Troponin T C. Troponin I D. Troponin C
E. ALP
12. A patient using warfarin should be monitored for?
I. INR
II. Prothrombin time
III. activated Prothrombin time (aPTT)
A. I only B. III only C. I and II only D. II and III only E. I, II, and III
13. A patient has dark urine, and muscle pains. Which of the following drug may be patient on?
A. Metronidazole
B. Atorvastatin
C. Rifampin
D. Bismuth
E. Lithium

Copyright © 2000-2014 TIPS Inc. Unauthorized reproduction of this manual is prohibited. This manual is being used
during review sessions conducted by PharmacyPrep. 13-2
www.Pharmacyprep.com Clinical Biochemistry
14. A patient creatinine clearance is 300 ml/hr, it is a?
A. chronic renal disease
B. normal renal function
C. excessive renal function
D. severe renal disease
E. Acute renal disease
15. What is the best urine sample for the urine analysis?
A. end of the sample
B. sample from midstream
C. sample from end of stream
D. Sample form beginning of stream
E. any one of them is ok
16) A patient creatinine clearance 300 ml/hour. This patient is categorized as?
A) normal CrCl
B) Chronic renal disease
C) Severe renal disease
D. Renal Failure
E. None of the above
17. Anticancer drugs in chemotherapy often cause the following side effects?
A. Neutropenia
B. Thrombocytopenia
C. Leukopenia
D. decrease in basophils
E. Increased monocyte
18. Which of the following test is the most sensitive to liver cirrhosis?
A. bilirubin
B. AST C. ALT D. ALP E. CK
19. Which of the following is the most sensitive test to detect cholestasis jaundice?
A. bilirubin B. AST C. ALT D. ALP E. CK
20. Which of the following is the most sensitive to detect acetaminophen hepatotoxicity?
A. bilirubin B. AST C. ALT D. ALP E. CK

Copyright © 2000-2014 TIPS Inc. Unauthorized reproduction of this manual is prohibited. This manual is being used
during review sessions conducted by PharmacyPrep. 13-3
www.Pharmacyprep.com Clinical Biochemistry
21. An osteoarthritis patient using NSAIDS to treat joint pain. Approaches to your pharmacy ask if
any pain reliever change dark brown stool color or "coffee ground". What is appropriate?
A) some NSAIDS can change stool color to dark brown
B) No, NSAIDS change stool color however if you notice dark brown of coffee ground stool color you
should contact doctor immediately
C) all NSAIDS give dark stool color
D) color change is quite normal and not a concern.
E) Tell patient to avoid taking coffee
22) Which of the following is the major ketone increase in blood diabetic ketoacidosis?
A) Acetone
B) Beta hydroxy butyric acid
C) Ethyl and methyl ketone
D) Long change ketones
E) Acetyl acetic acid
23. All of the following drugs can increase CK-MM, except?
A. Rosuvastatin B. Benzfibrate C. Atorvastatin plus niacin D. Simvastatin E) Ramipril
24. A laboratory test indicated increased number of neutrophils. This may be due to?
A. Viral infections
B. Inflammation
C. Bacterial infection in sinusitis
D) Seasonal flu
E. Parasites infections
24. Decreased CD4 count indicates?
A. Viral infections B. Inflammation C.Bacterial infection D. COPD E. Parasites infections
25. A patient approaches to you and ask if she should stop taking dark green vegetable because she
is using warfarin to treat thrombosis? What is appropriate?
A) sure, should stop taking all green vegetables
B) Take high quantity of dark green vegetables
C) Reduce taking dark green vegetables
D) If you're taking in moderation it should not effect on warfarin
E. none
26) A patient is diagnosed with chronic bronchitis and pneumonia in COPD. Which of the following is
increased?
A. Basophills
B. Neutrophil
C. Lymphocytes

Copyright © 2000-2014 TIPS Inc. Unauthorized reproduction of this manual is prohibited. This manual is being used
during review sessions conducted by PharmacyPrep. 13-4
www.Pharmacyprep.com Clinical Biochemistry
D. WBCs
E.RBCs
27. A patient laboratory shows increase in esinophils. This can be indicator of?
A. COPD B.Infection C. Asthma D. pneumonia E. ulcer
Ans: C
Tips: Asthma and allergies increase esinophils and basophils.
28. A 55 yo patient using warfarin to treat deep vein thrombosis. Which of the following factors
increase risk of bleeding?
A. Using vitamin K supplements
B. Using ASA 650 mg for pain
C. Using dark green vegetables such as broccoli, avacado
D. Using supplements containing vitamin K
E. Using iron supplements
29. A patient serum thyroid function test (Serum TSH) shows, 10 mU/L. Doctor should initiate
therapy of?
A. methimazole
B. Propylthiouracil
C. Levothyroxine
D. Radioactive iodide
E. Lugol's solution
30) Which of the following test are used determine anemia?
A) MCV
B) Hematocrit
C) Serum ferritin
D) MCHC
E) TIBC
31. A patient using overdose of acetaminophen for the fast few days. Which of the following enzyme
increase may indicate possibility of hepatotoxicity?
I) LDH
II) AST
III) ALT
A. I only B. III only C. I and II only D. II and III only E. I, II, and III
32. Patient is overdosed with acetyl salicylic acid. Call to your pharmacy at midnight. What is correct
of overdose management ?
I) refer patient to poison control centre
II) Refer to doctor
III) Recommend patient to ipecac
A. I only B. III only C. I and II only D. II and III only E. I, II, and III

Copyright © 2000-2014 TIPS Inc. Unauthorized reproduction of this manual is prohibited. This manual is being used
during review sessions conducted by PharmacyPrep. 13-5
www.Pharmacyprep.com Clinical Biochemistry
33. KP IS A 50 YEAR OLD MAN USING ATORVASTATIN 40 MG DAILY. GET MUSCLE PAIN. WHICH OF
THE FOLLOWING LAB TEST INCREASE?
A) ALT
B) AST
C) CK-MB
D) CK-MM
E) TROPONIN I
ANSWERS:
1. E
2. B
3. A
4. E
5. A
6. E
7. A
8. A
Tips: CK-MM indicator of skeletal muscle, CK-BB is for brain and CK-MB for cardio.
9. B
10. E
11. C
12. C
13. B
14. D
15. B
16. C
17. C

Copyright © 2000-2014 TIPS Inc. Unauthorized reproduction of this manual is prohibited. This manual is being used
during review sessions conducted by PharmacyPrep. 13-6
www.Pharmacyprep.com Clinical Biochemistry
18. B
19. D
20. B
21.
22. B
23. E
24. C
24. A
25. D
26. B
Tips: chronic bronchitis is bacterial infection in COPD, so increase neutrophils
27. C
Tips: Asthma and allergies increase eosinophils and basophils.
28. B
29. C
30. C
31.
32.
33.

Copyright © 2000-2014 TIPS Inc. Unauthorized reproduction of this manual is prohibited. This manual is being used
during review sessions conducted by PharmacyPrep. 13-7
www.Pharmacyprep.com Clinical Biochemistry
Copyright © 2000-2014 TIPS Inc. Unauthorized reproduction of this manual is prohibited. This manual is being used
during review sessions conducted by PharmacyPrep. 13-8
www.Pharmacyprep.com Nutrition

PHARMACY PREP
NUTRITION
1) The heat labile factor in the vitamin B complex?
A) Thiamine
B) Riboflavin
C) Cyanocobalamine
D) biotin
E) nicotinic acid
2) Vitamin A are chemically described as?
A) Carotenoids
B) Ecosonides
C) Prostaglandins
D) Leukotrienes
E) Serotonins
3) Source of vitamin A
A) Fish
B) Green vegetables
C) Carrots
D)
Ans: E
4. Forms of vitamin A found in rods of retina:
A. Cis-retinal
B. Trans retinal
C. Rhodopsin
D. Retinoic acid
E) Cis-13-retinoic acid
5. Topical acne treatment tretinoin (Retin A) is:
I) 13-cis retinoic acid
II) 12-isomer of retinoic acid
III) 13-trans retinoic acid
A. I only B. III only C. I and II only D. II and III only E. I, II and III
6. Which of the following drugs that cause vitamin B6 deficiency:
I-Isoniazid
II- Penicillamine
III- Resins
A. I only B. III only C. I and II only D. II and III only E. I, II and III

Copyright © 2000-2014 TIPS Inc. Unauthorized reproduction of this manual is prohibited. This manual is being used
during review sessions conducted by PharmacyPrep. 14-1
www.Pharmacyprep.com Nutrition
7. Which of the following should be avoided with vitamin B6?
A. Isoniazid
B. Penicillamine
C. Levodopa
D. Dopamine
E. Carbidopa
8. Which of the following vitamin decreases in chronic alcoholics?
A. Vitamin A
B. Thiamine
C. Vitamin C
D. Vitamin B12
E. Vitamin E
9. Which of the following vitamin should you recommend for smokers?
A. Vitamin A
B. Vitamin B
C. Vitamin C
D. Vitamin D
E. Vitamin E
10. Which vitamin structure is related to steroidal hormone structures?
A. vitamin A
B. vitamin B
C. Vitamin D
D. Vitamin K
E. Vitamin E
11. Ergocalciferol is:
A. Vitamin C
B. Vitamin D2
C. Vitamin D3
D. Vitamin E
E. Vitamin K
12. Cholecalciferol:
A. Vitamin C
B. Vitamin D2
C. Vitamin D3
D. Vitamin E
E. Vitamin K

Copyright © 2000-2014 TIPS Inc. Unauthorized reproduction of this manual is prohibited. This manual is being used
during review sessions conducted by PharmacyPrep. 14-2
www.Pharmacyprep.com Nutrition
13. Vitamin:
A. It is cofactor
B. It is inorganic substance
C. It is catalyst
D. It is organic substance
E. It is enzyme
14. Chemical ring structure in folic acid that binds to para aminobenzoic acid is;
A. PABA ring
B. pteridine ring
C. Glutamate
D. Quinolone ring
E. Epoxide ring
15. Seniors have deficiency of vitamins?
A. Vitamin A
B. Vitamin B12
C. Vitamin C
D. Vitamin D
E. Vitamin E
16. What vitamin overdose could cause toxicity?
A. Vitamin A
B. Vitamin B
C. Vitamin C
D. Vitamin D
E. Vitamin E
17. Vegetarians have deficiency of vitamins?
A. Vitamin A
B. Vitamin B12
C. Vitamin C
D. Vitamin D
E. Vitamin E
18. A breast fed infant should recieved which of the following vitamin drops?
A. Vitamin A drops
B.Vitamin B12
C.Vitamin C
D.Vitamin D drops
E. Vitamin E drops

Copyright © 2000-2014 TIPS Inc. Unauthorized reproduction of this manual is prohibited. This manual is being used
during review sessions conducted by PharmacyPrep. 14-3
www.Pharmacyprep.com Nutrition
19. Age over 70 years and elderly have deficiency of vitamin?
A. Vitamin A
B.Vitamin B12
C. Vitamin C
D. Vitamin D
E. Vitamin E
20. Chronic renal disease can cause deficiency of?
A. Vitamin D2
B. Vitamin B12
C. Vitamin C
D. Vitamin D3
E. Vitamin D
21. Which of the following condition have deficiency of vitamin B1(thiamine)
A. smoking
B. old age
C. chronic alcoholism
D. Infants
E. Pregnancy
22. Vitamin B12 deficiency is more common in?
A. smoking
B. old age
C. chronic alcoholism
D. Infants
E. Pregnancy
Ans: B
23. What is incorrect combination about vitamin deficiencies in:
A. Smokers Vitamin C
B. Seniors vitamin B12
C. Chronic alcoholics Vitamin B1
D. Infants (on breast milk) vitamin D
E. Chronic renal failure Vitamin E
24. What are essential fatty acids?
A) Linoleic acid (omega-6)
B) Linolenic acid (omega-3)
C) Arachidonic acid
D) Cholesterol
E) A and B

Copyright © 2000-2014 TIPS Inc. Unauthorized reproduction of this manual is prohibited. This manual is being used
during review sessions conducted by PharmacyPrep. 14-4
www.Pharmacyprep.com Nutrition
25. What is omega 6 essential fatty acids?
I-Linoleic acid
II-Linolenic acid
III-Arachidonic acid
A. I only B. III only C. I and II only D. II and III only E. I, II and III
26. What fats are safe to use?
A. Trans fats
B. Saturated fats
C. Poly unsaturated essential fats
D. Animal fats
E. Vegetable fats
27. A person is allergic gluten, which of the following should not recommend?
A. Milk
B. Rice
C. Wheat
D. Wine
E. Eggs
28. A person have milk allergy, which may be due to?
A. Protein
B. gluten
C. Enzyme
D. vitamins
E. None of the above
29. What is incorrect about calcium carbonate supplements?
A. Calcium carbonate, take it with meals. This type of supplement is absorbed best with food.
B. Take no more than 500-600 mg of calcium at a time.
C. If you take both iron and calcium supplements, do not take them at the same time. This is because
calcium affects your body’s ability to absorb iron.
D. If milk allergy, there can cross allergy to calcium supplement, so calcium carbonate should be
avoided in milk allergy.
E. Calcium carbonate regimen should taken with food 2 to 4 times a day
30) Which of the following vitamin is synthesized by gas trointestinal bacteria?
A) Vitamin D B) Vitamin K C) Vitamin A D) Vtiamin B1 E) Vitamin C
31. Which is true about the needs of vitamins in elderly:
A. Vitamin E deficiency is common in elderly
B. Vitamin D deficiency is common in elderly
C. Iron deficiency is common in elderly

Copyright © 2000-2014 TIPS Inc. Unauthorized reproduction of this manual is prohibited. This manual is being used
during review sessions conducted by PharmacyPrep. 14-5
www.Pharmacyprep.com Nutrition
D. Vitamin B12 is needed
32. Essential amino acid deficiency can cause?
A. Deficiency of protein
B. Deficiency of vitamins
C. Deficiency of nucleic acids
D. No change in primary metabolites
E. All of the above
33. Rhodopsin is essential for vision. Its chemical structure related is?
A. Ecosonides
B. Steroid hormone
C. Carotenoids
D. Omega fats
E. Saturated fat
34. Which of the following fats in the diet has the greatest influence on blood cholesterol?
A. Polyunsaturated fats
B. saturated fats
C. Cholesterol
D. Omega 3
E. Omega 6
35. What is the best source of omega 3?
A. meat
B. Green vegetable
C. Color vegetables
D. Fish
E. whole grains
36. Which of the following has the highest glycemic index?
A. whole grain
B. Vegetables
C. fruits like apple
D. White bread
E. Green beens
37. What is the closes option to breast milk?
A. formula milk
B. cow milk
C. soy milk
D. lactose free formula milk
E. fat free milk

Copyright © 2000-2014 TIPS Inc. Unauthorized reproduction of this manual is prohibited. This manual is being used
during review sessions conducted by PharmacyPrep. 14-6
www.Pharmacyprep.com Nutrition
38. A person with celiac disease can take?
A. milk cereal
B. wheat
C. soy
D. oat
E. rice
39. Vitamin K clotting factors are formed in?
A. blood
B. liver
C. colon
D. skin
E. renal
40. Where does warfarin an oral anticoagulation mechanism action takes place?
A. blood
B. liver
C. colon
D. skin
E. renal
41) What is correct about iron absorption?
A) Iron is transported across the intestinal mucosa by active transport.
B) It is absorbed as ferrous iron and is converted to ferric iron within the mucosal cell.
C) Iron is absorbed better on empty stomach
D) Ferrous gluconate has highest absorption
E) All of the above
42) Choose correct answer about linolenic acid and linoleic acid:
A) Linolenic acid is should be taken with diet
B) Linolenic acid and linoleic acid have only difference of double bond in chemical structure.
C) Linolenic acid is omega 3 essential fatty acid
D) Linoleic acid omega 6 essential fatty acid
E) All of the above
43) What the correct about the following chemical structure?
A) Vitamin A B) Rodhopsin C) Retinol D) Trans retinoic acid E) Cis retinoic acid

Copyright © 2000-2014 TIPS Inc. Unauthorized reproduction of this manual is prohibited. This manual is being used
during review sessions conducted by PharmacyPrep. 14-7
www.Pharmacyprep.com Nutrition
ANSWERS:
1. B
Tips: Vitamin B2 (riboflavin) is heat labile factor in vitamin B complex. Other vitamin Bs are heat and
light stable. Thiamine is vitamin B1, cyanocobalamine is vit. B12. Nicotoinic acid is B3 and biotic vitamin
H.
2. A
Tips. Vitamin A are chemically carotenoids.
3. E
4. C
5. B
Tips: Topical Tretinoin is 13-Trans retinoic acid (RetinA)TTT; Oral isotretinoin is 13-cis retinoic
acid (Accutane)
6. C
Tips: Penicillamine and Isoniazid may cause vitamin B6 deficiency.
7. C
8. B
9. C
10. C
11. B
12. C
13. D
14. B
15. B
16. A
17. B
18. D

Copyright © 2000-2014 TIPS Inc. Unauthorized reproduction of this manual is prohibited. This manual is being used
during review sessions conducted by PharmacyPrep. 14-8
www.Pharmacyprep.com Nutrition
19. B
20. D
Tips: Chronic renal deficiency can cause deficiency of vitamin D3 i.e 1, 25 dihydroxy ergocalceferol
which is active vitamin D.
21. C
22. B
23. E
24. E
25. A
26. C
Tips: unsaturated or mono saturated fats are recommended.
27. C
Tips: Gluten is present in wheat, rye
28. A
29. D
Ref: Eatrightontario.ca
30. D
Tips. Vitamin K is synthesized by bacteria in gastrointestinal tract.
31. D
32. A
33. C
34. B
35. D
36. D
37. A

Copyright © 2000-2014 TIPS Inc. Unauthorized reproduction of this manual is prohibited. This manual is being used
during review sessions conducted by PharmacyPrep. 14-9
www.Pharmacyprep.com Nutrition
38. E
39. B
40. B
41. E
Tips: Iron is transported across the intestinal mucosa by active transport. It is absorbed as ferrous
iron and is converted to ferric iron within the mucosal cell.
42. C
Tips: Linolenic is omega 3 essential fatty acids
43. C
Tips: Trans retinoic acid is a topical product of tretinoin.

Copyright © 2000-2014 TIPS Inc. Unauthorized reproduction of this manual is prohibited. This manual is being used
during review sessions conducted by PharmacyPrep. 14-10
PharmacyPrep.Com Microbiology

PHARMACY PREP
MICROBIOLOGY
1. Which of the following are the correct statement about Staphylococcus aureus
I-It contains techoic acid
II-Gram stain test is positive
III-It contains lipopolysaccharide layer
A. I only B. III only C. I and II only D. II and III only E. All are correct
2. Obligate anaerobic bacteria:
I-Generate hydrogen peroxide
II-Generate superoxide dismutase
III-Do not generate superoxide dismutase
A. I only B. III only C. I and II only D. II and III only E. All are correct
3. Colon is large intestine which contains which of the following type of bacteria:
A. 95% to100% anaerobic
B. 95% to 100% aerobic
C. 30 to 50% anaerobic
D. 30 to 50% aerobic
E.There is no bacteria in lower gut
4. Which of the following clinical condition is the prophylactic use of anti bacterial is NOT required?
A. Preoperative treatment for GI conditions
B. Frequent respiratory diseases
C. Preoperative treatment of hip surgeries
D. Prevention of meningitis among individuals in close contact with infected patients
E. Patient with tooth surgeries or tooth removed.
Tips: Certain clinical situation require the use of antibiotics for prevention rather than treatment of
infections, such as:
Prevention of strep infections in patient with rheumatic heart disease
Patient under going dental extraction, who have prosthetic devices such as heart valve etc.
Prevention of Tuberculosis, meningitis (close contacts) Prior to certain surgical procedures. HIV
infected pregnant woman
5. Drugs used in the treatment of meningitis are all of the following, EXCEPT:
A. Penicillins
B. Cephalosporins
C. Gentamicin
D. Streptomycin

Copyright © 2000-2014 TIPS Inc. Unauthorized reproduction of this manual is prohibited. This manual is being used
during review sessions conducted by PharmacyPrep. 14-1
PharmacyPrep.Com Microbiology
E. Sulfonamides
6. The most common causative organism of community acquired pneumonia (CAP) is:
A. S. pneumonia
B. M. pneumonia
C. H. influenza
D. S. aureus
E. E. coli
7. The following organism least likely causes pneumonia:
A. S. pneumonia
B. M. pneumonia
C. H. influenza
D. M.catharhalis
E. E. coli
8. Bacteria that cause community acquired pneumonia infections include:
I-S.pneumonia
II-H.influenza
III-M.catarrhalis
A. I only B. III only C. I and II only D. II and III only E. All are correct
9. The most common cause of urinary tract infections include:
A. N. gonnorhea
B. E. coli
C. Chlamydea
D. S. pneumonia
E. M. catarhalis
10. The most common pathogen that associated with infections in travelers diarrhea:
A. Protozoa
B. E. coli
C. Salmonella sp.
D. Shigella
E. Corynebacterium
11. Lyme disease a tick born infections commonly occur in the months of July-August and in the area
of Mississippi river valley, is caused by:
A.Treponema pallidum
B. B. burgdorferi
C. Dermatophyte
D. Giardia lamblia
12. Which of the following are common pathogens that cause nosocomial infections:

Copyright © 2000-2014 TIPS Inc. Unauthorized reproduction of this manual is prohibited. This manual is being used
during review sessions conducted by PharmacyPrep. 14-2
PharmacyPrep.Com Microbiology
I-S. aureus
II-P. aeruginosa
III-E. coli
A. I only B. III only C. I and II only D. II and III only E. All are correct
13. Drug of choice to treat nosocomial infection that caused by P. aeruginosa.
A. Ciprofloxacin
B. Aminoglycosides
C. Amoxicillin
D. Erythromycin
E. Cephalosporins
14. Clindamycin, may cause severe diarrhea, this condition can be treated by:
A. Pepto-Bismol
B. Loperamide
C. OTC antidiarrheal
D. Metronidazole
E. Oral rehydration salts
15. Which of the following is mycolic acid synthesize inhibitor?
A. Rifampin
B. Ethambutol
C. Isoniazid
D. Cotrimazole
E. Ketoconazole
16. Syphilis is a sexually transmitted infections that can occur with;
A. E. coli
B. Salmonella
C. T. pallidum
D. Spirochete
E. Diphtheria
17. The fungal infection that commonly occurs on skin between foot toes is:
A. Jack itch
B. Onychomycosis
C. Athletes foot
D. Thrush
E. Ringworm
18. Toxic shock syndrome (TSS) is caused by:
A) E. coli
B) It is type of UTI
C) S. aureus

Copyright © 2000-2014 TIPS Inc. Unauthorized reproduction of this manual is prohibited. This manual is being used
during review sessions conducted by PharmacyPrep. 14-3
PharmacyPrep.Com Microbiology
D) It is type of STD
E) It is not a infection
19. Pseudomembranous colitis is a condition that occurs as result of antibiotics use, is caused by:
A. C. tetani
B. C. difficile
C. C. botulinum
D. C. perfringes
E: All of the above
20. All of the following cause food poisoning EXCEPT:
A. E. coli
B. Solmonella sp
C. Shigella sp
D. Nisseria
E. C. botulinum
21. Seasonal flu is caused by:
I) Influenza A
II) Influenza B
III) H. Influenza type B only
A. I only
B. III only
C. I and II only
D. II and III only
E. All are correct
Tips: Influenza A and B are virus that cause seasonal flu and H. influenza is gram –ve bacteria
commonly cause pneumonia and otitis media
22. Hepatitis A viral infections may spread by:
I-Sexual contact
II-Blood transfusion
III-Food contamination
A. I only
B. III only
C. I and II only
D. II and III only
E. All are correct
23. The common cold (runny nose) is caused by:
A. Influenza A
B. Influenza B
C. Rhinovirus

Copyright © 2000-2014 TIPS Inc. Unauthorized reproduction of this manual is prohibited. This manual is being used
during review sessions conducted by PharmacyPrep. 14-4
PharmacyPrep.Com Microbiology
D. Pox virus
E. Papilloma virus
Tips: Rhino has runny nose
24. Hepatitis B and C are chronic viral infections may spread by?
I-Sexual contact
II-Blood transfusion
III-Food contamination
A. I only
B. III only
C. I and II only
D. II and III only
E. All are correct
25. Which of the following NOT associated with Toxic shock syndrome?
A. Tampons
B. Condoms
C. Contraceptive sponges
D. Intra uterine devices (IUD)
E. None of the above
26) Endotoxins are product of?
A. Lipopolysaccharides
B. Techoic acid
C. Proteins
D. Fats
E. bacterial cell wall
Tips: Gram negative bacteria cell wall produce endotoxins, thus this bacteria cell wall have
lipopolysaccharide.
27. Which of the following methods are used for
removal endotoxins?
A) lyophylization
B) Rabbit test
C) LAL test
D) Sterilization
E) None of the above
Tips: Distillation method is used for removal of endotoxins.
28. A person with H. pylori positive may have -->
I-Peptic ulcer
II-Ulcerative colitis
III-Chrons disease
A. I only

Copyright © 2000-2014 TIPS Inc. Unauthorized reproduction of this manual is prohibited. This manual is being used
during review sessions conducted by PharmacyPrep. 14-5
PharmacyPrep.Com Microbiology
B. III only
C. I and II only
D. II and III only
E. All are correct
29. Which of the following is least detected in meningococcal meningitis?
A. S. pneumonia
B. Nisseria meningitis
C. H. influenza
D. S. aureus
E. E. coli
30) Pneumococcal vaccination prevent all except?
A. Meningitis
B. Otitis media
C. Community acquired pneumonia
D. osteomyelitis
E. sinusitis
31. The drug of choice in treatment of lyme disease?
A.doxycycline in children
B.doxycycline adults
C.amoxicillin in children
D.B and C
32. Which of the following drug is tuberculosis prophylaxis?
A. isoniazid (INH)
B. Rifampin
C.ethambutol
D.Ciprofloxacin
33. Which is true about Chancre disease?
A. primary symptoms of syphilis infection
B. painful ulcer at infection site
C. patient infected with Yersinia pestes have this symptoms
34. Which of the following symptoms are vaginal candidiasis?
I) fishy odor II) itchy, burning sensation III) Cottage cheese discharge
A. I only
B. III only
C. I and II only
D. II and III only
E. All are correct

Copyright © 2000-2014 TIPS Inc. Unauthorized reproduction of this manual is prohibited. This manual is being used
during review sessions conducted by PharmacyPrep. 14-6
PharmacyPrep.Com Microbiology
35. A patient receiving treatment of gonorrhea infections. Should also treatment of?
A. meningitis
B. chlamydia
C. syphilis
D. Herpes
E. UTI
36. What infections is more common in new born in Canada?
A. meningitis
B. chlamydia
C. syphilis
D. Herpes
E. UTI
37. Chancre or chancroids on genitals are symptoms of?
A. Athletes foot
B. gonorrhea
C. syphilis
D. herpes
E. warts
38. Which of the following is NOT sexually transmitted? which does not require partner treatment?
A. syphilis
B. trichomonas
C. Canadida
D. bacterial vaginitis
E. herpes
39. Which of the following viral strain infection can cause cervical cancer?
A. herpes
B. papilloma virus
C. hepatitis
D. warts
E. all
40. Annually flu season in Canada. Flu is caused by?
A. Influenza A only
B. Influenza B only
C. Influenza A and B
D. Influenza A and Flu vaccine
E. Flu vaccine only

Copyright © 2000-2014 TIPS Inc. Unauthorized reproduction of this manual is prohibited. This manual is being used
during review sessions conducted by PharmacyPrep. 14-7
PharmacyPrep.Com Microbiology
41. Which of the following infections required to be treated all close contact as prophylaxis?
A. Chlamydia
B. Yeast
C. Athletes foot
D. meningitis
E. Salmonella
42. Which of the following infection are commonly associated in Nosocomial infections?
I) S. aureus
II) P. aureugunosa
III) C. difficle
A. I only
B. III only
C. I and II only
D. II and III only
E. All are correct
43. What is true about cervical cancer?
I) caused by oral contraceptives pills
II) Caused by papilloma virus
III) It is sexually transmitted infection
A. I only
B. III only
C. I and II only
D. II and III only
E. All are correct
44. Which of the following infections is NOT related to virus?
a) Prion disease
b) Measles
c) Shingles
d) Keratoconjuntivitis
e) Cold sore
ANSWERS:
1. C
2. B
3. A
4. B

Copyright © 2000-2014 TIPS Inc. Unauthorized reproduction of this manual is prohibited. This manual is being used
during review sessions conducted by PharmacyPrep. 14-8
PharmacyPrep.Com Microbiology
Tips: Certain clinical situation requires the use of antibiotics for prevention rather than treatment of
infections, such as: Prevention of strep infections in patient with rheumatic heart disease. Patient
under going dental extraction, who have prosthetic devices such as heart valve etc. Prevention of
Tuberculosis, meningitis (close contacts) Prior to certain surgical procedures. HIV infected pregnant
woman.
5. D
Tips: Streptomycin is not used in the treatment of meningitis due to 8th cranial nerve toxicity. The
main treatment of bacterial meningitis may include penicillin's, cephalosporin's and vancomycin
while viral meningitis is mainly treated by acyclovir IV.
6. A
7. E
8. E
9. B
10. B
11. B
Tips. Lyme disease infections are caused by B. burgdorferi
12. C
Tips: nosocomial infections = hospital acquired infections.
13. B
14. D
15. C
16. C
17. C
Tips: Jack itch is fungal infections commonly occurs on foot toe Onchomycosis is fungal nail
infections. Thrush is fungal infections mainly caused by Candida albicans . Ringworm is a fungal
infection, it caused by dermatophytes (it is not a worm).
18. C
19. B

Copyright © 2000-2014 TIPS Inc. Unauthorized reproduction of this manual is prohibited. This manual is being used
during review sessions conducted by PharmacyPrep. 14-9
PharmacyPrep.Com Microbiology
20. D
21. C
Tips: Influenza A and B are virus that cause seasonal flu and H. influenza is gram –ve bacteria
commonly cause pneumonia and otitis media
22. B
23. C
Tips: Rhino has runny nose
24. C
25. B
26. A
Tips: Gram negative bacteria cell wall produce endotoxins, thus this bacteria cell wall have
lipopolysaccharide.
27. E
Tips: Distillation method is used for removal of endotoxins.
28. A
29. D
30. D
31. D
32. A
33. A
34. D
35. B
36. B
37. C
38. C

Copyright © 2000-2014 TIPS Inc. Unauthorized reproduction of this manual is prohibited. This manual is being used
during review sessions conducted by PharmacyPrep. 14-10
PharmacyPrep.Com Microbiology
39. B
40. C
41. D
Tips: rifampicin is used as prophylaxis to prevent meningitis.
42. E
43. D
44. A
Tips: Prion is a small infectious particle composed of abnormal folded protein. Prion disease.
(Creutzfeldt-Jakob) caused by prions. Or transmissible spongiform encephalitis. It can occurs in
human and animals (mad cow disease).

Copyright © 2000-2014 TIPS Inc. Unauthorized reproduction of this manual is prohibited. This manual is being used
during review sessions conducted by PharmacyPrep. 14-11
www.Pharmacyprep.com Cell and Molecular Biology

PHARMACY PREP
CELL AND MOLECULAR BIOLOGY
1. Genetic information is stored on?
A. RNA
B. DNA
C. Gene
D. chromosome
E. Ribosome
2. What base is NOT found in ribonucleic acid (RNA):
A. Thymine
B. Adenine
C. Guanine
D. Cytosine
E. Uracil
3. The DNA exists as a double helix in which polynucleotide chains consist of a sequence of
nucleotides linked together by phosphodiester bonds, joining adjacent deoxyribose moieties. The
hydrogen bonding between holds the two polynucleotide strands together bases in opposing strands
called as the complementary base-pairing. The base-pairs are of high specificity such that:
I. Adenine (A) is always paired with Thymine (T)
II. Guanine (G) is always paired with Cytosine (C)
III. Adenine (A) is always paired with Cytosine (C)

A. I only
B. III only
C. I and II only
D. II and III only
E. All are correct
4. Translation occurs:
A. Before transcription
B. After transcription
C. Same time as transcription
D. in mitochondria
E. in chromosomes
5. Glycoproteins are linked to:
A. DNA
B. RNA
C. mRNA

Copyright © 2000-2014 TIPS Inc. Unauthorized reproduction of this manual is prohibited. This manual is being used
during review sessions conducted by PharmacyPrep. 16-1
www.Pharmacyprep.com Cell and Molecular Biology
D. Carbohydrates
E. Lipids
6) An enzyme that cleaves deoxyribonucleic acid (DNA)
at a specific site is called:
A) Trypsin
B) Restrictive ribonuclease
C) Restrictive endonuclease
D) Reverse transcriptase
E) Ribonulceases
7. Coupling of two DNA helicals is:
A. Lyase
B. Ligase
C. Endonuclease
D. Ribonuclease
E. Reverse Transcriptase
8. Separating two DNA helicals is:
A. Lyase
B. Ligase
C. Endonuclease
D. Ribonuclease
E. Reverse Transcriptase
9. Which of the following is NOT a type of RNA;
A. mRNA
B. tRNA
C. cRNA
D. rRNA
E. None of the above
10. Which of the following is the highest percent of RNA type:
A. mRNA
B. tRNA
C. cRNA
D. rRNA
E. dRNA
11. Which of the following is recombinant DNA
A. rDNA
B. R-DNA
C. cDNA
D. mDNA

Copyright © 2000-2014 TIPS Inc. Unauthorized reproduction of this manual is prohibited. This manual is being used
during review sessions conducted by PharmacyPrep. 16-2
www.Pharmacyprep.com Cell and Molecular Biology
E. tDNA
12) Hemophilia type A results from deficiency of
clotting factor?
A) factor 5
B) factor10
C) factor 8
D) factor 2
E) factor 9
13. In the manufacture of drugs via DNA technology, the DNA material is microscopically inserted
into the:
A. Cytoplasm
B. Nucleus
C. Endoplasmic reticulum
D. Cell wall
E. Cell membrane
14. A woman carrying hemophilia and has a
child from a man with hemophilia.
Which of the following is NOT her child?
A. A girl with hemophilia
B. A boy with hemophilia
C. A girl having no gene of hemophilia
D. A girl having the gene of hemophilia
E. A boy having no hemophilia
15. Bases found in DNA is/are
I-Uracil
II-Cytosine
III-Thymine
A. I only
B. III only
C. I and II only
D. II and III only
E. All are correct
16) What is not a complimentary pair?
A) G-C
B) T-A
C) A-G
D) A-U
E) C-G

Copyright © 2000-2014 TIPS Inc. Unauthorized reproduction of this manual is prohibited. This manual is being used
during review sessions conducted by PharmacyPrep. 16-3
www.Pharmacyprep.com Cell and Molecular Biology
17. What is codon?
A. specific set of three bases are codons and these are expressed at transcription
B. specific set of DNA and RNA
C. specific set of DNA
D. Specific set of DNA bases
E. specific set of RNA bases
18. What is cDNA?
A. DNA template produced by mRNA, is complementary DNA (cDNA)
B. specific set of DNA and RNA
C. specific set of DNA
D. Specific set of DNA bases
E. specific set of RNA bases
19) Functions of restrictive endonuclease include -->
A. Cut DNA into piece
B. Cut RNA into pieces
C. Cut DNA and RNA into pieces
D. Join DNA pieces
E. Join RNA pieces
20. What is true regarding DNA?
I-It stores genetic information of a person
II-Genetic inheritance
III-Some viruses have NO DNA.
A. I only
B. III only
C. I and II only
D. II and III only
E. All are correct
21) Which of the following is NOT used as plasmid?
A) Bacteria
B) Virus
C) DNA
D) Eukaryotic cell
E) Prokaryotic cell
22. Which of the following is NOT a complimentary base pair?
A. G-C and A-T
B. C-G and A-T
C. G-C and T-A
D.T-G and G-C
E. A-U and G-C

Copyright © 2000-2014 TIPS Inc. Unauthorized reproduction of this manual is prohibited. This manual is being used
during review sessions conducted by PharmacyPrep. 16-4
www.Pharmacyprep.com Cell and Molecular Biology
23) What is NOT used in inserting DNA sample into a plasmid?
A. DNA
B. Plasmid
C. DNA ligase
D. Topoisomerase II
E. All of the above
24. Single set of unpaired chromosomes is referred as?
A. Hapten
B. Haploid
C. Diploid
D. Immunogens
E. Antigen
25. Low molecular weight compounds that act as immunogens after chemically complexing to a
larger molecule or cell surface.
A. Hapten
B. Haploid
C. Diploid
D. Immunogens
E. Antigen
26. What base is NOT found in deoxyribose nucleic acid (DNA):
A. Thymine
B. Adenine
C. Guanine
D. Cytosine
E. Uracil
27. Information transfer from DNA to M-RNA is referred as?
A. Transcription
B. Translation
C. DNA-gyrase
D. protein synthesis
E. DNA-Recombination
28. Complimentary DNA (cDNA) produced from?
A. mRNA
B. tRNA
C. Cell
D. Gene
E. rRNA

Copyright © 2000-2014 TIPS Inc. Unauthorized reproduction of this manual is prohibited. This manual is being used
during review sessions conducted by PharmacyPrep. 16-5
www.Pharmacyprep.com Cell and Molecular Biology
29. Which of the following cell organ in eukaryotic cells cholesterol synthesis takes place?
A. Mitochondria
B. Ribosome
C. Centrosome
D. Endoplasmic reticulum
E. Cytoplasm

30) Genetic diseases such as cystic fibrosis, sickle cell anemia, hemophilia can be treated by using
gene therapy, in which defected genes are identified and altered or by altering gene expression to
prevent or cure genetic diseases. This kind of therapy is defined as?
A) Gene therapy
B) Pharmacotherapy
C) Biological therapy
D) Biomedical therapy
E) Cognitive behavioral therapy
31. Peroxisomes?
A) contains enzymes that produces and decomposes hydrogen peroxides.
B) site for protein synthesis
C) site for ATP synthesis
D) site for cholesterol synthesis
E) site microtubule in cell division
32) DNA differs from RNA in that it lacks?
A) DNA contains Thiamine rather than Uracil
B) Hydroxyl group at the pentose C5 position
C) DNA contains uracil rather than thiamine
D) Polymerase chain reaction magnifies DNA
E) DNA is oxyribose hexose at C5 position
33) Chimeric antibody is?
A) Human antibody
B) derived from T cells
C) have four group attached into a antibody
D) type T cells
E) Monoclonal antibody
34) HAMA stands for?
A) Human antigen Mouse antibody
B) Human anatomy and mouse anatomy
C) Human antibiotics developed from mouse

Copyright © 2000-2014 TIPS Inc. Unauthorized reproduction of this manual is prohibited. This manual is being used
during review sessions conducted by PharmacyPrep. 16-6
www.Pharmacyprep.com Cell and Molecular Biology
D) Human Assay for Mouse antigens
E) Human AntiMouse Antibody
ANSWERS:
1. C
2. A
3. C
Tips: AT and GC
4. B
5. D
Tips: glycoprotein's are linked to carbohydrates
6. C
Tips: Restrictive endonuclease cleaves the DNA at a specific site.
7. B
Tips: ligase is coupling of DNA helicals where as lyase is splitting of helicles.
8. A
9. C
Tips: RNA classified as rRNA, tRNA and mRNA. However, there is no cRNA.
10. D
Tips: rRNA80%, tRNA15%, mRNA5%
11. A
Tips: rDNA is recombinant DNA
12. C
13. B
X 14. C m
w
PHARMACY PREP
PHARMACOGENETICS
1. What is definition of Pharmacogenetics?
A) The pharmacogenetics is integration of pharmacology and genetics.
B) The study of pharmacogenetics allows designing and developing drugs that are customized to
each person’s genetic mark up.
C) The pharmacogenetics also utilized to study cytochrome enzymes that are responsible for drug
interactions.
D) The pharmacogenetics identifies the genetic variation that cause drug cause differences in drug
response.
E) All of the above
2). A single base differences that exist between individual. This is the most common genetic variation
in DNA occurs when one base pair of nucleotide replaces another. This is termed as?
A. Single nucleotide polymorphism (SNP)
B. Genetic variation
C. Gene therapy
D) Polymorphism
E) Pharmacogenetics
3) Enzymes that catalyze coupling of two molecules are classified as
A. Hydralases
B. Ligases

C. Oxidoreductases
D. Transferases
E. Isomerases

4. If you know person genetics mark up, and target a medicine is defined as?
A. pharmaceutics
B. biotechnology
C. Nanotechnology
D. pharmacogenetics
E. Molecular biology
5. Trastuzumab, is used for?
A. a HER2 receptor RNA protein inhibitor used for breast cancer
B. contraceptives
C. Rheumatoid arthritis
D. Chemotherapy
E. Chron’s disease
6. Antisense technology is?

Copyright © 2000-2014 TIPS Inc. Unauthorized reproduction of this manual is prohibited. This manual is being used
during review sessions conducted by PharmacyPrep. 17-1
A) Codons that stops proteins synthesis are used to stop formation of defected proteins.
B) Antisense technology targets mRNA
C) Antisense technology target DNA synthesis
D) Antisense technology target translation in protein synthesis
E) A and B
ANSWERS:
1. D
2. A
Tips. Single nucleotide polymorphism (SNP). occurs when one base pair of nucleotide replaces
another. A single base differences that exist between individual. This is the most common genetic
variation in DNA.
3. B
4. D
5. A
6. E

Copyright © 2000-2014 TIPS Inc. Unauthorized reproduction of this manual is prohibited. This manual is being used
during review sessions conducted by PharmacyPrep. 17-2
PharmacyPREP.Com Immunology

PHARMACY PREP.
IMMUNOLOGY AND TOXICOLOGY
1. The FALSE statement about vaccines is/are:
I- It is given only after the person suffer exposition to the virus
II- Passive vaccines promote immunization for short period of time
III- Active vaccines promote long-term immunization
A. I only
B. III only
C. I and II only
D. II and III only
E. I, II and III
2. Differences between passive and active immunization:
I- Passive enhance and active stimulate the patient’s immune response.
II- In passive antibody injection are given and in active antigen injections are given
III- Passive is effective for some months while active is longer lasting
A. I only
B. III only
C. I and II only
D. II and III only
E. I, II and III
3) Correct statement regarding α-Fetoprotein-AFP include
I- Produced by the fetal liver
II- May cause hepatocellular carcinoma
III Cause death of fetus
A. I only
B. III only
C. I and II only
D. II and III only
E. I, II and III
4. A woman carrying hemophilia and has a child from a man with hemophilia. Which of the following
is NOT her child?
A. A girl with hemophilia
B. A boy with hemophilia
C. A girl having no gene of hemophilia
D. A girl having the gene of hemophilia
E. A boy having no hemophilia

Copyright © 2000-2014 TIPS Inc. Unauthorized reproduction of this manual is prohibited. This manual is being used
during review sessions conducted by PharmacyPrep. 18-1
PharmacyPREP.Com Immunology
5. All of the following can be differentiated from a mast cell, EXCEPT
A. Eosinophil
B. Basophile
C. Neutrophil
D. Platelet
E. Erythrocytes
6. Basophiles, neutrophils and eosinophils are originated from
A. Myeloblasts
B. Lymphoblasts
C. Monoblasts
D. Proerythroblasts
E. Megakaryoblasts
7) Which of the following is/are considered autoimmune disorders?
I) Grave’s disease
II) Systemic Lupus Erythromatosus
III) Osteoporosis
A) I only
B) III only
C) I and II only
D) II and III only
E) I, II, III
8. True statements regarding the use of monoclonal antibodies include which of the following?
I- Flu-like symptoms commonly occurs at the start of therapy
II- T-cells are blocked from initiating the rejection process
III- Orthoclone is a human immunoglobulin product
A. I only
B. III only
C. I and II only
D. II and III only
E. I, II and III
9. Which of the following is not related to lymphocytes?
A. Lymph nodes
B. Thymus gland
C. Thoracic duct
D. Spleen
E. Thyroid gland

Copyright © 2000-2014 TIPS Inc. Unauthorized reproduction of this manual is prohibited. This manual is being used
during review sessions conducted by PharmacyPrep. 18-2
PharmacyPREP.Com Immunology
10. Attenuated viruses used for vaccination:
I- Poliomyelitis virus
II- MMR-Mumps, measles and rubella virus
III- Influenza
A. I only
B. III only
C. I and II only
D. II and III only
E. I, II and III
11. Drugs that stabilize mast cells are used in the treatment of:
A. Organ implants rejection
B. Cancer therapy
C. Allergic rhinitis
D. Asthma
E. Myocardial infaction
12. All of the following biological are used for active immunization, EXCEPT:
A. Bacterial vaccine
B. Bacterial antigen
C. Multiple antigen preparations
D. Toxoids
E. Toxins
13. Correct statements regarding Technetium 99m may include?
I- Is the most used radionuclide in nuclear pharmacy
II- It is produced by the radioactive decay of molybdenum-99
III- It has a half-life of 6hours
A. I only
B. III only
C. I and II only
D. II and III only
E. I, II and III
14. Correct statements regarding Klein filter syndrome include:
I- Male with enlarge breast, small testis and infertility
II- The male has 3 chromosomes
III- Chromosomes forming XXY, presence of one extra female chromosome.
A. I only
B. III only
C. I and II only
D. II and III only
E. I, II and III

Copyright © 2000-2014 TIPS Inc. Unauthorized reproduction of this manual is prohibited. This manual is being used
during review sessions conducted by PharmacyPrep. 18-3
PharmacyPREP.Com Immunology
15. Correct statements regarding infantile genetic agranulocytosis include?
I- Type I hypersensitivity
II- Caused by a decrease in bone marrow activity
III- Treatment can be done by transfer of WBCs or large doses of antibodies.
A. I only
B. III only
C. I and II only
D. II and III only
E. I, II and III
16. Which of the following is the organ responsible for the immune system?
A. Thymus
B. Lymphoid organ
C. Megakaryocyte
D. Lynphocyte
E. Spleen
17. The “key” operative of immune system is known as:
A. Megakaryocytic
B. Monocyte
C. Lymphocyte
D. Proerythrocyte
E. Myelocyte
18. Which of the following is/are types of lymphocytes?
I- B cells
II- T cells
III- Natural killer cell
A. I only
B. III only
C. I and II only
D. II and III only
E. I, II and III
19. The stem cell of immune system is developed in:
A. Bone marrow
B. Lymphoid organ
C. Liver
D. Spleen
E. Lymph nodes
20. Which of the following is a pro-inflammatory cell responsible for initiation of an acute
inflammation?
A. Eosinophils

Copyright © 2000-2014 TIPS Inc. Unauthorized reproduction of this manual is prohibited. This manual is being used
during review sessions conducted by PharmacyPrep. 18-4
PharmacyPREP.Com Immunology
B. Platelets
C. Macrophages
D. Basophiles
E. Neutrophils
21. Antidote of mercury may include:
A. Dimercaprol
B. EDTA
C. Deferoximine
D. Succimer
E. Naloxone
22. Which of the following is a benzodiazepines antagonist:
A. Naloxone
B. Physostigmine
C. Flumazenil
D. Naltrexone
E. Penicillamine
23. What would be the best treatment for cyanide poisoning?
A. Antivenin
B. Sodium thiosulfite
C. Acetylcystein
D. Oxygen therapy
E. Sodium bicarbonate
24) Carbon monoxide can be highly toxic because
it easily binds to:
I) Hemoglobin
II) Myoglobin
III) Cytochrome oxidase
A. I only
B. III only
C. I and II only
D. II and III only
E. I, II and III
25. Acetaminophen toxicity most well known complication is:
A. Cardiovascular failure
B. Pulmonary edema
C. CNS lethargy
D. Liver necrosis
E. Sedation

Copyright © 2000-2014 TIPS Inc. Unauthorized reproduction of this manual is prohibited. This manual is being used
during review sessions conducted by PharmacyPrep. 18-5
PharmacyPREP.Com Immunology
26. Salicylate toxicity treatment include which of the following?
I- Decontamination with syrup of IPECAC until 30 minutes of overdose
II- CHARCOAL every 6 hours
III- Alkalinization of urine with sodium bicarbonate
A. I only
B. III only
C. I and II only
D. II and III only
E. I, II and III
27. Hyperphosphatemia is best treated by:
A. Magnesium hydroxide
B. Calcium carbonate
C. Aluminium hydroxide
D. Sodium Bicarbonate
E. Sodium phosphate
28. Correct statements regarding IPECAC may include:
I- It has an emetic effect when large doses are administrated
II- It has a expectorant effect when small doses (1 to 2mls) is administrated
III- Used in decontamination procedures during toxic treatments
A. I only
B. III only
C. I and II only
D. II and III only
E. I, II and III
29. The organophosphates commonly found in insecticides are thought to act by witch of the
following mechanisms?
A. Combining with acetylcholine
B. Potentiating the action of acetylcholinesterase
C. Forming a very stable complex with acetylcholinesterase
D. Reacting at the cholinergic receptor
E. Preventing the release of acetylcholine from the nerve ending
30. All of the following can be recognized as atropine poisoning, EXCEPT:
A. Dry skin
B. Mydriasis
C. Flushed appearance
D. Diarrhea
E. Delirium

Copyright © 2000-2014 TIPS Inc. Unauthorized reproduction of this manual is prohibited. This manual is being used
during review sessions conducted by PharmacyPrep. 18-6
PharmacyPREP.Com Immunology
31) Poison ivy is what type of hypersensitive reactions?
A) Type I
B) Type II
C) Type III
D) Type IV
E) Type V
32. Monocytes are white blood cells that can swallow and digest microscopic organisms and particles
in a process known as?
A. phagocytosis
B. Allergic reaction
C. Natural killer cells
D.Gene therapy
E. Monoclonal antibodies
33. What is normal average life time of patient after HIV infection?
A. 10 yrs
B. 2 to 3 yrs
C. 1 yrs
D. long life
E. 5 yrs
34. A 70 yo elderly patient is treated with antibiotics for 7 days. However, still have signs of
symptoms of infections such as fever. The patient neutrophil count is still increased over 70%. What
kind of infections patient have?
A. parasitic
B. viral
C. fungal
D. inflammation
E. bacterial
35. If doctor suspect parasites infection, which cells are increased?
A. neutrophils
B. eosinophils
C. basophils
D. platelets
E. lymphocytes
36. A 50 yo COPD patient is diagnosed pneumonia. What type cells are increased?
A. neutrophils
B. eosinophils
C. basophils
D. platelets
E. lymphocytes

Copyright © 2000-2014 TIPS Inc. Unauthorized reproduction of this manual is prohibited. This manual is being used
during review sessions conducted by PharmacyPrep. 18-7
PharmacyPREP.Com Immunology
37. All of the following can transmit HIV, except?
A) Sexual contact
B) Maternal transmission
C) Drug abuse and sharing needle
D) cut and wounds contact
E) Orofecal
38. A HIV patient currently has CD4 T cell count is 300.
What treatment doctor may target for prophylaxis?
A) M. Tuberculosis
B) Cytomegalvoirus
C) Pneumocytisis pneumonia jerovicii or carinii
D) Non-Hodgkins lymphoma
E) Kaposis sarcoma
39. Amoxicillin suspension was prescribed to treat otitis media to a child. The next day child was
hospitalized due severe urticaria/eczema. Doctor diagnosed as amoxicillin allergy. What type of
hypersensitive reaction?
A) type 4
B) type 3
C) type 2
D) type 1

Copyright © 2000-2014 TIPS Inc. Unauthorized reproduction of this manual is prohibited. This manual is being used
during review sessions conducted by PharmacyPrep. 18-8
PharmacyPREP.Com Immunology
ANSWERS:
1. A
Tips: Vaccines can be given as prophylaxis before the person suffers exposition to the
microorganisms.
2. E
Tips: Passive vaccines: Normally IV and IM injections of antibodies to enhance patient’s immune
competence; protection depends on serum half-life of the injected antibody that may be effective
for days or months. Active vaccinations: Normally IV, IM, SC and oral administration of antigen to
stimulate the immune response. Immunity is long lasting.
3. C
Tips: α-Fetoprotein-AFP is produced by the fetal liver and may cause hepatocellular carcinoma of
fetus, however it does not cause the death of fetus.
4. C
Tips: If the mother and father have hemophilia their child may not have the disease hemophilia but
the child will have the gene of the disease.
5. B
Tips: Basophile is a pro-inflammatory cell that initiates acute inflammation and cannot be
differentiated from mast cells.
6. A
Tips: Basophiles, neutrophilis and eosinophiles are leukocytes-White Blood Cells-WBC originated
from myeloblasts.
7. C
Tips: Osteoporosis is not an autoimmune disorder. It is a disease characterized by the loss of bone
mass.
8. C
Tips: Monoclonal antibodies can deliver toxins specifically to cancer cells and destroy them. They are
also used with radioisotopes to diagnose and visualize cancer cells and may cause flu-like symptoms
in the beginning of the treatment.
9. E
Tips: Thyroid gland is not part of lymphocytes. Lymphocytes include: Tonsils and adenoid gland,
thymus, lymphonodes, spleen, bone marrow and lymphatic vessels.
10. C

Copyright © 2000-2014 TIPS Inc. Unauthorized reproduction of this manual is prohibited. This manual is being used
during review sessions conducted by PharmacyPrep. 18-9
PharmacyPREP.Com Immunology
Tips: Attenuated vaccines are made from whole microbes that have been killed by heat or chemical.
MMR-Mumps, Measles, Rubella virus and TB-Mycobacterium are live vaccines.
11. C
Tips: Drugs that stabilize mast cells are normally used in the treatment of allergic rhinitis as per
example cromoglycate.
12. E
Tips: Toxins are used to refer specifically to a protein produced by some higher plants, certain
animals, and pathogenic bacteria, which is highly toxic for other living organisms therefore cannot be
used for vaccination.
13. E
Tips: Is the most used radionuclide in nuclear pharmacy. This radionuclide is produced by the
radioactive decay of molybdenum-99. A generator is a devise used to separate a short half-life
radionuclide from the longer-lived parent nuclide, while retaining the parent to produce more of the
daughter nuclide. In this way, short half-life nuclides can be made available continuously at great
distances from the sites of generator production. Technetium-99m has a half-life of 6hours
14. E
Tips: Klein filter syndrome is a condition where the male have enlarged breast, small testis and
infertility. The male chromosome is constituted of 3 chromosomes forming XXY, presence of one
extra female chromosome.
15. D
Tips: It is an autosomal recessive disorder characterized by the early onset of recurrent, severe
pyogenic infections, especially of the skin and lung, total absence of neutrophilis in the blood or
presence in reduced numbers, absolute monocytosis and eosinophilia, markedly decreased numbers
of mature neutrophilic precursors in the bone marrow.
16. B
Tips: Lymphoid organ is the organ responsible for the immune system concerned with growth,
development and deployment of lymphocytes.
17. C
Tips: Lymphocytes are WBC-White Blood Cells, the key operative of immune system.
18. E
Tips: Lymphocytes are WBC-White Blood Cells found in the blood and in many other part of body and
may include B and T cells, and natural killer cells.
19. A

Copyright © 2000-2014 TIPS Inc. Unauthorized reproduction of this manual is prohibited. This manual is being used
during review sessions conducted by PharmacyPrep. 18-10
PharmacyPREP.Com Immunology
Tips: Steam cells are blood cells progenitor, or mother cell, having the capacity for both replication
and differentiation originated from bone marrow.
20. D
Tips: Basophiles are white blood cells and pro-inflammatory cell responsible for initiation of an acute
inflammation. It also stimulated the release of heparin and histamine
21. A
Tips: Antidote of elemental mercury inhalation is dimercaprol and inorganic salt mercury
gastrointestinal absorption the antidote may be dimercaprol and penicillamine as well.
22. C
Tips: Flumazenil is a benzodiazepinic antagonist therefore used as antidote in benzodiazepinic
overdoses.
23. B
Tips: Treatment for cyanide poisoning may be done by sodium thiosulfite, amyl and sodium nitrile as
well.
24. E
Tips: Carbon monoxide can be highly toxic due to its ability to rapidly bind to hemoglobin, myoglobin
and cytochrome oxidase. Essential components for the vital functions in human body.
25. D
Tips: The main and more serious implication concerning acetaminophen toxicity is liver necrosis
26. E
Tips: Salicylate toxicity treatment can first be done by control of the vital functions following
administration of sodium bicarbonate to alkalinize the urine and correct the acidosis,
decontamination with syrup of IPECAC until 30 minutes of overdose and CHARCOAL every 6 hours
may be considered. In case of failure of these methods, hemodialysis should be considered.
27. C
Tips: Aluminum hydroxide causes hypophosphatemia as side effect therefore may be used in the
treatment of hyperphosphatemia.
28. E
Tips: IPECAC is a gastric decontaminant agent used in the treatment of overdoses and poisonings of
many different agents. It acts by inducing vomiting and is best used within

Copyright © 2000-2014 TIPS Inc. Unauthorized reproduction of this manual is prohibited. This manual is being used
during review sessions conducted by PharmacyPrep. 18-11
PharmacyPREP.Com Immunology
60 minutes of agent ingestion. It also has an expectorant effect when used in very low doses.
29. C
Tips: Organophosphates are substances commonly found in insecticides in very toxic to humans due
to formation of a very stable complex with acetylcholinesterase
30. D
Tips: Constipation is a common side effect of atropine poisoning, therefore we should not consider
diarrhea as atropine symptoms of overdose.
31. D
32. A
33. B
34. E
35. B
36. A
37. E
38.
39. D

Copyright © 2000-2014 TIPS Inc. Unauthorized reproduction of this manual is prohibited. This manual is being used
during review sessions conducted by PharmacyPrep. 18-12
www.Pharmacyprep.com Pharmacy Prep Review Questions

PHARMACY PREP
IMMUNIZATION
1) Which of the following populations is LEAST considered
high risk for influenza so that vaccination with influenza
vaccine is not necessary?
A) Person age > 5 yrs to 50 yrs
B. Residents of long-term care facilities (e.g., nursing homes)
C. Patients with diabetes mellitus
D. Patients over the age of 65
E. Patients with COPD or asthma
2) Flu season in Canada?
A. March to December
B. October to April
C. November to December

D. October to Mid November


E. October through November

3) Flu immunization season in Canada?


A) November to April
B) October to Mid November
C) October to April
D) October through December
E) October through April
4. Flu vaccine active against, which of the following virus.
I-Influenza A
II-Influenza B
III-H. influenza type A
A. I only
B. III only
C. I and II only
D. II and III only
E. I,II,III
5. What is incorrect statement about Oseltamavir (Tamiflu)?
A. Oseltamavir is effective against A and B
B. Oseltamavir is neuramidase inhibitor
C. Oseltamavir is used in treatment of influenza A and B
D. Oseltamavir therapeutic antiviral spectrum is limited to influenza A only
E. Treatment is particularly useful high risk patient who have not been vaccinated.

Copyright © 2000-2014 TIPS Inc. Unauthorized reproduction of this manual is prohibited. This manual is being used
during review sessions conducted by PharmacyPrep. 19-1
www.Pharmacyprep.com Pharmacy Prep Review Questions
6. Who should not receive flu shot?
I) Allergies to eggs
II) Child under 6 months age
III) Pregnant women
A. I only
B. III only
C. I and II only
D. II and III only
E. I,II,III
7. Which of the following population least likely to receive flu shots:
A. Elderly over 65 years
B. A 23-year-old worker in long term care facility
C. A 44 year with asthma condition
D. A 25 year old nurse working in elderly home
E. A 45 year old worker in hotel
8. Hepatitis B vaccine provides immunity against:
I-Hepatitis A II-Hepatitis D III-Hepatitis B
A. I only
B. III only
C. I and II only
D. II and III only
E. I,II,III
9. Poison ivy causes contact dermatitis, which of the following type of hypersensitive reaction:
A. Type 1
B. Type 2
C. Type 3
D. Type 4
E. Type 5
10. Dukoral vaccine is indicated for travelers to prevent?
I-E. coli II-Cholera III-Hepatitis A and B
A. I only
B. III only
C. I and II only
D. II and III only
E. I,II,III

Copyright © 2000-2014 TIPS Inc. Unauthorized reproduction of this manual is prohibited. This manual is being used
during review sessions conducted by PharmacyPrep. 19-2
www.Pharmacyprep.com Pharmacy Prep Review Questions
11) All of the following are NOT recommended for flu vaccine, except?
A) Child under 6 months age
B) A person allergy to eggs
C) Person with currently acute bronchitis symptoms
D) Pregnant
E) Person with current symptoms of pneumonia
12. All of the following should not take flu vaccine, except?
A. under 6 months age
B. pregnancy
C. Allergies to egg
D. Persons with flu symptoms
E. A person have taken flu vaccine a month ago
13. Which of the following is the most predominant immune cells?
A. Basophils
B. Neutrophils
C. Esinophils
D. Mast cells
E. Macrophages
14) Influenza vaccine is contraindication in all of the following,
EXCEPT
A) A person flu symptoms
B) Child under 6 months
C) A person with egg allergy
D) Pregnant women
E) A person with fever
15. Hepatitis B vaccine may also prevent infections of?
A. Hepatitis A
B. Hepatitis C
C. Hepatitis D
D. Hepatitis E
E. All types of hepatitis
16. A mother approached to your pharmacy she is asking about varicella vaccine for her 11 mo child.
What you should tell her?
I) It is not suitable because he is 11 months old, should wait until 12 months
II) Should take 1 dose at age of 12 mo
III) May cause signs of the disease at first
A. I only B. III only C. I and II only D. II and III only E. I,II,III

Copyright © 2000-2014 TIPS Inc. Unauthorized reproduction of this manual is prohibited. This manual is being used
during review sessions conducted by PharmacyPrep. 19-3
www.Pharmacyprep.com Pharmacy Prep Review Questions
17. Which of the following is a NOT symptom of flu?
A. Muscle pain
B. chills
C. Fever
D. Hives
E. Headache
18. HSV1 is spread by?
I) direct contact with lesion
II) Sexual contact
III) Oro-fecal transmission
A. I only
B. III only
C. I and II
D. II and III
E. All
19. Which of the following vaccine is used at birth?
A. Hepatitis A
B. Flu vaccine
C. Hepatitis B
D. Vericella
E. All
20. A pregnant women approaches to your pharmacy. She wants to take flu vaccine. She says she is
allergic egg. What is the best recommendation?
A. Take low dose of flu vaccine
B. Take pneumococcal vaccine
C. Give flu vaccine to other close contacts
D. Stay away from people with flu symptoms
E. Give oral osaltemavir (Tamiflu) antiviral therapy
21. The above customer comes after 1 year. She says has 4 month old baby. She want to give flu
vaccine? What to do?
A. Recommend flu vaccine
B. Recommend close contact take flu vaccine
C. Recommend to protect child from flu
D. advise her to give lot of fluid and rest
E. none
22. A customer of your pharmacy travelling to mountain. She has
history of anaphylaxis. Which of the following product pharmacist
should recommend to take?
A) travelers diarrhea vaccine

Copyright © 2000-2014 TIPS Inc. Unauthorized reproduction of this manual is prohibited. This manual is being used
during review sessions conducted by PharmacyPrep. 19-4
www.Pharmacyprep.com Pharmacy Prep Review Questions
B. hepatitis A vaccine
C) Flu vaccine
D. DEET mosquito repellant
E. Epipen
23. JD is a 69 year person travelling to south east Asia. what vaccines are recommended?
I) Hepatitis A
II) Hepatitis B
III) Flu vaccine
A) I only
B) III only
C) I and II
D) II and III
E) I, II, III

Copyright © 2000-2014 TIPS Inc. Unauthorized reproduction of this manual is prohibited. This manual is being used
during review sessions conducted by PharmacyPrep. 19-5
www.Pharmacyprep.com Pharmacy Prep Review Questions
ANSWERS:
1. A
2. B
3. B
4. C
Tips: Flu vaccine is effective against influenza A and B. H. influenza is a gram –ve bacterial infections,
flu vaccine is ineffective against bacterial infections.
5. D
6. C
7. E
8. D
Tips: Hepatitis B patient is susceptible to hepatitis D.
9. D
10. C
11. D
12. B
13. B
14. D
15. C
16. E
17. D
18. C
19. C
Tips: Hep -A is given at age minim 12 mo, Flu vaccine min age 6 mo, Hepatitis at the birth and
varicella min 12 mo.

Copyright © 2000-2014 TIPS Inc. Unauthorized reproduction of this manual is prohibited. This manual is being used
during review sessions conducted by PharmacyPrep. 19-6
www.Pharmacyprep.com Pharmacy Prep Review Questions
20. C
21. B
22. E
23.

Copyright © 2000-2014 TIPS Inc. Unauthorized reproduction of this manual is prohibited. This manual is being used
during review sessions conducted by PharmacyPrep. 19-7
www.Pharmacyprep.com Pharmacy Prep Review Questions
Copyright © 2000-2014 TIPS Inc. Unauthorized reproduction of this manual is prohibited. This manual is being used
during review sessions conducted by PharmacyPrep. 19-8
PharmacyPrep.Com Biotechnology

PHARMACY PREP
BIOTECHNOLOGY
1. Herceptin (Trastuzumab), is
A. Treatment of HER2-overexpressing metastatic breast cancer
B. Tumor necrosis factor alpha inhibitor
C. Glycoprotein inhibitor
D. Interleukin
E. Interferon's
Ans: A
2. Erythropoietin's are?
A. Colony stimulating factors
B. Circulating iron storage
C. Interleukins
D. Immunosuppressant
E. Biotechnological products
Ans: A
Tips: Erythropoietin's are CSF. Products epoeitin alpha and beta are used for the treatment of
chemotherapy induced anemia.
3. Hematocrit is?
A. Circulating iron storage
B. Measures the total iron binding capacity
C. Measures the total free iron
D. Mean volume of iron
E. It is transferrins
Ans: A
4. What is correct about erythropoietin's?
I- Epoeitin alpha are used for anemia associated chronic kidney diseases.
II-Helps in to production of RBCs
III-Produced in bone marrow
A. I only
B. III only
C. I and II only
D. II and III only
E. All of the above
Ans: C
5. Erythropoeitins are produced from?
A. kidney

Copyright © 2000-2014 TIPS Inc. Unauthorized reproduction of this manual is prohibited. This manual is being used
during review sessions conducted by PharmacyPrep. 20-1
PharmacyPrep.Com Biotechnology
B. Liver C. Bone marrow
D. Muscles
E. Thymus gland
Ans: A
6) Filgrastim is a granulocytes CSF indicated in the treatment of?
A) renal, cardiac and hepatic graft rejection
B) neutropenia associated with cancer chemotherapy
C) thrombocytopenia associated with cancer chemotherapy
D) anemia associated with chronic renal diseases
E) chemotherapy induced anemia
Ans: B
7. Filgrastim is?
A. Colony stimulating factor
B. Erythropoietin's
C. Interferon's
D. Human growth hormones
E. Monoclonal antibodies
Ans: A
8. Operlaveukin is?
A. Interleukin 11
B. Colony stimulating factor
C. Interleukin 3
D. Interleukin 2
E. Epoeitin
Ans: A
9) Operlaveukin (IL-11) is indicated for?
A) Chemotherapy induced thrombocytopenia
B) Bone marrow suppression
C) Acute graft rejection
D) Neutropenia
E) Growth factor
Ans: A
10. Muromonab (OKT3) is monoclonal antibody is used for treatment of ?
A) Renal, cardiac and hepatic graft rejection
B. Neutropenia associated with cancer chemotherapy
C. Thrombocytopenia associated with cancer chemotherapy
D. Anemia associated with chronic renal diseases
E. None of the above
Ans: a

Copyright © 2000-2014 TIPS Inc. Unauthorized reproduction of this manual is prohibited. This manual is being used
during review sessions conducted by PharmacyPrep. 20-2
PharmacyPrep.Com Biotechnology
11. CD4 cells represents?
A. Helper T cells
B. Suppressor T cells
C. Cytotoxic T cells
D. Immunoglobulins
E. None of the above
Ans: A
12) A HIV patient how long life span is?
A. 1 to 2 years
B. 2 to 3 years
C. 3 to 4 years
D. 4 to 5 years
E. Can live longer than 10 years
Ans: B
13. Genetic disease hemophilia A is due to deficiency of clotting factor?
A. Factor 11
B. Factor 10
C. Factor 12
D. Factor 8
E. Factor 9
Ans: D
14. Interferons are good examples of:
I) Cytokines
II) Biotechnological products
III) Colony stimulating factors (CSF)
A. I only
B. III only
C. I and II only
D. II and III only
E. All of the above
Ans-C
15. HAMA is?
A) Human antigen and Mouse antibody
B) Human antimouse antibody
C) Murine antibody
D) Humanized antibody
E) Human antibody

Copyright © 2000-2014 TIPS Inc. Unauthorized reproduction of this manual is prohibited. This manual is being used
during review sessions conducted by PharmacyPrep. 20-3
PharmacyPrep.Com Biotechnology
16. Transtuzumab is produced by?
A) cell culture
B) monoclonal antibody method
C) chimeric monoclonal antibody
D) murine monoclonal antibody
E) Humanized monoclonal antibody
17. Chimeric antibody chain is?
A) humanized chain linked to mouse chain.
B) Humanized chain linked to rat chain
C) Humanized chain binding with different antigen
D) One half the antibody is from one species origin where as the other half from other species
18. All of the following are biological derived drugs, except?
A. infliximab
B. Anakinra
C. trastuzumab
D. Methotrexate
E. Adelimumab
19, Rituximab is used for the treatment of?
A. B-Cell follicular lymphoma
B. contraceptives
C. Rheumatoid arthritis
D. Chemotherapy
E. Crohn's disease

Copyright © 2000-2014 TIPS Inc. Unauthorized reproduction of this manual is prohibited. This manual is being used
during review sessions conducted by PharmacyPrep. 20-4
PharmacyPrep.Com Biotechnology
ANSWERS:
1. A
2. A
3. A
4. C
5. A
6. B
7. A
8. B
9. A
10. A
11. A
12. B
13. D
14. C
15. B
16. C
17. D
18. D
19. A

Copyright © 2000-2014 TIPS Inc. Unauthorized reproduction of this manual is prohibited. This manual is being used
during review sessions conducted by PharmacyPrep. 20-5
PharmacyPrep.Com Biotechnology
Copyright © 2000-2014 TIPS Inc. Unauthorized reproduction of this manual is prohibited. This manual is being used
during review sessions conducted by PharmacyPrep. 20-6
www.Pharmacyprep.com Toxicology

PHARMACY PREP
TOXICOLOGY
1. Antidote of mercury may include:
A. Dimercaprol
B. EDTA
C. Deferoximine
D. Succimer
E. Naloxone
2. Which of the following is a benzodiazepinics antagonist:
A. Naloxone
B. Physostigmine
C. Flumazenil
D. Naltrexone
E. Penicillamine
3. What would be the best treatment for cyanide poisoning?
A. Antivenim
B. Sodium thiosulfite
C. Acetylcystein
D. Oxygen therapy
E. Sodium bicarbonate
4) Carbon monoxide can be highly toxic because it easily binds to:
I) Hemoglobin
II) Myoglobin
III) Cytochrome oxidase
A) I only
B) III only
C) I and II only
D) II and III only
E) All are correct
5. Acetaminophen toxicity most well known complication is:
A. Cardiovascular failure
B. Pulmonary edema
C. CNS lethargy
D. Liver necrosis
E. Sedation

Copyright © 2000-2014 TIPS Inc. Unauthorized reproduction of this manual is prohibited. This manual is being used
during review sessions conducted by PharmacyPrep. 21-1
www.Pharmacyprep.com Toxicology
6. Salicylate toxicity treatment include which of the following?
I- Induce emesis with syrup of IPECAC until 30 minutes of overdose
II- Decontamination with CHARCOAL every 6 hours
III- Alkalinisation of urine with sodium bicarbonate
A. I only
B. III only
C. I and II only
D. II and III only
E. All are correct
7. Hyperphosphatemia is best treated by:
A. Magnesium hydroxide
B. Calcium carbonate
C. Aluminium hydroxide
D. Sodium Bicarbonate
E. Sodium phosphate
8. Correct statements regarding IPECAC may include:
I- It has an emetic effect when large doses are administrated
II- It has a expectorant effect when small doses (1 to 2mls) is administrated
III- Used in decontamination procedures during toxic treatments
A. I only
B. III only
C. I and II only
D. II and III only
E. All are correct
9. The organophosphates commonly found in insecticides are thought to act by witch of the
following mechanisms?
A. Combining with acetylcholine
B. Potentiating the action of acetylcholinesterase
C. Forming a very stable complex with acetylcholinesterase
D. Reacting at the cholinergic receptor
E. Preventing the release of acetylcholine from the nerve ending
10. All of the following can be recognized as atropine poisoning, EXCEPT:
A. Dry skin
B. Mydriasis
C. Flushed appearance
D. Diarrhea
E. Delirium

Copyright © 2000-2014 TIPS Inc. Unauthorized reproduction of this manual is prohibited. This manual is being used
during review sessions conducted by PharmacyPrep. 21-2
www.Pharmacyprep.com Toxicology
11. Antidote for atropine poisoning may include:
A. Physostigmine
B. Aminophillin
C. Pralidoxine
D. Flumazenil
E. Dimercaprol
12. Poison Ivy can be treated by which of the following agents?
A) Topical antipruritic like calamine lotion
B) Topical Antihistaminics
C) Systemic antibiotics
D) Topical antibiotics
E) Topical anesthetics
13. A patient is stabilized on heparin as anticoagulant. After one hour of administration of the drug,
he experienced bleeding from his gums, while he was brushing his teeth. The pharmacist can refer to
doctor to get:
A. Ascorbic acid
B. Vitamin K
C. Protamine sulfate
D. Warfarin
E. Quinolone
14. N-acetylcysteine may be the antidote of choice for which of the following agents?
A. Aspirin
B. Warfarin
C. Digoxin
D. Lead
E. Acetaminophen
15. Dimercaprol (BAL) + mercury (Hg) combination characteristics include:
I- Dimercaprol is the best antidote for elemental mercury inhalation poisoning
II- They form a stable complex excreted renally
III- Penicillamine is an alternative when mercury suffer GIT absorption
A. I only
B. III only
C. I and II only
D. II and III only
E. All are correct
16. Naloxone is the best antidote for which of the following overdose agents?
A. Opioid overdose
B. Treatment of alcohol overdose symptoms

Copyright © 2000-2014 TIPS Inc. Unauthorized reproduction of this manual is prohibited. This manual is being used
during review sessions conducted by PharmacyPrep. 21-3
www.Pharmacyprep.com Toxicology
C. Benzodiazepinics overdose
D. Organophosphate overdose
E. Digitalis overdose
17- Management and characteristics of Digoxin toxicity may include:
I- May cause heart failure, cardiac dysrrhythmias, nausea, anorexia, vomiting, and confusion
II- Treated by decontamination and supportive therapy
III- Treated with a digoxin specific antidote known as FAB antibodies
A. I only
B. III only
C. I and II only
D. II and III only
E. All are correct
18. Death due to cyanide poisoning is due to:
A. Cyanide-RBCs complex formation
B. Cyanide-hemoglobin complex formation
C. Cyanide inhibiting of cytochrome oxidase
D. Cyanide increasing hemoglobin levels
E. Coronary vessel oclusion
19. The toxicity of methyl alcohol is due to formation of:
A. Ketones
B. Formic acid
C. Free alcohol radicals
D. Ethylene
E. Aldehyde
20. Acetaminophen toxicity is due to:
A. Oxidation stress
B. Active metabolite
C. Free radical chain
D. Reactive metabolite
E. Reduction of metabolite
21. Salicylate toxicity excessive respiration is due to:
I- Excess production of CO2
II- Pulmonary irritation
III- Central stimulation in the brain
A. I only
B. III only
C. I and II only
D. II and III only
E. All are correct

Copyright © 2000-2014 TIPS Inc. Unauthorized reproduction of this manual is prohibited. This manual is being used
during review sessions conducted by PharmacyPrep. 21-4
www.Pharmacyprep.com Toxicology
22. Grayish mouth and loose of teeth are toxic symptoms of:
I- Fe salts poisoning
II- Cu poisoning
III- Lead poisoning
A. I only
B. III only
C. I and II only
D. II and III only
E. All are correct
23. Vomiting (emesis) is contra indicated if poisoning is due to:
I- Bleaching
II- Gasoline
III- Light petroleum
A. I only
B. III only
C. I and II only
D. II and III only
E. All are correct
24. Universal antidote is a mixture of:
I- Activated charcoal
II- Magnesium oxide
III- Tannic acid
A. I only
B. III only
C. I and II only
D. II and III only
E. All are correct
25. Amphetamines overdose is best treated with administration of:
A. Salicylates
B. Benzodiazepinics
C. Barbiturates
D. Naloxone
E. B and C are right.
26. True statements regarding the treatment of hyperkalemia:
I- If change in ECG is detected give Ca to counteract the excess of K on the heart
II- Bicarbonate and insulin administration can shift K from extra to intracellular
III- Enema of kafexolate (exchange resin) or dialysis helps to remove excess of K from the body
A. I only
B. III only

Copyright © 2000-2014 TIPS Inc. Unauthorized reproduction of this manual is prohibited. This manual is being used
during review sessions conducted by PharmacyPrep. 21-5
www.Pharmacyprep.com Toxicology
C. I and II only
D. II and III only
E. All are correct
27. Cardiovascular drug that overdose can cause cyanide intoxication include:
A. Nitrates
B. Nitroglycerin
C. Nitroprusside
D. Isosorbide
E. Isosorbide dinitrate
28. Lead poisoning can be treated by:
I- BAL-Dimercaprol
II- EDTA-Edentate
III- Penicillamine
A. I only
B. III only
C. I and II only
D. II and III only
E. All are correct
29. Specific antidote for iron preparations overdose include:
A. Dimercaprol
B. Deferoxamine
C. Penicillamine
D. Naloxone
E. Sucimmer
30. Atropine, an anticholinergic drug is used as antidote in the poisoning treatment of:
A. Organophosphates
B. Heavy metal
C. Salicylates
D. Tricyclic antidepressants
E. Iron
31. Treatment of tricyclic antidepressants overdose is done by all the following procedures, EXCEPT:
A. Control the seizures and cardiotoxicity
B. Administration of benzodiazepinics or phenytoin to control seizures
C. Activated charcoal can be used
D. MAO antidepressants can helps to overcome the patient’s seizure
E. Control acidosis with administration of sodium bicarbonate
32. Specific antidote used for poisoning caused by snake and black spider bits
A. Deferoxamine

Copyright © 2000-2014 TIPS Inc. Unauthorized reproduction of this manual is prohibited. This manual is being used
during review sessions conducted by PharmacyPrep. 21-6
www.Pharmacyprep.com Toxicology
B. Ancrod
C. Antivenin
D. Psysostigmine
E. Naloxone
33. Best treatment for carbon monoxide intoxication:
A. Alkalinization of urine
B. Oxygen therapy
C. Supportive therapy
D. Acidification of urine
E. Gastric lavage
34. Which of the following is the isoniazide antidote?
A. Vitamin A
B. Bitamin B6
C. Vitamin B12
D. Folic acid
E. Vitamin E
35. Which of the following is considered the warfarin antidote?
A. Vitamin K
B. Heparin
C. Protamine
D. Filgrastim
E. Vitamin E
36. The major first step in the initial management of ANY intoxication and poisoning is the supportive
care treatment that involves:
A. Detoxification
B. Decontamination
C. Hydration
D. Evaluation and support of vital function (Airway, Breath and Circulation-ABC)
E. Toxicology laboratory tests
37- General management in treating patients with depressed mental status include:
I- Treat hypoglycemia with 50mls of dextrose 50%
II- Administration of thiamine 100mg IV push
III- Administration of naloxone
A. I only
B. III only
C. I and II only
D. II and III only
E. All are correct
38. Decontamination procedures may include:

Copyright © 2000-2014 TIPS Inc. Unauthorized reproduction of this manual is prohibited. This manual is being used
during review sessions conducted by PharmacyPrep. 21-7
www.Pharmacyprep.com Toxicology
A. Gastric lavage
B. Emesis
C. Administration of activated charcoal
D. Administration of adsorbent agents
E. All are correct
39. Intravenous calcium administration is used in which of the following situations?
A. Verapamil overdoses
B. Cocaine overdose
C. Verapamil overdose and hyperkalemia
D. Hyperkalemia
E. Nifedipine overdose
40. What metabolic reaction saturation can lead to acetaminophen toxicity?
A. Glucuronidation
B. Sulfonation
C. Oxidation
D. Glutathione conjugation
E. Reduction
41. Ethanol metabolizes to acetic acid by the enzyme called?
A. Alcohol dehydrogenase
B. Alcohol reductase
C. Glucorunidation
D. Alcohol decarboxylation
42. Glutathione conjugation produce the product called?
A. Mercapturic acid
B. Sulfonation
C. Oxidation
D. Glutathione conjugation
E. Reduction
43. Which of the following is antidote of ASA?
A. N-acetyl cysteine
B. Flumazenil
C. Pentoxifilin
D. Nalaxone
E. None of the above
44. Heparin antidote is?
A. Vitamin K
B. Heparin
C. Protamine sulphate
D. Filgrastim

Copyright © 2000-2014 TIPS Inc. Unauthorized reproduction of this manual is prohibited. This manual is being used
during review sessions conducted by PharmacyPrep. 21-8
www.Pharmacyprep.com Toxicology
E. Vitamin E
45. What is true about charcoal?
I. Increase surface area increase adsorption,
II. Decrease impurities on surface of charcoal increase adsorption.
III. Increase temperature on surface of charcoal increase adsorption.

A. I only
B. III only
C. I and II only
D. II and III only
E. All are correct
46) A mother approaches to your pharmacy. Her 3 yr child swallowed lots of acetaminophen. Which
are initial symptoms?
A. Diarrhea
B. Headache
C. Nausea
D. Fever
E. Vomiting

47. Which of the following is used as urine acidifier in alkaline toxicities?


A. Ammonium chloride
B. Ascorbic acid
C. Sod. Bicarbonate
D. all of the above
E. none of the above
48. In hospital ward a nurse mistakenly administered high dose of heparin dose. Which of the
following is used to treatment of overdose?
A) Vitamin K oral
B) Vitamin K injection
C) Protamine sulphate
D) Vitamin K oral
E) other oral anticoagulants
49. A grandmother administered 2 tbsp tid of amoxicillin 300 mg /5ml instead of 2 tsp tid to 9 mo old
child. What to do?
A) refer to emergence
B) refer to doctor
C) wait and watch
D) Give antidote
E) Give oral rehydration solution
ANSWERS:

Copyright © 2000-2014 TIPS Inc. Unauthorized reproduction of this manual is prohibited. This manual is being used
during review sessions conducted by PharmacyPrep. 21-9
www.Pharmacyprep.com Toxicology
1. A
Tips: Antidote of elemental mercury inhalation is dimercaprol and inorganic salt mercury
gastrointestinal absorption the antidote may be dimercaprol and penicillamine as well.
2. C
Tips: Flumazenil is a benzodiazepinic antagonist therefore used as antidote in benzodiazepinic
overdoses.
3. B
Tips: Treatment for cyanide poisoning may be done by sodium thiosulfite, amyl and sodium nitrile as
well.
4. E
Tips: Carbon monoxide can be highly toxic due to its ability to rapidly bind to hemoglobin, myoglobin
and cytochrome oxidase. Essential components for the vital functions in human body.
5. D
Tips: The main and more serious implication concerning acetaminophen toxicity is liver necrosis
6. E
Tips: Salicylate toxicity treatment can first be done by control of the vital functions following
administration of sodium bicarbonate to alkalinize the urine and correct the acidosis, induce emesis
with syrup of IPECAC until 30 minutes of overdose and decontamination CHARCOAL every 6 hours
may be considered. In case of failure of these methods, hemodialysis should be considered.
7. C
Tips: Aluminum hydroxide causes hypophosphatemia as side effect therefore may be used in the
treatment of hyperphosphatemia.
8. E
Tips: IPECAC is a gastric decontaminant agent used in the treatment of overdoses and poisonings of
many different agents. It acts by inducing vomiting and is best used within 60 minutes of agent
ingestion. It also has an expectorant effect when used in very low doses.
9. C
Tips: Organophosphates are substances commonly found in insecticides in very toxic to humans due
to formation of a very stable complex with acetylcholinesterase
10. D

Copyright © 2000-2014 TIPS Inc. Unauthorized reproduction of this manual is prohibited. This manual is being used
during review sessions conducted by PharmacyPrep. 21-10
www.Pharmacyprep.com Toxicology
Tips: Constipation is a common side effect of atropine poisoning, therefore we should not consider
diarrhea as atropine symptoms of overdose.
11. A
Tips: Physostigmine is considered as the best antidote for atropine poisoning and for antihistaminic
overdose as well.
12. A
Tips: Poison Ivy can be treated by topical antipruritic, oral antihistaminic and systemic
corticosteroids in case of severe poisoning.
13. C
Tips: Protamine sulfate is the recommended antidote for heparin
14. E
Tips: N-acetyl-cysteine is the specific antidote for acetaminophen overdose or poisoning.
15. E
Tips: Antidote of elemental mercury inhalation is dimercaprol and inorganic salt mercury
gastrointestinal absorption the antidote may be dimercaprol and penicillamine as well.
16. A
Tips: Naloxone is a pure opioid antagonist used in opioids overdose.
17. E
Tips: Digoxin toxicity may cause heart failure, cardiac dysrrhythmias, nausea, anorexia, vomiting, and
confusion. The first step in the treatment of overdoses should include decontamination and
supportive therapy. Digoxin has a specific antidote known as FAB antibodies (Digibind).
18. B
Tips: Death due to cyanide poisoning is primarily due to cyanide-hemoglobin complex formation.
19. B
Tips: The toxicity of methyl alcohol is mainly due to formation of formaldehyde.
20. D
Tips: Acetaminophen toxicity is mainly due to reactive metabolite.
21. A
Tips: 21- Salicylate toxicity excessive respiration is due to excess production of CO2.

Copyright © 2000-2014 TIPS Inc. Unauthorized reproduction of this manual is prohibited. This manual is being used
during review sessions conducted by PharmacyPrep. 21-11
www.Pharmacyprep.com Toxicology
22- B
Comments: Lead poisoning is mainly characterized by grayish mouth and loose of teeth. The best
antidote for lead poisoning includes dimercaprol, edentate, penicillamine and succimer.
23. E
Tips: Vomiting induction normally done by administration of IPECA is contra indicated if poisoning is
due to bleaching, gasoline, light petroleum and/or any other corrosive substance.
24. E
Tips: Universal antidote is a mixture of activated charcoal, magnesium oxide and tannic acid.
25. E
Tips: The main concerning in amphetamines overdose is the control of the seizures that can be done
by either benzodiazepinic or barbiturate agents.
26. E
Tips: Hyperkalemia should be immediately treated if change in ECG is detected; give Ca to counteract
the excess of K on the heart and other treatments may be considered such as bicarbonate and
insulin administration that can shift K from extra to intracellular. Alternative treatment of
hyperkalemia include enema of kafexolate (exchange resin) or dialysis that helps to remove excess of
K from the body.
27. C
Tips: Nitroprusside is a direct vasodilator agent widely used in cardiovascular complications. It over
dosage may lead to cyanide intoxication because its end metabolism product is cyanide.
28. E
Tips: The best antidote for lead poisoning includes dimercaprol, edentate, penicillamine and
succimer.
29. B
Tips: Deferoxamine is the only specific antidote used as antidote for iron preparations overdose or
poisoning.
30. A
Tips:Atropine is considered to be the best antidote in the treatment of organophosphates poisoning
31. D

Copyright © 2000-2014 TIPS Inc. Unauthorized reproduction of this manual is prohibited. This manual is being used
during review sessions conducted by PharmacyPrep. 21-12
www.Pharmacyprep.com Toxicology
Tips: MAO antidepressants administration in tricyclic antidepressant overdoses would intensify the
overdose of the patient and would increase the chances of the patients in develop a syndrome called
“serotoninergic syndrome”.
32. C
Tips: Antivenin is the antidote of choice in poisoning due to bits of snake and black spider.
33. B
Tips: Oxygen therapy is the most appropriate method to treat carbon monoxide intoxication avoiding
it to bind to hemoglobin.
34. B
Tips: Vitamin B6 also known as pyridoxine is considered the best antidote in isoniazide overdoses.
35. A
Tips: Vitamin K is considered the specific antidote for warfarin overdose.
36. D
Tips: Evaluation and support of vital function such as airway, breath and circulation should be
considered as the major first step in the initial management of any intoxication and poisoning.
37. E
Tips: Patient with depressed mental status and unknown drug overdose should be general treated by
control of hypoglycemia with 50mls of dextrose 50%, administration of thiamine 100mg IV push and
administration of naloxone
38. E
Tips: Decontamination procedures involve removal of the ingesting agent with gastric lavage, emesis
by IPECA administration, and adsorbent agents administration-charcoal.
39. C
Tips: Verapamil overdose associated with hyperkalemia may be treated by intravenous calcium
administration
40. D
41. A
42. A
43.E

Copyright © 2000-2014 TIPS Inc. Unauthorized reproduction of this manual is prohibited. This manual is being used
during review sessions conducted by PharmacyPrep. 21-13
www.Pharmacyprep.com Toxicology
Tips: there is no antidote for ASA, as overdose has to treat by alkaline diuresis using NaHCO3.
44. C
45. E
46. E
Tips: acetaminophen large doses can cause toxicity. Initially for 2 to 4 hr no symptoms. Then toxic
symptoms can occur in 4 stages:
Stage 1: vomiting can occur
Stage 2: after 24 hr nausea, vomiting can get severe
Stage 3: can cause toxicity, symptoms bleeding, jaundice.
Stage 4: can recover or if not recovered it can cause liver failure
47. B
48. C
49. B
BIBLIOGRAPHIC REFERENCE
1- COMPREHENSIVE PHARMACY REVIEW – Lippincott William & Wilkins – Fourth edition
2- CPS-COMPENDIUM OF PHARMACEUTICALS AND SPECIALITIES - Canadian Pharmacist Association –
2001 edition.
th
3- MEDICAL DICTIONARY – Dorland’s illustrated – 27 edition.
4- PHARMACY PREP – Lectures series & study guide for Evaluating Examination-TIPS - 2003/2004
5- THERAPUTIC CHOICES – Canadian Pharmacist Association -Third edition
th
6- USP DI – Drug Information for the Health Care Professional–15 edition – Volume I.

Copyright © 2000-2014 TIPS Inc. Unauthorized reproduction of this manual is prohibited. This manual is being used
during review sessions conducted by PharmacyPrep. 21-14
Pharmacyprep.com Dilutions and Concentrations

Pharmacy prep
DILUTIONS AND CONCENTRATIONS
Q& A
1) In dosing the drug gentamicin in pediatric patients, for every 1 mg/kg of gentamicin administered,
serum drug concentrations are expected to increase by 2.5 μg/ml. What would be the expected
serum drug concentration following an administration of a 2.5 mg/kg dose of gentamicin?
A) 5 μg/ml B) 6.25 μg/ml C) 10 μg/ml D) 2.5 μg/ml
Ans: B
(X mcg/ml/ 2.5 mg/kg) = (2.5mcg/ml/ 1mg/kg )
2.5 mg/kg x 2.5 mcg/ml
1mg/kg
X = 6.25 μg /ml
2) An elixir is to contain 250 mg of an alkaloid in each teaspoonful dose. How many grams of the
alkaloid will be required to prepare 5 litres of the elixir?
A) 0.25 g B) 5 g C) 250 g D) 2.5 g
Ans: C
= __X g___ 0.25 g
5000 ml 5 ml
X= 250 g
3) A pediatric product contains 100 mg of erythromycin ethyl succinate in each dropperful (2.5 ml) of
the product. How many kilograms of erythromycin ethyl succinate would be required to prepare
5000 pint-size bottles?
A) 74.6 kg B) 84.6 kg C) 99.5 kg D) 94.6 kg
Ans: D
1 pint = 473 ml
473 ml x 5000 = 2, 365,000 ml
0.0001 kg = ____ X_kg___ X = 94.6 kg
2.5 ml 2, 365,000 ml
4) A physician places a patient on a daily dose of 48 units of U80 insulin (80units/mL). How many ml
should the patient inject each day?
A) 0.4 ml B) 0.5 ml C) 0.6 ml D) 0.25 ml
Ans: C

Copyright © 2000-2014 TIPS Inc. Unauthorized reproduction of this manual is prohibited. This manual is being used
during review sessions conducted by PharmacyPrep. 24-1
Pharmacyprep.com Dilutions and Concentrations
U- 80 insulin
80 U = 48 units
1ml X
X= 0.6 ml
5) A 20 ml vial of biologic solution is labelled “2 megaunits.” How many units of drug are present in
every ml of solution?
A) 2000 units
B) 1000 units
C) 100, 000 units
D) 10,000 units
Ans:C
1Megaunit = 1,000,000 units
2 mega units is = 2000,000 units
X= 2 x 1000 x 1000 = 100,000 units
20
6) A prescription calls for 10 units of a drug to be taken 3 times a day. How much will the patient
have taken after 7 days?
A) 21.0 units
B) 0.21 units
C) 2.10 units
D) 210 units
Ans: D
10 units x 3 x 7 = 210 units
7) A physician orders Meprobamate 0.2 g. How much is to be administered if the dose on hand is 400
mg in each tablet?
A) do not dispense
B) give 2 tablets
C) give 1 tablet
D) give ½ tablet
Ans: D
200 mg = _1_ tab
400mg 2

Copyright © 2000-2014 TIPS Inc. Unauthorized reproduction of this manual is prohibited. This manual is being used
during review sessions conducted by PharmacyPrep. 24-2
Pharmacyprep.com Dilutions and Concentrations
8)The usual initial dose of chlorambucil is 150 μg/kg of body weight once a day. How many
milligrams should be administered to a person weighing 154 lbs.?
A) 10.5 mg
B) 18 mg
C) 15 mg
D) 8 mg
Ans: A
154 lbs x_1 kg__ = 70 kg
2.2 lbs
150 μg_ = _ X_
1 kg 70 kg
X = 10, 500 μg = 10.5 mg
9) An initial heparin dose of not less than 150 units/kg of body weight has been recommended of
open-heart surgery. How many ml of an injection containing 5000 heparin units per milliliter should
be administered to a 300-pound patient?
A) 5.1 μl B) 4.1 μl C) 5.1 ml D) 4.1 ml
Ans: D
300 lbs x _1kg_ = 136.36 kg
2.2
150 units = _ X___ = 20, 454.54 units
1 kg 136.36 kg
_5000 u_ = 20454.54 u
1 ml X
X= 4.1 ml
10) The pediatric dose of cefadroxil is 30 mg/kg/day. If a child is given a daily dose of 2 teaspoonful
of a suspension containing 125 mg of cefadroxil per 5 ml, what is the weight in lb. of the child?
A) 19.5 lbs. B) 18.8 lbs. C) 18.3 lbs. D) 18.1 lbs.
Ans: C
(125 mg/ 5 ml)x 2 = 250 mg
250 mg/ X = 30mg/1 kg
X = 8.33 kg
8.33 kg x 2.2 lbs = 18.33 lbs

Copyright © 2000-2014 TIPS Inc. Unauthorized reproduction of this manual is prohibited. This manual is being used
during review sessions conducted by PharmacyPrep. 24-3
Pharmacyprep.com Dilutions and Concentrations
11) If the loading dose of Kanamycin is 7 mg/kg of body weight, how many grams should be
administered to a patient weighing 130 lbs.?
A) 0.492 g
B-0.414 g
C-414 g
D-0.485 g
Ans: B
130 lbs /2.2 = 59.09 kg
7 mg /1 kg = X/ 59.90 kg
X= 413.63 mg
= 0.414 g
12) The adult dose of a liquid medication is 0.1 ml/kg of body weight as single dose. How many
teaspoonfuls should be given to a patient weighing 220 lbs.?
A) 2 tsp.
B) 2.5 tsp.
C) 2 tbsp.
D) 2.5 tbsp.
Ans: A
(220 lbs/2.2 lbs) x 1 kg = 100 kg
X ml/100 kg=0.1 mL/1 kg
X = 10 ml
= 2 tsp
13) Ans: D
14) If the dose of a drug is 0.5-mg/kg body weight/day, how many mg will a 35 lb infant receive per
24 hours?
A) 7.9 mg
B) 7.1 mg
C) 7.2 mg
D) 7.4 mg
Ans: A
(35 lbs/2.2 lbs) ×1 kg = 15.9 kg
0.5 mg/1 kg = ×/15.9 kg
X = 7.9 mg

Copyright © 2000-2014 TIPS Inc. Unauthorized reproduction of this manual is prohibited. This manual is being used
during review sessions conducted by PharmacyPrep. 24-4
Pharmacyprep.com Dilutions and Concentrations
15) What is the weight of 60 ml of oil whose density is 0.9624 g/ml?
A) 5.770 g
B-57.7 g
C-6.0 g
D-0.577 g
Ans: B
V = 60 ml
D = 0.9624 g/ml
D = M/V
M=DxV
= 0.9624 x 60
= 57.7 g
16) A prescription calls for 0.3 g of phosphoric acid with a specific gravity of 1.71. How many
millilitres should be used in compounding the prescription?
A-0.5 ml
B-0.7 ml
C-0.18 ml
D-0.3 ml
Ans: C
D = _M/V = g/ ml
V = M/ D = 0.3 g/1.71 g/ml
X = 0.18 ml
17) How many ml of 0.9% (w/v) NaCl solution should be prepared from 250 ml of 25% (w/v)
solution?
A) 3750 ml
B) 2500 ml
C) 6944 ml
D) 9 ml
Ans: C
C1V1 = C2V2
V2 = (C1V2)/C2
V2 =[ (250 x 25)/0.9 ] = 6944 ml
Step 1: Unknown take as Q1C1
Q1 = ?
C1 =0.9% w/v

Copyright © 2000-2014 TIPS Inc. Unauthorized reproduction of this manual is prohibited. This manual is being used
during review sessions conducted by PharmacyPrep. 24-5
Pharmacyprep.com Dilutions and Concentrations
Step 2: Known take as Q2C2
Q2 = 250 ml
C2 = 25 % w/v
Q1 = ( Q2 x C2)/C1
= (250 ml x 25%)/0.9% = 6944 ml
(X) (0.9)= 250 (25)
X = 6944.4 ml
18) A patient is determined to have 0.8 mg of glucose in each millilitre of blood. Express the
concentration of glucose in the blood as mg%.
A) 800 mg%
B) 0.8 mg%
C) 8 mg%
D) 80 mg%
Ans: D
mg% = X mg/ 100 ml
0.8 mg = X/1 ml
X = 80mg %
19) How many mL of a 1:400 (w/v) stock solution should be used to make 4 litres of a 1:2000 (w/v)
solution?
A-1000 mL
B-200 mL
C-800 mL
D-1600mL
Ans: C
_1 g_ = __X__ = 2 g
2000 ml 4000mL
XmL/2g = 400mL/ 1g
= 800 mL
C1V1 = C2V2
1g x 2000mL = C2 x 4000mL
20 ) If a patient is determined to have 100 mg % of blood glucose, what is the equivalent
concentration in terms of mg/dL?
A) 1 mg/dL
B) 10 mg/dL
C) 40 mg/dL
D) 100 mg/dL
Ans:D
1dL = 100 mL
mg/ dL = mg/100 mL
=100 mg /dL

Copyright © 2000-2014 TIPS Inc. Unauthorized reproduction of this manual is prohibited. This manual is being used
during review sessions conducted by PharmacyPrep. 24-6
Pharmacyprep.com Dilutions and Concentrations
21) Strong Iodine Solution USP contains 5% w/v iodine. How many mg of iodine are consumed daily if
the usual dose is 0.3 mL tid.?
A-15 mg
B-90 mg
C-22.5 mg
D-45 mg
Ans: D
5/0.9 = X
X = 0.045 g x 1000
= 45 mg
22-Express in percentage the fluoride concentration in drinking given in 0.6 ppm.
A-0.06%
B-0.00006%
C-0.0006%
D-0.006%
E-0.06%
Ans: B
0.6g = X
1,000,000 100
X = 0.00006%
23-How many grams of dextrose are required to prepare 4 litres of a 5% solution?
A-0.2 g
B-200 g
C-2 g
D-20 g
Ans-B
x/ 4000 mL = 5 g/100 mL
24-Change to percent the number 1/300.
A-3%
B-33%
C-3.3%
D-1/3%
E-3/1%
Ans:D
1/300 =X/100
(1/3)/100 = 0.33% = 1/3%

Copyright © 2000-2014 TIPS Inc. Unauthorized reproduction of this manual is prohibited. This manual is being used
during review sessions conducted by PharmacyPrep. 24-7
Pharmacyprep.com Dilutions and Concentrations
25) A Pharmacy tech adds 75 mL of strong iodine solution USP (5.0% w/v) to 1 litre of sterile water
for irrigation. What is the % w/v of iodine present?
A) 0.35% B) 0.45% C) 0.37% D) 0.60% E) 70%
Ans: A
C1V1 = C2V2 so
C1 = 5%, V1 = 75 ml, C2=?, V2;1075 ml
C2 = (C1V1)/V2
Technician adds 75 ml to 1000 ml so total volume (V2) is 1075 ml
(5) 75 = X (1075)
X = 0.35%
26-How many grams of potassium citrate are needed to prepare 1 liter of 10%?
A-1000 g
B-50 g
C-100 g
D-10 g
E-1 g
Ans: C
X/1000 = 10/100
X = 100 g
27) How many grams of a drug are required to make 120 mL of a 25% solution?
A-30 g
B-10 g
C-12.0 g
D-120 g
E-25 g
Ans: A
_X_ = _25 g
20 ml 100 ml
X = 30 g
28-Calcium hydroxide topical solution contains 170 mg of calcium hydroxide per 100 mL at 15º C.
Express this concentration as ratio strength.
A-1: 688
B-1: 888
C-1: 588
D-1: 788
Ans: C
170 mg → 0.17 g = 1_
100 ml 100 ml X
X = 588 ≈ 1:588

Copyright © 2000-2014 TIPS Inc. Unauthorized reproduction of this manual is prohibited. This manual is being used
during review sessions conducted by PharmacyPrep. 24-8
Pharmacyprep.com Dilutions and Concentrations
29) How many mg of isofluorophate are contained in 15 g of a 1: 10,000 ophthalmic solution of
isoflurophate in peanut oil?
A-1.7 mg
B-1.9 mg
C-1.8 mg
D-1.5 mg
E-150 mg
Ans: D
___1 g__ = _ X_
10,000 mg 15 g
X = 1.5 mg
30-Express 0.2 % in a ratio strength.
A-1:5000
B-1:50
C-1:500
D-1:5
Ans:C
_0.2_ = _1_
100 x
0.2 X = 100
X = 500
31) How much of a substance is needed to prepare 1L of a 1: 10,000 solution?
A-0.1 g
B-10 g
C-0.01 g
D-1.0 g
Ans: A
_ 1_ = _ X _
10,000 1000
X = 0.1 g
32) A cupric chloride injection (0.4 mg Cu/mL) is used as an additive to IV solution for TPN. What is
the final ratio strength of copper in the TPN solution if 2.5 mL of the injection is added to enough of
the IV solution to prepare 500 mL?
A-1: 500
B-1:5000
C-1: 500,000
D-1: 50,000
Ans: C

Copyright © 2000-2014 TIPS Inc. Unauthorized reproduction of this manual is prohibited. This manual is being used
during review sessions conducted by PharmacyPrep. 24-9
Pharmacyprep.com Dilutions and Concentrations
_0.4 mg__ = X___ 0.001 g = _1_
1 ml 2.5 ml 500 ml X
X = 1 mg ≈0.001 g X = 500,000
1: 500,000
33) How many milliliters of a 23.5% (w/v) concentrate of Sodium Chloride solution should be used in
preparing 650 mL of a stock solution such that 30 mL diluted to liter will yield a 1: 5000 solution?
A-0.2 mL
B-4.33 mL
C) 18.44 mL
D-11.75 mL
Ans: C
_ 1_ = __X_ X = 0.2 g
1000 5000
0.2 g_ = _ X _ X = 4.33 g
30 ml 650 ml
23.5 = 4.33 g X = 18.44 ml
100 X
34) You have a stock solution of 50% Sodium citrate and you were asked to prepare 300 mL of a 10%
solution. How many mL is needed?
A-20 ml
B-15 ml
C-30 ml
D-60 ml
Ans: D
Q1C1 = Q2C2
X (50) = 300(10)
X = 60 ml
35) How many milliliters of 1:16 solution of sodium hypochlorite should be used in preparing 5,000
mL of a 5% solution of sodium hypochlorite for irrigation?
A) 800 ml
B) 2500 ml
C) 4000 ml
D) 300 ml
Ans: C
C1V1 = C2V2
(V1) 6.25% = (5000)(5%)
V1 = (5% x 5000mL)/ 6.25%
X = 4000 ml

Copyright © 2000-2014 TIPS Inc. Unauthorized reproduction of this manual is prohibited. This manual is being used
during review sessions conducted by PharmacyPrep. 24-10
Pharmacyprep.com Dilutions and Concentrations
36)Prepare 1000 ml of KMnO4 1:12,000 compresses out of KMnO4 1: 8000.
A)Add 333.3 mL water to 1000 mL KMnO4 1: 8000
B)Add 666.6 mL water to 333.3 mL KMnO4 1: 8000
C)Add 333.3 mL KMnO4 1: 8000 and enough water to make final volume 1000 mL
D) Add 333.3 mL water to 666.6 mL KMnO4 1: 8000
Ans.D
C1V1 = C2V2
0.0125. X = 0.0083 x 1000
x = 664 ml
37-How many millilitres of 24% (w/v) concentrate of saline solution should be used in preparing 600
mL of a solution such that 10mL diluted to a litre will yield a 0.09% solution?
A-300 ml
B-150 ml
C-50.0 ml
D-225 ml
Ans-D
0.09/ 100 = X/1000
X = 0.9 g
0.9 g/10 ml = X/600 ml X = 54 g
24/100 = 54/ X
X = 225 ml
38-The only source of Sodium Chloride is in the form of tablets, each containing 5.0 g. How many
tablets should be used in preparing 3000 liters of a solution of such strength that 20 mL diluted to
100 mL with water will yield a 0.9% (w/v) solution?
A-60,000 tablets
B-27,000 tablets
C-12,000 tablets
D-9,000 tablets
Ans-B
0.9/100 = X/100 X = 0.9 g
0.9 g/20 ml = X/3,000,000 ml
X = 135,000 g
135,000 g/5 g = 27, 000 tabs
39)How many grams of 10% (w/w) ammonia solution can be made from 1800 g of 28% (w/w) strong
ammonia solution?
A) 6428 g
B)5040 g
C) 50,400 g
D) 642.86 g

Copyright © 2000-2014 TIPS Inc. Unauthorized reproduction of this manual is prohibited. This manual is being used
during review sessions conducted by PharmacyPrep. 24-11
Pharmacyprep.com Dilutions and Concentrations
Ans:
C1V1 = C2V2
V2 = C1 V1/C2
V2 = (28 x 1800 g)/10 = 5040g

(10) X = (1800)(28)
X = 5040 g
40-If a prescription order requires 25 g of concentrated HCI (density 1.18 g/ml), what volume should
the pharmacist measure?
A-29.50 ml
B-0.0212 ml
C-23.0 ml
D-21.2 ml
Ans: D
W= 25 g
D= 1.17 g/ml
V= M/ D = (25/1.18) = 21.2 ml

Copyright © 2000-2014 TIPS Inc. Unauthorized reproduction of this manual is prohibited. This manual is being used
during review sessions conducted by PharmacyPrep. 24-12
www.pharmacyprep.com

PHARMACY PREP.
PHARMACOKINETIC CALCULATIONS
1) A dose of 240 mg was given to a patient, his total body clearance is 3.5 L/min and the drug
excreted unchanged in the urine is 80 mg. What is his non-renal clearance?
a) 240 ml/min
b) 2.3 L/min
c) 3.3 L/min
d) 1.16 L/min
e) 160 ml/min
2) A dose was given, the plasma concentration was 64 mg, t1/2=0.7 hrs. After 7 hours from the initial
dose, what is the drug's concentration?
a) 0.0mg
b) 1mg
c) 2mg
d) 3mg
e) 4mg
3- What is the absolute bioavailability of tablet A?
TABLET A-----------------50mg---------------AUC 40
IV PUSH -------------------10mg---------------AUC 50
a) 80%
b) 0.16%
c) 16%
d) 8%
e) 25%
4) If the rate of infusion of the drug is 500 mg q8hours and clearance is 7.3 L/hr. Find steady state
concentration
a) 85.6 mg/L
b) 856 mg/L
c) 8.56 g/L
d) 8.56 mg/L
e) 68.4 mg/L
5) A drug was found to have zero order kinetic 100 mg ⇒ after 6 days ⇒ 99 mg. How long it will take
to eliminate the entire drug from the body?
a) 100 days
b) 200 days
c) 300 days
d) 600 days

Copyright © 2000-2014 TIPS Inc. Unauthorized reproduction of this manual is prohibited. This manual is being used
during review sessions conducted by PharmacyPrep. 25-1
www.pharmacyprep.com
e) 900 days
6- Ampicillin in 5% dextrose degrades by first order kinetic, at rate constant of 0.026 h. What is the
shelf life of ampicillin?
a) 4 hr
b) 8hr
c) 2hr
d) 16hr
e) 12hr
7) Approximately 50% of cloxacillin is excreted unchanged in the urine. If the normal dosage
schedule for cloxacillin is 125 mg q6h, a patient with renal function 20% of normal should receive?
a) 25 mg q6h
b) 31.25 mg q6h
c) 62.5 mg q6h
d) 75 mg q6h
e) 125 mg q12h
8) Drug which dose is 1500 mg is given every 24 hours. The renal clearance of this drug is 1.2 mg/dL.
Calculate the clearance in ml/min.
a) 98.6 ml/min
b) 76.8 ml/min
c) 86.8 ml/min
d) 66.8 ml/min
e) 43.6 ml/min
1.2 mg --------100 ml 125000 mls--------1440 min (60 min x 24 hrs)
1500 mg---------X X---------------1 min
X = 125000 mls X = 86.8 ml/min
9) If a drug is 50% metabolized and half life drugs is 1 hr . What is the percentage of blood
concentration after 4hrs?
a) 50%
b) 25%
c) 12.5%
d) 6.25%
e) 3.12%
10) A new antifungal was given at a dose of 5mg/Kg by a single intravenous bolus injection to a 32
years old female who weighed 75Kg. The antifungal has an elimination half life of 2 hours and
apparent volume of distribution of 0.28L/Kg.
What is the initial plasma drug concentration in this patient?
a) 36 mg/L
b) 1.8 mg/L

Copyright © 2000-2014 TIPS Inc. Unauthorized reproduction of this manual is prohibited. This manual is being used
during review sessions conducted by PharmacyPrep. 25-2
www.pharmacyprep.com
c) 17.8 mg/L
d) 1.79 mg/L
e) 19 mg/L
11- Following the anterior information, calculate the predicted plasma concentration at 8 hours after
the dose:
a) 15mg/L
b) 111mg/L
c) 11.1mg/L
d) 1.11mg/L
e) 2.64mg/L
12-Following the same anterior information, calculate how much drug remains in the patient body, 8
hours after the administrated dose:
a) 23mg
b) 112mg
c) 100mg
d) 15.3mg
e) 84.4mg
13- Following the anterior information, how long after the dose is exactly 75% eliminated from the
patient body?
a) 2 hours
b) 4 hours
c) 6 hours
d) 8 hours
e) 10 hours
14- What is the recommended rate of infusion for this drug?
a) 85mg/min
b) 58mg/min
c) 0.85mg/min
d) 8.5mg/min
e) 0.085mg/min
15- What is the total body clearance for this patient?
a) 33L/min
b) 43mls/min
c) 43L/min
d) 0.43L/min
e) 0.43mls/min
16- Still following the information of the anterior questions, calculate the corresponding loading
dose:

Copyright © 2000-2014 TIPS Inc. Unauthorized reproduction of this manual is prohibited. This manual is being used
during review sessions conducted by PharmacyPrep. 25-3
www.pharmacyprep.com
a) 98mg
b) 160mg
c) 48mg
d) 24mg
e) 100mg
Two drugs that metabolized by the kidney and liver. Which one has first pass metabolism?
drug x drug y
CLt 1100 460
CLr 110 20
Vd 3.3 2
PB 85% 90%
A) Drug x = oral
B) drug y = oral
C) drug y = rectal
6- Mr. Florence Backs is a 72-year-old patient weighing 71.8 kg who is taking aminophylline 250mg
by i.v. at stat. and then was given immediately by i.v. drip at a rate of 40mg/hour. Theophylline level
at stat = 7.5mg/ml and Vd of theophylline is 0.5L/kg.
What is the expected serum concentration after the initial dose?
a. 5.55mcg/ml
b. 13.1mcg/ml
c. 7.5mcg/ml
d. 6.8mcg/ml

Ans-
What is the expected serum concentration after the initial dose?
Co = Dose x F x S
Vd
= 250 x 1 x 0.8 = 5.55 ug/ml
(0.5) (71.8 kg)
5.55 + 7.5 = 13.05 = 13.1 ug/ml
CLINICAL PHARMACOKINETICS
1. Which of the following are the monitoring parameters of warfarin therapy?
I total blood count
II PT (prothrombin time)
III INR (international normalization ratio)

Copyright © 2000-2014 TIPS Inc. Unauthorized reproduction of this manual is prohibited. This manual is being used
during review sessions conducted by PharmacyPrep. 25-4
www.pharmacyprep.com
a. I only
b. III only
c. I and II
d. II and III
e. I, II and III

Ans D
2. During gentamycin therapy which parameter would you measure?
a.creatinine clearance for renal toxicity (nephrotoxicity)
b. serum creatinine, BUN, blood concentration
c.auditory function because of ototoxicity
d. all of the above
ans: D
3
misoprostol 200mg Tid
Nifedipine 10mg TID
Hydrochlorothiazide 50mg QID
Chlorpropamide 500mg QID
The patient is experiencing transient tachycardia. What is the most appropriate course of action?
a. D/C hydrochlorthiazide
b. Change indomethacin to diclofenac
c. Change misoprostol to sucralfate
d. Change nifidipine to diltiazem
e. D/C chlorpropamide and start insulin

Ans: D
4. Two drugs are in the form of tablet. One of them has lower bioavailability than the other
a. due to lower dissolution rate
b. due to lower disintegration rate
ans: A and B
5. Absolute bioavailability is the result of comparing
a. oral dosage from and IV dosage form
b. oral bioavailability from and sublingual form
c. oral bioavailability from and IM dosage form
d. oral bioavailability from and rectal form

Copyright © 2000-2014 TIPS Inc. Unauthorized reproduction of this manual is prohibited. This manual is being used
during review sessions conducted by PharmacyPrep. 25-5
www.pharmacyprep.com
ans: A
6. What is post antibiotic phenomena(PAP)
a. It is an immunity of the body for a certain period of time against certain microorganism after
treatment with antibiotics.
7. A patient is taking aminophylline IV soln. In order to get higher theophylline level you must
I give the patient IV soln. of aminophylline in a rapid rate
II give a partial loading dose of aminophylline 3mg/kg over 20- 30minutes
III give theophylline instead of aminophylline
a. I only
b. III only
c. I and II
d. II and III
e. I, II and III
ans: D
8. What is the F value for an experimental drug tablet based on the
following data? Drug Dose
AUC (μg/ml/mL/h) Dose Form
20 100 mg po Tablet
30 100 mg po Solution (control)
40 50 mg IV push Injection (control)
www.pharmacyprep.com

PHARMACY PREP.
RATES AND ORDERS OF REACTIONS
1. Plot of log of concentration against time produces a straight line with a:
A. Slope of –K/2.303
B. Slope of –K/t1/2
C. Slope of –Ko
D. slope of K/2.303
E. slope of K/0.693
2. AUC of drug can be determined by a graph using the following rule
I-Trapezoidal rule
II-Fick’s first law
III-Rule of nine
A. I only B. III only C. I and II only D. II and III only E. All of the above
3. Determine the F value for a capsule AUC 20 mg/dL/hour with 100 mg dose when i.v of same drug
AUC is 25 mg/dL/hour with 100 mg dose.
A. 20%
B. 60%
C. 40%
D. 80%
E. 100%
4. Determine the F value for a capsule AUC 20 mg/dL/hour with 50 mg dose when i.v of same drug
AUC is 25 mg/dL/hour with 100 mg dose.
A-20% B-60% C-40% D-80% E-160%
5. Two drugs to be pharmaceutical equivalent, the drugs must:
I-contain same exipient
II-Contain it salts
III-Same therapeutic moiety
A. I only B. III only C. I and II only D. II and III only E. All of the above
6. The time it takes to infused drug to reach plasma steady state concentration (Css) depends on:
I- Elimination half-life of drug.
II-Fraction of free excreted in urine
III-Plasma concentration curve
A. I only B. III only C. I and II only D. II and III only E. All of the above

Copyright © 2000-2014 TIPS Inc. Unauthorized reproduction of this manual is prohibited. This manual is being used
during review sessions conducted by PharmacyPrep. 24
www.pharmacyprep.com
7. A 40 year old patient who weighs 70 kg, needs intravenous infusion of amoxicillin. The desired Css
of drug is 15mg/dL. The physician ordered antibiotic infused for 10 hours. Amoxicillin has t1/2 = 1 hour
and Vd = 9L.
What rate of IV is recommended for this patient? (No loading dose was given);
A. 135 mg/hr
B.936 mg/hr
C. 1000 mg/hr
D. 333 mg/hr
E. 400 mg/hr
8. A drug with zero order reaction eliminating, which is independent of initial concentration. What is
correct about slope?
A. ko
B k/2.303
C. 0.693/k
D. 1/ka
E. None
9. Use of loading dose?
I-To achieve therapeutic concentration immediately
II-Loading dose = desired concentration x Vd
III-Steady state will be achieved immediately and maintained
A. I only B. III only C. I and II only D. II and III only E. All of the above
10. Time to reach steady state is determined by the?
A. Elimination half life
B. Loading dose
C. Maintenance dose
D. The absorption rate
E. dose interval
11. A drug completely renally eliminated. The patient has renal failure. This drug is hepatic
metabolized. What will happen to drug?
A. Drug metabolite accumulates in blood
B. Drug is completely in metabolized in liver
C. Drug is completely eliminate renally
D. Drug is completely protein bound
E. Drug have linear kinetic
12) What is correct about a drug with first order elimination?
A) Drug elimination depends on initial concentration
B) Drug elimination depends on time only

Copyright © 2000-2014 TIPS Inc. Unauthorized reproduction of this manual is prohibited. This manual is being used
during review sessions conducted by PharmacyPrep. 25
www.pharmacyprep.com
C) Drug elimination depends and concentration and time
D) Drug elimination depends renal and hepatic function
E) Drug elimination depends on renal function only
13) What is correct about a drug with zero order elimination?
A) Drug elimination depends on initial concentration
B) Drug elimination depends on time only
C) Drug elimination does NOT depends and concentration and time
D) Drug elimination depends renal and hepatic function
E) Drug elimination depends on renal function only

Copyright © 2000-2014 TIPS Inc. Unauthorized reproduction of this manual is prohibited. This manual is being used
during review sessions conducted by PharmacyPrep. 26
www.pharmacyprep.com
ANSWERS:
1. A
2. A
3. D
AUC po x 100 = 80%
AUC iv
4. E
AUC po x 100
AUC iv
AUC po = 20 x 2 = 40 mg = 1.6 x 100 = 160%
25 mg
5. D
6. A
7. B ion
Tips K = first order
: elimination constant
C Vd = volume of
s
s distribution
R = Css x k x Vd
=

R
_
_
_

V
d

R
a
t
e

o
f

i
n
f
u
s
R = 15 0.693 x 9000 =
100 1 hour
= 935.54 mg/hr
8.
9. C
10. A
11. A
12) B

Copyright © 2000-2014 TIPS Inc. Unauthorized reproduction of this manual is prohibited. This manual is being used
during review sessions conducted by PharmacyPrep. 27
www.pharmacyprep.com
13)

Copyright © 2000-2014 TIPS Inc. Unauthorized reproduction of this manual is prohibited. This manual is being used
during review sessions conducted by PharmacyPrep. 28
www.pharmacyprep.com
Copyright © 2000-2014 TIPS Inc. Unauthorized reproduction of this manual is prohibited. This manual is being used
during review sessions conducted by PharmacyPrep. 29
www.pharmacyprep.com

PHARMACY PREP
PHARMACODYNAMICS
1. In competitive (reversible) antagonism
* *
Ag (agonist) + E + Ag (antagonist)AgE + Ag E
If we use more concentration of Ag what happen?
A. Concentration of AgE increase and Ag*E decrease
B. Concentration of Ag*E increase and AgE decrease
C. Concentration of AgE and Ag*E decrease
D. Concentration of AgE and Ag*E increase
E) No change in AgE and Ag*E rate
2. Lethal dose LD50 is classified as:
I-Dose causes 50% of toxic response in tests
II-Dose causes 50% of effect response in tests
III-Dose cause 50% of death response in tests
A. I only
B. III only
C. I and II only
D. II and III only
E. All of the above
3) The km value of an enzyme is numerically equal to, in first order kinetics
A. Half the maximum velocity (Vmax) expressed in moles/liter
B. Velocity of a reaction divided by substrate concentration
C. Substrate concentration in moles/liter necessary to achieve half the maximum velocity of a
reaction
D. Maximum velocity divided by half the substrate concentration in moles necessary to achieve
maximum velocity
E. Substrate concentration divided by velocity of a reaction

Copyright © 2000-2014 TIPS Inc. Unauthorized reproduction of this manual is prohibited. This manual is being used
during review sessions conducted by PharmacyPrep. 27-1
www.pharmacyprep.com
ANSWERS:
1. A
Tips: Ag = agonist, Ag* = antagonist, E = enzyme
In competitive antagonism, increase agonist concentration, increase agonist complex product
2. B
3. C
First order : Km = 1/2 Vmax
Zero order : Km = Vmax

Copyright © 2000-2014 TIPS Inc. Unauthorized reproduction of this manual is prohibited. This
manual is being used during review sessions conducted by PharmacyPrep. 27-2
Pharmacyprep.com Medicinal Chemistry

PHARMACY PREP.
MEDICAL CHEMISTRY
1. Which of the following drug structure can form epimers?
A.B-lactams antibiotics
B. Quinolones
C. Tetracycline
D. Sulfanilamide
E. Erythromycin
2. Pyrrolidine and piperidine are important heterocyclic compounds in the structure of drugs. Which
of the following drugs has these heterocyclic rings on its structure?
A. Folic acid
B. Clonidine
C. Tetrodotoxin
D. Imipramine
E. Atropine
3. Allylic is a common group in the molecular structure of drugs. Which of the following groups is
commonly called a vinyl group?
A. H2C=CH
B. H3C−O−
C. H2C=CH−CH2−
D. H3C−C−
E. HC≡C−
4. Diastereoisomers that have the opposite configuration at only one of two or more tetrahedral
stereogenic centres present in the respective molecular entities are epimers. Interconversion of
epimers are epimerization. Which of the following class of antibiotics known to have epimerization
properties?
A. B-lactams
B. Quinolones
C. Vancomycin
D. Tetracycline
E. Cephalosporins
5. Oxidation of alkyl group results into alcohol, aldehyde and finally carboxylic acids. What type of
alkyl group that mostly undergoes oxidation to an acid is known as:
A. Terminal alkyl group of aliphatic structure
B. Terminal Aromatic alkyl group of structure
C. Benzylic alkyl group attached to aromatic ring structure
D. Heterocyclic ring structure carbons
E. Cyclic ring structure carbons

Copyright © 2000-2014 TIPS Inc. Unauthorized reproduction of this manual is prohibited. This manual is being used
during review sessions conducted by PharmacyPrep. 28-1
Pharmacyprep.com Medicinal Chemistry
6. Amines are one of the most common group present in the molecular structure of drugs. Primary
amines are mainly metabolized by phase II metabolism:
A. Deamination
B. Dealkylation
C. Acetylation
D. Hydrolysis
E. Oxidation
7. Compound considered a constitutional isomer of hexane include:
A. Cyclohexane
B. Cyclopentane
C. 1-methyl-pentane
D. 1-methyl-cyclohexane
E. None of the above
8. Structure activity relationship (SAR) studies of muscarinic blockers indicate that:
I) Maximum activity is achieved when the quaternary or tertiary nitrogen is three carbons removed
from the ester group
II) Structures which display a quasi-ring conformation exhibit antimuscarinic activity
III) One methyl group of the trimethylammonium group can be replaced by an ethyl group with only
slight loss of activity.
A. I only
B. III only
C. I and II only
D. II and III only
E. All are correct
9. Molecular structures (drugs) having carboxylic acid, amide, alcohol and/or thio group are mainly
metabolized by:
A. Oxidation
B. Hydrolysis
C. Conjugation
D. Etherification
E. Deamination
10. For an antihistaminic drug be effective, it must have which of the following molecular
characteristics on its structure?
I- A terminal tertiary nitrogen
II- More than one aryl or heteroaryl group
III- Presence of C−O−C bonds
A. I only B. III only C. I and II only D. II and III only E. All are correct

Copyright © 2000-2014 TIPS Inc. Unauthorized reproduction of this manual is prohibited. This manual is being used
during review sessions conducted by PharmacyPrep. 28-2
Pharmacyprep.com Medicinal Chemistry
11. Arrange the following groups from the most reactive to the least reactive:
COOH CH4 OCH3 COOˉ
A. COOˉ→COOH→OCH3→CH4
B. COOH→CH4 →COOˉ→OCH3
C. OCH3 →COOH→COOˉ→CH4
D. CH4→OCH3→COOˉ→COOH
E. COOH→OCH3→CH4 →COOˉ
12. Conformational isomerism are stereo isomers produced by;
I- rotation around single bond
II- typically rapidly interconverting at room temperature
III- They can be enantiomers
A. I only B. III only C. I and II only D. II and III only E. All are correct
13) Geometrical isomerism may be classified in CIS and TRAS depending the spatial position of the
molecule around a bond with restricted rotation, which of the following is an example of geometrical
isomerism?
A) Butene
B) 1-Butene
C) 2-Butene
D) 3-Butyne
E) 1-Butane
14. Cylcohexane may form chair and boat conformations. At each carbon has equatorial and axial
conformations. For compounds 1,4-dimethyl-cyclohexane what is the conformation of the 2-methyl
groups in is:
A. Equatorial-equatorial
B. Conformational-conformational
C. Equatorial-spatial
D. Geometric-geometric
E. Spatial-spatial
15. Strength of acids and base may depends on several electronic factors of molecule, Which of the
following electronic factors makes cloroacetic acid is stronger than acetic acid due to:
A. Higher ionization effect
B. Higher molecular weight
C. Higher disintegration rate
D. Electron withdrawal effect
E. All are right

Copyright © 2000-2014 TIPS Inc. Unauthorized reproduction of this manual is prohibited. This manual is being used
during review sessions conducted by PharmacyPrep. 28-3
Pharmacyprep.com Medicinal Chemistry
16) Pyridine is a mono heterocyclic compound very important in the activity of many different drugs.
The molecular structure 1,4-dihydropyridine is found in:
A. Verapamil
B. Digoxin
C. Clonidine
D. Nifedipine
E. Diltiazem
17) Uinolone structure may be found in which of the following substances?
A. Vitamin K
B. Thiamine
C. Cyanocobalamin
D. Folic acid
E. Vitamin A
18. What is incorrect about structure activity of acetylcholine agonist?
A) acetylcholine dissociate into acetyl and choline by acetylcholinesterase enzyme
B) methanocoline has one extra methyl group than Ach
C) Carbacholine has one carbmate group
D) Bethanacholine has one carbamate and methyl group than Ach
E) acetylcholine is the major neurotransmitter of sympathetic system
19. Steroid structures are common in hormones, such estrogen, progesterone, and testosterones.
Steroid contains how many cyclohexane and cyclopentane respectively in it skeleton.
A. 2 and 3 B. 3 and 2 C. 4 and 1 D. 3 and 1 E. 1 and 3
20. Phase I metabolism reaction also known as functionalization metabolism, during this phase drug
will metabolized to obtain functional groups such as hydroxyl group or carboxyl group and these
group in phase II are best metabolized by which of following conjugation reaction:
A. Acetylation
B. Glucuronidation
C. Reduction
D. Hydrolysis
E. Methylation
21. Which of the following is the strongest acid?
A. Acetic acid
B. Hydrochloric acid
C. Perchloric acid
D. Benzylic acid
E. Carboxylic acid

Copyright © 2000-2014 TIPS Inc. Unauthorized reproduction of this manual is prohibited. This manual is being used
during review sessions conducted by PharmacyPrep. 28-4
Pharmacyprep.com Medicinal Chemistry
22. The most common reactions in stomach is hydrolysis. Which of the following functional group is
more susceptible to hydrolysis?
A. R-COO-R
B. R-O-R
C. R-NHCH3
D. R-COOH
E. R-CH=O
23. The Bronsted-Lowry theory describe acid is a proton (hydrogen ion) donor. Phenol is an acid. The
acidic properties of phenol is due to:
A. Stable Cation resonance
B. Weak dissociation
C. Common ion effect
D. Stable anion resonance
E. Fast dissociation is lipophilic solutions
24. What is incorrect in catecholamine biosynthesis?
A) tyrosine is the precursor
B) dopamine produce norepinephrine hydroxylation
C) epinephrine produce norepinephrine by methylation
D) levodopa catalyzed by dopadecarboxylase to dopamine
E) norepinephrine catalyzed by methyl transferase to epinephrine
25. Wrong statement regarding optical isomerism includes:
A. To be optimal isomer molecule should contain minimum one chiral or stereogenic center
B. Diastereomers are non superimposable mirror image
C. Optical isomer rotate plane polarized light
D. Enantiomers are non-superimposable mirror images
E. Has one or more chiral center
26. R-COOH is an acid while R-OH is not due to:
A. Presence of dipole
B. Presence of stable anionic resonance
C. Presence of stable cationic resonance
D. Presence of stable non-ionic resonance
E. Complexation
27. The pH value is calculated mathematically as the:
A. Log of the hydroxyl ion (OH ˉ) concentration
B. Negative log of the OH ˉ concentration
+
C. Log of the hydrogen ion (H ) concentration
+
D. Negative log of the H concentration
E. Ratio of H+/OH ˉ concentration

Copyright © 2000-2014 TIPS Inc. Unauthorized reproduction of this manual is prohibited. This manual is being used
during review sessions conducted by PharmacyPrep. 28-5
Pharmacyprep.com Medicinal Chemistry
28. This reaction is known as: NH4+ + OH ˉ → H2O + NH3
A. First order reaction
B. Ionization reaction
C. Neutralization reaction
D. Equilibrium reaction
E. Protonated reaction
29. Correct statements regarding bioisosters may include:
I- Molecules containing one different group or atom but electronically equivalent
II- May act as antagonist to a normal metabolite
III- Molecules that differs only in the length of the side chain
A. I only
B. III only
C. I and II only
D. II and III only
E. All are correct
30. Which of the following is the molecular structure of isotretinoin, used in oral treatment of acne?
A. 13-CIS Retinoic acid
B. 13-TRANS Retinoic acid
C. 13-CIS-TRANS Retinoic acid
D. Retinoic acid
E. None of the above
31. Morphine is the prototype of opioid analgesics. The analgesic activity of morphine is dependent
of which molecular structure?
A. Phenolic hydroxyl group
B. p-phenyl-N-alkylpiperidine
C. Methyl substitution
D. Piperidine
E. Phenolic amino group
32. Imidazoline is a molecular structure that contains a heterocyclic ring and belongs the molecular
structure of:
A. Prazosin
B. Nifedipine
C. Atropine
D. Clonidine
E. Allopurinol
33. Which of the following cardiovascular medications chemically classified as glycoside?
A. Digoxin

Copyright © 2000-2014 TIPS Inc. Unauthorized reproduction of this manual is prohibited. This manual is being used
during review sessions conducted by PharmacyPrep. 28-6
Pharmacyprep.com Medicinal Chemistry
B. Warfarin
C. Nitrates
D. Nitrites
E. Cholestyramine
34. Nitroglycerine is classified as?
A. Nitrites B. Nitrates C. Nitriles D. Nitrol E. None of the above
35) What functional group is responsible for metallic taste or taste changes in captopril?
A) Sulfonamide
B) Sulfide
C) Sulfhydril group
D) Proline ring
E) Carbamate group
36) Fexofenadine has 1 chiral or asymmetric centers, how many optical isomers are possible?
NH3CCH3OHHONH3CCH3CH3HOOTerfinadineFexofenadineOHOH
A) 1 B) 2 C-3 D-4 E-None
37. Which of the following drug cause bradycardia?
A. Amlodipine B. Nicardipine C. Nifedipine D. Verapamil E. Felodipine
If a drug chemical structure has 3 chiral carbons. How many optical isomers are possible?
A) 2 B) 4 C) 8 D) 9 E) 12
n 2
2 =2 =2x2=4
n= number of chiral centers
3
2 =2 x 2 x 2 = 8

Copyright © 2000-2014 TIPS Inc. Unauthorized reproduction of this manual is prohibited. This manual is being used
during review sessions conducted by PharmacyPrep. 28-7
Pharmacyprep.com Medicinal Chemistry
38) What is incorrect about structure activity of acetylcholine agonist?
A) acetylcholine dissociate into acetyl and choline by acetylcholinesterase enzyme
B) methanocoline has one extra methyl group than Ach
C) Carbacholine has one carbmate group
D) Bethanacholine has one carbamate and methyl group than Ach
E) acetylcholine is the major neurotransmitter of sympathetic system
39) What is incorrect in catecholamine biosynthesis?
A) tyrosine is the precursor
B) dopamine produce norepinephrine hydroxylation
C) epinephrine produce norepinephrine by methylation
D) levodopa catalyzed by dopadecarboxylase to dopamine
E) norepinephrine catalyzed by methyl transferase to epinephrine

Copyright © 2000-2014 TIPS Inc. Unauthorized reproduction of this manual is prohibited. This manual is being used
during review sessions conducted by PharmacyPrep. 28-8
Pharmacyprep.com Medicinal Chemistry
CORRECT ANWERS
MEDICAL CHEMISTRY
1. C
Tips: tetracycline epimerizes to epitetracyclin, which is more stable than tetracyclins
2. E
Tips: Atropine is an antimuscarinic agent having pyrrolidine and piperidine heterocyclic rings on its
molecular structure.
3. A
Tips: Vinyl group is the univalent group CH2:CH.
4. E
Tips: Epimerization is the reaction responsible for the transfer of tetracycline to epi-tetracycline
5. C
Tips: Benzylic carbon group undergo oxidation easily undergoes oxidation to an acid.
6. C
Tips: Acetylation is a phase II metabolism reaction characterized by the gain of an acyl group, and the
main reaction that undergoes primary amines.
7. C
Tips: Isomers are compounds that have the same molecular formula but different chemical structure,
hexane and 1-methyl-pentane are examples of isomerism.
8. D
Tips: Parasympathetic cholinergic drugs have their effect based on nature of their structural
molecule. To be effective, cholinergic drugs most display a quasi-ring conformation. One methyl
group of the trimethylammonium group can be replaced by an ethyl group with only slight loss of
activity
9. C
Tips: Conjugations are phase II metabolism reactions characterized by an incorporation of
endogenous substrate on the molecular structure of drugs normally coming from phase I metabolism
reaction. Molecular structures (drugs) having carboxylic acid, amide, alcohol and/or thio group are
mainly metabolized by a conjugation.
10. E
Tips: In order for an antihistaminic drug be effective, they must have a terminal tertiary nitrogen and
more than one aryl or heretoaryl group on their molecular structure.

Copyright © 2000-2014 TIPS Inc. Unauthorized reproduction of this manual is prohibited. This manual is being used
during review sessions conducted by PharmacyPrep. 28-9
Pharmacyprep.com Medicinal Chemistry
11. A
Tips: Molecules may be classified as more or less reactive depending in the presence of hydrogen
and oxygen. Less hydrogen molecules and more oxygen molecules increase the reactivity of the
compound or substance.
12. C
Tips: Conformational isomerisms are stereoisomers produced by rotation around single bond,
typically rapidly interconverting at room temperature. They are normally classified as enantiomers
and diastereomers.
13. C
Tips: Geometric isomers may be classified as CIS and TRANS depending the spatial position of the
molecule around a bond with restricted rotation. 2-Butene is an example of molecule having
geometric isomerism.
14. A
Tips: Equatorial-equatorial is a structural formation of a molecule occurring at the same distance
from each extremity of an axis.
15. B
Tips: Choracetic acid is stronger than acetic acid due to higher molecular weight
16. D
Tips: Nifedipine is calcium channel blocker that has 1,4-dihydropyridine on its structure responsible
for its mechanism of action.
17. A
Tips: Vitamin K has a quinolone on its molecular structure.
18. E
19. D
Tips: Most of the steroid compounds have three cyclohexanes and one cyclopentane ring on their
molecular structure.
20. B
Tips: Glucuronidation is the main reaction of phase II metabolism reactions, and are responsible
mainly for the metabolism of drugs having hydroxyl, carboxyl and amide group on their molecular
structure.
21. C
Tips: Chloracetic acid is one of the strongest acids due to its molecular structure.

Copyright © 2000-2014 TIPS Inc. Unauthorized reproduction of this manual is prohibited. This manual is being used
during review sessions conducted by PharmacyPrep. 28-10
Pharmacyprep.com Medicinal Chemistry
22. A
Tips: Amides and esters are the only groups undergoing hydrolysis.
23. D
Comments: Phenol is a weak acid due to stable anion resonance
24. C
25. B
Tips: Diastereomers must have more than one stereogenic center and are not mirror image, they
have superimposed image.
26. B
Tips: R-COOH is an acid while R-OH is not due to presence of stable anionic resonance, an indication
of the extent of activity of free radicals in an organic reaction.
The factors to consider: Two of the factors which influence the ionization of an acid are, the strength
of the bond being broken, the stability of the ions being formed.
o Acetic acid: pKa=4.76
o Phenol: pKa=10.00
o Ethanol pKa=16.00

27. D
Tips: pH is the measure of the acidity or basicity of a solution that is calculated mathematically as the
negative log of the H+ concentration
28. C
Comments: NH4+ + OH ˉ → H2O + NB3 is an example of neutralization reaction.
29. C
Tips: Bioisosters are functional group or atoms that impact similar physical and chemical properties
on biological activity of molecule.
30. A
Tips: The molecular structure of isotretinoin, used in oral treatment of acne is 13-CIS Retinoic acids.
The TRANS molecular structure is only used in topical preparations.
31. B
Tips: The analgesic activity of morphine is dependent of p-phenyl-N-alkylpiperidine
32. D
Tips: Clonidine is an antihypertensive drug that depends of its imidazoline molecular structure to be
effective.

Copyright © 2000-2014 TIPS Inc. Unauthorized reproduction of this manual is prohibited. This manual is being used
during review sessions conducted by PharmacyPrep. 28-11
Pharmacyprep.com Medicinal Chemistry
33) A
34) B
35) C
36) B
37) D
38) E
39) C
BIBLIOGRAPHIC REFERENCE
1- COMPREHENSIVE PHARMACY REVIEW – Lippincott William & Wilkins – Fourth edition
2- CPS-COMPENDIUM OF PHARMACEUTICALS AND SPECIALITIES - Canadian Pharmacist Association –
2001 edition.
th
3- MEDICAL DICTIONARY – Dorland’s illustrated – 27 edition.
4- PHARMACY PREP – Lectures series & study guide for Evaluating Examination-TIPS - 2003/2004
5- THERAPEUTIC CHOICES – Canadian Pharmacist Association -Third edition
th
6- USP DI – Drug Information for the Health Care Professional–15 edition – Volume I

Copyright © 2000-2014 TIPS Inc. Unauthorized reproduction of this manual is prohibited. This manual is being used
during review sessions conducted by PharmacyPrep. 28-12
Pharmacyprep.com Medicinal Chemistry
Copyright © 2000-2014 TIPS Inc. Unauthorized reproduction of this manual is prohibited. This manual is being used
during review sessions conducted by PharmacyPrep. 28-13
PharmacyPREP.Com Medicinal Chemistry of autonomic nervous system

Pharmacy Prep
MEDICINAL CHEMISTRY OF AUTONOMIC NERVOUS SYSTEM
1. Which of the following cytochrome is mainly the benzodiazepine action?
A. CYP1A2
B. CYP3A4
C. CYP2D6
E. CYP2C9
E. CYP3C2
2. When the use of MAO antidepressant is associated with some medications cause a serotonergic
syndrome, an example of class of medication that normally cause this syndrome
A. Tricycle antidepressants
B. Birth control pills
C. ß-Blockers
D. Digitalis
E. H1 inhibitors
3. Scabies preferred treatment in pregnancy, lactation and children less than two months old;
I. Permethrin 5%
II. Lindane
III.Precipitated sulfur 6% in petrolatum
A.I only B.III only C.I and II only D.II and III only E. All of the above
4. Which of the following is not a side effect of an opioid analgesic for example morphine:
A. C.V depression
B. Constipation
C. Sedation
D. CNS depression
E. Dilatation of eye pupil
5. Example of peptide neurotransmitter that binds to opioids receptors include:
A. Acetylcholine
B. Norepinephrine
C. Epinephrine
D. Enkephalins
E. GABA

Copyright © 2000-2014 TIPS Inc. Unauthorized reproduction of this manual is prohibited. This manual is being used
during review sessions conducted by PharmacyPrep. 29-1
PharmacyPREP.Com Medicinal Chemistry of autonomic nervous system
6. Alzheimer is a progressive disease characterized by relentless and loss of mental function due to
lack of:
A. Thiamine
B. Dopamine
C. Acetylcholine
D. Norepinephrine
E. Serotonin
7. Topical yeast preparations may be necessary in a diaper dermatitis that has been ongoing for
more than three days since it is likely colonized with Canadida and may need preventative anti yeast
therapy. What are the topical anti yeast preparation causes stains:
I.miconozole 1%
II.Clotrimazole 2%
III.Nystatin
A.I only B.III only C.I and II only D.II and III only E. All of the above
8. CNS depressant that cause retinal damage (ocular damage) includes:
A. Ethanol
B. Alcohol
C. Methanol
D. Propylalcohol
E. Isopropylalcohol
9. Thiopental is used in anesthesia due to high distribution in body tissues and fast onset of action,
we can classify thiopental as:
A. General anesthetic
B. Local anesthetic
C. Opiate
D. Barbiturate
E. Benzodiazepine
10. Which of the following agents may be used as thiopental antagonist?
A. Flumazenil
B. Naloxone
C. Naltrexone
D. Atropine
E. Aminophilin
11. Which of the following is not classified as SSRI’s antidepressant?
A. Fluoxetine
B. Phenelzine
C. Paroxetine
D. Citalopram
E. Sertraline

Copyright © 2000-2014 TIPS Inc. Unauthorized reproduction of this manual is prohibited. This manual is being used
during review sessions conducted by PharmacyPrep. 29-2
PharmacyPREP.Com Medicinal Chemistry of autonomic nervous system
12. The effect of inhalation anesthesia may be determined by:
I- Concentration of the drug in the alveolar airway
II- MAC-Minimum Alveolar Concentration
III- Inhalation anesthesia in not used in children
A.I only B.III only C.I and II only D.II and III only E. All of the above
13. Sumatriptan is widely used in migraine treatment acting on:
A. 5HT1D/1B agonist
B. 5HT1D/1B antagonist
C. 5HT2 agonist
D. Enkephalin
E. Dopamine
14. Benztropine is used in treatment of extra pyramidal syndrome acting as:
A. Cholinergic
B. Anticholinergic
C. Dopaminergic
D. Dopamine antagonist
E. Antiviral drug
15. Correct statements regarding chlorpromazine may include:
I- A phenothiazine anti-psychotic drug
II- Long-term treatment produces tardive dyskinesia
III- Contra indicated in Parkinson’s disease because decrease dopamine.
A.I only B.III only C.I and II only D.II and III only E. All of the above
16. Benzodiazepines as antianxiety agent action:
I- Enhance GABA transmission
II- Act on GABA-A
III- Binds with BZ1 and BZ2 receptors
A.I only B.III only C.I and II only D.II and III only E. All of the above
17. Agent classified as short acting antidepressant may include:
A. Lorazepan
B. Amitriptiline
C. Fluoxetine
D. Trazodone
E. Citalopram
18. Toxin that inhibits sodium sensitivity channels may include:
A. Tetrodotoxin
B. Bungarotoxin
C. Botulism toxin
D. Edrophonium
E. None of the above

Copyright © 2000-2014 TIPS Inc. Unauthorized reproduction of this manual is prohibited. This manual is being used
during review sessions conducted by PharmacyPrep. 29-3
PharmacyPREP.Com Medicinal Chemistry of autonomic nervous system
19. Drug considered as the drug of choice in the treatment of status epileptics include:
A. Oral diazepam
B. Diazepam IV
C. Carbamazepine IV
D. Oral phenytoin
E. Chlorazepate IV
20. Examples of drug(s) used in petit mal epilepsy treatment may include:
I- Ethosuximide
II- Gabapentin
III- Diazepam
A.I only B.III only C.I and II only D.II and III only E. All of the above
21. Dopamine deficiency in the nigrostriatal pathway cause:
A. Alzheimer’s disease
B. Parkinson’s disease
C. Schizophrenia
D. Serotoninergic syndrome
E. Seizures
22. Characteristics regarding Parkinson’s disease may include which of the following?
I- Primary: Loss of substantia nigra neurons
II- Secondary: Loss or interference in the action of dopamine in the basal gland
III- Secondary Parkinson may be the results of long-term treatment with anti-psychotic drugs
A.I only B.III only C.I and II only D.II and III only E. All of the above
23. Parkinson’s disease may be treated with all of the following agents, EXCEPT:
A. Amantadine
B. Selegiline
C. Pergolide
D. Levodopa
E. Donezepil
24. The action of acetylcholine in the motor endplates and in the celiac ganglia is described as:
A. Nicotinic stimulation
B. Muscarinic stimulation
C. Dopaminic stimulation
D. Adrenergic stimulation
E. Serotonin stimulation

Copyright © 2000-2014 TIPS Inc. Unauthorized reproduction of this manual is prohibited. This manual is being used
during review sessions conducted by PharmacyPrep. 29-4
PharmacyPREP.Com Medicinal Chemistry of autonomic nervous system
25. Correct statements regarding schizophrenia may include:
I- Characterized by long term-standing paranoid delusion
II- Treated by anti-psychotic medication
III- Onset of age incidence is normally in later 20’s early 30’s.
A.I only B.III only C.I and II only D.II and III only E. All of the above
26. Which of the following factors would be important in determining the concentration of drug that
would be reached in the cerebrospinal fluid for treatment of meningitis?
I- Oil/Water partition coefficient
II- Plasma binding protein
III- pKa of the drug
A.I only B.III only C.I and II only D.II and III only E. All of the above
27. Select the drug whose major metabolites have therapeutically significant pharmacological
activity
A. Oxazepan
B. Nitrazepan
C. Diazepan
D. Triazolan
E. Clonazepan
28.Leukotriene modifiers, zafirleukast indicated in long term care or mild asthma symptoms over 15
years of age patient; zafirleukast administration with meals:
I.Decreases bioavailability
II.Take atleast 1 hour before and 2 hours after meals
III.Increases bioavailbility
A.I only B.III only C.I and II only D.II and III only E. All of the above
29. Drug commonly used with local anesthetic to decrease the absorption and increase the time of
action of anesthesia includes:
A. Norepinephrine
B. Adrenaline SC
C. Isoproterenol
D. Aminophiline
E. Salmeterol
30. The mechanism of antipsychotic effect of phenothiazines is thought to:
A. Improvement of cholinergic transmission
B. Blockage of catecholamines re-uptake
C. Blockage of dopamine receptors
D. Blockage of enkephalinergic neurons
E. Prolactin release
31. Correct statements regarding codeine may include:
I- Used as antitussive especially in children not less than 2 years old

Copyright © 2000-2014 TIPS Inc. Unauthorized reproduction of this manual is prohibited. This manual is being used
during review sessions conducted by PharmacyPrep. 29-5
PharmacyPREP.Com Medicinal Chemistry of autonomic nervous system
II- Opiod analgesic
III- Cause extreme addiction and drug dependency
A.I only B.III only C.I and II onl D.II and III only E. All of the above
32. Pilocarpine normally used in the treatment of glaucoma may cause which of the following
effects?
I- Mydriasis effect
II- Cholinergic drug
III- Miosis effect
A.I only B.III only C.I and II only D.II and III only E. All of the above
33. Correct statements regarding carbidopa + levodopa include:
I- Used as drug of choice in Alzheimer’s disease
II- Carbidopa inhibit dopa-decaboxylase enzyme allowing levodopa reach the brain without suffer
alterations
III- Levodopa is a pro-drug that can cross blood brain barrier
A.I only B.III only C.I and II only D.II and III only E. All of the above
34. Mechanism of action of levodopa may include:
A. Inhibit the dopamine re-uptake
B. The small amount that reaches the brain is decarboxylated to dopamine
C. Inhibit dopa-decarboxylase enzyme increasing the levels of dopamine
D. Inhibit the action of dopamine
E. Improve the balance between cholinergic and dopaminergic activity resulting in an increased level
of acetylcholine
35. Tardive dyskinesia is mainly characterized by an involuntary movement of mouth and tongue
that can be a side effect of long term treatment with:
A. Phenothiazine
B. Chlorpromazine
C. Dopamine antagonist drugs
D. Antispychotic drugs
E. All are correct
36. Competitive non-depolarizing neuromuscular junction blocker may include:
A. Pancuronium
B. Succinylcholine
C. Nicotine
D. Bungarotoxin
E. Tetradotoxin
37. Correct statements regarding succinylcholine may include which of the following?
I- Non-competitive depolarizing neuromuscular blocker

Copyright © 2000-2014 TIPS Inc. Unauthorized reproduction of this manual is prohibited. This manual is being used
during review sessions conducted by PharmacyPrep. 29-6
PharmacyPREP.Com Medicinal Chemistry of autonomic nervous system
II- Used in general anesthesia providing skeletal relaxation
III- It is a sympathetic agent
A.I only B.III only C.I and II only D.II and III only E. All of the above
38. Examples of drug(s) that may cause secondary-Parkinson’s disease include:
A. Haloperidol
B. Phenothiazines
C. Chlorpromazine
D. Clozapine
E. All can cause secondary Parkinson’s disease
39. Opioids may be used as all the following agents, EXCEPT:
A. Antitussive
B. Analgesic
C. Antidiarrheal
D. Anti-inflammatory
E. Pre-anesthetic
40. Which of the following is the strongest morphine analog opioid agent?
A. Methadone
B. Heroine
C. Codeine
D. Hydrocodone
E. Oxycodone
41. Which of the following opioids can be used as antidiarrheal agent?
A. Diphenoxylate
B. Butorphanol
C. Propoxyphene
D. Pentazocine
E. Methadone
42. Morphine may be used in adjunct treatment of which of the following conditions?
A. Increase intracranial pressure
B. High blood pressure
C. Emphysema
D. Asthma
E. Pulmonary edema secondary to left ventricular failure
43. Anticholinergic drugs may be used to treat which of the following conditions?
I- Parkinson’s disease

Copyright © 2000-2014 TIPS Inc. Unauthorized reproduction of this manual is prohibited. This manual is being used
during review sessions conducted by PharmacyPrep. 29-7
PharmacyPREP.Com Medicinal Chemistry of autonomic nervous system
II- Spasms and ulcers
III- Organophosphates overdose
A.I only B.III only C.I and II only D.II and III only E. All of the above
44. Example of antianxiety drug with great antihistaminic effect may include:
A. Diazepan
B. Hydroxazine
C. Memprobamate
D. Chlorodiazepoxide
E. Oxazepam
45. For a sympathomimetics drug to be effective it should:
I- Fits in receptor
II- Bind to plasma proteins
III- Compete at the site where the neurotransmitter is released
A.I only B.III only C.I and II only D.II and III only E. All of the above
46. Damage of the auxiliary nerve is thought to produce:
th
A. Paralysis of the 5 finger
B. Impaired sensation
C. Numbness of extremities
D. Inability to bend the elbow
E. Paralysis of hand
47. A hypertensive patient suffering from depression can be taking all of the following, EXCEPT:
A. Clonidine
B. Methyldopa
C. Reserpine
D. Guanethidine
E. Hydralazine
48. Drug considered as the drug of choice in the treatment of bipolar-maniac depression may
include:
A. Carbamazepine
B. Lithium
C. Phenytoin
D. Diazepam
E. Valproic acid
49. Which of the following vitamins accelerate the metabolism of phenytoin?
A. Vitamin A

Copyright © 2000-2014 TIPS Inc. Unauthorized reproduction of this manual is prohibited. This manual is being used
during review sessions conducted by PharmacyPrep. 29-8
PharmacyPREP.Com Medicinal Chemistry of autonomic nervous system
B. Vitamin B
C. Vitamin C
D. Folic acid
E. Vitamin E
50. Drugs classified as long acting benzodiazepine include:
A. Diazepam
B. Lorazepam
C. Triazolam
D. Alprazolam
E. Temazepam
51. Drugs classified as intermediate acting benzodiazepine include:
A. Triazolam
B. Midazolam
C. Diazepam
D. Lorazepam
E. Oxazepam
52. All the following are examples of long acting benzodiazepine, EXCEPT:
A. Clonazepam
B. Diazepam
C. Chlorazepate
D. Flurazepam
E. Temazepam
53. Chemical mediator that is released from the end nerve of sympathetic system:
I- Adrenaline
II- Noradrenaline
III- Acetylcholine
A.I only B.III only C.I and II only D.II and III only E. All of the above
54. Chemical mediator that is released from the end nerve of parasympathetic system:
I- Adrenaline
II- Noradrenaline
III- Acetylcholine
A.I only B.III only C.I and II only D.II and III only E. All of the above
55. One of the major inhibitory neurotransmitter in the brain and mediates include:
A. GABA
B. Serotonin
C. Acetylcholine
D. Enkephalin
E. Epinephrine

Copyright © 2000-2014 TIPS Inc. Unauthorized reproduction of this manual is prohibited. This manual is being used
during review sessions conducted by PharmacyPrep. 29-9
PharmacyPREP.Com Medicinal Chemistry of autonomic nervous system
56. Which of the following may stimulates adrenaline release from adrenal medulla?
A. Acetylcholine in very low doses
B. Nicotine in very low doses
C. Bilirubin
D. Noradrenaline
E. None of the above
57. Tolerance of narcotics is normally characterized by which of the following statements?
I- Decrease duration of analgesia
II- Decrease sensitivity to receptors
III- The dose should be first adjusted instead to change for a stronger narcotic.
A.I only B.III only C.I and II only D.II and III only E. All of the above
58. All are examples of amides local anesthetics, EXCEPT:
A. Mepivacaine
B. Bupivacaine
C. Lidocaine
D. Procaine
E. Prilocaine
59. Which of the following anesthetics metabolites is characterized by its PABA structure?
A. Amides local anesthetics
B. Ester type local anesthetics
C. All local anesthetics
D. All general anesthetics
E. Lidocaine
60. The CNS – Central Nervous System is mainly constituted of:
I- Brain
II- Spinal cord
III- Thoracic Lumbar
A.I only B.III only C.I and II only D.II and III only E. All of the above
61. All the following group of drugs has direct action on CNS, EXCEPT:
A. Cardiotonics
B. Analgesics
C. Anesthetics
D. Antidepressives
E. Antipsychotics
62. All of the following are important neurotransmitters, EXCEPT:
A. Acetylcholine

Copyright © 2000-2014 TIPS Inc. Unauthorized reproduction of this manual is prohibited. This manual is being used
during review sessions conducted by PharmacyPrep. 29-10
PharmacyPREP.Com Medicinal Chemistry of autonomic nervous system
B. Dopamine
C. Norepinephrine
D. Muscarinic-M1
E. Serotonin
63. Catecholamines neurotransmitter may include:
I- Dopamine
II- Norepinephrine
III- Epinephrine
A.I only B.III only C.I and II only D.II and III only E. All of the above
64. Agents well known as precursor of dopamine include:
A. Epinephrine
B. Tyrosine
C. Tyramine
D. Norepinephrine
E. Thiamine
65) Reaction that converts norepinephrine to epinephrine is normally called:
A) Catecholinization
B) Methylation
C) Oxidation
D) Deamination
E) Glutathione conjugation
66) Neurotransmitter with an indole group in its structure includes:
A) Serotonin
B) Acetylcholine
C) Histamine
D) Dopamine
E) PABA
67. Example of amino acid neurotransmitter may include:
A. Tyrosine
B. Thiamine
C. Histamine
D. Glycine
E. Glutamate
68. Examples of cholinergic neuroreceptors may include:
I- Muscarinic receptors
II- Nicotinic receptors
III- Serotonergic receptors
A.I only B.III only C.I and II only D.II and III only E. All of the above

Copyright © 2000-2014 TIPS Inc. Unauthorized reproduction of this manual is prohibited. This manual is being used
during review sessions conducted by PharmacyPrep. 29-11
PharmacyPREP.Com Medicinal Chemistry of autonomic nervous system
69. The major dopaminergic receptor that controls the extrapyramidal system includes:
A. D1 B. D2 D. D4 E. D5 C. D3
70. Which of the following is an example of pure narcotic antagonist?
A. Pentazocine
B. Butorphanol
C. Naltrexone
D. Naloxone
E. C and D are pure narcotic antagonists
71. Example of opioid used in the treatment of alcoholism includes:
A. Morphine
B. Naloxone
C. Hydrocodone
D. Naltrexone
E. Codeine
72. Opioid widely used during labor due to its less liability to induce neonate respiratory depression
includes:
A. Meperidine
B. Morphine
C. Methadone
D. Oxycodone
73. Intense pruritus and vasodilatation are side effects of classic opioids due to stimulation and
release of:
A. Adrenaline
B. Histamine
C. Serotonin
D. Mu receptors
E. Histadine
74. Examples of neurotransmitters used by barbiturates drugs include:
I- Benzodiazepines neurotransmitters
II- GABA
III- Glycine neurotransmitters
A.I only B.III only C.I and II only D.II and III only E. All of the above
75. Primidone is a prodrug that when biotransformed become:
A. Carbamazepine
B. Risperidone
C. Phenobarbital
D. Lorazepan

Copyright © 2000-2014 TIPS Inc. Unauthorized reproduction of this manual is prohibited. This manual is being used
during review sessions conducted by PharmacyPrep. 29-12
PharmacyPREP.Com Medicinal Chemistry of autonomic nervous system
E. Phenytoin
76. MAO-Monoamine Oxidase inhibitor increases the bioavailability of biogenic amines and are
mainly used to treat depression. A classic example of MAO is:
A. Doxepine
B. Reserpine
C. Venlafaxine
D. Desipramine
E. Phenelzine
77. Example of less sedating TCAs-Tricyclic Antidepressants include:
A. Desipramine
B. Clomipramine
C. Amitriptiline
D. Imipramine
E. Nortriptiline
78. Tricyclic antidepressant with effect in serotonin system used to treat maniac compulsive
disorders includes:
A. Lithium
B. Phenelzine
C. Clomipramine
D. Desipramine
E. Nortriptiline
79. Which of the following is a wrong statement regarding lithium?
A. Most effective psychiatric medication used to treat bipolar depression
B. Interfere with trans membrane Na exchange
C. Takes 2 to 3 weeks to start to show pharmacological effect
D. Weight gain and weight loss is a side effect of lithium therapy
E. Blood levels less than 1.5mEq/L cause over dose symptoms.
80. Antipsychotic drugs are effective in treating all of the following conditions, EXCEPT:
A. Bizarre behavior
B. Dellutions and hallucinations
C. Schizophrenia
D. Parkinson’s disease
E. Alzheimer’s disease
81. Mechanism that best describes the action of general anesthetics include:
A. Reversible block nerve impulse conduction
B. Reversible loss of sensation at their administration site
C. Binds to specific membrane Na ion, restricting Na permeability in response to partial
depolarization

Copyright © 2000-2014 TIPS Inc. Unauthorized reproduction of this manual is prohibited. This manual is being used
during review sessions conducted by PharmacyPrep. 29-13
PharmacyPREP.Com Medicinal Chemistry of autonomic nervous system
D. Inhibit sensory, autonomic reflex and skeletal muscles inducing analgesia, amnesia and loss of
consciousness
E. Do not produce loss of consciousness
82. Mechanism of action of local anesthetics may include:
A. Reversible block nerve impulse conduction
B. Reversible loss of sensation at their administration site
C. Binds to specific membrane Na ion, restricting Na permeability in response to partial
depolarization
D. Do not produce loss of consciousness
E. All are correct
83. Example of general anesthetic with malignant hyperthermia as side effect include:
A. Isoflurane
B. Halothane
C. Sevoflurane
D. Nitrous oxide
E. Fentanyl
84. Volatile liquid general anesthetic that is more recommended to be used in children includes:
A. Halothane
B. Sevoflurane
C. Isoflurane
D. Enflurane
E. Nitrous oxide
85) Endorphins may be classified as which of the following?
A) Endogenous neurotransmitter
B) A new class of topical anti-inflammatory agents
C) Neuromuscular blocking agents
D) Biogenic amines believed to cause schizophrenia
E) Endogenous opioid peptides
86. Example of MAO-B inhibitor used to treat Parkinson’s disease include:
A. Hydralazines
B. Phenelzine
C. Selegeline
D. Clomipramine
E. Levodopa
87. Characteristics of agonist drugs may include which of the following statements?
A. Bind to a receptor site and produces 100% of biological response

Copyright © 2000-2014 TIPS Inc. Unauthorized reproduction of this manual is prohibited. This manual is being used
during review sessions conducted by PharmacyPrep. 29-14
PharmacyPREP.Com Medicinal Chemistry of autonomic nervous system
B. Bind to a receptor site but do not produces 100% of the biological response
C. Bind to a receptor site and produces 100% biological response only when given in high doses
D. Bind to receptor site and block the biological response
E. Bind to the receptor site and block the biological response in 50%
88. Drug dosages that can kills 50% of the experimental animals receiving the drug is called:
A. Effective dose
B. Minimum dose effect
C. Lethal dose
D. Maximum dose effect
E. Therapeutic index
89. Inhaled carticosteroids indicated in asthma long term prevention symptoms, suppression, control
of inflammation: space/holding chamber devices and mouth washing after inhalation.
I.Decreases local side effects
II.Decrease systemic absorption
III.Increases systemic absorption
A.I only B.III only C.I and II only D.II and III only E. All of the above
90. The action of acetylcholine released from parasympathetic fibers in visceras are mediated by
which receptor?
A. Muscarinic
B. Nicotinic
C. Adrenergic
D. Dopaminic
E. Serotonergic
91. Which of the following neurotransmitter is used by all preganglionic fibers of the autonomic
nervous system?
A. Dopamine
B. Acetylcholine
C. Epinephrine
D. Norepinephrine
E. GABA
92. The major neurotransmitter for sympathetic postganglionic fibers is known as:
A. Dopamine
B. Acetylcholine
C. Epinephrine
D. Norepinephrine
E. GABA

Copyright © 2000-2014 TIPS Inc. Unauthorized reproduction of this manual is prohibited. This manual is being used
during review sessions conducted by PharmacyPrep. 29-15
PharmacyPREP.Com Medicinal Chemistry of autonomic nervous system
93) Stimulation of blood
vessels by α1-receptors
may causes:
A) Dilation
B) Constriction
C) Increase in blood flow
D) Vasodilatation
E) All are wrong
94. The peripheral nervous system is anatomically divided in:
I- Somatic Nervous System
II- Autonomic Nervous System
III-Spinal cord
A.I only B.III only C.I and II only D.II and III only E. All of the above
95. Part of nervous system responsible for body homeostasis:
A. Central Nervous System
B. Somatic Nervous System
C. Spinal cord
D. Autonomic nervous system
E. Brain
96. Acetylcholine is an ester neurotransmitter synthesized from:
A. Gluthamine
B. Tyrosine
C. Acetyl CoA
D. Dopa
E. Cathecolamines
97. Nicotinic receptors are mainly found in:
I- Motor end plate
II- Adrenal medulla
III-Autonomic ganglia
A.I only B.III only C.I and II only D.II and III only E. All of the above
98. Prototype of muscarinic antagonist class of drugs:
A. Atropine
B. Scopolamine
C. Ipratropium
D. Benztropine
E. Oxybutynin
99. Botulism toxins are used in the treatment of prolonged muscle spasm where a small amount of the
toxin is injected directly into a muscle fiber, causing the muscle to relax. What is the action of
botulism toxins in acetylcholine?

Copyright © 2000-2014 TIPS Inc. Unauthorized reproduction of this manual is prohibited. This manual is being used
during review sessions conducted by PharmacyPrep. 29-16
PharmacyPREP.Com Medicinal Chemistry of autonomic nervous system
A. It blocks the release of acetylcholine
B. It increase the release of acetylcholine
C. It inhibit the nicotinic receptors
D. It stabilizes the serum acetylcholine
E. It increase the action of acetylcholine
100. Malignant hyperthermia can be a side effect of general anesthetics and may be treated by:
A. Aminophylline
B. Terbutaline
C. Dantrolene
D. Pancuronium
E. Dobutamine
101. Which of the following antianxiety agents causes the least sedation?
A. Diazepam
B. Buspirone
C. Chlordiazepoxide
D. Oxazepam
E. Chlorazepate
102. Which of the following does NOT cause miosis?
A. Bethanechol B. Methanachol C. pilocarpine D. Tiotropium E. Physostigmine
103. Which of the following does NOT increase blood pressure?
A. pseudoephedrine B. ephedrine C. phenylephrine D. clonidine E. Tyramine
104. Physostigmine is antidote of?
A. Organophosphate B. Neostigmine C. Atropine D. Pilocarpine E. benzodiazepine
105. Atropine is antidote of?
A. organophosphate B. Neostigmine C. Atropine D. Pilocarpine E. benzodiazepine
106. Antidote of atropine is?
A. organophosphate B. Neostigmine C. Atropine D. physostigmine E. benzodiazepine
107)The difference between dopamine and amphetamine?
A) one methyl group
B) one hydroxyl group
C) 2 hydroxyl group and 1 methyl group
D) 2 methyl group and 1 hydroxyl group
E) One methyl and one hydroxyl group

Copyright © 2000-2014 TIPS Inc. Unauthorized reproduction of this manual is prohibited. This manual is being used
during review sessions conducted by PharmacyPrep. 29-17
PharmacyPREP.Com Medicinal Chemistry of autonomic nervous system
108) Difference between pseudoephedrine and methylamphetamine is?
A) one methyl group
B) one hydroxyl group
C) 2 hydroxyl group and 1 methyl group
D) 2 methyl group and 1 hydroxyl group
E) One methyl and one hydroxyl group

Copyright © 2000-2014 TIPS Inc. Unauthorized reproduction of this manual is prohibited. This manual is being used
during review sessions conducted by PharmacyPrep. 29-18
PharmacyPREP.Com Medicinal Chemistry of autonomic nervous system
CORRECT ANSWERS
NERVOUS SYSTEM
1. B
Tips: Generally benzodiazepinics binds to the most abundant cytochrome P-450 in the body: 3A4.
2. A
Tips: Serotoninergic syndrome is characterized by agitation, neuromuscular rigidity, hypotension,
coma and death. It mainly happens with the concomitant use of MAO antidepressant with other
medication that also have interference in the serotonin neuroreceptors an example of drugs that
may cause this syndrome is clomipramine, meperidine and Selective serotonin re-uptake inhibitors
antidepressives.
3. B
Tips: Scabies preferred treatment in pregnancy, lactation and children less than two months old is
Precipitated sulfur 6% in petrolatum. Pemethrin 5% is currently preferred treatment for adults and
children older than two months. Lindane 1% has convulsions as side effect.
st
Ref: Patient Selfcare, P.613 1 edition, CPhA, Canada
4. E
Tips: Morphine causes a miosis side effect therefore diminishes eye pupil.
5. D
Tips: Enkephalins are pentapeptides that bind to opioid receptors. They act as neurotransmitter or
neuromodulators at many locations in the brain and spinal cord and play a part in pain perception,
movement, mood, behaviour and neuroendocrine regulation.
6. C
Tips: Comments: Alzheimer’s is a progressive disease characterized by a imbalance of acetylcholine
and dopamine, low levels of acetylcholine and high levels of dopamine.
7. B
Tips: Nystatin known to cause staining
8. C
Tips: Methanol when is metabolized form formaldehyde that is a very toxic substance causing retinal
damage.
9. D
Tips: Thiopental is a barbiturate mainly used as adjunct in general anesthesia.

Copyright © 2000-2014 TIPS Inc. Unauthorized reproduction of this manual is prohibited. This manual is being used
during review sessions conducted by PharmacyPrep. 29-19
PharmacyPREP.Com Medicinal Chemistry of autonomic nervous system
10. E
Tips: Aminophylline is considered the thiopental antagonist drug.
11. B
Tips: Phenelzine is a MAO-Monoamino Oxidase inhibitor antidepressant.
12. C
Tips: Inhalation anesthesia is the most used method of general anesthesia for children
13. A
Tips: Sumatriptan activate serotonin receptors-5HT1B/1D agonists. It is used in the abortive
treatment of migraine.
14. B
Tips: Benztropine is a parasympathetic anti-cholinergic agent used specifically in the treatment of
extra pyramidal syndrome-Parkinson’s disease.
15. E
Tips: Chlorpromazine is a phenothiazine anti-psychotic drug completely contra indicated in
Parkinson’s disease because decrease dopamine levels and may cause tardive diskinesia, abnormal
movements of tongue and mouth, as side effect of its long-term therapy.
16. E
Tips: Benzodiazepinics bind to GABA-A and enhance its transmission.
17- D
Comments: Trazodone is the shortest acting antidepressant agent.
18. A
Tips: Tetradotoxin is a toxin that inhibits sodium sensitivity channels.
19. B
Tips: Diazepam IV is the most appropriate drug in treating status epileptics due to its fastest onset of
action.
20. A
Tips: Valproic acid is the drug of choice in the treatment of petit-mal epilepsy. Phenobarbital and
clonazepan are also used.
21. B
Tips: Parkinson’s disease is mainly caused by hypofunction of nigrostriatal pathway resulting in
decrease of dopamine levels and increase of acetylcholine levels.

Copyright © 2000-2014 TIPS Inc. Unauthorized reproduction of this manual is prohibited. This manual is being used
during review sessions conducted by PharmacyPrep. 29-20
PharmacyPREP.Com Medicinal Chemistry of autonomic nervous system
22. E
Tips: Parkinson’s disease is characterized by a decrease in dopamine levels causing tremors and
muscular rigidity among other symptoms. It can be classified as primary, when occurs loss of
substantia nigra neurons and secondary, when occurs loss or interference in the action of dopamine
in the basal gland, normally due to drug induction as per example long-term treatment with anti-
psychotic drugs.
23. E
Tips: Donezepil is used in the treatment of Alzheimer’s disease and it is completely contra-indicated
in Parkinson’s because of decrease in dopamine levels.
24. A
Tips: Nicotinic receptors are found at the motor end plate, in all autonomic ganglia and in the
adrenal medulla.
25. E
Tips: Schizophrenia is a mental disorder characterized by disturbances in form and content of
thought, delusions, hallucinations, mood disorder, loss of sense of self and relationship to the
external world and behavior. Antipsychotic drugs mainly treat it.
26. E
Tips: Oil/Water partition coefficient is one of the most important considerations when choosing a
drug for treatment of meningitis because completely water-soluble drugs cannot cross blood brain
barrier. As much the drug is bound to plasma protein less drug will reach the site of action. PKa of
the drug is other very important factor to be considered when determining the concentration of
drug that would reach the cerebrospinal fluid
27. C
Tips: Diazepam metabolites have therapeutically significant pharmacological activity.
28. C
Tips: Zafirleukast with meals decreases bioavailability.
29. A
Tips: Epinephrine is a vasoconstrictor used in adjunct with local anesthesia to increase the duration
of action of local anesthetic agents.
30. C
Tips: Antispychotic drugs are used to reduce the intensity and frequency of psychotic symptoms.
They act primarily by blocking dopamine receptors.
31. C

Copyright © 2000-2014 TIPS Inc. Unauthorized reproduction of this manual is prohibited. This manual is being used
during review sessions conducted by PharmacyPrep. 29-21
PharmacyPREP.Com Medicinal Chemistry of autonomic nervous system
Tips: Codeine is not a strong opioid analgesic. It is classified as moderate. Although it may cause
addiction as most of all opioids, codeine is not well known as a drug that causes extremely
dependency or strong drug dependency.
32. D
Tips: Pilocarpine is a parasympathetic cholinergic agent that causes miosis.
33. D
Tips: Carbidopa + Levodopa is the drug of choice in the treatment of Parkinson’s disease, levodopa
when used alone loss part of its action due suffer decarboxylation from dopa-decarboxylase enzyme,
thus carbidopa is used in combination to inhibit dopa-decarboxylase enzyme allowing levodopa
reach the brain without suffer alterations
34. B
Tips: Levodopa is a prodrug that cross blood brain barrier however it loss most of its action in the
way to reach the brain crossing blood brain barrier just a small amount of the drug that is
decarboxylated to dopamine.
35. E
Tips: Tardive dyskinesia is mainly characterized by an involuntary movement of mouth and tongue
that can be a side effect of long-term treatment with phenothiazines, dopamine antagonist drugs
and other antispychotic drugs.
36. A
Tips: Pancuronium is a nicotinic antagonist neuromuscular junction blocker acting as competitive
non-depolarizing agent.
37. C
Tips: Succinylcholine is a nicotinic antagonist neuromuscular junction blocker acting as non-
competitive depolarizing agent. It is mainly used as adjunct in general anesthesia providing skeletal
relaxation.
38. E
Tips: Secondary Parkinson’s disease can be induced by most of antipsychotic drugs due to decrease
in dopamine levels by the blockage of dopamine neurotransmitters.
39. D
Tips: Opioids have no action as anti-inflammatory agents.
40. B
Tips: Heroine is the strongest morphine analog.
41. A

Copyright © 2000-2014 TIPS Inc. Unauthorized reproduction of this manual is prohibited. This manual is being used
during review sessions conducted by PharmacyPrep. 29-22
PharmacyPREP.Com Medicinal Chemistry of autonomic nervous system
Tips: Diphenoxylate, well known as lomotil, and loperamide are opioids used in the treatment of
diarrhea.
42. E
Tips: Morphine also may be used in adjunct treatment of pulmonary edema secondary to left
ventricular failure.
43. E
Tips: Anticholinergic drugs as benztropine can be used to treat Parkinson’s disease. Scopolamine
another anticholinergic drug is used to treat spasms and ulcers and atropine, the anticholinergic
prototype can be used to treat organophosphates overdose, so all are correct answers.
44. B
Tips: Hydroxazine is an antianxiety drug with great antihistaminic effect
45. A
Tips: One of the most important considerations in the effectiveness of a drug is fit it into receptor to
produce the biological or pharmacological response.
46. D
Tips: Damage of the auxiliary nerve will lead to inability to bend the elbow.
47. C
Tips: Reserpine is used in the treatment of hypertension and should be avoid in patients suffering
depression because it acts depleting tissues and central nervous system stores of catecholamines
and serotonin.
48. B
Tips: Lithium is the most effective psychiatric medication used to treat bipolar depression disorder
49. D
Tips: Phenytoin deplete the body of folate stores, supplementation with folic acid may result in
lowered serum phenytoin concentration and possible loss of seizure control, therefore, an increase
in phenytoin dosage is necessary in patients who receives folate supplementation.
50. A
Tips: Diazepam is one of the longest acting benzodiazepinic with duration of action up to 48 hours.
51. D

Copyright © 2000-2014 TIPS Inc. Unauthorized reproduction of this manual is prohibited. This manual is being used
during review sessions conducted by PharmacyPrep. 29-23
PharmacyPREP.Com Medicinal Chemistry of autonomic nervous system
Tips: Intermediate acting benzodiazepinics include: Alprazolam, bromazepam, lorazepam and
temazepam.
52. E
Tips: Temazepam is an intermediate acting benzodiazepinic.
53. C
Tips: Noradrenaline and adrenaline are chemical mediator that are released from the end nerve of
sympathetic system
54. B
Tips: Acetylcholine is the only chemical mediator that is released from the end nerve of
parasympathetic system
55. B
Tips: GABA is a neurotransmitter permeable to Cl ions and the major inhibitor of Central Nervous
System.
56. B
Tips: Nicotine in low doses stimulates the release of adrenaline from adrenal medulla.
57. E
Tips: Tolerance of narcotics results in a decrease of sensitivity to receptors and therefore decreases
in the duration of analgesia.
58. D
Tips: Procaine is an ester local anesthetic.
59. B
Tips: Ester local anesthetics have similar structure with GABA neurotransmitter.
60. C
Comments: Thoracic lumbar is part of peripheral nervous system.
61. A
Tips: Cardiotonics interfere with autonomic-peripheric nervous system.
62. D
Tips: Muscarinic-M1 is a cholinergic neuroreceptor, not neurotransmitter.
63. E
Tips: Dopamine, epinephrine and norepinephrine are catecholamines neurotransmitters due the
presence of a catecholamine group on their molecular structure.

Copyright © 2000-2014 TIPS Inc. Unauthorized reproduction of this manual is prohibited. This manual is being used
during review sessions conducted by PharmacyPrep. 29-24
PharmacyPREP.Com Medicinal Chemistry of autonomic nervous system
64. B
Tips: Tyrosine is the precursor of dopamine.
65. B
Tips: Norepinephrine is converted to epinephrine through mediation of phenyl-ethanol-amine-
methyl transferase enzyme by a reaction called methylation.
66. A
Tips: Serotonin is 5HT-5-Hydroxyl-Tryptamine permeable to Na and K ions with an indole group on its
molecular structure.
67. D
Tips: Glycine is the only amino acid neurotransmitter.
68. C
Tips: Cholinergic neuroreceptors include only muscarinic and nicotinic receptors.
69. B
Tips: D2 dopamine neuroreceptor is the one responsible for the control of extra-pyramidal
syndrome.
70. E
Tips: Pentazocine and buthorphanol are mixed agonist and antagonist agents therefore cannot be
considered pure as naltrexone and naloxone.
71. D
Tips: Naltrexone is given orally in the treatment of alcoholism.
72. A
Tips: Meperidine is the opioid analgesic of choice administrated during labor due to its less liability to
induce neonate respiratory depression.
73. B
Tips: Classic opioid can prompt the release of histamine causing intense pruritus and vasodilatation.
The best example for these symptoms is morphine.
74. D
Tips: Barbiturates use glycine and GABA neurotransmitters. Benzodiazepinic neurotransmitters are
the use exclusive of benzodiazepinic drugs.
75. C
Tips: Primidone is a barbiturate prodrug that when biotransformed in the body become
Phenobarbital, the longest acting barbiturate.

Copyright © 2000-2014 TIPS Inc. Unauthorized reproduction of this manual is prohibited. This manual is being used
during review sessions conducted by PharmacyPrep. 29-25
PharmacyPREP.Com Medicinal Chemistry of autonomic nervous system
76. E
Tips: Phenelzine is a hydralazine monoamine oxidase inhibitor used to treat depression.
77. A
Tips: Desipramine is the least sedating tricyclic antidepressant while amitriptiline is the most
sedating one.
78. C
Tips: Clomipramine enhance the action of biogenic amines by inhibiting their re-uptake. It also exert
action on serotonin system, therefore it may be used to treat maniac compulsive disorders.
79. E
Tips: Main side effects of lithium therapy include: Loss of appetite, nausea, mild diarrhea, dizziness,
tremors, excessive urination, thirst and diabetes insipidus.
80. D
Tips: Antipsychotic drugs act by blocking dopamine receptors; therefore they cannot be used to treat
Parkinson’s disease that is already characterized by decrease in dopamine levels.
81. D
Tips: General anesthetics induce a combined state of analgesia, amnesia, loss of consciousness,
inhibition of sensory and autonomic reflex and skeletal muscle relaxant.
82. E
Tips: Local anesthetics reversible block nerve impulse conduction producing reversible loss of
sensation only at administration site, therefore they do not produce loss of consciousness.
83. B
Tips: Halothane is a volatile liquid used in inhaled anesthesia and may cause malignant
hyperthermia.
84. A
Tips: Halothane is the most recommended general anesthetic in the anesthesia induction of children.
85. E
Tips: Endorphins are opioids endogenous neurotransmitters.
86. C

Copyright © 2000-2014 TIPS Inc. Unauthorized reproduction of this manual is prohibited. This manual is being used
during review sessions conducted by PharmacyPrep. 29-26
PharmacyPREP.Com Medicinal Chemistry of autonomic nervous system
Tips: Selegeline is the only MAO-B used in the treatment of Parkinson’s disease. It blocks the central
catabolism of dopamine; it is indicated in the early stage of Parkinson’s and in adjunct of levodopa
treatment.
87. A
Tips: An agonist drug is a compound that binds to a receptor and produces the biological response.
88. C
Tips: The lethal dose is the dose that kills 50% of the animals that receive it. The lethal dose is mainly
compared with the effective dose to determine the therapeutic window of a drug. Effective
methodology in determining both the range of effective drug concentration and the safety of the
drug.
89. C
Comments: Decrease local side effect, and systemic absorption
90. A
Tips: All of the parasympathetic postganglionic fibers release acetylcholine. At the target organ
acetylcholine interacts with muscarinic receptors.
91. B
Tips: All preganglionic neurons release acetylcholine as their transmitter. The acetylcholine binds to
nicotinic receptors on the postganglionic cell.
92. D
Tips: Most of the sympathetic postganglionic fibers release norepinephrine. At the target organ
norepinephrine interacts with a variety of receptors.
93. B
Tips: Activation of alpha-receptors causes contraction or constriction, mostly vasoconstriction.
94. C
Tips: Spinal cord is a part of central nervous system.
95. D
Tips: The autonomic nervous system is responsible for maintaining the internal environment of the
body called “homeostasis”.
96. C
Tips: Acetylcholine is synthesized from acetyl coenzyme A (acetyl CoA) and choline. It is action is
terminated by acetylcholinesterase.

Copyright © 2000-2014 TIPS Inc. Unauthorized reproduction of this manual is prohibited. This manual is being used
during review sessions conducted by PharmacyPrep. 29-27
PharmacyPREP.Com Medicinal Chemistry of autonomic nervous system
97. E
Tips: Nicotinic receptors are found at the motor end plate, in all autonomic ganglia, and in the
adrenal medulla.
98. A
Tips: Atropine is the prototype of parasympathetic anticholinergic group of drugs.
99. A
Tips: Botulism toxins prevent and/or completely block acetylcholine release.
100. C
Tips: Dantrolene may be used in the treatment of malignant hyperthermia, a side effect associated
with the use of general anesthetics.
101. B
Tips: Buspirone is an atypical anxiolitic agent that binds to dopamine and serotonin. It is very well
preferred when compared with other anxiolytic agents due to its least side, non-drowsiness, no
addiction and no CNS depressant activity.
102. D
Tips: tiotropium is a muscarinic blockers and cause mydriasis. However other answers Ach agonist.
103. D
104. C
105. A
106. D
107. C
108.

Copyright © 2000-2014 TIPS Inc. Unauthorized reproduction of this manual is prohibited. This manual is being used
during review sessions conducted by PharmacyPrep. 29-28
PharmacyPREP.Com Medicinal Chemistry of autonomic nervous system
BIBLIOGRAPHIC REFERENCE
1- COMPREHENSIVE PHARMACY REVIEW – Lippincott William & Wilkins – Fourth edition
2- CPS-COMPENDIUM OF PHARMACEUTICALS AND SPECIALITIES - Canadian Pharmacist Association –
2001 edition.
th
3- MEDICAL DICTIONARY – Dorland’s illustrated – 27 edition.
4- PHARMACY PREP – Lectures series & study guide for Evaluating Examination-TIPS - 2003/2004
5- THERAPEUTIC CHOICES – Canadian Pharmacist Association -Third edition
th
6- USP DI – Drug Information for the Health Care Professional–15 edition – Volume I.

Copyright © 2000-2014 TIPS Inc. Unauthorized reproduction of this manual is prohibited. This manual is being used
during review sessions conducted by PharmacyPrep. 29-29
PharmacyPREP.Com Medicinal Chemistry of autonomic nervous system
Copyright © 2000-2014 TIPS Inc. Unauthorized reproduction of this manual is prohibited. This manual is being used
during review sessions conducted by PharmacyPrep. 29-30
www.pharmacyprep.com

PHARMACY PREP.
MEDICINAL CHEMISTRY AND PHARMACOLOGY OF CARDIOVASCULAR SYSTEM
1. Diuretic characterized for the ↓ Na, K, Cl and Ca.
A. Acetazolamide
B. Spironolactone
C. Hydrochlorothiazide
D. Furosemide
E. Triamterene
2. Thrombolytics (fibrinolytic) agents are contraindicated in
I-Myocardial infarction
II-Hypertensive patients
III-Bleeding from the GIT
A. I only B. III only C. I and II only D. II and III only E. All are correct
3. Patient with renal failure and normal liver function is showing sensibility to the drug, this means
that
A. High accumulation of metabolite
B. High concentration of drug protein binding
C. High volume of distribution
D. High concentration of drug in the systemic circulation
E. High accumulation of drugs into tissues
4. Normal osmotic blood pressure in a health adult:
A. 280mOsm/L
B. 280Osm/L
C. 280mOsm/ml
D. 280Osm/ml
E. 280mOsm/100ml
5. Which increase water permeability at collecting tubules?
A. ADH
B. Hydroclorothiazide
C. Aldosterone
D. Triametrene
E. Spirolactone
6. Digoxin is cardiac digitalis widely used in CV which toxicity is increased by
I- Erythromycin
II- Quinidine
III- Cholestyramine
A. I only B. III only C. I and II only D. II and III only E. All are correct

Copyright © 2000-2014 TIPS Inc. Unauthorized reproduction of this manual is prohibited. This manual is being used
during review sessions conducted by PharmacyPrep. 31-1
www.pharmacyprep.com
7. Acetazolamide, a carbonic anhydrase diuretic and hydroclorothiazide, a thiazide diuretic have an
identical molecular group very important in their structure knows as:
A. Sulfonamide group
B. Pyrimidine group
C. Purine group
D. Pyrrolidine group
E. Ether group
8) A man with blood group AB can receive blood from:
A) A
B. B
C. AB
D. O
E. All the above
9. When would will advise patient that he may suffer from hypertension and he may need to see a
doctor?
I- Doctor blood pressure monitor shows BP180/110
II- The pharmacy blood pressure monitor shows BP180/110
III- His monitor shows BP 120/80
A. I only B. III only C. I and II only D. II and III only E. All are correct
10. During absolute refractory period the myocardial fiber:
A. Receive the strongest stimulation
B. It is relaxed but is able to respond to any stimulus
C. Respond to the most sensible stimulus
D. Do not respond to any stimulus
E. None of the above
11. Abnormal persistent dilatation of blood vessels is known as:
A. Embolism
B. Aneurism
C. Emphysema
D. Starvation
E. Atherosclerosis
12. Correct statements regarding embolism include:
I- Abnormal dilatation of blood vessel
II- Obstruction of a blood vessel by a mass transported in the circulation
III- An embolus can be a clot or a foreign material transported in the circulation
A. I only B. III only C. I and II only D. II and III only E. All are correct

Copyright © 2000-2014 TIPS Inc. Unauthorized reproduction of this manual is prohibited. This manual is being used
during review sessions conducted by PharmacyPrep. 31-2
www.pharmacyprep.com
13. Which of the following statements regarding HEPARIN is/are right
I- It is a mucopolysacaride, organic acid present naturally in many tissues especially in lungs and liver.
II- It has anticoagulant proprieties that inhibit the conversion of prothrombim to thrombin and
fibrinogen to fibrin
III- It has antithrombin III as it co-factor
A. I only B. III only C. I and II only D. II and III only E. All are correct
14. Heparin and warfarin are very popular anticoagulant agents. What is wrong regarding heparin
and warfarin?
A. Both have anticoagulant proprieties
B. Warfarin has activity only in vivo
C. Heparin has activity only in vivo
D. Heparin has activity in vivo and in vitro
E. All are correct
15. Which of the following calcium channel blockers cause bradycardia
I- Nifedipine
II- Verapamil
III- Diltiazem
A. I only B. III only C. I and II only D. II and III only E. All are correct
16. Important characteristics regarding the blood formed elements may include:
I- Red Blood Cells (RBC) is originated from erythroblast and has a life span of 120 days (4 months)
II- Platelets are involved in blood clot and have a life span is 7 to 10 days
III- Platelets are originated from megakariocyts
A. I only B. III only C. I and II only D. II and III only E. All are correct
17. Warfarin, is an anticoagulant agent has its effect is potentiates by:
I- Deficiency of Vitamin K
II- Adjunct therapy with heparin
III- Adjunct therapy with salicylates
A. I only B. III only C. I and II only D. II and III only E. All are correct
18. All are drugs that decrease Low Density Lipoproteins, EXCEPT:
A. Clofibrate
B. Gemfibrozil
C. Cimetidine
D. Simvastatin
E. Pravastatin

Copyright © 2000-2014 TIPS Inc. Unauthorized reproduction of this manual is prohibited. This manual is being used
during review sessions conducted by PharmacyPrep. 31-3
www.pharmacyprep.com
19. A patient with deep vein thrombosis (DVT) can be treated by:
I- Streptokinase
II- Heparin
III- Warfarin
A. I only B. III only C. I and II only D. II and III only E. All are correct
20. α-blockers are mainly used in CV complications. One of the biggest cautions in using α-blockers is
the development of:
A. Tachycardia
B. Bradycardia
C. Orthostatic hypotension
D. Hypotension
E. Increase in cardiac output
21. All of the following are ACE-Angiotensin Covertase Inhibitor, EXCEPT
A. Enalapril
B. Captopril
C. Ramipril
D. Lisinopril
E. Losartan
22. During inspiration the lowest blood pressure in supine position is:
A. Vena cava
B. Internal carotid artery
C. Aorta artery
D. Sciatic artery
E. Pulmonary artery
23. Which of the following theories relates the tension, radius of vessel and intraluminal pressure
achieved in a blood vessel wall?
A. Fick’s law
B. Arrhenius
C. LaPlace’s law
D. Newtonian law
E. Henderson law
24. The first location where blood is present in case of left ventricular heart failure:
A. Liver
B. Lung
C. Heart
D. Kidney
E. Brain

Copyright © 2000-2014 TIPS Inc. Unauthorized reproduction of this manual is prohibited. This manual is being used
during review sessions conducted by PharmacyPrep. 31-4
www.pharmacyprep.com
25. The blood weights 7 to 8% of the total body weight of a health adult. How much of these
percentages are represented approximately in liter of blood?
A. 4 to 6L
B. 7 to10L
C. 7 to 8L
D. 10L
E. 15L
26. Nitrates are mainly used in angina treatment. True statement regarding nitrates may include:
I- Nitrates are esters of nitric acid.
II- Nitrates are prodrugs and must be denitrated to exert their therapeutic effect
III- Nitrates relaxes vascular smooth muscle and in addition relax bronchial, biliary, gastrointestinal,
urethral and uterine smooth muscles.
A. I only B. III only C. I and II only D. II and III only E. All are correct
27. Which of the following diuretics is used to block Na/H exchange system of the renal tubule:
A. Indapamide
B. Ouabain
C. Spirolactone
D. Acetazalamide
E. Amiloride
28. Example of diuretic that increases the levels of Cl, causing hypercloremia is:
A. Acetazalamide
B. Amiloride
C. Furosemide
D. Ouabain
E. Hydroclorotiazide
29. Dobutamine is a direct acting inotropic agent parenteraly used in cardiac emergences and
characterized by:
I- Increase myocardial contractility, very useful in myocardial infarction.
II- Increase cardiac output
III- Increase glomerular filtration rate, renal blood flow and sodium excretion
A. I only B. III only C. I and II only D. II and III only E. All are correct
30) Spironolactone is an aldosterone antagonist diuretic that competes with aldosterone at:
A. Proximal tubule
B. Early distal tubule
C) Early collecting tubule
D. Collecting tubule

Copyright © 2000-2014 TIPS Inc. Unauthorized reproduction of this manual is prohibited. This manual is being used
during review sessions conducted by PharmacyPrep. 31-5
www.pharmacyprep.com
E. End of collecting tubule
31. Examples of drugs inhibitor of platelet aggregation may include:
I- Ticlopidine
II- Clopidrogel
III- Epoprosterenol
A. I only B. III only C. I and II only D. II and III only E. All are correct
32. What kind of arrhythmia can be caused by an increase in temperature?
A. Ventricular premature depolarization
B. Atrial fibrillation
C. Paroxysmal arrhythmia
D. Ventricular tachycardia
E. None of the above
33. Prazosin is a sympathetic agent used in hypertension acting as:
A. β-adrenergic
B. α-adrenergic
C. β-adrenergic blocker
D. α-adrenergic blocker
E. α and β-adrenergic blocker
34. All are side effects of hydroclorothiazide, a thiazide diuretic, EXCEPT:
A. Hypokalemia
B. Hypoglycemia
C. Hyponatremia
D. Hypercalcemia
E. Hyperuricemia
35. CHF-Congestive Heart Failure is a cardiac disorder:
I- Resulted from any cardiac disorder that impairs the ability of the ventricle to deliver adequately
quantity of blood to the metabolizing tissues
II- Result in poor pump function
III- β-blockers are the BEST treatment for CHF.
A. I only B. III only C. I and II only D. II and III only E. All are correct
36. Drugs that may affect the normal activity of warfarin include:
A. Ibuprofen
B. Aspirin
C. Vitamin K
D. Acetaminophen
E. All are correct

Copyright © 2000-2014 TIPS Inc. Unauthorized reproduction of this manual is prohibited. This manual is being used
during review sessions conducted by PharmacyPrep. 31-6
www.pharmacyprep.com
37. Glycosides are widely used in the treatment of CHF, glycosides molecular structure consist of
steroidal nucleon and:
A. Sugars
B. Polypeptides
C. Carbohydrates
D. Proteins
E. Hormones
38. Stenosis:
A. Dilatations of blood vessels
B. Increase in cardiac output
C. Accumulation of cholesterol in blood vessels
D. Narrowing of blood vessels
E. High pressure in blood vessels
39. The first tissue that receive the greatest cardiac output at rest:
A. Lung
B. Liver
C. Kidney
D. Muscle
E. Fat
40. Calcium channel blockers contraindicated in patient with heart blockade because it may cause
bradycardia and heart blockade
I- Diltiazem
II- Verapamil
III- Nifedipine
A. I only B. III only C. I and II only D. II and III only E. All are correct
41. Bradycardia and orthostatic hypotension are characteristic side effects of:
I- Guanabenz
II- Hydralazine
III- Vasodilators
A. I only B. III only C. I and II only D. II and III only E. All are correct
42) An important advantage of using dobutamine in cardiac shock may include:
A) It will not cross blood brain barrier and cause CNS effect
B) It has no effect on α and β-receptors
C) It produces dose-dependent increasing cardiac output and renal perfusion
D) It will not increase blood pressure
E) It can be given orally

Copyright © 2000-2014 TIPS Inc. Unauthorized reproduction of this manual is prohibited. This manual is being used
during review sessions conducted by PharmacyPrep. 31-7
www.pharmacyprep.com
43. Which of the following diuretics may show a hyperurecemia?
A. Spironolactone
B. Furosemide
C. Chlortalidone
D. Hydrochlorthiazide
E. Chlorothiazide
44. Ancrod, an antithrombolitic agent is characterized by:
I- It is derivated from snake venom
II- It depletes fibrinogen
III- It is an alternative to heparin
A. I only B. III only C. I and II only D. II and III only E. All are correct
45. The use of sildenafil together with atenolol may result in:
I- Tremor
II- Hypotension
III- Visual disturbance
A. I only B. III only C. I and II only D. II and III only E. All are correct
46. A diabetic patient suffering from hypertension is having enalapril, however he is complaining
from severe dry cough that we can associated as a side effect of the enalapril. Which would be the
best choice for changing this medication?
A. Ramipril
B. Losartan
C. Amlodipine
D. Indapamide
E. Captopril
47. Drug of choice when treating hypertension in a diabetic patient may include:
A. ACE inhibitors
B. β-Blockers
C. Calcium Channel Blockers
D. Digitalis
E. Nitrates
48) Angina, a cardiovascular complication characterized by the oxygen insufficiency may be
aggravated by all of the following, EXCEPT:
A. Exercise
B. Hot places
C. Cold places
D. Anger
E. Over weight

Copyright © 2000-2014 TIPS Inc. Unauthorized reproduction of this manual is prohibited. This manual is being used
during review sessions conducted by PharmacyPrep. 31-8
www.pharmacyprep.com
49) Drugs used in the treatment of hypertension associated with renal failure may include:
I) Hydralazine
II) Furosemide
III) Atenolol
A. I only B. III only C. I and II only D. II and III only E. All are correct
50) Digitalis is NOT used in the treatment of:
A. Myocardial infarction
B. Atrial fibrillation
C. Congestive Heart Failure (CHF)
D. Arrhythmias
E. Venous congestion
51. Systemic lupus or erythromatus lupus may be a side effect of:
A. Propanolol
B. Digoxin
C. Cyclophosphamide
D. Hydralazine
E. Hydroxazine
52. Which is the best describe the proprieties of labetolol:
A. β-blocker
B. α-blocker
C. β1 and α-blocker
D. β non selective and α-blocker
E. β2-blocker
53. Correct statements regarding blood pressure may include:
I- Systolic pressure is the highest arterial pressure during a cardiac cycle.
II- Diastolic pressure is the lowest arterial pressure during a cardiac cycle
III- Diastolic pressure is crucial in verifying if a person suffers of high blood pressure
A. I only B. III only C. I and II only D. II and III only E. All are correct
54. Cholesterol:
I- VLDL-Very Low Density Lipoproteins are formed in the liver
II- LDL-Low Density Lipoproteins are formed from VLDL and carry LDL cholesterol that is the “bad
cholesterol”
III- HDL-High Density Lipoproteins are formed in the liver and small intestine. They carry HDL
cholesterol that is the “good cholesterol”
A. I only
B. III only
C. I and II only

Copyright © 2000-2014 TIPS Inc. Unauthorized reproduction of this manual is prohibited. This manual is being used
during review sessions conducted by PharmacyPrep. 31-9
www.pharmacyprep.com
D. II and III only E. All are correct
55. Abnormal levels of lipids can cause all of the following, EXCEPT:
A. Atheroscherosis
B. Heart attack
C. Pancreatitis
D. Bone dystrophy
E. Stroke
56. Lipid lowering group of drugs include all of the following, EXCEPT:
A. Bile acid binders
B. Fibric acid derivatives
C. Angiotensin inhibitors
D. Lipoprotein synthesis inhibitors
E. HMG-CoA reductase inhibitors
57. Niacin mechanism of action as lipoprotein synthesis inhibitor is characterized by:
A. Decrease the production rate of VLDL that synthesizes LDL
B. Block the synthesis of cholesterol
C. Decrease lipoproteins production by increasing the breakdown of lipids
D. Remove VLDL from bloodstream
E. Binds to bile acids in the intestine
58. Lipid lowering drug that binds to bile acids in the intestine causing the acids to be excreted
include:
A. Clofibrate
B. Cholestyramine
C. Niacin
D. Genfibrozil
E. Atorvastatin
59.The safest lipid lowering drug that may be used in pregnancy include:
A. Pravastatin
B. Genfibrozil
C. Cholestyramine
D. Sinvastatin
E. Clofibrate
60. Absorbable agents successfully used in patients with high cholesterol includes.
I) Grapefruit juice
II) Nicotinic acid
III) Fish oil
A. I only
B. III only

Copyright © 2000-2014 TIPS Inc. Unauthorized reproduction of this manual is prohibited. This manual is being used
during review sessions conducted by PharmacyPrep. 31-10
www.pharmacyprep.com
C. I and II only
D. II and III only
E) I,II,III
61. Digoxin may be used in dysrhythmias, however CANNOT be used in one specific type of
dysrhythmias that is:
A. Atrial tachycardia
B. Ventricular tachycardia
C. Paroxysmal tachycardia
D. Atrial fibrillation
E. Flutter
62. Class I sodium channel blocker antidysrhythmic agents include all of the following, EXCEPT:
A. Quinidine
B. Lidocaine
C. Phenytoin
D. Propafenone
E. Amiodarone
63. Correct statements regarding antidysrhythmic agents may include:
I- β-blockers, Na, K and Ca channel blockers are used in treat dysrhythmias
II- They are classified according to their ability to alter the action potential of cardiac cells
III- Reduce abnormalities of impulse generation and modify the disturbance of impulse conduction
within cardiac tissue
A. I only B. III only C. I and II only D. II and III only E. All are correct
64. Diuretic that act in the loop of henle include:
A. Acetazolamide
B. Hydroclorothiazide
C. Furosemide
D. Spirolactone
E. Chlortalidone
65. All the following diuretics may cause metabolic acidosis, EXCEPT:
A. Hydroclorothiazide
B. Acetozalamide
C. Spirolactone
D. Amiloride
E. Triametrene
66. Diuretic with pteridine and pyrazine hererocycles rings used in the treatment of nephrogenic
diabetes insipidus caused by lithium:

Copyright © 2000-2014 TIPS Inc. Unauthorized reproduction of this manual is prohibited. This manual is being used
during review sessions conducted by PharmacyPrep. 31-11
www.pharmacyprep.com
A. Spironolactone
B. Amiloride
C. Indapamide
D. Furosemide
E. Acetazolamide
67. Best diuretic used in the adjunct treatment of hepatic cirrhosis and renal diseases include:
A. Spironolactone
B. Amiloride
C. Indapamide
D. Furosemide
E. Acetazolamide
68. Agent with a sulphonamide group on its structure responsible for its functional action and that is
considered the first line treatment for mild hypertension?
A. β-Blockers
B. α-Blockers
C. Thiazides diuretics
D. Vasodilators
E. ACE inhibitors
69. Antihypertensive drug most likely to cause syncope effect or first-dose effect:
A. Prazosin
B. Clonidine
C. Minoxidil
D. Guanabenz
E. Labetolol
70. Antihypertensive α-blocker used in BPH-Benign Prostate Hypertrophy: Except
A. Prazosin
B. Terazosin
C. Doxazosin
D. Finasteride
E. Tamsulosin
71. Drug used in the treatment of hypertension that may cause hemolytic anemia.
A. Clonidine
B. Guanabenz
C. Thiazides
D. Atenolol
E. Methyldopa
72. One of the safest antihypertensive drugs used in pregnancy is:

Copyright © 2000-2014 TIPS Inc. Unauthorized reproduction of this manual is prohibited. This manual is being used
during review sessions conducted by PharmacyPrep. 31-12
www.pharmacyprep.com
A. Metoprolol
B. Captopril
C. Sodium nitroprusside
D. Methyldopa
E. Prazosin
73. Direct vasodilator drug used in the treatment of baldness may include:
A. Hydralazine
B. Sodium nitroprusside
C. Minoxidil
D. Enalapril
E. Captopril
74. All of the following may be examples of calcium channel blocker, EXCEPT:
A. Enalapril
B. Amlodipine
C. Nifedipine
D. Diltiazen
E. Verapamil
75. First line treatment in mild to moderate CFH-Congestive Heart failure include:
A. β-Blockers and diuretics
B. Glycosides and ACE inhibitors
C. Calcium channel blockers
D. Vasodilators
E. Nitrates
76. Drug used in cardiovascular complications which metabolism produce cyanide:
A. Hydralazine
B. Nitrates
C. Nitroglycerine
D. Isosorbide
E. Nitroprusside
77. Nitrates used in cardiovascular complications mainly in CHF induce the formation of:
A. Nitric oxide
B. Cyanide
C. Zinc oxide
D. Nicotinic acid
E. Nalidixic acid
78. Angina is a type of:
A. Congestive heart failure disease
B. Hypertension disease

Copyright © 2000-2014 TIPS Inc. Unauthorized reproduction of this manual is prohibited. This manual is being used
during review sessions conducted by PharmacyPrep. 31-13
www.pharmacyprep.com
C. Ischemic disease
D. Atherosclerosis disease
E. Dysrhytmiac disease
79. Examples of common used thrombolytic or fibrinolytics agents may include:
I- Streptokinase
II- Recombinant tissue plasminogen activators (tPA)
III- Warfarin
A. I only B. III only C. I and II only D. II and III only E. All are correct
80. All are examples of low molecular weigh heparins (LMWH), EXCEPT:
A. Ardeparin
B. Enoxaparin
C. Dalteparin
D. Tinzaparin
E. Fondaprinaux
81. Which of the following β-agonist drugs has no effect in α-agonists receptors?
A. Isoproterenol
B. Dopamine
C. Norepinephrine
D. Epinephrine
E. Clonidine
82. Which of the following antihypertensive drugs is considered a prodrug:
I- Minoxidil
II- Methyldopa
III- Clonidine
A. I only B. III only C. I and II only D. II and III only E. All are correct
83. Predominant sympathetic adrenergic receptor in the heart may include:
A. α-receptors
B. β-receptors
C. Mu receptors
D. M1-receptors
E. D3-receptors
84. What is appropriate action should be advised to the patient if his blood pressure is over 180/120
A. Ask patient to relax
B. Refer to Dr
C. Ask patient seek medical attention within a week
D. Offer him antihypertensive drugs
E. contact his family member to inform about his blood pressure

Copyright © 2000-2014 TIPS Inc. Unauthorized reproduction of this manual is prohibited. This manual is being used
during review sessions conducted by PharmacyPrep. 31-14
www.pharmacyprep.com
85. Use of nitroglycerine with sildenafil will cause:
A.Hypotension
B.hypertension
C.angina
D.CHF
E.stroke
86. Thrombolytic therapy is used in all except?
A.STEMI
B. DVT
C. PE
D. Acute MI
E. Stable Angina
87. Phase 0 is?
A. Repolarization
B. Depolarization
C.Prolong repolarization
D. Plateau
E. None of the above
88. All of the following are therapeutic use of antiplatelet drugs, except?
A. STEMI
B. NSTEMI
C. Prophylaxis of angina
D. Stroke
E. Hypertension
89. Which of the following drug prolong the life of person in congestive heart failure?
A. Metoprolol
B. Ramipril
C. Digoxin
D. Furosemide
E. Potassium supplements
90. What class of drug represents the following chemical structure?
HS - H2CONCH3CO2H

Copyright © 2000-2014 TIPS Inc. Unauthorized reproduction of this manual is prohibited. This manual is being used
during review sessions conducted by PharmacyPrep. 31-15
www.pharmacyprep.com
A) Diuretics
B) Angiotensin converting enzyme (ACE) Inhibitors
C) Calcium channel blockers
D) Selective Serotonin Receptor Inhibitors
E) Angiotensin Receptor Blockers (ARBs)
91) What is incorrect about ACE Inhibitors?
A) sulfhydril group is present in captopril
B) captopril and fosinopril are NOT prodrugs because these do not require hydrolysis to produce
active products
C) it essential to have esterase for the activation of ACE Inh.
D) ACE Inh. prevent formation of angiotensin II,
E) Angiotensin II is strong vasodilator
92) PK is a 55 yo man is using ramipril 10 mg daily to treat hypertension, recently reports cough.
What is incorrect?
A) recommend to see doctor to change to ARBs
B) recommend cough suppressant dextromethorphan
C) ARBs have no cough SEs
D) cough is induced by bradykinins, in ACEi
E) CCBs have no cough side effects
93) What is active pharmacophore in Angiotensin receptor blockers?
NNNNHNNClOHTetrazole ringLosartanImidazole ring (essential)
A) Tetrahydropyrazole ring
B) On position 2 ethyl alcohol on imidazole ring
C) Two ring structure
D) Alkyl long chain on imidazole ring
E) AT1 receptors
94) HMG Co A reductase inhibitor like statins effective lowers LDL. What is incorrect?
A) 3,5-dihydroxycarboxylate is essential for SARs
B) Myopathy SE is the associated with statins. This is monitored by CK-MM
C) Initial SEs include stomach upset like diarrhea
D)Statins increase liver enzymes, so monitor LFTs

Copyright © 2000-2014 TIPS Inc. Unauthorized reproduction of this manual is prohibited. This manual is being used
during review sessions conducted by PharmacyPrep. 31-16
www.pharmacyprep.com
E) Statins combined with niacin decrease the risk of myopathy
95) SK is a 55 year old male patient has been prescribed amlodipine 10 mg daily. After 2 weeks of
initiating this patient reports fluid retention of feet, head ache, flushing, dry mouth, and palpitation.
Pharmacist suspect these are side effect of amlodipine. Which of the following is not side effects of
CCBs?
A) ankle edema
B) flushing
C) headache
D) palpitation
E) dry mouth
96) Which of the following drugs represent the structure below?
A. Leukotriene antagonists
B. Beta blockers
C. Coumarins
D. Calcium channel blockers
E. Benzodiazepines
97) Angina precipitated by exercise or emotional stress, drug of choice:
A. Propranolol
B. Nitroglycerine SL
C. Diltiazem
D. Nifedipine
98) Clopidogrel is classified as an:
A) Anti-hypertensive. B. Inhibitor of platelets' aggregation. C. Anti-diabetic. D. Anti-Parkinson’s.

Copyright © 2000-2014 TIPS Inc. Unauthorized reproduction of this manual is prohibited. This manual is being used
during review sessions conducted by PharmacyPrep. 31-17
www.pharmacyprep.com
CORRECT ANSWERSCARDIOVASCULAR SYSTEM
1. D
Tips: Furosemide diuretic may cause hyponatremia, hypokalemia, hypocalcemia, and
hypomagnesaemia. It also causes hyperglycemia, hyperuricemia and can increase levels of
triglicerideos.
2. B
Tips: Antithrombolitic agents are completely contra indicated in patients suffering from any internal
bleeding.
3. A
Tips: The liver with normal function can completely metabolize a drug, however a kidney with poor
function-renal failure cannot properly eliminate the drug from the body leading to accumulation of
metabolites.
4. A
Comments: The osmotic blood pressure may vary from 280 to 300mmol/Kg or mOsm/Kg
5. A
Tips: ADH-Anti-Diuretic Hormones increase water reabsorption by increasing the cellular
permeability of the collecting ducts. Resulting in a decrease in urine volume with resultant increase
in osmolarity.
6. C
Tips: Both quinidine and erythromycin have drug interaction with digoxin which induce toxic effects
of digoxin.
7. A
Tips: Both hydroclorothiazide and acetozalamide have a heterocyclic sulphonamide group on their
molecular structure responsible for their functional action.
8. E
Tips: A person with blood AB is considered the “universal acceptor” because can accept blood
donation from any other group, A, B, AB and O.
9. C
Tips: The normal blood pressure in a health adult is 80/120. Any big difference between these valors,
mostly in the diastolic pressure the patient should be indicated for medical assistance.
10. D

Copyright © 2000-2014 TIPS Inc. Unauthorized reproduction of this manual is prohibited. This manual is being used
during review sessions conducted by PharmacyPrep. 31-18
www.pharmacyprep.com
Tips: During absolute refractory period the myocardial fiber do not respond to any stimulus.
11. B
Tips: Aneurysm is an abnormal dilatation of a blood vessel while embolism is an obstruction of
airway by a mass transported in the circulation. Emphesema is a reversible form of airway
obstruction.
12. D
Tips: Embolism is an obstruction of a blood vessel by a mass transported in the circulation. The
embolus is this mass transported in the circulation that can be a clot or a foreign material. Abnormal
dilatation of a vessel artery is known as aneurysm
13. E
Tips: Heparin is amino acid polysaccharide naturally present in the body predominantly in lung and
liver tissues. It inhibits the conversion of prothrombim to thrombin and fibrinogen to fibrin using
antithrombin III as it co-factor therefore been therapeutically used as anticoagulant.
14. C
Tips: Heparin is already naturally present in the body and also act in vitro, it is very useful as
anticoagulant in blood donations conservation. Warfarin has only in vivo activity
15. D
Tips: Nifedipine is mainly the only calcium channel blocker that does not have bradicardia as side
effect.
16. E
Tips: Blood formed elements are erythrocytes, leukocytes and platelets. Erythrocytes also known as
Red Blood Cells form 2% of the whole blood, has a half-life of approximately 120 days and play
important rules in the O2 and CO2 transportation. Platelets are originated from megakariocytes and
are involved in blood clotting.
17. E
Tips: Vitamin K is considered the antidote for Warfarin. When the body has lower vitamin K the
tendency is increase the effect of Warfarin. Heparin and salicylates also act as anticoagulant; in
adjunct therapy with Warfarin they will have a synergism effect.
18. C
Tips: Cimetidine is a H2-antagonist therefore do not decrease LDL
19. D

Copyright © 2000-2014 TIPS Inc. Unauthorized reproduction of this manual is prohibited. This manual is being used
during review sessions conducted by PharmacyPrep. 31-19
www.pharmacyprep.com
Tips: Patient with “deep” vein thrombosis can be treated with both Warfarin and heparin.
Streptokinase would be mainly used in acute myocardial infarction or in other acute coronary artery
diseases.
20. C
Tips: Orthostatic hypotension is a fall in blood pressure associated with dizziness, syncope and
blurred vision occurring upon standing or when standing motionless in a fixed position, one of the
most common side effects with α-blockers therapy
21. E
Tips: Losartan is an AT-1, Angiotensin receptor blocker.
22. A
Tips: During inspiration the lowest blood pressure in supine position is in vena cava
23. C
Tips: LaPlace’s Law explains the tension achieved in a blood vessel wall to the radius of vessel and
intraluminal pressure.
24. B
Tips: Blood leaves the lung and reaches the left ventricle of the heart through pulmonary vein. In
case of left ventricular failure the tendency is accumulation of blood in the lung due to blockage of
blood circulation from lung through the heart
25. A
Tips: The blood is composed of 90% water and 8% plasma protein. A health adult has approximately
5L of blood circulating in the body.
26. E
Tips: Nitrates are esters of nitrous or nitric acid that are rendered nonexplosive by the addition of
inert excipient such as lactose. Nitrates are prodrugs and must be denitrated to exert their
therapeutic effect, denitration liberates nitric oxide, which cause vasodilatation. In addition to the
relaxation of vascular smooth muscle, these drugs relax bronchial, biliary, gastrointestinal, urethral
and uterine smooth muscles.
27. D
Tips: Acetozalamide is a carbonic anhydrase diuretic that blocks Na/H exchange system at the
proximal tubule of kidney.
28. A
Tips: A common side effect seen with long-term treatment with acetozalamide is hypercloremia and
metabolic acidosis.
29. E

Copyright © 2000-2014 TIPS Inc. Unauthorized reproduction of this manual is prohibited. This manual is being used
during review sessions conducted by PharmacyPrep. 31-20
www.pharmacyprep.com
Tips: Dobutamine is indicated to improve cardiac function during cardiac decompensation or
depressed contractility from cardiac or major vascular complications
30. C
Tips: Spirolactone is an aldosterone antagonist that binds to aldosterone receptor as competitive
antagonist at early collecting tubules of kidney.
31. E
Tips: Ticlopidine, clopidrogel and aspirin may be used as antiplatelet agents definitely affecting
platelet aggregation.
32. C
Tips: Paroxysmal arrhythmia is characterized by sudden recurrence or intensification of arrhythmias
that may be caused by an increase in temperature.
33. D
Tips: Prazosin is a α-adrenergic blocker used to treat hypertension due to its action in the arteries
and veins dilatation.
34. B
Tips: Hydroclorothiazide may have hyperglycemia as side effect.
35. C
Tips: CHF is a complex clinical syndrome that can results from any cardiac disorder that impairs the
ability of the ventricle to deliver adequate quantities of blood to the metabolizing tissues during
normal activity or at rest.
36. E
Tips: Ibuprofen inhibits the enzymatic metabolism of Warfarin. Acetozalamide and aspirin increases
Warfarin activity and vitamin K inhibits the activity of Warfarin.
37. A
Tips: Digoxin is a classic example of cardiac glycosides that consist of sugar and steroidal nucleon.
38. D
Tips: Stenosis is the narrowing of blood vessels
39. B
Tips: The first tissue that receives the greatest cardiac output at rest is liver, following by kidney and
lung.
40. C

Copyright © 2000-2014 TIPS Inc. Unauthorized reproduction of this manual is prohibited. This manual is being used
during review sessions conducted by PharmacyPrep. 31-21
www.pharmacyprep.com
Tips: Heart block is a contraindication for the nondihydropyridines. The most common side effects
are bradycardia and heart block (for the nondihydropyridines). Patient with MI using diltiazem and
verapamil should be monitored for blood pressure and heart rate and signs of clinical heart failure.
Diltiazem and verapamil side effects include Hypotension, bradycardia, heart block, heart failure,
gingival hyperplasia, and constipation.
In patients with LV dysfunction (LVEF ≤40%), IV diltiazem or verapamil should be avoided because of
their potent negative inotropic effects.
41. A
Tips: Guanabenz is a sympathomimetic α2-adrenergic agent having bradycardia and orthostatic
hypotension as main side effect.
42. C
Tips: Dobutamine improve cardiac function, it increases cardiac contraction and vasodilatation of
smooth muscles
43. D
Tips: Triamterene is a diuretic that exert its effect on the distal renal tubule to inhibit the
reabsorption of sodium in exchange for potassium and hydrogen ions. Triamterene may reduce
glomerular filtration and renal plasma flow. Via this mechanism it may reduce uric acid excretion.
44. E
Tips: Ancrod is designated to use as an antithrombolitic in patients with heparin-induced
thrombocytopenia or thrombosis, who requires immediate and continued anticoagulation
45. E
Tips: The currently use of sildenafil and β-blockers may result in exacerbated hypotension, blurred
vision and tremors.
46. B
Tips: Losartan is an AT-1 inhibitor and would be the best choice for treatment of hypertension in a
diabetic patient with dry cough that is definitely a side effect of ACE inhibitor, enalapril.
47. A
Tips: Although ACE inhibitors increase the risk of hyperkalemia in diabetes mellitus; it is still the most
appropriated agent in treating hypertension in diabetic patients because it does not interfere with
glucose or insulin metabolism.
48. B

Copyright © 2000-2014 TIPS Inc. Unauthorized reproduction of this manual is prohibited. This manual is being used
during review sessions conducted by PharmacyPrep. 31-22
www.pharmacyprep.com
Tips: Angina occurs when myocardial oxygen availability is insufficient to meet the myocardium
oxygen demand. Any condition or factor impairing the oxygen supply such as cold places and
exercises can predispose or increase the risk of angina.
49. C
Tips: Furosemide, and bumetanide are the most frequently used loop diuretics in patients with ARF.
Ethacrynic acid is typically reserved for patients who are allergic to sulfa compounds. Atenolol is a β-
blocker and may not be used in patients with renal failure. Hypernatremia and fluid retention are
frequent complications of ARF, and thus sodium restriction is a necessary intervention. In general,
patients should receive no more than 3 g of sodium per day from all sources, including intravenous
fluids, drugs, and enteral intake. Clinicians should be vigilant about sources of sodium. Excessive
sodium intake is a common reason diuretic therapy fails.
50. A
Tips: Digitalis are mainly used in the treatment of all degrees of congestive heart failure and some
kind of arrhythmias as atrial fibrillation and atrial flutter.
51. D
Tips: Systemic lupus or erythromatus lupus is a side effect of hydralazine.
52. D
Tips: Labetalol is a β-non-selective blocker and α-blocker agent used to treat hypertension. It dilates
blood vessels without increase the heart rate.
53. E
Tips: The normal blood pressure in a health adult is 80/120, where 80 is the systolic and 120 is the
diastolic pressure. Systolic pressure is the highest arterial pressure during a cardiac cycle and is
measured when the heart contracts while diastolic pressure is the lowest arterial pressure during a
cardiac cycle and is measured when the heart is relaxed.
54. E
Tips: VLDL-Very Low Density Lipoproteins are formed in the liver and it is the origin of LDL the Low
Density Lipoproteins, carry LDL cholesterol that is well known as the “bad cholesterol”, while HDL the
High Density Lipoproteins are formed in the liver and small intestine, carry HDL cholesterol and is
known as the “good cholesterol.
55. D
Tips: Abnormal levels of cholesterol or lipids there is no association with bone dystrophy that is a
condition normally called osteoporosis.
56. C

Copyright © 2000-2014 TIPS Inc. Unauthorized reproduction of this manual is prohibited. This manual is being used
during review sessions conducted by PharmacyPrep. 31-23
www.pharmacyprep.com
Tips: Angiotensin covertazing inhibitors are used to treat hypertension.
57. A
Tips: Niacin act as lipoprotein synthesis inhibitor by decreasing the production rate of VLDL, which
synthesizes LDL
58. B
Tips: Cholestyramine is an insoluble, non-absorbable agent used as bile acid binder in the intestine
causing the acids to be excreted.
59. C
Tips: Cholestyramine is the primary prevention of hyperlipidemia and can be safely used in
pregnancy.
60. D
Tips: Grapefruit juice interacts with most of the antihyperlipidemic agents.
61. B
Tips: Digoxin as antidysrhythmic agent is only used in atrial fibrillation, paroxysmal atrial tachycardia
and flutter.
62. E
Tips: Class I antidysrhythmics agents are Na channel blockers agents and do not include amiodarone
that belongs to class III antidysrhythmics agents inhibiting K channel.
63. E
Tips: Antidysrhytmic agents are used to reduce abnormalities of impulse generation and modify the
disturbance of impulse conduction within cardiac tissue, they are classifies by classes I, II, III and IV
which include β, Na, K and Ca channel blockers
64. C
Tips: Furosemide and ethacrynic acid act by inhibition of Na/K dichloride transport system. The are
the only diuretics with action on the loop of henle
65. A
Tips: Hydroclorothiazide may cause metabolic alkalosis not acidosis.
66. B
Tips: Amiloride, a potassium sparing diuretic is used for treatment of nephrogenic diabetes insipidus
caused by lithium.
67. D
Tips: Furosemide may be the best diuretic used in acute pulmonary edema, hepatic cirrhoses, renal
failure and edema associated with CHF.

Copyright © 2000-2014 TIPS Inc. Unauthorized reproduction of this manual is prohibited. This manual is being used
during review sessions conducted by PharmacyPrep. 31-24
www.pharmacyprep.com
68. C
Tips: Hydrochlorothiazide prevents the reabsorption of sodium chloride at distal tubule of kidney
and is the primarily treatment of mild hypertension.
69. A
Tips: Prazosin is a α1-adrenergic antagonist normally given in doctor’s office due to its first pass
effect and syncope effect.
70. D
Tips: All alpha blockers, like prazosin, doxozocin, Terazosin and tamsulosin is indicated in the
treatment of BPH. It has been show to improve urinary flow and symptoms of BPH. However
Finasteride 5-alpha reductase inhibitor.
71. E
Tips: Methyldopa long-term treatment may be associated with anemia. A comb’s test is
recommended every six months for patients in methyldopa therapy.
72. D
Tips: Methyldopa is a prodrug considered as one of the safest drug for treatment of hypertension
during pregnancy.
73. C
Tips: Minoxidil is a direct vasodilator used to treat hypertension and alternatively used to treat
baldness.
74. A
Tips: Enalapril is an ACE-II, Angiotensin Convertazing Enzyme.
75. B
Tips: First line treatment of CHF is normally the association of a cardiac glycoside that inhibit Na/K
(Na pump) exchange and a vasodilator as ACE inhibitors that inhibits the conversion of angiotensin I
to angiotensin II
76. E
Tips: Nitroprusside is a direct vasodilator used in cardiac emergences. It is metabolism has cyanide as
end product.
77. A
Tips: Nitrates act directly relaxing vascular smooth muscle by transformation of nitric oxide causing a
peripheral pooling of blood.
78. C

Copyright © 2000-2014 TIPS Inc. Unauthorized reproduction of this manual is prohibited. This manual is being used
during review sessions conducted by PharmacyPrep. 31-25
www.pharmacyprep.com
Tips: Angina is a kind of ischemic disease characterized by symptoms that occurs when myocardial
oxygen availability is insufficient to meet the myocardium oxygen demand.
79. C
Tips: Warfarin is an anticoagulant agent.
80. E
Tips: All are low molecular weight heparin, which are normally used in the prevention of deep vein
thrombosis and reduce the risk of pulmonary embolism.
81. A
Tips: Isoproterenol is an adrenergic β1-agonist with no effect in α-receptors.
82.C
Tips: Prodrugs are drugs that become active only after bioactivation by the body. Minoxidil and
methyldopa are examples of prodrugs.
83. B
Tips: β-adrenergic receptors are the most abundant receptor in the heart.
84. C
85. A
86. E
87. B
88. E
89. C
90. B
Tips: the above structure is for captopril, this contain sulfhydril group.
91. E
Tips: Angiotensin II is strong vasoconstrictor
92. B
93. B
94. E
95. E
96. C
97. B
Tips: nitroglycerine SL or spray
98. B
Tips: Clopidogrel (Plavix) is a blood thinning agent. Agents of this class generally act by inhibiting
aggregation of platelets which is thought to be the primary mechanism of blood clotting. It is
indicated for prevention of heart stroke in patients with CHF.

Copyright © 2000-2014 TIPS Inc. Unauthorized reproduction of this manual is prohibited. This manual is being used
during review sessions conducted by PharmacyPrep. 31-26
www.pharmacyprep.com
BIBLIOGRAPHIC REFERENCE
1- COMPREHENSIVE PHARMACY REVIEW – Lippincott William & Wilkins – Fourth edition
2- CPS-COMPENDIUM OF PHARMACEUTICALS AND SPECIALITIES - Canadian Pharmacist Association –
2001 edition.
th
3- MEDICAL DICTIONARY – Dorland’s illustrated – 27 edition.
4- PHARMACY PREP – Lectures series & study guide for Evaluating Examination-TIPS - 2003/2004
5- THERAPUTIC CHOICES – Canadian Pharmacist Association -Third edition
th
6- USP DI – Drug Information for the Health Care Professional–15 edition – Volume I.

Copyright © 2000-2014 TIPS Inc. Unauthorized reproduction of this manual is prohibited. This manual is being used
during review sessions conducted by PharmacyPrep. 31-27
PharmacyPrep.Com Medicinal Chemistry and Pharmacology of Endocrine System

PHARMACY PREP
MEDICINAL CHEMISTRY AND PHARMACOLOGY OF ENDOCRINE SYSTEM
1. Thyroid stimulating hormone (TSH) secretes from?
A. Anterior pituitary gland
B. Posterior pituitary gland
C. Thyroid gland
D. Parathyroid gland
E. None of the above
2) Aromatized ring (phenolic ring) is present in?
A. Progesterone
B. Estrogen
C. Testosterone
D. Androgen
E. Aldosterone
3) Primary adrenocorticoids insufficiency can lead to:
A) Cushing syndrome
B) Myxedema
C) Addison disease
D) Graves disease
E) Hashimoto disease
4. What is incorrect about comparison of insulin and glucagons
A. Insulin ↓blood glucose levels and glucagons ↑ blood glucose levels
B. ↑ blood glucose levels stimulates production of insulin
C. ↓ blood glucose levels stimulates production of glucagons
D. Insulin increase glucose uptake and glycogen formation
E. Insulin increase gluconeogenesis
5. In pregnancy progesterone is produced by?
I-Corpus luteum
II-Placenta
III- Ovaries
A. I only B. III only C. I and II only D. II and III only E. All are correct
6. First three months of pregnancy, what hormone is elevated?
A. Estrogen B. hCG C. Progesterone D. LH E. FSH
7. What hormones are elevated in ovulation?

33-1 Copyright © 2000-2014 TIPS Inc. Unauthorized reproduction of this manual is prohibited. This manual is
being used during review sessions conducted by PharmacyPrep.
PharmacyPrep.Com Medicinal Chemistry and Pharmacology of Endocrine System
A. Estrogen B. hCG C. Progesterone D. LH E. FSH
8. Intracellular enzyme that generate free radicals include:
A. Cysteine
B. Superoxide desmutase
C. Hydrogen peroxide
D. Hydroxylation
E. None of the above
9. Free radicals include:
I-Hydrogen peroxide
II-Benzoyl peroxide
III-Superoxide desmutase
A. I only B. III only C. I and II only D. II and III only E. All are correct
10. Calcium ion is:
A. Intracellular ion B. Extracellular ion C. Extracellular anion D. Intracellular anion
E. None of the above
11. Amenorrhea may be described as:
A. Impotence
B. Premenstrual symptoms
C. Menopause
D. Painful menstruation
E. Missing menstruation
12. Primary dysmenorrhea is caused by:
A. Decreased levels of prostaglandin
B. Increased levels of prostaglandin
C. Endometriosis
D. Improper posture
E. None of the above
13. A wasting due to lack of nutrition to a body part is?
A. Hypertrophy
B. Atrophy
C. Aplasia
D. Hyperplasia
E. Gernotopia
14. Dyspnea means:
A. Pain in muscles
B. Painful breathing

33-2 Copyright © 2000-2014 TIPS Inc. Unauthorized reproduction of this manual is prohibited. This manual is
being used during review sessions conducted by PharmacyPrep.
PharmacyPrep.Com Medicinal Chemistry and Pharmacology of Endocrine System
C. Painful heart
D. Painful lung
E. Pain in bone joints
15. Hyperthyroidism causes?
A. slow speech
B. impaired memory
C. hyper excitability
D. decreased mental capacity
E. somnolence
16. Epinephrine (adrenalin) is released from?
A. Adrenal medulla
B. Adrenal cortex
C. Brain
D. Lungs
E. Pituitary gland
17. Corticosteroids are released from?
A. Adrenal medulla
B. Adrenal cortex
C. Brain
D. Lungs
E. Pituitary gland
18. All of the following released from anterior pituitary gland, except:
A. ACTH B. LH C. ADH D. Prolactin E. TSH
19. Aldosterones are released from?
A. Kidney
B. Adrenal cortex
C. Adrenal medulla
D. Anterior pituitary gland
E. Posterior pituitary gland
20. What are the main risks associated with diabetes?
I-Cardiovascular diseases
II-Nephropathy
III-Liver necrosis
A. I only B. III only C. I and II only D. II and III only E. All are correct
21. Erythropoeitins are released from?
A. Liver B. bone marrow C. kidney D. Lungs E. Heart

33-3 Copyright © 2000-2014 TIPS Inc. Unauthorized reproduction of this manual is prohibited. This manual is
being used during review sessions conducted by PharmacyPrep.
PharmacyPrep.Com Medicinal Chemistry and Pharmacology of Endocrine System
22. All of the following hormones are released from anterior pituitary gland, except?
A. Leutinizing hormone
B. Follicular stimulating hormones
C. Antidiuretic hormone
D. Prolactin hormone
E. Thyroid stimulating hormone
23. What hormone is secreted from hypothalamus?
A. Thyroid B. Oxytocin C. ACTH D. FSH E. GH
24. What is the lab test for hypothyroidism?
A. Serum TSH
B. Increase FT4
C. Decrease TT3
D. Increase TT3
E. Decrease FT4
25. What is true in the laboratory test for hyperthyroidism?
I-serum TSH II-Increase FT4 III-Increase TT3
A. I only B. III only C. I and II only D. II and III only E. I, II, III
26. Which of the following stimulates that production of calcitonin?
A. hypocalcemia
B. Hypercalcemia
C. hypothyroidism
D. hyperthyroidism
27. A patient is on Synthroid 50 mcg daily for the treatment of hypothyroidism. Complains the
palpitation and sweating. Her serum levels were 0.02 mU/L. What is appropriate action to do?
A. Increase dose of Synthroid 100 mcg
B. decrease dose of Synthroid
C. treat palpitation and sweating
D. discontinue Synthroid as palpitation and sweating are adverse drug reactions
E. none of the above
28. A patient is insulin therapy, which of the following conditions require low dose of insulin?
A. when takes heavy meals
B. Before exercise
C. Before high carbohydrate meals
D. Before bedtime
E. After breakfast

33-4 Copyright © 2000-2014 TIPS Inc. Unauthorized reproduction of this manual is prohibited. This manual is
being used during review sessions conducted by PharmacyPrep.
PharmacyPrep.Com Medicinal Chemistry and Pharmacology of Endocrine System
29. What is the most serious side effect of oral contraceptives?
A. nausea and vomiting
B. breast tenderness
C. weight gain
D. Thrombotic events
E. acne
30. Estrogen cuases, except?
A. late bleeding
B. Mid cycle bleeding
C. brest tenderness
D. Nausea & vomiting
E. weight gain
31. Hormone replacement therapy (HRT) is controversial therapy used in post menopausal woment
to prevent osteoporosis. How long HRT is given?
A. 4 yrs
B. life time
C. 1 months
D. 1 weeks
E. None of the above
32. Steroid structures are common in hormones, such estrogen, progesterone, and testosterones.
Steroid contain how many cyclohexane and cyclopentane respectively in it skeleton.
A. 2 and 3 B. 3 and 2 C. 4 and 1 D. 3 and 1 E. 1 and 3
33. Phenolic ring is present in?
A. Progestin
B. Testosterone
C. Estrogen only
D. Finasteride
E. Estrogen and Progesterone
34. A patient using glyburide 5 mg bid, and acarbose daily to treat diabetes. Sometimes, patient
experiencing hypoglycemic symptoms. What is the appropriate suggestion?
A. take fruit juice to treat hypoglycemia
B. take sugar solution to treat hypoglycemia
C. take glucose tablets to treat hypoglycemia
D take sweet beverages to treat hypoglycemia
E. take sugar candy to treat hypoglycemia

33-5 Copyright © 2000-2014 TIPS Inc. Unauthorized reproduction of this manual is prohibited. This manual is
being used during review sessions conducted by PharmacyPrep.
PharmacyPrep.Com Medicinal Chemistry and Pharmacology of Endocrine System
35. A patient using metformin 500 mg bid and glyburide 2.5 mg to treat diabetes. However, still
blood sugar are high. What in-appropriate to counsel patient?
A. increase physical activities upto 150 min/wk
B. Increase carbohydrate diet to decrease hypoglycemia
C. Decrease sugar containing beverages
D. Decrease starch containing diet
E. All of the above
36. Which of the following is incorrect about mechanism of antithyhroid drugs such as methimazole
and propylthiouracil?
A. Prevent the uptake of I by the gland they inhibit the synthesis of T3 and T4 by inhibiting the
iodination of tyrosine in the thyroglobulin
B. Blocks the coupling of the iodo thyroxine
C. Inhibits the conversion of T4 to T3
D. Thioamide group is essential for antithyroid activity
E. None
37. An asthma patient using prednisone 5 mg daily. Which of the following least likely cause?
A. hyperglycemia
B. fungal infection
C. dermatitis
D. weight gain
E. Hypertension
38. A 45 year old man using metformin 500 mg bid, and glyburide 5 mg bid. Patient often gets
symptoms of sweating, tremors, and dizziness after taking medications. Which of the following
appropriate recommendation pharmacist should give?
A. take sugar candies to manage symptoms
B. take juice to manage symptoms
C. take medications with food to manage symptoms
D. do exercise after taking medications
E. do exercise before taking medications
39. Which of the following conditions increase risk of lactic acidosis?
I) renal diseases
II) Liver diseases
III) excessive intake of alcohol
A. I only B. III only C. I and II D. II and III E. I, II, III
40. A patient reports to emergence with diabetic keto acidosis medical condition. What incorrect
about diabetic ketoacidosis?
A. the major ketone is beta hydroxybutyric acid
B. it is mainly occurs in type 1 DM

33-6 Copyright © 2000-2014 TIPS Inc. Unauthorized reproduction of this manual is prohibited. This manual is
being used during review sessions conducted by PharmacyPrep.
PharmacyPrep.Com Medicinal Chemistry and Pharmacology of Endocrine System
C. Fruity smell is symptoms of diabetic ketoacidosis
D. diabetic ketoacidosis is NOT life threatening
E. diabetic ketoacidosis occurs due to severe hypoglycemia
41. A 46 year old female using Synthroid 75 mcg to treat hypothyroidism. What is monitored?
I) Increase serum TSH
II) Decrease FT4
III) Decrease TT3
A. I only B. III only C. I and II D. II and III E. I, II, III
42. The above same patient which of the symptoms indicates she is taking higher dose?
A. constipation
B. weight gain
C. Palpitation
D. hypertension
E. sensitivity to cold
43. What is true about corticosteroids?
I) Adrenal cortex synthesizes adrenocorticosteriods
II) A double bond between C-1 and C-2 increases only the glucocorticoid activity
III) Fluorination at position C-9 decreases glucocorticiod activity but increases mineralocorticoid
activity
A. I only B. III only C. I and II D. II and III E. I, II, III
44. Which of the drug can be given in the mild to moderate hypertension with CHF?
A) Beta blockers B) CCB C) HCTZ D) ACE I E) Nitrates
45. Antiplatelets fab fragment of human monoclonal antibody act on the following types of
glycoprotein's?
A. GP IIb/IIIa B. GP III/X C. GP IX/X D. GP XII E. None of the above
46. Which of the following drugs are contraindicated in first trimester of pregnancy?
A. antacids
B. diclectin
C. dimenhydrinate

33-7 Copyright © 2000-2014 TIPS Inc. Unauthorized reproduction of this manual is prohibited. This manual is
being used during review sessions conducted by PharmacyPrep.
PharmacyPrep.Com Medicinal Chemistry and Pharmacology of Endocrine System
D) methyl dopa
E) warfarin
47. Which of the following are NOT steroidal hormones?
A. estrogen
B. Progesterone
C. vitamin D3
D. Aldosterone
E. levothyroxine

33-8 Copyright © 2000-2014 TIPS Inc. Unauthorized reproduction of this manual is prohibited. This manual is
being used during review sessions conducted by PharmacyPrep.
PharmacyPrep.Com Medicinal Chemistry and Pharmacology of Endocrine System
ANSWERS:
1. A
Tips: Anterior pituitary hormone secretes TSH, FSH, LH, prolactin, ACTH and growth hormones.
Whereas posterior stimulated ADH and oxytocin only.
Ref: (Read table in endocrine system)
2. B
3. C
Tips: Primary adrenocorticoids deficiency cause Addison disease, where excessive production
corticoids cause Cushing syndrome. Myxedema and Hashimoto is diseases of hypothyroidism. Graves
disease is hyperthyroidism.
4. E
Tips: Insulin prevents or decrease formation of glucose from fats and proteins.
5. E
Tips: In pregnancy progesterone is produced by corpus luteum and placenta. In general progesterone
are produced by ovaries.
6. B
7. D
8. B
9. C
10. B
Tips: Calcium, Sodium and Chloride are extracellular. Potassium, magnesium and phosphate are
intracellular ions.
11. E
Tips: Amenorrhea is absence of menstruation and dysmenorrhea is painful menstruation
12. B
Tips: Increased levels of prostaglandin’s gives menstrual pain, which is also referred as
dysmenorrhea
13. B
Tips: wasting is loss of function and tissue death.

33-9 Copyright © 2000-2014 TIPS Inc. Unauthorized reproduction of this manual is prohibited. This manual is
being used during review sessions conducted by PharmacyPrep.
PharmacyPrep.Com Medicinal Chemistry and Pharmacology of Endocrine System
14. B
15. C
16. A
Tips: epinephrine and norepinephrine is secreted from adrenal medulla
17. B
Tips: middle of layer adrenal cortex zona fasciculate secrete corticosteroids
18. C
Tips: Antidiuretic hormone (ADH) and oxytosin is secreted from posterior pituitary hormone.
19. B
Tips: aldosterone are produced from outer layer of cortex. Aldosterone, cortisol, androgen, where as
adrenal medulla secretes, epinephrine and norepinephrine.
20. C
Tips: complications of diabetes in include nephropathy, retinopathy, neuropathy, cardiopathy. and GI
conditions.
21. C
Tips: Erythropoeitins are produced from kidney, and helps in formation of red blood cells synthesis.
22. C
23. B
24. A
Tips: serum TSH increase more than 5 mU/L in hypothyroidism.
25. D
26. B
27. B
Tips: Serum TSH 0.02 mU/L indicate hyperthyroidism and palpitation is indicator of hyperthyroidism.
So this patient may need less dose of Synthroid.
28. B
29. A

33-10 Copyright © 2000-2014 TIPS Inc. Unauthorized reproduction of this manual is prohibited. This manual is
being used during review sessions conducted by PharmacyPrep.
PharmacyPrep.Com Medicinal Chemistry and Pharmacology of Endocrine System
30. B
31. A
32. D
33. C
34. C
Tips: Glucose absorption is not impaired by acarbose. However other answers have sucrose and
starch. Break down of these products impaired by acarbose.
35. B
35. A
36. C
37. C
38. E
39. D
40. E
41. C
42. C
43. D
44. A
45. E
46. E
47.

33-11 Copyright © 2000-2014 TIPS Inc. Unauthorized reproduction of this manual is prohibited. This manual is
being used during review sessions conducted by PharmacyPrep.
PharmacyPrep.Com Medicinal Chemistry and Pharmacology of Endocrine System
33-12 Copyright © 2000-2014 TIPS Inc. Unauthorized reproduction of this manual is prohibited. This manual is
being used during review sessions conducted by PharmacyPrep.
www.pharmacyprep.com Med Chem and Pharma of Musculoskeletal Drugs

PHARMACY PREP
MEDICINAL CHEMISTRY AND PHARMACOLOGY OF MUSCULOSKELETAL DRUGS
1. What are the correct statement about rheumatoid arthritis?
I) Occurs on weight bearing joints
II) Occurs on non weight bearing joints
III) Morning stiffness is common symptom
A. I only
B. III only
C. I and II
D. II and III
E. I, II and III

2. All of the following are disease modifying anti rheumatoid arthritis drugs (DMARDs), except?
A. Minocycline B. Sulfasalazine C. Leflunamide D. Sulfinpyrazone
E. Hydroxy choloroquine
3. What test is used for diagnosis of rheumatoid arthritis?
A. rheumatic factor B. erythrocyte sedimentation rate C. serology tests D. All of the above
4. What drug is not recommended in acute gout attacks?
A. ibuprofen B. ASA C. Prednisone D. allopurinol E. colchicine
5. Which of the following drug decrease uric acid production?
A. Probenecid B. Indomethacin C. Colchicine D. Allopurinol
E. Sulfinpyrazone
6) A patient using the following dose of acetaminophen can cause hepatotoxicity?
A) 500 mg x 8 tab daily
B) 500 mg x 14 tab daily
C) 500 mg x 20 tab daily
D) 500 mg x 30 tab daily
7) Acetaminophen toxicity initial symptoms in children?
A. diarrhea B. nausea C. vomiting D. fever E. coma
8. What is the common side effect of acetaminophen?
A. Hepatotoxicity B. renal toxicity C. Rye syndrome D. Rash E. corneal deposits

Copyright © 2000-2014 TIPS Inc. Unauthorized reproduction of this manual is prohibited. This manual is being used
during review sessions conducted by PharmacyPrep. 35-1
www.pharmacyprep.com Med Chem and Pharma of Musculoskeletal Drugs
9) A patient approaches to your pharmacy with prescription of methotrexate 25 mg/daily to treat
rheumatoid arthritis. What should you do?
A) Dispense methotrexate and counsel
B) Call doctor to change the dose to 25 mg/week
C) Call doctor to add infliximab with methotrexate
D) Call doctor to decrease dose 7.5 mg daily
E) Call doctor to add folic acid supplement
10. A patient with fever, intense pain, warmth swelling, and inflammation of foot has been
diagnosed as gout arthritis. What is correct?
I) reduce purine rich diet
II) reduce protein rich diet
III) reduce fluid intake
A. I only B. III only C. I and II D. II and III E. I, II and III
11. Which of the following drugs should be taken with full glass of water?
I) Allopurinol
II) Sulfinpyrazone
III) Colchicine
A. I only B. III only C. I and II D. II and III E. I, II and III
12) What is incorrect about capsaicin?
A) Slow onset of action
B) Used for neuropathic pain
C) Used for osteoarthritis pain
D) Gives burning effect on skin
E) Obtained from ginger root
13) Bisphosphonates mechanism by permanently binding surfaces of mineralized bone?
A) Prevent osteoblast activity
B) Prevent osteoclast activity
C) Prevent bone minerilization
D) Prevent bone deminerilzation
E) Increase calcium deposits on bone

14) Teriparatide increases cortical thickness and bone size. It is?


A) Anabolic agent
B) Bisphosphonates
C) SERM
D) Hormone replacement therapy

Copyright © 2000-2014 TIPS Inc. Unauthorized reproduction of this manual is prohibited. This manual is being used
during review sessions conducted by PharmacyPrep. 35-2
www.pharmacyprep.com Med Chem and Pharma of Musculoskeletal Drugs
ANSWERS:
1. E
2. D
Tips: sulfinpyrazone is uricosuric agent.
3. D
4. D
5. D
Tips: Probenecid and sulfinpyrazones increase uric acid excretion (uricosuric agents), allopurinol
inhibit formation of uric acid. Colchicine, inhibit mitotic tubules, gives anti-inflammatory action.
6. B
Tips: maximum daily dose of acetaminophen to treat osteoarthritis is 4 g/daily. Using 7g/daily or
more can cause hepatotoxicity
7. C
8. D
9. B
10. C
11. C
12. E
13. B
Tips: osteoclasts cause bone resorption thus preventing osteoclast increase bone mineral
density.
14. A

Copyright © 2000-2014 TIPS Inc. Unauthorized reproduction of this manual is prohibited. This manual is being used
during review sessions conducted by PharmacyPrep. 35-3
www.pharmacyprep.com Med Chem and Pharma of Musculoskeletal Drugs
Copyright © 2000-2014 TIPS Inc. Unauthorized reproduction of this manual is prohibited. This manual is being used
during review sessions conducted by PharmacyPrep. 35-4
www.pharmacyprep.com
PHARMACY PREP.
METABOLISM
1) Which of the following is not a drug-metabolizing enzyme?
A) CYP3A
B) CYP2D6
C) CYP2C9
D) CYP1A2
E) Alcohol dehydrogenase
2) Pyrrolidine and piperidine are important heterocyclic compounds in the structure of drugs. Which
of the following drugs has these heterocyclic rings on its structure?
A. Folic acid
B. Clonidine
C. Tetrodotoxin
D. Imipramim
E. Atropine
3. Oxidation of alkyl group results into alcohol, aldehyde and finally carboxylic acids. What type of
alkyl group that mostly undergoes oxidation to an acid is known as:
A. Terminal alkyl group of aliphatic structure
B. Terminal Aromatic alkyl group of structure
C. Benzylic alkyl group attached to aromatic ring structure
D. Heterocyclic ring structure carbons
E. Cyclic ring structure carbons
4. Amines are one of the most common group present in the molecular structure of drugs. Primary
amines are mainly metabolized by phase II metabolism:
A. Deamination
B. Dealkylation
C. Acetylation
D. Hydrolysis
E. Oxidation
5. Molecular structures (drugs) having carboxylic acid, amide, alcohol and/or this group are mainly
metabolized by:
A. Oxidation
B. Hydrolysis
C. Conjugation
D. Estherification
E. Deamination

Copyright © 2000-2014 TIPS Inc. Unauthorized reproduction of this manual is prohibited. This manual is being used
during review sessions conducted by PharmacyPrep. 37-1
www.pharmacyprep.com
6. Pyridine is a mono heterocyclic compound very important in the activity of many different drugs.
The molecular structure 1,4-dihydropyridine is found in:
A. Verapamil
B. Digoxin
C. Clonidine
D. Nifedipine
E. Diltiazem
7. Quinolone structure may be found in which of the following substances?
A. Vitamin K
B. Thiamine
C. Cyanocobalamin
D. Folic acid
E. Vitamin A
8. Steroid structures are common in hormones, such estrogen, progesterone, testosterones. Steroid
contains how many cyclohexane and cyclopentane respectively in it skeleton.
A. 2 and 3 B. 3 and 2 C. 4 and 1 D) 3 and 1 E) 1 and 3
9) Phase I metabolism reaction also known as functionalization metabolism, during this phase drug
will metabolized to obtain functional groups such as hydroxyl group or carboxyl group and these
group in phase II are best metabolized by which of following conjugation reaction:
A) Acetylation
B) Glucuronidation
C) Reduction
D) Hydrolysis
E) Methylation
10. Imidazoline is a molecular structure that contain a heterocyclic ring and belongs the molecular
structure of:
A. Prazosin
B. Nifedipine
C. Atropine
D. Clonidine
E. Allopurinol
11. Diazepam is a long acting benzodiazepine metabolized by:
A. CYP3A
B. CYP2D6
C. CYP2C9
D. CYP1A2
E. CYP0

Copyright © 2000-2014 TIPS Inc. Unauthorized reproduction of this manual is prohibited. This manual is being used
during review sessions conducted by PharmacyPrep. 37-2
www.pharmacyprep.com
12. Conjugation reactions are the most common reactions in Phase II metabolism. Glycine contain
hydroxyl group and may undergoes conjugation with:
A. Carboxylic acid
B. Glutathione conjugates
C. Cytochrome P450
D. Sulfonamides conjugates
E. Alcohol conjugates
13. Which of the following statins can be taken with grapefruit juice:
I-Atorvastatin
II-Pravastatin
III-Fluvastatin
A) I only B) III only C) I and II only D) II and III only E. All are correct
14. The most common metabolic reaction in human being;
A. Oxidation
B. Hydroxylation
C. Glucuronidation
D. Glutathione conjugation
E. Hydrolysis
15) Acetaminophen is considered to be toxic at higher dose (4g/day) and may cause hepatotoxicity.
This is due to accumulation metabolic intermediate. Which of the following is enzyme would most
likely be deficient?
A) Glucuronyl transferase
B) Azo reductase
C) Methyl transferase
D) Glutathione S-transferase (GST)
E) Pseudo cholinesterase
16. All of the following statement about aspirin metabolism are correct except:
A. Aspirin undergo ester glucuronidation
B. Aspirin undergo ether glucuronidation
C. Aspirin undergo hydroxylation
D. Aspirin undergo glycine conjugation
E. Aspirin excretes unchanged drug
17. Glucuronidation reactions is phase II metabolic reaction, and it is catalyzed by:
A. Glutathione S- transferase
B. Azo reductase
C. UDP-Glucuronyl transferase
D. Methyl transferase
E. N-Acetyl transferase

Copyright © 2000-2014 TIPS Inc. Unauthorized reproduction of this manual is prohibited. This manual is being used
during review sessions conducted by PharmacyPrep. 37-3
www.pharmacyprep.com
18) Which of the following substances has affinity to carbon monoxide:
I-Hemoglobin
II-Myoglobin
III-Cytochrome oxidase
A. I only B. III only C. I and II only D. II and III only E. All of the above
19. If a patient taking atorvastatin 80 mg daily, and brings a prescription of erythromycin for 3 days.
What type problem is expected?
A. Drug and Disease interaction
B. Drug and food interaction
C. Drug and Drug interaction
D. Drug and lab interactions
20. Which of the following is not a drug-metabolizing enzyme?
A. CYP 3A4 B. CYP 450 C. UDP-GT D. N-acetyl transferase
E. Alcohol dehydrogenase
21) St John wort is inducer?
A) CYP 3A4 B) CYP 2D6 C) CYP2C9 D) CYP2C19 E) CYP1A2
22. Neonates have insufficient?
A. Glucoronidation conjugation
B. Azo reductase
C. UDP-Glucuronyl transferase
D. Methyl transferase
E. N-Acetyl transferase
23. If patient is using excessive alcohol which of the following enzyme is responsible for side effects?
A. UDP-GT B. CYP 3A4 C. CYP450 D. Alcohol dehydrogenase
E. Sulfotransferase
24. All of the following are CYP3A4 inhibitors, except?
A. Erythromycin
B. Carbamazepine
C. Itraconazole
D. Ketoconazole
E. Cimetidine
25. What is the metabolite of ester type local anesthetics?
A. Para amino benzoic acid B. acroliene C. mercapturic acid D. epinephrine E. none of the above

Copyright © 2000-2014 TIPS Inc. Unauthorized reproduction of this manual is prohibited. This manual is being used
during review sessions conducted by PharmacyPrep. 37-4
www.pharmacyprep.com
26. Penicillin G is metabolized by
A. Renal metabolism B. Biliary metabolism C. Liver metabolism
D. GI metabolism E. All of the above
27. All of the following are prodrugs, except?
A. Captopril B. Enalapril C. Moexipril D. Fosinopril E. Ramipril
28. All of the following are substrate of CYP3A4, except?
A. HMG Co A reductase inhibitors
B. Grapefruit juice
C. Sildenafil
D. Nifedipine
E. Amlodipine
29. All of the following are the potent CYP3A4 inhibitors, except A. Grapefruit juice
B. HMG Co A reductase inhibitors
C. Erythromycin
D. Azole antifungal
E. Quinidine
30) Glutathione is intracellular antioxidants, consist of
A. g-glutamylcysteinyl glycine
B. Glutaric acid
C. 2 cysteine
D. glycine and cysteine
E. glutamyl and cysteine
31. A patient is suffering from COPD and asthma. He is on theophylline. His lifestyle include high
protein diet and smoking? What is correct
A-Protein diet and smoking theophylline clearance
B-Protein diet and smoking theophylline clearance
C) Protein diet and smoking do not effect theophylline clearance
D) None of the above
32. A patient using propranolol to treat hypertension and using salbutamol to treat mild asthma.
What is the pharmacist concern?
I) Drug and drug interaction
II) Drug and disease interaction
III) Drug and food interactions
A. I only B. III only C. I and II D. II and III E. I, II, III

Copyright © 2000-2014 TIPS Inc. Unauthorized reproduction of this manual is prohibited. This manual is being used
during review sessions conducted by PharmacyPrep. 37-5
www.pharmacyprep.com
33. Which of the following enzyme catalyzes glycine conjugation?
A) UDP glucoronyl transferase
B) Glutathione S- transferase
C) Acetyl co-A
D) Cytochrome CYP 450
E) Esterase
34. In hepatic detoxification related to glutathione conjugation, which amino acid related?
A) Glycine
B) Cysteine
C) Phenylephrine
D) Glutathione
E) Glutamic acid

Copyright © 2000-2014 TIPS Inc. Unauthorized reproduction of this manual is prohibited. This manual is being used
during review sessions conducted by PharmacyPrep. 37-6
www.pharmacyprep.com
CORRECT ANSWERS
1. E
2. E
Tips: Pyrrolidine is a five membered and piperidine six membered, important heterocyclic
compounds in the structure of drugs. Atropine is an antimuscarinic agent having pyrrolidine and
piperidine heterocyclic rings on its molecular structure.
3. C
Tips: 5- Alkyl group attached to aromatic rings would undergo faster oxidation because of resonance
effect of aromatic ring.
4. C
Tips: Acetylation is a phase II metabolism reaction characterized by the gain of an acyl group, and the
main reaction that undergoes primary amines.
5. C
Tips: Conjugations are phase II metabolism reactions characterized by an incorporation of
endogenous substrate on the molecular structure of drugs normally coming from phase I metabolism
reaction. Carboxylic acids are one of the most common groups present in the molecular structure of
drugs mainly metabolized by conjugation.
6. D
Tips: Nifedipine is calcium channel blocker that has 1,4-dihydropyridine on its structure responsible
for its mechanism of action. It is also contain nitro fuctional group
7. A
Tips: Vitamin K has a quinolone on its molecular structure.
8. D
Tips: Most of the steroid compounds have three cyclohexanes and one cyclopentane ring on their
molecular structure.
9. B
Tips: Glucuronidation is the main reaction of phase II metabolism reactions, and are responsible
mainly for the metabolism of drugs having hydroxyl, carboxyl and amide group on their molecular
structure. Conjugation reactions are the most common reactions of phase II drug metabolism.
10. D
Tips: Clonidine is an antihypertensive drug that depends of its imidazoline molecular structure to be
effective.

Copyright © 2000-2014 TIPS Inc. Unauthorized reproduction of this manual is prohibited. This manual is being used
during review sessions conducted by PharmacyPrep. 37-7
www.pharmacyprep.com
11. A
12 A
Tips: Glycine is a phase II metabolism reaction also known as amino acids conjugation reaction that
undergoes conjugation with carboxylic acid
13. D
Tips: Atorvastatin, are metabolized by CYP3A enzyme and this enzyme is inhibited by grapefruit juice,
therefore they should not be combined. Fluvastatin, pravastatin, Rosuvastatin are not metabolized
by CYP3A
14. C
15. D
Tips: Glutathione conjugation metabolizes N-acetyl benzoquinone to mercapturic acid, hence this
reduce toxicity of intermediate.
16. E
17. C
18. E
Tips: Hemoglobin, Myoglobin and cytochrome oxidase contain porphyrin ring, and this ring has
affinity to oxygen, carbon monoxide and carbon dioxide, cyanide etc.
19. C
Tips: Atorvastatin is substrate of CYP3A4, which means it is essential for its metabolism. Where as
erythromycin is inhibitor of CYP3A4, that means, it lower the enzyme levels.
20. E
21. A
22. A
23. D
24. B
25. A
26. A

Copyright © 2000-2014 TIPS Inc. Unauthorized reproduction of this manual is prohibited. This manual is being used
during review sessions conducted by PharmacyPrep. 37-8
www.pharmacyprep.com
27. A
Tips: captopril and lisinopril are active drugs and listed 7 other ACE inhibitors approved in the United
States that are inactive prodrugs until metabolized in the liver.
28. B
29. B
Tips: ABCDEFs are substrates of CYP3A4 Amitriptyline, Benzodiazepine, Clarithromycin,
Cyclosporins, Dihydropyridine, Estrogen, TerFinadine, and Statins
30. A
31. B
32. C
33. C
34. B

Copyright © 2000-2014 TIPS Inc. Unauthorized reproduction of this manual is prohibited. This manual is being used
during review sessions conducted by PharmacyPrep. 37-9
www.pharmacyprep.com
Copyright © 2000-2014 TIPS Inc. Unauthorized reproduction of this manual is prohibited. This manual is being used
during review sessions conducted by PharmacyPrep. 37-10
www.pharmacyprep.com

PHARMACY PREP
BIOPHARMACEUTICS
1) Acetyl salicylic acid (ASA) pH is 4.5 and pKa is 3.5, what percent drug will ionized in the body.
A) 90%
B) 99%
C) 100%
D) 50%
E) 0%
2. Aspirin pH is 3.5 and pKa is 3.5, what percent drug will ionized in the body.
A. 90% B. 99% C. 100% D. 50% E. 9%
3. Acetyl salicylic acid is pH-Pka = 0, what percent drug will ionized in the body.
A) 90% B) 99% C) 100% D) 50% E) 0%
4) The pH of blood is 7.4, after administering drug it becomes 6.2 the drug is:
I-weak acidic II-weak base III-amphoteric
A. I only B. III only C. I and II D. II and III E. I, II, III
5. ASA pH is 3.5 and pKa is 3.5, what percent drug will unionized in the body.
A. 90% B) 99% C. 100% D. 50% E. 10%
6) If the pKa of weak base drug is 6.4 and you have an ambient temperature with pH 4.4.
I) The drug will be almost completely ionized
II) The drug will be rapidly excreted
III) The drug will not be ionized neither excreted
A) I only
B) III only
C) I and II
D) II and III
E) I, II, III
7. Critical solution temperature represents:
A. The temperature which concentration of phenol is homogenous solution
B. The temperature at phenol vaporizes
C. The temperature above which homogenous solution is formed for all concentration of phenol
D. Temperature which phenol vapor pressure equals the vapor pressure of water.

Copyright © 2000-2014 TIPS Inc. Unauthorized reproduction of this manual is prohibited. This manual is being used
during review sessions conducted by PharmacyPrep. 38-1
www.pharmacyprep.com
8. The rate of chemical reactions is enhanced by:
A. Increase the stirring
B. Increasing the temperature of reaction mixture
C. Removing the inert solvent
D. Increase the rate of cooling
E. None of the above
9. Correct statement about physiological factors affecting the rate and extent for topical drug
absorption include:
I-The type of ointment base
II-the partition coefficient of the drug molecule
III-surfactant in formulations
A. I only
B. III only
C) I and II
D) II and III
E) I, II, III
10. If the total body clearance of a drug is 200ml/min in normal healthy adult and renal clearance is
10ml/min then, what is correct in patient.
A. Drug is deposited in adipose tissue
B. Drug is increased in liver and have recycling
C. Drug metabolisms is increased
D. Greater accumulation
E. drug have no plasma protein binding
11. Compared to the sublingual tablet, nitroglycerine ointment is:
A. More stable
B. More effective
C. More prolong effect
D. More absorbed
E.More local effect
12) pH-pka = 1.5, for acidic drug what percent of drug is ionized?
A) 3.06%
B) 96%
C) 2.5%
D) 100%
50%
13. If a medication if pH = Pka?
A-100% solute is ionized
B-40% solute is ionized
C-50% solute is ionized (1/2)

Copyright © 2000-2014 TIPS Inc. Unauthorized reproduction of this manual is prohibited. This manual is being used
during review sessions conducted by PharmacyPrep. 38-2
www.pharmacyprep.com
D-99% solute is ionized
E-95% solute is ionized
14. A drug degradation at room temperature is determined by?
A. Noye-Whittney equation
B. Arrhenius equation
C. Hasselbach-Handerson equation
D. Ficks law
E. Laplaws Law
15) The pka of acetic acid, barbituric acid and ASA are 5.21, 4.371 and 3.31 respectively. Arrange
these acids in their respective strengths (from lowest to strongest)
A. Acetic acid, Barbituric acid and ASA
B. ASA, Barbituric acid and Acetic acid
C. Barbituric acid, Acetic acid and ASA
D. Acetic acid, ASA, and Barbituric acid
E. Barbituric acid, ASA, and Acetic acid
16. Which of the following forms buffer?
A. weak acid and salt of acid
B. Weak base and salt of acid
C. Weak acid and salt of base
D. Strong acid and salt of weak base
E. Strong base and salt of weak acid
17) What is the absolute bioavailability of tablet A?
TABLET A-----------------50 mg---------------AUC 40
IV PUSH ------------------10 mg---------------AUC 50
A) 80% B) 0.16% C) 16% D) 8% E) 25%
18) The AUC for an oral dose of a drug is 4.5 μg/mL/hr and for an IV dose 11.2 μg/mL/hr. What is the
bioavailability of an oral dose of this drug?
A) 100% B) 50% C) 40% D) 99% E) 9%
19. If clearance is decreased by half, how will that influence AUC?
A. AUC decrease half
B. AUC increase half
C. AUC is doubled increase
D. AUC is doubled decrease
E. No change in AUC

Copyright © 2000-2014 TIPS Inc. Unauthorized reproduction of this manual is prohibited. This manual is being used
during review sessions conducted by PharmacyPrep. 38-3
www.pharmacyprep.com
20. Phenobarbita (acidic) pH is 3.4 and pKa is 3.4, what percent drug will ionized in the body.
A. 90% B. 99% C. 100% D. 50% E. 10%
21. If the pKa of weak base drug is 4.4 and you have an ambient temperature with
ph 7.4
I-The drug will be almost completely ionized
II-The drug will be rapidly excreted
III-The drug will not be ionized neither excreted
A. I only B. III only C. I and II only D. II and III only E. All are correct
22. Absorption of water-soluble drug:
A. Well absorbed all over the body
B. Cross BBB very fast
C. Poor absorption through BBB
D. Poor absorption through placenta
E. C and D are right
23. Fick’s law is a theory very used in pharmacokinetic that describes:
A. The movement of the molecules across barriers
B. The stability of a drug molecule in room temperature
C. The rate of drug absorption versus the rate of drug elimination
D. The process of bioavailability of drugs
E. The drugs mechanism of action at its receptor site
24. According to the Noyes-Whitney equation, which of the following statements is not true?
A. The greater the difference in concentration across the diffusion layers the faster the dissolution
rate.
B. Decreasing surface area increases dissolution rate.
C. The greater the solubility, the greater the dissolution rate.
D. Agitation will generally increase dissolution rate.

25) Phenobarbital (acidic) pH is 3.4 and pKa is 3.4, what percent drug will ionized
in the body.
A. 90% B. 99% C. 100% D. 50% E. 10%
26. All can be done to ensure higher systemic absorption of a drug that shows high first-pass effect,
except?
I-Increase the dose
II-Change the route of administration
III-Increase the frequency of dosing
A. I only B. III only C. I and II only D. II and III only E. All are correct

Copyright © 2000-2014 TIPS Inc. Unauthorized reproduction of this manual is prohibited. This manual is being used
during review sessions conducted by PharmacyPrep. 38-4
www.pharmacyprep.com
27. An immediate release tablet and sustained released tablets have the same?
A. AUC B. Tmax C. Cmax D. Duration of action E. time to release
28. Which of the following is pharmacokinetics factor is least concern for a drug immediate release
and sustain release dosage form?
A. Tmax B. Cmax C. AUC D. Time E. Duration of action
29. In the above questions, immediate release dosage form has?
A. short Tmax B. long Tmax C. long duration of action D. Slow action E. High potency
30. Factors affecting the dissolution rate of drug from a dosage form can be related to, except?
A. physiochemical properties of drug
B. particle size and crystalline structures
C. Partition coefficient
D. dissociation constant
E. rate of disintegration

Copyright © 2000-2014 TIPS Inc. Unauthorized reproduction of this manual is prohibited. This manual is being used
during review sessions conducted by PharmacyPrep. 38-5
www.pharmacyprep.com
ANSWERS:
1. A
Tips: Hasselbach-Handerson equation:
charge (pH-Pka)
% ionization = 100 / 1+10
Charge = -1 for acid
-1 (1)
= 100 / 1+10
= 100 / 1.1 = 90%
1 -1
10 = 10 10 = 0.1
2 -2
10 = 100 10 = 0.01
3 -3
10 = 1000 10 = 0.001
2. D
3. D
4. A
Tips: pH of blood is 7.4 turns into 6.2 after addition drug that means, decrease pH is acidic drug.
Example of amphoteric solutions is: H2O H + OH
+ -

5. D
6) C
1(4.4-6.4)
100/1+10
1(-2)
100/1+10
= 99% ionized
Tips: In the above questions just 99% is ionized which is rapidly excreted.
7. C
8. B
9. E
10. C
11. C
12. B
–1 (1.5)
100/1+10
100 / 1 + 0.0316
100/1.0316 = 96%

Copyright © 2000-2014 TIPS Inc. Unauthorized reproduction of this manual is prohibited. This manual is being used
during review sessions conducted by PharmacyPrep. 38-6
www.pharmacyprep.com
How to use calculator?
enter 100
Press divide /
Press (
Press 1+10
x
Press y
Press (-)
Press 1.5
Press )
Press =
13. C
How to use calculator? SHARP EL-510R
Enter: 10
x
Press : y
Press: (-)

Enter 1.5 =
14. B
15. B
16. A
17. C
AUCpo/AUCiv = [10 x 40]/[50 x 50] x 100 = 16%
or
[40/250] x 100 = 16%
18. C
F = AUCoral = 4.5 μg/mL/hr = 0.4 or 40%
AUCIV 11.2μg/mL/hr
Tips:
To determine the AUC? AUC = [F x Dose]/V x K
19. C
Clt = Amount absorbed/AUC or Clt = FD/AUC
F = bioavailability
D= Dose
AUC = Area Under the Curve

Copyright © 2000-2014 TIPS Inc. Unauthorized reproduction of this manual is prohibited. This manual is being used
during review sessions conducted by PharmacyPrep. 38-7
www.pharmacyprep.com
20. D
Tips: pH – pKa = 0 is 50% ionized and 50% unionized. If PH = pKa is 50% drug is ionized and 50%
unionized.
21. B
Tips: 99.9% drug is unionized and do not excrete. 0.09% drug is ionized excrete but slow.
22. E
Tips: The drug cannot be too acid or too base or too water-soluble or too lipid soluble because these
interfere with the absorption. Completely water-soluble drug will have poorly absorption through
blood brain barrier and through placenta because these tissues are lipid constituted and water-
soluble drugs tends to cross these barrier very slow due to have no affinity with these barriers.
23. A
Tips: Fick’s law is a theory very used in pharmacokinetic that describes the movement of the
molecules across barriers
24. B
25. D
Tips: pH – pKa = 0 is 50% ionized or unionized or PH = pKa is 50%
26. B
27. A
28. C
29. A
30. E

Copyright © 2000-2014 TIPS Inc. Unauthorized reproduction of this manual is prohibited. This manual is being used
during review sessions conducted by PharmacyPrep. 38-8
PharmacyPREP.Com Physical Pharmacy

PHARMACY PREP.
PHYSICAL PHARMACY
1. Buffer salts are added to many drug solutions to maintain the formulation at optimum pH. Which
of the following is/are characteristics of a buffer solution:
I- Formed from a weak acid or weak base and its salts.
II- Addition of a little bit of water or a little increase in the temperature will not affect the pH.
III: Addition of a strong acid or a strong base to the solution will not affect the pH.
A. I only B. III only C. I and II only D. II and III only E. All are correct
2. Parenteral solutions are used for internal use. What is the most common used vehicle for
parenteral drugs manufacturing?
A. Purified water USP
B. Water for injection USP
C. Distillate water
D. Sterile water for injection USP
E. Bacteriostatic water
3. All the following are oral drug solutions, EXCEPT
A. Elixirs
B. Spirits
C. Tinctures
D. Fluidextracts
E. Lotion
4. All of the following are characteristics of on emulsion, EXCEPT
A. Most of emulsions are constituted of two phases: Aqueous and oily phase
B. If water is the internal phase, the emulsion is classified as W/O
C. If water is the internal phase, the emulsion is classified as O/W
D. If water is the external phase, the emulsion is classified as O/W
E. Emulsifying agents are added to emulsion to lower the interfacial tension
5. Correct statements regarding emulsifying agents include:
I- Lower the interfacial tension stabilizing emulsions
II- Acacia and methyl cellulose are examples of emulsifying agents
III- Synthetic emulsifying agents are not used in emulsion
A. I only B. III only C. I and II only D. II and III only E. All are correct
6. In which method of the following pharmaceutics techniques a substance is reduced by addition of
a solvent material that is easily removed.
A. Trituration
B. Pulverization by intervention

Copyright © 2000-2014 TIPS Inc. Unauthorized reproduction of this manual is prohibited. This manual is being used
during review sessions conducted by PharmacyPrep. 39-1
PharmacyPREP.Com Physical Pharmacy
C. Levigation
D. spatulation
E. Sifting
7. Correct statements regarding Trituration include:
A. Is the process of mixing powders in a large container rotated by a motorized process
B. Is the process when potent substance must be mixed with a large amount of diluents.
C. Is the process of reducing the substance to small particles by rubbing it in a mortar with a pestle.
D. Is the process of reducing the substance by addition of a solvent that is easily removed after
pulverization.
E. Is the process of mixing powders by passing them through sifters
8. All of the following are used as diluents in tablet formulation, EXCEPT
A. Talc B. Kaolin C. Lactose D. Mannitol E. Starch
9. Characteristics of use of lubricants in tablet formulation include:
I- It helps the patient to swallow the tablet.
II- Lubricants, glidants and anti adherents have overlapping functions in tablet formulation.
III- It reduces the friction that occurs between the walls of the tablet and the walls of the die cavity.
A. I only B. III only C. I and II only D. II and III only E. All are correct
10- Which of the following tablet(s) allow absorption through the oral mucosa?
I-Sublingual tablets
II- Lozenges
III- Sugarcoated tablet
A. I only B. III only C. I and II only D. II and III only E. All are correct
11. A manufacture tablet processing problem that is characterized by the partial or complete
separation of the top or bottom crown from the main body of the tablet is know as:
A. Lamination B. Picking C. Mottling D. Capping E. Sticking
12. Tablet manufacture problems include:
I-Mottling is unequal colour distribution on the tablet surface.
II- Sticking is adhesion of tablet material to a die wall
III- Humidity is the melting of the tablet
A. I only B. III only C. I and II only D. II and III only E. All are correct
13. One of the most common lubricant used in tablet manufacture include:
A. Theobroma oil
B. Glycerol

Copyright © 2000-2014 TIPS Inc. Unauthorized reproduction of this manual is prohibited. This manual is being used
during review sessions conducted by PharmacyPrep. 39-2
PharmacyPREP.Com Physical Pharmacy
C. Aluminium stearate
D. Sodium stearate
E. Magnesium stearate
14. Which of the following propellant used in aerosol products was put out of market due to
depletation of the ozone layer?
I-Chlorofluorocarbons-CFC
II-Dimethyl-Ether
III-Hydrofluorocarbons-HFC
A. I only B. III only C. I and II only D. II and III only E. All are correct
15. Which of the following equations can be used to easily calculate the pH of a buffer solutions
constituted of a weak acid and its salts.
A. Hixson-Crowell equation
B. Arrhenius equation
C. Stokes equations
D. Henderson-Hasselbalch equation
E. Noyes-Whitney equation
16. Arrhenius equation is used when we need calculate the:
A. pKa of an acid solution
B. Stability of a drug at room temperature
C. pH of a buffer solution
D. Rate of ionization of a drug
E. Solubility of a solution
17. The HLB-Hydrophilic-Lipophilic Balance is applied to:
A. Emulsifying agents
B. Humectants agent
C. Thermo labile agents
D. Surfactant agent
E. Solubilizing agents
18. An agent with HLB value range of 0-3 is known as:
A. Antifoaming agent
B. W/O emulsifying agent
C. O/W emulsifying agent
D. Wetting agent
e. Detergents
19. An agent classified as oil in water (O/W) emulsifying agent has its HLB value ranged in:
A. 0-3 B. 4-6 C. 7-9 D. 8-18 E. 13-15

Copyright © 2000-2014 TIPS Inc. Unauthorized reproduction of this manual is prohibited. This manual is being used
during review sessions conducted by PharmacyPrep. 39-3
PharmacyPREP.Com Physical Pharmacy
20. Suppository base vary well know of its polymorph characteristics:
A. Cocoa butter
B. Polyethylene Glycol
C. Glycerin
D. Mineral Oil
E. Cotmar
21. All are characteristics of a polymorph drug, EXCEPT:
A. It has a two different physical form
B. It melts in some determinate temperature, normally high temperatures (>30°C)
C. Theobroma oil is a classic example of polymorph drug
D. Polymorph drugs are widely used as suppository base
E. Polymorph drugs melt in very low temperature (<10°C)
22. A pharmacist would like to make a compound prescription where he needs to mix 1% of camphor
into white petroleum. Which would be the more appropriated technique to be used by the
pharmacist?
A. Trituration
B. Levigation
C. Pulverization by intervention
D. Geometric dilution
E. Attrition
23. We can increase the bioavaliability of an ophthalmic preparation by
I-Increasing viscosity of preparation
II-Addition of a sticking agent
III-Increasing volume of the inserted dose
A. I only B. III only C. I and II only D. II and III only E. All are correct
24. Autoclaving is sterilization process of:
A. Dry heat sterilization
B.Steam Sterilization
C.Filtration
D.Gas sterilization
E-Irradiation
25. Acetyl Salicylic Acid is easily hydrolyzed during tablet manufacturing. To avoid manufacture
problems we should use:
I- Dry granulation
II- Direct compression
III Wet granulation
A. I only B. III only C. I and II only D. II and III only E. All are correct

Copyright © 2000-2014 TIPS Inc. Unauthorized reproduction of this manual is prohibited. This manual is being used
during review sessions conducted by PharmacyPrep. 39-4
PharmacyPREP.Com Physical Pharmacy
26- Flocculated suspension upon sedimentation will produce
I-Large volume sedimentation
II-Clear boundary when particles settle
III-Easily resuspended when shaking
A. I only B. III only C. I and II only D. II and III only E. All are correct
27. Methylparaben is used in ophthalmic solution as
A. Preservative
B. Surfactant
C. Viscosity enhancer
D. Humectants
E. Solubilizing agent
28. The exipient used in tablet formulation have a large variety of functions. Which of the following
do not describe a tablet excipient?
A. Diluents are fillers designated to make up the required bulk of the tablet
B. Binders promote granulation during the dry granulation process
C. Disintegrates facilitate disintegration when the tablet reaches the GIT
D. Glidants help to promote the flow of tablet granulation
E. Emollients help to moisturize the tablet to avoid irritation of oral mucosa
29. What is true regarding DELIQUESCENCE?
I- Occurs when the vapor pressure of the saturated aqueous solution is less than the vapor pressure
of the water in the ambient air.
II- It’s the condition of becoming moist
III- It’s the result of the absorption of water from the air
A. I only B. III only C. I and II only D. II and III only E. All are correct
30. Substances that have the capacity of absorbing moisture is well known as:
A. Hygroscopic
B. Hydromorphic
C. Hydrophilic
D. Hydrophobic
E. Amphiphilic
31. Van Der Waal’s forces is an example of
A. Hydrogen bound
B. Dipole-dipole interaction
C. Covalent bound
D. Cation-cation interaction
E. Anion-anion interaction
32. In order to have hydrogen bound, the molecule should have the following characteristics

Copyright © 2000-2014 TIPS Inc. Unauthorized reproduction of this manual is prohibited. This manual is being used
during review sessions conducted by PharmacyPrep. 39-5
PharmacyPREP.Com Physical Pharmacy
I- The molecule should be formed of hydrogen and strong electronegative elements
II- F, N, O, S, Cl are examples of strong electronegative elements
III- Hydrogen bounds can be classified in INTER and INTRA depending the position of the bound in
the molecule
A. I only B. III only C. I and II only D. II and III only E. All are correct
33. Hydrogen bound can be formed with all the following molecules, EXCEPT:
A. Ether-ether
B. Alcohol-alcohol
C. Esther-alcohol
D. Water-alcohol
E. Esther-Water
34. Correct statements regarding Eutectic Mixture may include:
I- Is the mixture of two or more substances that melt at the highest possible temperature
II- Is the mixture of two or more substances that melt at the lowest possible temperature
III- Eutectic point is the melting point of a eutectic mixture
A. I only B. III only C. I and II only D. II and III only E. All are correct
35. Correct statements regarding Partition Coefficient may include:
I- It measures the solubility of a substance in octane and water
II- It specifies how hydrophilic and hydrophobic a substance is.
III- A substance with high partition coefficient is hydrophilic
A. I only B. III only C. I and II only D. II and III only E. All are correct
36. All are true statements regarding ADSORPTION, except:
A. Separation of a substance from one phase accompanied by its concentration at the surface of
another.
B. Separation of a substance from one phase to another phase
C. An adsorbent agent is capable of holding other molecule onto its surface
D. Examples of adsorbent agents are charcoal and powdered cellulose
E. Adsorption and absorption are two different processes
37. Which of the following tablets suffer disintegration only when it reaches the small intestine?
A. Coated tablet
B. Sugar coated tablet
C. Enteric coated tablet
D. Caplets
E. Lozenges

Copyright © 2000-2014 TIPS Inc. Unauthorized reproduction of this manual is prohibited. This manual is being used
during review sessions conducted by PharmacyPrep. 39-6
PharmacyPREP.Com Physical Pharmacy
38. Binders are used in tablet formulation to make the adhesion of the powdered drug to the
inactive drug. Which of the following binder can be used in both wet and dry tablet formulation?
A. Cellulose B. Gelatin C. Starch D. Sucrose E. Methyl cellulose
39. What does EFFLORESCENCE means?
A. Absorb moisture
B. Maintain moisture
C. Loss of moisture
D. Absorb water
E. Loss of carbon dioxide
40. Flocculating agents are widely used in suspensions:
I- They are used to enhance the particles dispersability of a suspension
II- They make the suspension easily to redisperse by shaking the suspension
III- Flocculating agents are used in solution, emulsions and suspensions
A. I only B. III only C. I and II only D. II and III only E. All are correct
41. Which of the following labels would be the most appropriate to be used in your pharmacy when
dispensing a suspension
A. “Take with food”
B. “Shake well before use”
C. “Take with lots of fluids”
D. “Rinse mouth after use”
E. All the above are correct
42. Antioxidants are added to drug formulation in order to avoid degradation. All of the following are
antioxidants, EXCEPT
A. Tocopherol
B. Ascorbic acid
C. EDTA
D. Sodium bisulfite
E. Sodium benzoate
43. Correct statements regarding Tocopherol include:
I- It is an antioxidant mainly used in water-soluble formulations
II- It is also known as vitamin E
III- α-Tocopherol is the strongest tocopherol antioxidant
A. I only B. III only C. I and II only D. II and III only E. All are correct
44. You received a new drug in your pharmacy and the manufacture instructions tell you to keep this
drug in a cool place. Which temperature should you keep this drug?
A. <0°C B. 2 to 8°C C. 8 to 15°C D. 15 to 30°C E. 30°C

Copyright © 2000-2014 TIPS Inc. Unauthorized reproduction of this manual is prohibited. This manual is being used
during review sessions conducted by PharmacyPrep. 39-7
PharmacyPREP.Com Physical Pharmacy
45. Correct statements regarding vapor pressure may include:
I- Vapor pressure is not a colligative propriety
II- Increasing solute, decrease the vapor pressure
III- Increasing solute, increase the boiling point
A. I only B. III only C. I and II only D. II and III only E. All are correct
46. Boric acid is often used to adjust the isotonicity of some drug preparations because of its buffer
capacity and anti-infective proprieties. Boric acid is widely used in:
A. Ophtalmic solutions
B. Nasal solutions
C. Otic solutions
D. Topical formulations
E. Oral solutions
47. Burrow’s solution – Aluminum acetate is known as:
A. Colloidion
B. Astringent
C. Adsorbent
D. Absorbent
E. Surfactant
48. Which of the following best describe the function of propylene glycol in this formulation?
Diazepan IV----------2mls
Propylene glycol-----2%
Ethanol----------------5mls
A. Preservative
B. Solvent
C. Co-solvent
D. Surfactant
E. None of the above
49. Sodium stearate is classified as:
A. Anionic Surfactant
B. Cationic Surfactant
C. Non-ionic Surfactant
D. Hydrophobic surfactant
E. Amphiphilic surfactant
50. Compared to the sublingual tablet, nitroglycerin ointment:
A. Provides only local effect
B. It a more stable dosage form
C. Provides a more prolonged effect
D. Is more rapidly absorbed

Copyright © 2000-2014 TIPS Inc. Unauthorized reproduction of this manual is prohibited. This manual is being used
during review sessions conducted by PharmacyPrep. 39-8
PharmacyPREP.Com Physical Pharmacy
E. Is more rapidly eliminated
51. The dissolution rate of nitrofurantoin tablets may be increased by:
A. Micronizing the nitrofurantoin powder
B. Decreasing viscosity of the dissolution medium
C. Heating the solution medium
D. Mechanical agitation
E. All of the above increase the dissolution rate of nitrofurantoin tablets
52. Benzocaine with caffeine can for
A. Ion-dipole
B. Ion-Ion
C. Dipole-dipole
D. Complex molecule
E. No reaction is formed
53. Antioxidants, which mechanistically act as reducing agent, include:
I- Ascorbic acid
II- Sodium bisulfite
III- Citric acid
A. I only B. III only C. I and II only D. II and III only E. All are correct
54. Reason for using coating tablets include all of the following, EXCEPT:
A. To mask the taste of the drug
B. To mask the odor of the drug
C. To improve the appearance of the drug
D. To increase the drug’s release rate
E. To protect the drug from stomach acid
55. An ophthalmic solution must be
A. Hypertonic
B. Isotonic
C. Hypotonic
D. Amphiphilic
E. Lipophilic
56. Use(s) Mineral oil may include:
A. Used as laxative
B. Used as antioxidant
C. In chronic use decrease the absorption of fat-soluble vitamins
D. Used as lubricant in suppositories
E. All are correct

Copyright © 2000-2014 TIPS Inc. Unauthorized reproduction of this manual is prohibited. This manual is being used
during review sessions conducted by PharmacyPrep. 39-9
PharmacyPREP.Com Physical Pharmacy
57. Which decrease tension between solid and liquid?
A. Solubilizing agent
B. Wetting agent
C. Hydrophilic agent
D. Emulsifying agent
E. Decreasing the particle size of the solid
58. An agent with HLB value range of 14-18 is known as:
A. Antifoaming agent
B. W/O emulsifying agent
C. O/W emulsifying agent
D. Wetting agent
E. Solubilizing agent
59. In preparing ZnSO4 suppositories using 200 mg of its USP value. Refers that displace value of
ZnSO4 is 4 means that:
A. ZnSO4 will displace 800 mg of cocoa butter
B. Cocoa butter displace 800 mg of ZnSO4
C. Only 4 mg of cocoa butter will be displaced
D. 200 mg of ZnSO4 will displace 50 mg of cocoa butter
60. Addition of mineral oil to a sulphur ointment preparation is called:
A. Levigation
B. Milling
C. Trituration
D. Solubilization
E. Emulsification
61. One part of solvent is required to dissolve one part of solute:
A. Slightly B. Sparingly C. Soluble D. Freely soluble E. Very soluble
62. Suitable antioxidants for an aqueous preparation:
I-EDTA
II-Ascorbic acid
III- Tocopherol
A. I only B. III only C. I and II only D. II and III only E. All are correct
63. Hydrogen peroxide for TOPICAL use solution:
A. 5V B. 10V C. 15V D. 20V E. 30V
64. Benzocaine as a local anesthetic in a TOPICAL preparation should be at least, not less than:
A. 1% B. 3% C. 5% D. 10% E. 15%

Copyright © 2000-2014 TIPS Inc. Unauthorized reproduction of this manual is prohibited. This manual is being used
during review sessions conducted by PharmacyPrep. 39-10
PharmacyPREP.Com Physical Pharmacy
65. Correct statements regarding Papain may include:
I- Proteolitic enzyme used in contact Lents to remove proteins
II- Used as preservative in soft contact Lents
III- Only available in the ophthalmic office
A. I only B. III only C. I and II only D. II and III only E. All are correct
66. The shell of soft gelatin capsules may make by addition of plastic like substance
A. Polyethylene glycol
B. Povidone
C. Lactose
D. Sorbitol
E. methyl cellulose
67. Incorrect statement for hard gelatine capsule?
A. hard gelatine capsules have preservatives
B. hard gelatine capsules shells are prepared by gelatine, sugar and water.
C. suitable to fill aqueous liquid in hard gelatine capsule
D. None of the above
68. What kind of capsules are used for fish liver oil?
A. Soft gel capsules B. Hard gel capsules C. sustain release capsules D.Time release capsules E. All of
the above
69. What is correct about maintaining moisture is?
A. Humectants B. Colloidion C. Elixir D. Gel E. Eutectic mixture
70. Method of drying heat sensitive preparation also known as?
A. Lyophylization or freeze drying method B. Levigation C. Trituration D. Sublimation E. Distillations
71. Transportation in open membrane system depends on?
I- saturable system
st
II-1 order
III-Independent of concentration
A. I only B. III only C. I and II only D. II and III only E. All are correct
72. Equation for colligative properties?
A. Raults law
B. Hasselbach-Handerson equation
C. Michelles Menten equation
D. Noyes Whitney equation
E. Arrhenius equation

Copyright © 2000-2014 TIPS Inc. Unauthorized reproduction of this manual is prohibited. This manual is being used
during review sessions conducted by PharmacyPrep. 39-11
PharmacyPREP.Com Physical Pharmacy
73. Polyethylene glycol (PEG) has all of the following uses except?
A. Ointment base
B. Plasticizer in capsule
C. Solvent in oral preparations
D. Suppository base
E. Binder in tablet
74. An aqueous 1.0 N solution of the following acids is prepared. Which will be the strongest acid?
A. Hydrochloric acid
B. Nitric acid
C. Perchloric acid
D. Sulfuric acid
E. None of the above

75. 1. Water-Proton Theory


2. Theory of Solvent Systems
3. Bronsted-Lowry Theory
4. Lewis theory
In which of the above concepts is the formation of the coordinate covalent bond a consequence?
A. 1&3
B. 1&4
C. 4
D. 3&4
E. 1, 2, 3 &4

76. According to the Bronsted-Lowry theory which of the following is untrue?


A. A strong acid gives rise to a weak conjugate base
B. Anions, cations and molecules may show acidic or basic behavior
C. Acids and bases react to form salts
D. Water can act either as an acid or as a base
E. The ionization of an acid in water is an example of an acid-base reaction

77. The force of attraction between molecules of different phases is referred to as:
A. electrostatic force
B. adhesive force
C. van der Waals force
D. cohesive force
78. The force which resists the increase in are of contact between two immiscible liquid phases is
referred as:
A. force of adhesion
B. surface tension

Copyright © 2000-2014 TIPS Inc. Unauthorized reproduction of this manual is prohibited. This manual is being used
during review sessions conducted by PharmacyPrep. 39-12
PharmacyPREP.Com Physical Pharmacy
C. interfacial tension
D. force of cohesion

79. Which of the following statements is true?


o
A. When the contact angle is 180 , wetting is complete.
B. Wetting agents are surfactants having an HLB of 7 to 9.
o
C. When the contact angle is 0 , the cohesive forces of the liquid are greater than the
adhesive forces between liquid and solid.
D. When a liquid has a high contact angle on a solid surface, it is said to have a high
spreading coefficient.

80. A liquid will rise in a capillary tube when the contact angle is:
o o
A. greater than 90 but less than 135
o o
B. greater than 135 but less than 180
o o
C. greater than 0 but less than 90
o
D. greater than 90

81. In adsorption at a solid interface the material which is adsorbed is referred to as the:
A. adsorbent
B. adsorbate
C. absorbent
D. eluate

82. The difference in potential between the particle surface an the electro-neutral region in the bulk
solution of a colloidal system defines:
A. Streaming potential
B. Zeta potential
C. Electrokinetic potential
D. Electrochemical potential

83. The use of an electrolyte to reduce the zeta potential of a dispersed system to almost zero in
order to stabilize a suspension is referred to as:
A. salting out effect
B. flocculating a suspension
C. deflocculation
D. adsorption bridging

84. According to the Schulze-Hardy Rule, Which of the following ions should be the most effective in
coagulating a colloid?
A. Sodium ion
B. Calcium ion
C. Aluminum ion
D. Chloride ion

Copyright © 2000-2014 TIPS Inc. Unauthorized reproduction of this manual is prohibited. This manual is being used
during review sessions conducted by PharmacyPrep. 39-13
PharmacyPREP.Com Physical Pharmacy
85. Which of the following is associated with the stability of flocculated suspension?
A. Primary minimum
B. Van der Waal’s attraction
C. Electrostatic repulsion
D. Secondary minimum

86. The ability of association colloids to enhance the solubility compounds which are normally
insoluble in the dispersion medium is referred to as:
A. micellization
B. flocculation
C. solubilization
D. stabilization

87. The protective effect of a colloid is expressed in terms of:


A. Schulze-hardy Rule
B. Gold number
C. Lyotropic series
D. Sedimentation volume

88. A substance dissolves to the extent of 30 mg/mL. The compendia designation of the extent of
solubility would be:
A. freely soluble
B. soluble
C. sparingly
D. slightly soluble

89. Which of the following solvents is considered to be a semipolar solvent?


A. Water
B. Methyl alcohol
C. Acetone
D. Carbon tetrachloride

89. The miscibility of acetone with water is due primarily to:


A. Hydration
B. Hydrogen bond formation
C. Ionization involving covalent bonds
D. Van der Waal’s forces

90. According to the Noyes-Whitney equation, which of the following statements is not true?
A. The greater the difference in concentration across the diffusion layer the faster the
dissolution rate.
B. Decreasing surface area increases dissolution rate.
C. The greater the solubility, the greater the dissolution rate.

Copyright © 2000-2014 TIPS Inc. Unauthorized reproduction of this manual is prohibited. This manual is being used
during review sessions conducted by PharmacyPrep. 39-14
PharmacyPREP.Com Physical Pharmacy
D. Agitation will generally increase dissolution rate.

91. When the rate of reaction is proportional to the first power of concentration of one reactant, the
reaction order is said to be:
A. first order
B. second order
C. zero order
D. any of the above

92. The decay of radioisotopes is an example of which of the following reaction orders?
A. first order
B. second order
C. zero order
D. none of the above

93. If the reactant in a first order reaction is disappearing a the rate of 23 percent in an hour, the
half-life of the reaction is:
A. 2 hours
B. 3 hours
C. 4.5 hours
D. 6 hours

94. If the half-life of a reaction which is first order is 4 hours, what percent of the reactant will
remain unreacted after 12 hours?
A. 12.5 percent
B. 25 percent
C. 75 percent
D. 87.5 percent

95. If one of the reactants in a second order reaction is kept at a large concentration compared to
the other reactant, the reaction is said to be:
a) First order
b) Zero order
c) pseudo first order
d) pseudo second order

96. The esterification of acetic acid with ethanol is a second order reaction of the reaction is
conducted in water. If the reaction were conducted in ethanol, the reaction order would be:
A. first order
B. pseudo first order
C. zero order
D. pseudo second order

Copyright © 2000-2014 TIPS Inc. Unauthorized reproduction of this manual is prohibited. This manual is being used
during review sessions conducted by PharmacyPrep. 39-15
PharmacyPREP.Com Physical Pharmacy
97. The absorption of a low solubility drug from the gastro intestinal tract will follow which of the
following order of kinetics?
A. zero order
B. first order
C. second order
D. Does not follow reaction kinetics
E.

98. The energy which must be absorbed by reactant molecules before they can rearrange to form
end products is called:
A. Bailsman factor
B. Frequency factor
C. Energy of reactivity
D. Energy of activation

99. The catalytic effect which depends on pH is referred to as:


A. specific acid-base catalysis
B. general acid-base catalysis
C. heterogenous catalysis
D. nonspecific catalysis

100. The rate constant k for first order reactions may be defined as:
A. The fraction of reactant present undergoing reaction per unit time.
B. The inverse relationship of rate with concentration of reactant.
C. Variation of reaction rate with half-life of the reaction.
D. The time required for half the molecules present to react.

101. If the rheogram for a solution of a drug is a straight line which passes through the origin, the
system is said to be:
A. plastic
B. pseudoplastic
C. Newtonian
D. dilettante

102. Which of the following flow systems shows a yield value?


A. plastic
B. pseudo plastic
C. Diletante
D. Newtonian

103. 70 gm of Ferric chloride in 100ml of water. Calculate the percent weight per volume (w/v).

A. 70% w/v

B. 30% w/v

Copyright © 2000-2014 TIPS Inc. Unauthorized reproduction of this manual is prohibited. This manual is being used
during review sessions conducted by PharmacyPrep. 39-16
PharmacyPREP.Com Physical Pharmacy
C. 7% w/v

D. None of the above

104. 10 gm of ZnO was dissolved in 100ml of glycerin to make 20% w/v (Density of glycerin is 1.2).
Find w/w of ZnO in glycerine.

A. 7.7% w/w

B. 14.4% w/w

C. 20% w/w

D. None of the above

105. If percent w/w of Zinc chloride (ZnCl2) inj. is 7% w/w in water. If the inj. contains 10ml volume,
what is the wt. of Zinc chloride (ZnCl2).

A. 1gm

B. 0.75 gm

C. 0.50 gm

D. 0.250 gm

106. If you incorporate 20 g (80% active ingredient) of a compound into 100 g of Aquaphor, what is
the new % w/w of the active ingredient in the final compound?
A. 8.0%
B. 13.3%
C. 16.0%
D. 16.7%
E. 20%
107. How many grams of opium containing 15% of morphine, and how many grams of lactose should
be used to prepare 150 g of powder containing 10% w/w morphine?
A. 10 g opium and 140 g lactose
B. 15 g opium and 135 g lactose
C. 50 g opium and 100 g lactose
D. 100 g opium and 50 g lactose
E. 135 g opium and 15 g lactose

Copyright © 2000-2014 TIPS Inc. Unauthorized reproduction of this manual is prohibited. This manual is being used
during review sessions conducted by PharmacyPrep. 39-17
PharmacyPREP.Com Physical Pharmacy
CORRECT ANSWERS
PHARMACEUTICS
1. C
Tips: Buffer salts are added to many drug solutions to maintain the formulation at optimum pH. It is
formed from a weak acid or weak base and its salts. Buffers solution will be mainly altered if suffer
addition of a strong acid or a strong base to the solution.
2. B
Tips: water for injection USP is the most common used vehicle for parenteral drugs, however Water
for injection and bacteriostatic water for injection are also used.
3. E
Tips: Lotions are only used topically.
4. C
Tips: Emulsions are considered two-phase system; if the water is the internal phase, the emulsion is
classified as W/O and if the water is the external phase, the emulsion is classified as O/W.
5. C
Tips: Emulsifying agents are any compound that lower the interfacial tension and forms a film at the
interface stabilizing emulsions. Acacia and methyl cellulose are some of the most common used
emulsifying agents and they can be classified as natural or synthetic emulsifying agents.
6. B
Tips: Pulverization by intervention is a widely used pharmaceutics technique in which a substance is
reduced and subdivided with an additional material (solvent) that is easily removed after
pulverization.
7. C
Tips: Trituration is a pharmaceutics technique in which the substance is reduced to small particles by
rubbing it in a mortar with a pestle. Trituration also describes the process by which fine powders are
intimately mixed in a mortar.
8. A
Tips: The most common diluents used in pharmacy industry include kaolin, lactose, mannitol,
cellulose, powdered sugar, starch and calcium phosphate.
9. D
Tips: Lubricants are used in tablet formulation to reduce the friction that occurs between the walls of
the tablet and the walls of the die cavity when tablet is ejected.

Copyright © 2000-2014 TIPS Inc. Unauthorized reproduction of this manual is prohibited. This manual is being used
during review sessions conducted by PharmacyPrep. 39-18
PharmacyPREP.Com Physical Pharmacy
10. C
Tips: Sugar coated should be swallowed because the coated sugar provides a barrier to a drug’s
objectionable taste and smell, or to improve the appearance of the drug.
11. D
Tips: Capping is a manufacture tablet processing problem characterized by the partial or complete
separation of the top or bottom crown from the main body of the tablet.
12. C
Tips: Manufacture common tablet processing problems involve mottling, sticking and picking.
13. E
Tips: Talc, magnesium stearate and calcium stearate are commonly lubricant used in tablet
manufacture
14. A
Tips: Chlorofluorocarbons-CFC is propellant used in the past in aerosol products, it was put out of
market due to depletation of the ozone layer. Nowadays the most used one is Hydro fluorocarbons-
HFC.
15. D
Tips: Henderson-Hasselbalch equations used to mathematically calculate the pH of a buffer solution.
16. B
Tips: Arrhenius equation is used to calculate the stability of a drug at room temperature.
17. D
Tips: The HLB-Hydrophilic-Lipophilic Balance is applied to surfactant agent
18. A
Tips: Agent with HLB value range of 0-3 is known as antifoaming agent.
19. D
Tips: Agent classified as oil in water emulsifying agent has its HLB value ranged between 8-18.
20. A
Tips: Cocoa butter is widely used as suppository base due to its polymorph activity.
21. E

Copyright © 2000-2014 TIPS Inc. Unauthorized reproduction of this manual is prohibited. This manual is being used
during review sessions conducted by PharmacyPrep. 39-19
PharmacyPREP.Com Physical Pharmacy
Tips: Polymorph drug has two different physical form depending the currently temperature that they
are present, normally melting in high temperatures (>30C), and keeping solid in low temperature
(<10C). Theobroma oil or cocoa butter is a classic example of polymorph drug
22. A
Tips: Trituration is a widely used pharmaceutics technique in which a substance is reduced and
subdivided with an additional material (solvent) that is easily removed after trituration
23. A
Tips: Increasing viscosity of an ophthalmic preparation increases its bioavailability.
24. B
Tips: Autoclaving is the sterilization process for steam sterilization.
25. C
Tips: Wet granulation tablet manufacturing method cannot be used in the preparation of Acetyl
Salicylic Acid because it is easily hydrolyzed during tablet manufacturing.
26. E
Tips: Flocculated suspension upon sedimentation will produce a large volume sedimentation and
clear boundary when particles settle.
27. A
Tips: Methylparaben is commonly used in ophthalmic solution as preservative.
28. E
Tips: Emollients are not used as excipients in tablet formulation
29. E
Tips: Deliquescence, the condition of become moist, occurs when the vapor pressure of the
saturated aqueous solution is less than the vapor pressure of the water in the ambient air as result of
the absorption of water from the air
30. A
Tips: Hygroscopic is the name given to substances that have the capacity of absorbing moisture. They
may be classified as anhydrates or hydrates.
31. B
Tips: Van Der Waal’s forces are permanent dipole interactions.
32. E

Copyright © 2000-2014 TIPS Inc. Unauthorized reproduction of this manual is prohibited. This manual is being used
during review sessions conducted by PharmacyPrep. 39-20
PharmacyPREP.Com Physical Pharmacy
Tips: Hydrogen bound occurs between strong electronegative elements such as F, N, O, S and Cl.
They may be classified in INTRA when the hydrogen bound is within the molecule and INTER when
the hydrogen bound is in between of two molecules
33. A
Tips: Ethers are groups having no hydrogen molecules therefore cannot form hydrogen bound
between them.
34. D
Tips: Eutectic Mixture is the mixture of two or more substances that melt at the lowest possible
temperature and the eutectic point is the melting point of a eutectic mixture
35. E
Tips: Partition Coefficient measures the solubility of a substance in octane and water specifying how
hydrophilic and hydrophobic a substance is.
36. B
Tips: Adsorption is a process of separating a substance from one phase accompanied by its
accumulation or concentration at the surface of another.
37. C
Tips: Enteric coated tablet are hardly coated tablet resistant to gastric acid, and suffer disintegration
only when it reaches the small intestine.
38. A
Tips: Binders are used in tablet formulation to make the adhesion of the powdered drug to the
inactive drug. Cellulose is an example of classic binder that can be use in both wet and dry tablet.
39. C
Tips: Efflorescence is the process of moisture lost.
40. C
Tips: Flocculated agents are widely used in suspensions to enhance the particles dispensability of a
suspension providing “no cake”, and making the suspension particles very easy to redisperse by
shaking the suspension. Flocculated agents are used only in suspensions
41. B
Tips: The most appropriated auxiliary label to use in dispensing a suspension is “Shake well before
use”
42. E

Copyright © 2000-2014 TIPS Inc. Unauthorized reproduction of this manual is prohibited. This manual is being used
during review sessions conducted by PharmacyPrep. 39-21
PharmacyPREP.Com Physical Pharmacy
Tips: Antioxidants are added to drug formulation in order to avoid degradation. All of the following
are antioxidants; sodium benzoate is an example of preservative not antioxidant.
43. D
Tips: Tocopherol also called vitamin E is widely used in pharmacy industry as antioxidant. α-
Tocopherol is the strongest tocopherol antioxidant.
44. C
Tips: Cool places include rooms wit temperature ranging between 8 to 15°C.
45. D
Tips: Vapor pressure is a colligative propriety.
46. A
Tips: Boric acid is widely used in ophthalmic solutions as buffer agent due to its capacity to adjust the
isotonicity
47. B
Tips: Astringent work by coagulating protein when applied to skin or mucous membrane. The protein
precipitate forming a protective coat, allowing new tissue to regenerate underneath, Burrow’s
solution is a classic example of astringent.
48. C
Tips: Propylene glycol is mainly used as co-solvent in parenteral drug formulation.
49. A
Tips: Sodium stearate is stiffening and emulsifying anionic agent used in pharmaceutical
preparations.
50. C
Tips: 50- Compared to the sublingual tablet, nitroglycerin ointment: Provides a more prolonged
effect
51. E
Tips: The dissolution rate of nitrofurantoin tablets may be increased by micronization of the
nitrofurantoin powder, decreasing viscosity of the dissolution medium, heating the solution medium
and mechanical agitation
52. D
Tips: Benzocaine with caffeine forms a complex molecule.
53. C
Comments: Citric acid does not act as reducing agent.

Copyright © 2000-2014 TIPS Inc. Unauthorized reproduction of this manual is prohibited. This manual is being used
during review sessions conducted by PharmacyPrep. 39-22
PharmacyPREP.Com Physical Pharmacy
54. D
Tips: Coating tablets has a long onset of action due the presence of the coated on its formulation,
therefore the coat decrease the drug’s release rate.
55. B
Tips: An ophthalmic solution must be isotonic to avoid eye irritation.
56. E
Tips: Mineral oil exerts many different functions in pharmaceutical manufacturing including
antioxidant, laxative and lubricant proprieties.
57. B
Tips: Wetting agents are normally used in pharmaceutical preparations to decrease tension between
solid and liquid.
58. E
Tips: Agent with HLB value ranging between 14-18 is known as solubilizing agent.
59. E
Tips: The “4” dispensing valour in the preparation of ZnSO4 suppositories means that 200mg of
ZnSO4 will displace 50mg of cocoa butter.
60. A
Tips: Levigation is the technique used by addition of a liquid to the substance as an intervening agent
to reduce the particle size of drug by grinding together.
61. E
Tips: Very soluble substances are called for substances that require only one part of solvent to
dissolve one part of solute (substance).
62. C
Tips: Tocopherol also known as vitamin E is a fat-soluble agent therefore cannot be used in aqueous
preparation.
63. B
Tips: Hydrogen peroxide is used topically on skin in a maximum concentration of 10V to avoid burns
on skin.
64. C
Tips: Benzocaine as a local anesthetic in a topical preparation should be at least, not less than 5%.
Lesser concentration will not be effective.
65. A

Copyright © 2000-2014 TIPS Inc. Unauthorized reproduction of this manual is prohibited. This manual is being used
during review sessions conducted by PharmacyPrep. 39-23
PharmacyPREP.Com Physical Pharmacy
Tips: Papain is a proteolitic enzyme used in contact Lens to remove proteins and is available in most
of OTC contact lents washers.
66. D
67. C
68. A
69. A
70. A
71. C
72. A
Tips: Raults law describes vapour pressure.
Hasselbach-Handerson equation for ionization of drugs at pH
Michelles menten for enzyme kinetics
Noyes Whitney equation rate of dissolution
Arrhenius equation for temperature effect on degradation
73. E
74. E
75. E
76. C
77. B
78. C
79. B
80. C
81. B
82. C
83. B

Copyright © 2000-2014 TIPS Inc. Unauthorized reproduction of this manual is prohibited. This manual is being used
during review sessions conducted by PharmacyPrep. 39-24
PharmacyPREP.Com Physical Pharmacy
84. C
85. D
86. C
87. B
88. C
89. C
89. B
90. B
91. A
92. A
93. B
Tips: t1/2 = 0.693 = 0.693 = 3 hrs
K 0.230
94. A
Tips: 0.5% → 4 hrs
0.25%→ 8 hrs
0.125%→ 12 hrs
0.125% x 100 = 12.5%
95. C
96. B
97. B
Tips: All drugs follow first order depending on their conc. in blood stream & reversibly binding to
receptor. In intoxication due to saturation of receptors, it follows zero order.
98. D
99. A
100. A

Copyright © 2000-2014 TIPS Inc. Unauthorized reproduction of this manual is prohibited. This manual is being used
during review sessions conducted by PharmacyPrep. 39-25
PharmacyPREP.Com Physical Pharmacy
101. C
102. A
103. A

Sol.

% wt. per volume = wt. of solute x 100

vol. of sol.

= 70 gm x 100 = 700 = 70% w/v 1


0
100 100 +
104. A 1
2
Sol. 0

D=M 1
3
V
0
.
1.2 = x . . x = 120gm

100

w/w = 10 x 10% = 1000 = 7.7%


105. ute x 100
B
wt. of solute + wt. of solvent
S .
o . . 7% = x x 100%
l
. x + wt. of water

D=M
%
V
w .
1 = x . . x = 10 g
/
w 10ml
.
= . . 7% = ( x ) 100%

w ( x + 10)
t
. 100 x = 7x + 70

o 93 x = 70
f x = 70 = 0.75 g
s 93
o
l
106. B
i. 20 grams of compound contain 16 g of active ingredient (20 x 80%)

Copyright © 2000-2014 TIPS Inc. Unauthorized reproduction of this manual is prohibited. This manual is being used
during review sessions conducted by PharmacyPrep. 39-26
PharmacyPREP.Com Physical Pharmacy
ii. % w/w strength of active ingredient in mixture is (16/(100 + 20)) x 100 = 13.33%
107. D
Working:
i. 150 grams of powder to be produced (containing 10% morphine) are equivalent to
(10/100) x 150 = 15 g
morphine
ii. Since the original opium contains 15% morphine, 100 grams of the original opium powder
are required to give the 15 g of morphine required for the final product. The rest of the
weight (150 g – 100 g) = 50 g should be derived from lactose.
BIBLIOGRAPHIC REFERENCE
1- COMPREHENSIVE PHARMACY REVIEW – Lippincott William & Wilkins – Fourth edition
2- CPS-COMPENDIUM OF PHARMACEUTICALS AND SPECIALITIES - Canadian Pharmacist Association –
2001 edition.
th
3- MEDICAL DICTIONARY – Dorland’s illustrated – 27 edition.
4- PHARMACY PREP – Lectures series & study guide for Evaluating Examination-TIPS - 2003/2004
5- THERAPUTIC CHOICES – Canadian Pharmacist Association -Third edition
th
6- USP DI – Drug Information for the Health Care Professional–15 edition – Volume I.

Copyright © 2000-2014 TIPS Inc. Unauthorized reproduction of this manual is prohibited. This manual is being used
during review sessions conducted by PharmacyPrep. 39-27
PharmacyPrep.Com Pharmaceutical Excipients

PHARMACY PREP
PHARMACEUTICAL EXCIPIENTS
1. Additives which promote the breaking up of tablets into smaller particles are referred to as:
A. Fillers
B. Disintegrant
C. Lubricants
D. Dispersants
2. Which of the following polyols should never be used as a sweetening agent because
of its severe toxicity?
A. Sorbitol
B. Glycerol
C. Propylene glycol
D. Ethylene glycol

3. Which of the following dosage forms is a concentrated sugar or sugar substitute?


A. Elixirs B. Syrups C. Solutions D) Cerates E) Tinctures
4. Certain cellulose derivatives are employed in syrups for which of the following reasons?
A. Sweetener
B. Viscosity enhancer
C. Flavoring
D. Caloric agent
E. Preservative

5. Antioxidants are added to drug formulation in order to avoid degradation. All of the following are
antioxidants, EXCEPT
A. Tocopherol
B. Ascorbic acid
C. EDTA
D. Sodium bisulfite
E. Sodium benzoate
6. Correct statements regarding tocopherol (vitamin E) include:
I- It is an antioxidant mainly used in water-soluble formulations
II- It is also known as vitamin E
III- α-tocopherol is the strongest tocopherol antioxidant
A. I only B. III only C. I and II only D. II and III only E. All are correct
7. Boric acid is often used to adjust the isotonicity of some drug preparations because of its buffer
capacity and anti-infective proprieties. Boric acid is widely used in:

Copyright © 2000-2014 TIPS Inc. Unauthorized reproduction of this manual is prohibited. This manual is being used
during review sessions conducted by PharmacyPrep. 40-1
PharmacyPrep.Com Pharmaceutical Excipients
A. Ophthalmic solutions
B. Nasal solutions
C. Otic solutions
D. Topical formulations
E. Oral solutions
8. Sodium stearate is classified as:
A. Anionic B. Cationic C. Non-ionic D. Hydrophobic E. Amphiphilic
9. Antioxidants, which mechanistically act as reducing agent, include:
I- Ascorbic acid
II- Sodium bisulfite
III- Citric acid
A. I only B. III only C. I and II only D. II and III only E. All are correct
10. Which decrease tension between solid and liquid?
A. Solubilizing agent
B. Wetting agent
C. Hydrophilic agent
D. Emulsifying agent
E. Decreasing the particle size of the solid
11. An agent with HLB value range of 14 to 18 is known as:
A. Antifoaming agent
B. W/O emulsifying agent
C. O/W emulsifying agent
D. Wetting agent
E. Solubilizing agent
12. As per USP which of the following is NOT a sterile product?
A. Nasal sprays B. Eye drops C. water for irrigation
D. iv fluid E. im injection
13. Magnesium stearate is?
A. levigating agent
B. suspending agent
C. emulsifying agent
D. anti-adherent
E. alkalizing agent
14. Tween is?
I-Polysorbate
II-Non ionic surfactant
III-Sorbic acid ester

Copyright © 2000-2014 TIPS Inc. Unauthorized reproduction of this manual is prohibited. This manual is being used
during review sessions conducted by PharmacyPrep. 40-2
PharmacyPrep.Com Pharmaceutical Excipients
A. I only B. III only C. I and II only D. II and III only E. All are correct
15. Which of the following is aerosol propellant is safe for environment?
A. Chloroflouro carbon (CFC)
B. Hydroflouro alkenes (HFA)
C. Chloroflouro bromo carbons (CFBrC)
D. Chloroflouro alkenes (CFA)
E. Propane
16. Magnesium stearate is the most commonly used for tablets preparation as?
A. lubricant B. Glidant C. Diluent D. Disintegrent E. Sticking agent
17. Diluents in pharmaceutical preparation, is used as?
A. to increase bulk of tablets
B. To increase flow of granules
C. To increase disintegration of solids
D. To increase antiadherent in tablet manufacturing
E. To increase lubrication of tablets
18. Which of the following is fillers in tablet manufacturing exipient, that can be interfere with
tetracyclin absorption?
A. Starch B. Calcium salts C. Lactose D. Glucose E. All of the above
19. If tablet which has more binder will present the following problems?
I-Excessive hardness
II-capping
III-friability problem
A. I only B. III only C. I and II only D. II and III only E. All are correct
20. What is incorrect about binders in tablet manufacturing?
A. Binder to cause the adhesion of the powdered drug and inactive ingredients
B. Excessive binder will present capping problem
C. Excessive binder will present excessive hardness
D. Corn starch is used as binder in tablet manufacturing
E. Magnesium sterate is used as binder in tablet manufacturing
21. Which is Not a method for blending the powder?
A. Milling B. Trituration C. Sifting D. Spatulation
E. None of the above
22. In a tablet formulation, Starch is used as:
I-Binder II-Glidant III-Lubricant
A. I only B. III only C. I and II only D. II and III only E. All are correct

Copyright © 2000-2014 TIPS Inc. Unauthorized reproduction of this manual is prohibited. This manual is being used
during review sessions conducted by PharmacyPrep. 40-3
PharmacyPrep.Com Pharmaceutical Excipients
23. What is Not true about capsule formulation?
A. Hard gelatine capsule can encapsulate between 65 mg to 1g of the powdered drug
B. If drug dose is not enough to fill the capsule, diluents is added
C. Lubricant is added to increase the flow property of the powder
D. Wetting agents may be added to enhance drug dissolution
E. Glidant may be added to reduce friction among the particles
24. Quaternary charged ammonium sulphate surfactants are?
A) Negatively charged B) Positive charged C) Zwitter ion D) No charge E) Non ionic
25. A liquid whose viscosity is increased when a agent is combined is called as:
A. Dilatant
B. Surfactant
C. Wetting agent
D. Emulsion
Ans.: A
26. Ophthalmic solutions must be:
A.Hypotonic
B.Isotonic
C.Hypertonic
D.Osmotic
27. If the particle size of the dispersed phase of a two-phase system is 50, the system is:
A. true solution
B. a colloidal dispersion
C. a coarse dispersion
D. a solid solution
28. The monolayer theory of adsorption on a solid surface is attributed to”
A. Freudlich
B. Einstein
C. Langmuir
D. Derjaquin
29. The chromatographic procedure which involves metathesis or double decomposition is:
A. partition
B. adsorption
C. vapor phase
D. ion exchange
30. A foam is an example of which type of disperse system?
A. liquid in liquid
B. gas in liquid

Copyright © 2000-2014 TIPS Inc. Unauthorized reproduction of this manual is prohibited. This manual is being used
during review sessions conducted by PharmacyPrep. 40-4
PharmacyPrep.Com Pharmaceutical Excipients
C. solid in gas
D. liquid in gas
31. A hydrophilic semisolid system consisting of a dispersion of small inorganic particles or larger
organic molecules enclosing and interpenetrated by a liquid defines which of the following?
A. Soil
B. Gel
C. Suspension
D. Colloid
32. A colloidal system in which the colloidal particles show little attraction for the dispersion medium
defines which of the following type colloids?
A. Lyophilic
B. Lyophobic
C. Hydrophilic
D. Association
33. Which of the following dosage forms is likely to show a yield value?
A. Ointments
B. Flocculated suspensions
C. Gels
D. All of the above
34. Which of the following dosage forms shows a high degree of elasticity?
A. Emulsions
B. Solutions
C. Gels
D. Elixirs
35. In which of the following systems does the rheogram plot not pass through the origin?
A. Pseudo plastic
B. Newtonian flow
C. Plastic
D. Dilatant
36. A concentrated suspension is likely to show which of the following to as “shear thinning”?
A. Plastic
B. Newtonian
C. Pseudo plastic
D. All of the above
37. A concentrated suspension is likely to show which of the following flow behaviours?
A.Thixotropy

Copyright © 2000-2014 TIPS Inc. Unauthorized reproduction of this manual is prohibited. This manual is being used
during review sessions conducted by PharmacyPrep. 40-5
PharmacyPrep.Com Pharmaceutical Excipients
B. Newtonian flow
C. Plastic flow
D. Dilatant flow
38. The comparatively slow recovery on standing of the consistency of a material lost through
shearing defines:
A. Rheopexy
B. Dilatancy
C. Thixotropy
D. Plasticity
39) An increase in apparent viscosity or the formation of a yield value resulting from gentle agitation
defines:
A. rheopexy
B. dilatancy
C. thixotropy
D. plasticity
40) The solubilization of mercuric iodide by adding potassium iodide to the solution is due to:
A. salt formation
B. dipole-dipole interaction
C. complex formation
D. salting-in effect
41) Autoclaving is sterilization process of:
A. Dry heat sterilization
B. Steam Sterilization
C. Filtration
D. Gas sterilization
42) Which of the following tablet manufacturing method is not suitable for aspirin tablet
formulations:
A. Wet granulation
B. Dry granulations
C. Direct compression method
D. None of the above
43) HLB value for solubilizing agent
A.1-4
B.5-6
C.6-10
D-10-18

Copyright © 2000-2014 TIPS Inc. Unauthorized reproduction of this manual is prohibited. This manual is being used
during review sessions conducted by PharmacyPrep. 40-6
PharmacyPrep.Com Pharmaceutical Excipients
44) HLB value for O/W emulsifying agent:
A.4-8
B.1-4
C.8-18
D.1-12
45) Which of the following terms indicates a loss of moisture? A. Deliquescence B. Efflorescence C.
Hygroscopic D. Polymorphism
46) Compared to sublingual tablet, nitroglycerin ointment :
A) Provides only local effect
B.Is a more stable dosage form
C) Provides more prolong effect.
D) Is more rapidly absorbed
47) What is advantage of enteric coated tablets?
A. taste better
B. does not hurt stomach
C. more effective
D. cheaper than drugs
48) What is friability?
A. Ability of the tablet to withstand abrasion in packing, handling and shipping
B. ability of the tablet to dissolve
C. ability of the tablet to disintegrate
D. none
49) What size of capsule is large?
A. 00
B. 1
C. 2
D.5
50) What is incorrect about hard gel capsule?
A. hard gel capsule can filled with powder
B. hard gel capsule can filled with granules
C. hard gel capsule can filled with liquid
D. hard gel capsule can be breakable

Copyright © 2000-2014 TIPS Inc. Unauthorized reproduction of this manual is prohibited. This manual is being used
during review sessions conducted by PharmacyPrep. 40-7
PharmacyPrep.Com Pharmaceutical Excipients
51) Fish oil capsules are?
A. hard gel
B. soft gel
C. hard and soft gel capsules
D. none
52) What are the example of levigating agents?
A. glycerine
B. mineral oil
C. ethanol
D. A and B
53) What label should be given to suspension?
A) prime before use
B. shake well before use
C. do not shake
D. shake gently
54) A substance that absorbs moisture from the air is termed as?
A) efflorescent
B) hygroscopic
C) eutectic
D) effervescent
55) All products should be label as for external use only, except?
A.ointment
B.lotions
C.astringents
D.inhalers
56. Which of the following is requiring to store in tightly closed container?
A. suspension
B. syrups
C. spirits
D. creams
57). Which of the following preparations may contain sugar?
A. tincture
B. solutions
C. spirit
D. syrup
58) What extemporaneous products expiration date is a label do not use after 6 months?
A. syrup

Copyright © 2000-2014 TIPS Inc. Unauthorized reproduction of this manual is prohibited. This manual is being used
during review sessions conducted by PharmacyPrep. 40-8
PharmacyPrep.Com Pharmaceutical Excipients
B. suspension
C. cream
D. ointments
59) What is not recommended against diaper rash management?
A. 0.5% hydrocortisone
B. petrolatum gel
C. Zinc oxide
D. alcohol wipes
60) All of the following are applied on skin, except?
A. lotion
B. transdermal patch
C. cream
D. elixir
61) Which of the following require sterile preparations?
A. nasal spray
B. ophthalmic drops
C. inhaler
D. sublingual spray

Copyright © 2000-2014 TIPS Inc. Unauthorized reproduction of this manual is prohibited. This manual is being used
during review sessions conducted by PharmacyPrep. 40-9
PharmacyPrep.Com Pharmaceutical Excipients
ANSWERS:
1. B
2. D
3. B
4. B
5. E
6. D
7. A
8. A
9. C
10. B
11. E
12. A
13. D
14. C
15. B
16. A
17. A
18. B
19. C
20. E
21. A

Copyright © 2000-2014 TIPS Inc. Unauthorized reproduction of this manual is prohibited. This manual is being used
during review sessions conducted by PharmacyPrep. 40-10
PharmacyPrep.Com Pharmaceutical Excipients
22. C
Tips: starch can be used as binder, filler, glidant and anti adherent
23. C & E
24. B
25. A
26. B
27. C
28. C
29. C
30. B
31. B
32. B
33. D
34. C
35. C
36. D
37. D
38. C
39. A
40. C
41. B
42. A
43. D
44. C
45. N
46. B
47. B
48. A
49. A
50. C
51. B
52. D
53. B
54. B
55. D
56. C
57. D
58. C AND D
59. D
60. D
61. B

Copyright © 2000-2014 TIPS Inc. Unauthorized reproduction of this manual is prohibited. This manual is being used
during review sessions conducted by PharmacyPrep. 40-11
www.pharmacyprep.com

PHARMACY PREP
RHEOLOGY
1. The diagram represents the following flow system:
A. Pseudo plastic flow B. Newtonian flow C. Thixotropy D. Dilatant flow E-Antithixotropy
2. The diagram represents which of the following flow system:
A. Pseudo plastic flow B. Newtonian flow C. Thixotropy D. Dilatants flow E. Antithixotropy
3) The diagram represents which of the following flow system:
A. Pseudo plastic flow
B. Newtonian flow
C. Thixotropy
D. Dilatants flow
E. Antithixotropy

Copyright © 2000-2014 TIPS Inc. Unauthorized reproduction of this manual is prohibited. This manual is being used
during review sessions conducted by PharmacyPrep. 41-1
www.pharmacyprep.com
Tips: substance “B” has yield value it is plastic flow. Substance “A” has begins from origin and it is
pseudoplastic or thixotropy.
4. Which of the following flow systems shows a yield value?
A. Thixotropy
B. Pseudoplastic flow
C. Dilatants flow
D. Newtonian flow
E. Plastic flow
5- Which of the following dosage forms is likely to show a yield value?
I-Ointments II-Flocculated suspensions III-Gels
A. I only B. III only C. I and II only D. II and III only E. All of the above
6. All are the important reasons for coating tablets: EXCEPT
A. To mask the odor of drug
B. to improve the flow of drug
C. To increase release rate
D. To protect from stomach acid
E. to mask taste of drug
7. Loss of moisture is:
A. Efflorescence
B. Deliquescence
C. Hygroscopic
D. Polymorphism
E. condensation
8) Which of the following would be referred to as a Newtonian type of flow?
A. Pseudo plastic
B. Dilatants
C. Plastic
D. None of the above

9. Which of the following is azo dye?


A. Lysol
B. pyridium
C. Argyrol
D. Nitrofurazone
E. Hexachlorphene
10. What is incorrect about thixotropy?
A. Gel to solution
B. Solution to gel

Copyright © 2000-2014 TIPS Inc. Unauthorized reproduction of this manual is prohibited. This manual is being used
during review sessions conducted by PharmacyPrep. 41-2
www.pharmacyprep.com
C. Viscosity decreased
D. Flow ability increased
E. None of the above
11. Gel to solution is?
A. Antithixotropy
B. thixotropy
C. dilatants flow
D. Newtonian flow
E. Plastic flow
12. Principles of thixotropic is applied on pharmaceutical products, except?
A. suspension
B. emulsion
C. lotion
D. Parenteral prep
E. Elixir
13. A liquid whose viscosity is increased when a agent is combined is called as:
A. Dilatant
B. Surfactant
C. Wetting agent
D. Emulsion
14. Ophthalmic solutions must be:
A.Hypotonic
B.Isotonic
C.Hypertonic
D.Osmotic
15. If the particle size of the dispersed phase of a two-phase system is 50, the system is:
A. true solution
B. a colloidal dispersion
C. a coarse dispersion
D. a solid solution
16. The monolayer theory of adsorption on a solid surface is attributed to”
A. Freudlich
B. Einstein
C. Langmuir
D. Derjaquin

Copyright © 2000-2014 TIPS Inc. Unauthorized reproduction of this manual is prohibited. This manual is being used
during review sessions conducted by PharmacyPrep. 41-3
www.pharmacyprep.com
17. The chromatographic procedure which involves metathesis or double
decomposition is:
A. partition
B. adsorption
C. vapor phase
D. ion exchange
18. A foam is an example of which type of disperse system?
A. liquid in liquid
B. gas in liquid
C. solid in gas
D. liquid in gas
19. In which of the following systems does the rheogram plot not pass through the origin?
A. Pseudoplastic
B. Newtonian flow
C. Plastic
D. Dilatant
20. A concentrated suspension is likely to show which of the following to as “shear thinning”?
A. Plastic
B. Newtonian
C. Pseudoplastic
D. All of the above
21. A concentrated suspension is likely to show which of the following flow behaviours?
A.Thixotropy
B. Newtonian flow
C. Plastic flow
D. Dilatant flow
22. The comparatively slow recovery on standing of the consistency of a material lost through
shearing defines:
A. Rheopexy
B. Dilatancy
C. Thixotropy
D. Plasticity
23. An increase in apparent viscosity or the formation of a yield value resulting from gentle agitation
defines:
A. rheopexy
B. dilatancy

Copyright © 2000-2014 TIPS Inc. Unauthorized reproduction of this manual is prohibited. This manual is being used
during review sessions conducted by PharmacyPrep. 41-4
www.pharmacyprep.com
C. thixotropy
D. plasticity
24. Which of the following dosage forms is likely to show a yield value?
A. Ointments
B. Flocculated suspensions
C. Gels
D. All of the above
25. Which of the following dosage forms shows a high degree of elasticity?
A. Emulsions
B. Solutions
C. Gels
D. Elixirs
ANSWERS:
1. B
2. A
3. C
4. E
5. E
6. C
7. A
8. D
Tips: Newtonian flow is dilute solutions, which obeys Newton’s Law.
9. B
10. B
Tips: solution to gel is antithixotropys
11. B
12. D
13. A

Copyright © 2000-2014 TIPS Inc. Unauthorized reproduction of this manual is prohibited. This manual is being used
during review sessions conducted by PharmacyPrep. 41-5
www.pharmacyprep.com
14. B
15. C
16. C
17. D
18. B
19. C
20. D
21. D
22. C
23. A
24. D
25. C

Copyright © 2000-2014 TIPS Inc. Unauthorized reproduction of this manual is prohibited. This manual is being used
during review sessions conducted by PharmacyPrep. 41-6
www.pharmacyprep.com

PHARMACY PREP
PHARMACEUTICAL DOSAGE FORMS
1. An enteric coated tablet or beads are designed to:
I-Pass through the stomach and break up in the intestine.
II-To reduce stomach irritation
III-Slow disintegration and sustain release dosage forms
A. I only B. III only C. I and II only D. II and III only E. All of the above
2. All of the following are the purpose of coating tablets EXCEPT:
A. Protection from by atmospheric oxygen or humidity
B. To provide desired release pattern for drug substance after administration.
C. To mask the taste and odour of the drug.
D. To allow ease in swallowing tablets
E. To increase the potency of drugs.
3. Methylparaben and propylparaben are:
A. Antifungal drugs
B. Antibacterial preservatives
C. Antifungal preservatives
D. Increase shelf life of drugs
E. increase stability of drugs
4. Aspirin tablets can be manufactured by the following methods:
I-Wet granulation methods
II-Dry granulation methods
III-Direct compression methods
A. I only B. III only C. I and II only D. II and III only E. All of the above
5. Picking is the problem in the manufacture of:
A. cream B. Lotions C. Tablets D. Capsules E. suspensions
6. Breaking is the problem in the manufacture of:
A-Cream B-Lotions C-Tablets D-Capsules E-suspensions
7. Lyophilization (freeze dry) method used for:
A. Drying wet drugs to powder
B. Powder sterilization methods
C. Hormone sterilization methods
D. Sublimation methods used in drying wet powder
E. Sterilization method used for thermo labile products

Copyright © 2000-2014 TIPS Inc. Unauthorized reproduction of this manual is prohibited. This manual is being used
during review sessions conducted by PharmacyPrep. 42-1
www.pharmacyprep.com
8. Which of the following indicated loss of carbon dioxide:
A. Deliquescent B. Efflorescent C. Effervescent
D. Hydroscopic E. Hygroscopic
9. In the presence of water sodium bicarbonate reacts with acid to liberate carbon dioxide:
A. Deliquescent B. Efflorescent C. Effervescent
D. Hydroscopic E. Hygroscopic
10. Upon combination of powder A and powder B, the mixture melting point is lower than powder A
and Powder B:
A. Efflorescent
B. Eutectic point
C. Hydroscopic mixture
D. Deliquescent mixture
E. Low melting point mixture
11. Flocculating agents:
I-Increase dispersion
II-Decrease sedimentation
III-Increase volume
A. I only B. III only C. I and II only D. II and III only E. All of the above
12. Veegum (Suspending agents)
I-decrease viscosity II-Increase viscosity III-Decrease sedimentation
A. I only B. III only C. I and II only D. II and III only E. All of the above
13. The flow ability of substance can be described by Thixotropy:
A. Increases gel viscosity
B. Increases fluidity
C. Decreases viscosity
D. Increase sedimentation
E. Decrease sedimentation
14. The primary pharmaceutical use of veegum is as a:
A. Emulsifying agent
B. Surfactant
C. Suspending agent
D. sweetener
E. Flocculating agent
15. The primary pharmaceutical use of surfactant is as a:
A. Emulsifying agent
B. Surfactant

Copyright © 2000-2014 TIPS Inc. Unauthorized reproduction of this manual is prohibited. This manual is being used
during review sessions conducted by PharmacyPrep. 42-2
www.pharmacyprep.com
C. Suspending agent
D. sweetener
E. Flocculating agent
16. Spans and tweens are:
A. Non ionic surfactants
B. Anionic surfactants
C. Cationic surfactants
D. Poly oxyalkylene derivatives
E. Suspending agents
17. Parenteral solution should be free from:
I-Microorganisms
II-Microparticulates
III-Pyrogen
A. I only B. III only C. I and II only D. II and III only E. All of the above
18. HEPA filters used in sterile preparations filtration:
I-Microorganisms
II-Particulates
III-Pyrogens
A. I only B. III only C. I and II only D. II and III only E. All of the above
19. All non-sterile surface areas in laminar airflow hood should be swabbed with:
I-70% isopropyl alcohol
II-70% Ethyl alcohol
III-Chlorhexidine
A. I only B. III only C. I and II only D. II and III only E. All of the above
20. Acacia is classified as:
A. suspending agent
B. natural emulsifying agent
C. Semi synthetic emulsifying agent
D. Flocculating agent
E. tablet filler
21) Rectal suppositories intended for adult use, approximately weigh.
A. 10 g B. 5 g C. 2 g D. 1 g E. 0.5 g
22) In a dispensing pharmacy, pharmacist usually mixes active powder with large amount of diluents
by:
A. Levigation
B. Geometrical dilution
C. Trituration

Copyright © 2000-2014 TIPS Inc. Unauthorized reproduction of this manual is prohibited. This manual is being used
during review sessions conducted by PharmacyPrep. 42-3
www.pharmacyprep.com
D. Sifting
E. Pulverization
23. The shell of soft gelatine capsules may made by addition of plastic like substance
A. Polyethylene glycol (PEG)
B. Povidone
C. Lactose
D. Sorbitol
E. methyl cellulose
24. Which is the smallest size of capsule:
A. 5 B. 2 C. 00 D. 1 E. 000
25. In extemporaneous preparations of suspensions, levigation is used:
A. Reduce particle size
B. Reduce viscosity
C. Increase particle size
D. Increase sedimentation rate
E. Reduce sedimentation rate
26. What are the suitable forms of water can be used in parenteral preparations
I-Sterile water for injection USP
II-Water for injection USP
III-Purified water USP
A. I only B. III only C. I and II only D. II and III only E. All of the above
27. Which of the following is the output of salbutamol Inhalation aerosol
I-Dry mist II-Wet mist III-Stable foam
A. I only B. III only C. I and II only D. II and III only E. All of the above
28. In the following formulations
Phenol 0.5%
Cimetidine HCl
Water for injection USP
What is the role of water for injection USP
A. Preservative B. Solvent C. Co-solvent D. Diluents E. Disinfectant
29. Absolute bioavailability is the result of comparing
A. oral dosage from and IV dosage form
B. oral bioavailability from and sublingual form
C. oral bioavailability from and IM dosage form
D. oral bioavailability from and rectal form

Copyright © 2000-2014 TIPS Inc. Unauthorized reproduction of this manual is prohibited. This manual is being used
during review sessions conducted by PharmacyPrep. 42-4
www.pharmacyprep.com
30. Which of the following labels would be the most appropriate to be used in your pharmacy when
dispensing a suspension
A. “Take with food”
B. “Shake well before use”
C. “Take with lots of fluids”
D. “Rinse mouth after use”
31. Antioxidants are added to drug formulation in order to avoid degradation. All of the following are
antioxidants, EXCEPT
A. Tocopherol
B. Ascorbic acid
C. EDTA
D. Sodium bisulfite
E. Sodium benzoate
32. Correct statements regarding tocopherol (vitamin E) include:
I- It is an antioxidant mainly used in water-soluble formulations
II- It is also known as vitamin E
III- α-tocopherol is the strongest tocopherol antioxidant
A. I only B. III only C. I and II only D. II and III only E. All of the above
33. You received a new drug in your pharmacy and the manufacture instructions tell you to keep this
drug in a cool place. Which temperature should you keep this drug?
A. <0°C B. 2 to 8°C C. 8 to 15°C D. 15 to 30°C
E. >30°C
34. Correct statements regarding vapor pressure may include:
I- Vapor pressure is not a Colligative propriety
II- Increasing solute, decrease the vapor pressure
III- increasing solute, increase the boiling point
A. I only B. III only C. I and II only D. II and III only E. All of the above
35. Boric acid is often used to adjust the isotonicity of some drug preparations because of its buffer
capacity and anti-infective proprieties. Boric acid is widely used in:
A. Ophthalmic solutions
B. Nasal solutions
C. Otic solutions
D. Topical formulations
E. Oral solutions
36. Burrow’s solution is aluminum acetate is categorized as:
A. Colloidion B. Astringent C. Adsorbent D. Absorbent
E. Surfactant

Copyright © 2000-2014 TIPS Inc. Unauthorized reproduction of this manual is prohibited. This manual is being used
during review sessions conducted by PharmacyPrep. 42-5
www.pharmacyprep.com
37. Which of the following best describe the function of propylene glycol in this formulation?
Lorazepan ----------2mls
Propileno glycol-----2%
Ethanol----------------5mls
A. Preservative
B. Solvent
C. Co-solvent
D. Surfactant
E. None of the above
38. Sodium stearate is a surfactant classified as:
A. Anionic B. Cationic C. Non-ionic D. Hydrophobic
E. Amphiphilic
39. Compared to the sublingual tablet, nitroglycerin ointment:
A. Provides only local effect
B. It a more stable dosage form
C. Provides a more prolonged effect
D. Is more rapidly absorbed
E. Is more rapidly eliminated
40. The dissolution rate of nitrofurantoin tablets may be increased by:
A. Micronizing the nitrofurantoin powder
B. Decreasing viscosity of the dissolution medium
C. Heating the solution medium
D. Mechanical agitation
E. All of the above increase the dissolution rate of nitrofurantoin tablets
41. Benzocaine with caffeine can form
A. Ion-dipole
B. Ion-Ion
C. Dipole-dipole
D. Complex molecule
E. No reaction is formed
42. Antioxidants, which mechanistically act as reducing agent, include:
I- Ascorbic acid II- Sodium bisulphite III- Citric acid
A. I only B. III only C. I and II only D. II and III only E. All of the above
43. Reason for using coating tablets includes all of the following, EXCEPT:
A. To mask the taste of the drug
B. To mask the odor of the drug
C. To improve the appearance of the drug

Copyright © 2000-2014 TIPS Inc. Unauthorized reproduction of this manual is prohibited. This manual is being used
during review sessions conducted by PharmacyPrep. 42-6
www.pharmacyprep.com
D. To increase the drug’s release rate
E. To protect the drug from stomach acid
44. An ophthalmic solution must be
A. Hypertonic
B. Isotonic
C. Hypotonic
D. Amphiphilic
E. Lipophilic
45. Uses of mineral oil may include:
A. Used as laxative
B. Used as antioxidant
C. In chronic use decrease the absorption of fat-soluble vitamins
D. Used as lubricant in suppositories
E. All are correct
46. Which decrease tension between solid and liquid?
A. Solubilizing agent
B. Wetting agent
C. Hydrophilic agent
D. Emulsifying agent
E. Decreasing the particle size of the solid
47. An agent with HLB value range of 14-18 is known as:
A. Antifoaming agent
B. W/O emulsifying agent
C. O/W emulsifying agent
D. Wetting agent
E. Solubilizing agent
48. In preparing ZnSO4 suppositories using 200mg of its USP value. Refers that displacement value of
ZnSO4 is 4 means that:
A. ZnSO4 will displace 800mg of cocoa butter
B. Cocoa butter displace 800mg of ZnSO4
C. Only 4mg of cocoa butter will be displaced
D. 200mg of ZnSO4 will displace 50mg of cocoa butter
49. Addition of mineral oil to a sulphur ointment preparation is called:
A. Levigation
B. Milling
C. Trituration
D. Solubilization
E. Emulsification

Copyright © 2000-2014 TIPS Inc. Unauthorized reproduction of this manual is prohibited. This manual is being used
during review sessions conducted by PharmacyPrep. 42-7
www.pharmacyprep.com
50. Tubular reabsorption is not dependent on?
A. pH of urine
B. Ionized form of drug
C. Speed of flow of urine
D. Metabolism
E. pKa
51. Renal nephrosis causes
I- Glomerular to be permeable
II-Leak the proteins to urine
III-The capillary membrane becomes impermeable
A. I only B. III only C. I and II only D. II and III only E. All of the above
52. What is the example of humectants?
I-Glycerine II-Propylene glycol III-Sorbitol
A. I only B. III only C. I and II only D. II and III only E. All of the above
53. Definition of colligative properties?
A. Elevation of boiling point
B. Decrease vapour pressure
C. Decrease in freezing point
D. osmotic pressure
E. All of the above
54. pH-Pka = 1.5, for acidic drug what percent of drug is ionized?
A-3.06% B-96% C-2.5% D-100% E-50%
55. What is the site for intraarticular injection?
A. injection in joint spaces
B. injection nerves
C. injection in spinal cord spaces
D. Injection in brain
E. injection in fatty tissues
56. Example of oils used in parenteral preparations, Except?
A. Sesame oil B. Olive oil C. Cotton seed oil D. Peanut oil
E. Mineral oil
57. What are the HLB values for O/W and w/O emulsions?
I-For O/W HLB is 8 – 18
II-For W/O HLB is 4 – 6
III-For O/W HLB is 1 – 3

Copyright © 2000-2014 TIPS Inc. Unauthorized reproduction of this manual is prohibited. This manual is being used
during review sessions conducted by PharmacyPrep. 42-8
www.pharmacyprep.com
A. I only B. III only C. I and II only D. II and III only E. All of the above
58. Sodium benzoate is used as?
A. Preservative
B. Solvent
C. Co-solvent
D. Filler
E. Humectants
59. Type of water is used in parenteral preparations manufacturing?
A. Sterile water for injection USP
B. Bacteriostatic water
C. Water for injection USP
D. All of the above
60. Steam (moisture) sterilization is carried out in?
A. Microwave
B. Heat sterilization
C. Autoclave
D. Ethylene oxide
E. None of the above
61. What are the methods of tablet disintegration?
I-Pulverization by intervention
II-Levigation
III-Trituration
A. I only B. III only C. I and II only D. II and III only E. All of the above
62. What are the factors effects on sedimentation of suspension?
A. Particle size
B. Density of particles
C. Density of vehicle
D. Viscosity of vehicle
E. All of the above
63. The primary pharmaceutical use of surfactant is as a:
A. Emulsifying agent
B. Surfactant
C. Suspending agent
D. Sweetener
E. Flocculating agent
64. Spans and tweens are:
A. Non ionic surfactants

Copyright © 2000-2014 TIPS Inc. Unauthorized reproduction of this manual is prohibited. This manual is being used
during review sessions conducted by PharmacyPrep. 42-9
www.pharmacyprep.com
B. Anionic surfactants
C. Cationic surfactants
D. Poly oxyalkylene derivatives
E. Suspending agents
65. Benzalkonium chlorides is
A. Non ionic surfactants
B. Anionic surfactants
C. Cationic surfactants
D. Poly oxyalkylene derivatives
E. Suspending agents
66. Parenteral solution should be free from:
I-Microorganisms
II-Microparticulates
III-Pyrogen
A. I only B. III only C. I and II only D. II and III only E. All of the above
67. HEPA filters used in sterile preparations filtration:
I-Microorganisms
II-Particulates
III-Pyrogens
A. I only B. III only C. I and II only D. II and III only E. All of the above
68. What is size of filters used in sterile water filtration?
A-0.22mm
B-0.33mm
C-0.4mm
D-0.55mm
E-0.66mm
69. Acacia is classified as:
A. suspending agent
B. natural emulsifying agent
C. Semi synthetic emulsifying agent
D. Flocculating agent
E. tablet filler
70. Rectal suppositories intended for adult use, approximately weigh.
A) 10 g B) 5 g C) 2 g D) 1 g E) 0.5 g
71. In a dispensing pharmacy, pharmacist usually mixes active powder with large amount of diluents
by:

Copyright © 2000-2014 TIPS Inc. Unauthorized reproduction of this manual is prohibited. This manual is being used
during review sessions conducted by PharmacyPrep. 42-10
www.pharmacyprep.com
A. Levigation
B. Geometrical dilution
C. Trituration
D. Sifting
E. Pulverization
72. The shell of soft gelatine capsules may make by addition of plastic like substance
A. Polyethylene glycol (PEG)
B. Povidone
C. Lactose
D. Sorbitol
E. methyl cellulose
73. Which is the smallest size of capsule:
A) 5 B) 10 C) 00 D) 1 E) 000
74. In extemporaneous preparations of suspensions, Levigating agent is used:
A. Reduce particle size
B. Reduce viscosity
C. Increase particle size
D. Increase sedimentation rate
E. Reduce sedimentation rate
75. Incorrect statement for hard gelatine capsule?
A. hard gelatine capsules have preservatives
B. hard gelatine capsules shells are prepared by gelatine, sugar and water.
C. suitable to fill aqueous liquid in hard gelatine capsule
D. None of the above
76. Incorrect statement about effervescent tablets?
A. It liberates CO2 in water
B. it always used by dissolving in water
C. It is mixture of acid and sodium bicarbonate
D. it is sensitive to moisture
E. When it comes in contact with moisture it swells up.
77. In tablet manufacturing what step before compression?
A. Granulation
B. Drying
C. Sifting
D. Lubrication and weighing
E. Milling

Copyright © 2000-2014 TIPS Inc. Unauthorized reproduction of this manual is prohibited. This manual is being used
during review sessions conducted by PharmacyPrep. 42-11
www.pharmacyprep.com
78. All of the following preparation require sterile, except?
A. ophthalmic drops
B. Ophthalmic ointments
C. Nasal spray
D. Intravenous
E. Otic preparations
79. Soft contact lenses are made of?
A. Hydroxymethyl methacrylate
B. Polymethyl methacrylate
C. glass
D. Plastics
E. cornea
80. Hard contact lens are made of?
A. Hydroxymethyl methacrylate
B. Polymethyl methacrylate
C. Glass
D. Plastics
E. Sorbital
81. Difference between plasma and interstitial fluid?
A. Albumin
B. Sodium ion (Na)
C. Bicarbonate
D. Potassium
E. All of the above
82. Dissolution rate increase by;
A. Increase particle size
B. Decrease particle size
C. Increase temperature
D. B&C
83. The solubilization of mercuric iodide by adding potassium iodide to the solution is due to:
A. salt formation
B. dipole-dipole interaction
C. complex formation
D. salting-in effect
84. Autoclaving is sterilization process of:
A. Dry heat sterilization

Copyright © 2000-2014 TIPS Inc. Unauthorized reproduction of this manual is prohibited. This manual is being used
during review sessions conducted by PharmacyPrep. 42-12
www.pharmacyprep.com
B.Steam Sterilization
C.Filtration
D.Gas sterilization
85. Which of the following tablet manufacturing method is not suitable for aspirin tablet
formulations:
A. Wet granulation
B. Dry granulations
C. Direct compression method
D. None of the above
86. Which of the following labels would be the most appropriate to be used in your pharmacy when
dispensing a suspension
A. “Take with food”
B. “Shake well before use”
C. “Take with lots of fluids”
D. “Rinse mouth after use”
87. HLB value for O/W emulsifying agent:
A.4-8
B.1-4
C.8-18
D.1-12
88. Which of the following terms indicates a loss of moisture? A. Deliquescence B. Efflorescence C.
Hygroscopicity D. Polymorphism
89. Compared to sublingual tablet, nitroglycerin ointment :
A.Provides only local effect
B.Is a more stable dosage form
C.Provides more prolong effect.
D.Is more rapidly absorbed
90. A hydrophilic semisolid system consisting of a dispersion of small inorganic particles or larger
organic molecules enclosing and interpenetrated by a liquid defines which of the following?
A. Soil
B. Gel
C. Suspension
D. Colloid

Copyright © 2000-2014 TIPS Inc. Unauthorized reproduction of this manual is prohibited. This manual is being used
during review sessions conducted by PharmacyPrep. 42-13
www.pharmacyprep.com
91. A colloidal system in which the colloidal particles show little attraction for the dispersion medium
defines which of the following type colloids?
A. Lyophilic
B. Lyophobic
C. Hydrophilic
D. Association
92. What is friability?
A. Ability of the tablet to withstand abrasion in packing, handling and shipping
B. ability of the tablet to dissolve
C. ability of the tablet to disintegrate
D. none
93. A substance that absorbs moisture from the air is termed as?
A. efflorescent
B. hygroscopic
C. eutectic
D. effervescent
94) In zinc oxide ointment preparation, using mineral oil and white petrolatum. Which of the
following is the correct step?
A) mix all three zinc oxide, mineral oil and white petrolatum
B) Mix white petrolatum and mineral oil and then add zinc oxide
C) Mix zinc oxide and mineral oil to form paste and then add white petrolatum
D) Mix zinc oxide and white petrolatum and then add mineral oil
95. A pharmacist considering recommending manages eczema in small child. What is the best
option?
A. cold cream
B. vanishing cream
C. petrolatum
D. zinc oxide cream
E. polysporin cream

Copyright © 2000-2014 TIPS Inc. Unauthorized reproduction of this manual is prohibited. This manual is being used
during review sessions conducted by PharmacyPrep. 42-14
www.pharmacyprep.com
ANSWERS:
1. C
2. E
3. C
Tips: Methylparaben and propylparaben are antifungal preservatives and combination may have
synergistic effect.
4. D
Tips: aspirin is gets degradation in moisture, therefore wet granulation methods is not used in aspirin
manufacturing.
5. C
6. A
Tips: breaking (irreversible) or creaming (reversible) is the problem in emulsions.
7. D
Tips: Freeze dry is drying method for thermo labile substances, mechanistically it works as
sublimation. However this is not a sterilization method.
8. C
Tips:
Deliquescentabsorb water
Efflorescentloss of moisture
Effervescentloss of carbon dioxide
Hydroscopicabsorb moisture
Hygroscopicabsorb moisture
9. C
Tips: Alka-seltzer tablets have effervescent
10. B
Tips: Camphor and menthol may have eutectic properties
11. C
Tips: flocculating agents increase dispersion and decrease sedimentation however do not change
volume

Copyright © 2000-2014 TIPS Inc. Unauthorized reproduction of this manual is prohibited. This manual is being used
during review sessions conducted by PharmacyPrep. 42-15
www.pharmacyprep.com
12. D
Comments: suspending agents increase viscosity and decrease sedimentation
13. B
14. C
Tips: veegum, agar, acacia, pectin, tragacanth are suspending agents
15. A
Tips: surfactants primarily used as emulsifying agents
16. A
Tips: benzalkonium chloride, spans, tweens, are examples of non ionic surfactants
17. E
Tips: parenteral solutions should be sterile (free from microorganism), microparticulates, pyrogens.
18. C
Tips: pyrogens cannot be filtered by HEPA filters
19. E
20. B
21. C
22. B
23. D
24. A
25. A
26. C
27. A
28. B
29. A

Copyright © 2000-2014 TIPS Inc. Unauthorized reproduction of this manual is prohibited. This manual is being used
during review sessions conducted by PharmacyPrep. 42-16
www.pharmacyprep.com
30. B
31. E
32. D
33. C
34. D
35. A
36. B
37. C
38. A
39. C
40. E
41. D
42. C
43. D
44. B
45. E
46. B
47. E
48. D
49. A
50. D
51. C (I and II)

Copyright © 2000-2014 TIPS Inc. Unauthorized reproduction of this manual is prohibited. This manual is being used
during review sessions conducted by PharmacyPrep. 42-17
www.pharmacyprep.com
52. E
53. E
54. B
Tips:
How to use calculator?
Enter: 10
x
Press : y
Press: (-)
Enter
1.5 =
55. A
56. E
57. C
58. A
59. C
60. C
61. E
62. E
63. A
Tips: surfactants primarily used as emulsifying agents
64. A
Tips: spans, tween, are examples of non-ionic surfactants.
65. C
66. E
Tips: parenteral solutions should be sterile (free from microorganism), micro particulates, and
pyrogen.
67. C
Tips: pyrogens cannot be filtered by HEPA filters
68. A

Copyright © 2000-2014 TIPS Inc. Unauthorized reproduction of this manual is prohibited. This manual is being used
during review sessions conducted by PharmacyPrep. 42-18
www.pharmacyprep.com
69. B
70. C
71. B
72. D
73. A
74. A
75. C
76. E
77. D
Milling Sifting Granulation Drying Lubrication and weighing compression.
78. C
79. A
80. B
81. E
82. D
Tips: Large particle size requires longer disintegration time
83. C
84. B
85. A
86. B
87. C
88. B

Copyright © 2000-2014 TIPS Inc. Unauthorized reproduction of this manual is prohibited. This manual is being used
during review sessions conducted by PharmacyPrep. 42-19
www.pharmacyprep.com
89. B
90. B
91. B
92. A
93. B
94. C
95. C

Copyright © 2000-2014 TIPS Inc. Unauthorized reproduction of this manual is prohibited. This manual is being used
during review sessions conducted by PharmacyPrep. 42-20
www.pharmacyprep.com

PHARMACY PREP.
DRUG DELIVERY SYSTEMS
1. What is meant by intrathecal injection?
A. Injection in between the spine
B. Injection underneath the tongue
C. Injection on thigh
D. Injection on bun

2. Which provides 100% bioavailability


A. Intramuscular B. Intravenous C. Extra venous D. Oral
E. Parenteral
3. What dosage form is NOT a sterile preparation?
A. Ophthalmic B. Intramuscular C. nasal spray D. otic
E. intravenous (IV)
4. Which of the following dosage form have first pass effect?
A. Rectal B. Sublingual C. Topical D. Oral solution E. Buccal
5. What is meant by intra articular injection?
A. Injection in between the spine
B. Injection underneath the tongue
C. Injection on thigh
D. Injection on bun
E. Injection in joints

6. What dosage form may NOT require sterile preparations?


A. Intrathecal B. Intraarticular C. nasal spray D. Ophthalmic suspension E. Sub cutaneous injections
7. We can increase the bioavailability of an ophthalmic preparation by
I-Increasing viscosity of preparation
II-Addition of a sticking agent
III-Increasing volume of the inserted dose
A. I only B. III only C. I and II only D. II and III only E. All are correct
8. If patient has to administer ophthalmic drops and ophthalmic ointment, what is correct
statement?
A. first administer drops and then next administer ointment
B. first administer ointment and then next drops
C. first administer drops and then after 2 hrs administer ointment
D. first administer ointment and then after 2 hrs administer drops

Copyright © 2000-2014 TIPS Inc. Unauthorized reproduction of this manual is prohibited. This manual is being used
during review sessions conducted by PharmacyPrep. 43-1
www.pharmacyprep.com
9. Which of the following is common allergic component in oral preparations?
A. alcohol B. starch C. Sodium lauryl sulfate D. Lactose E. Pyrogen
10. Nitro glycerine patch has to removed at least 4 to 6 hr, it is nitrate free period, because, it
decrease?
A. Resistance B. Headache C. Tolerance D. Dependence E. Side effects

11. Which of the following is taste awful if you stored in refrigerator?


A. Azithromycin B. Clindamycin C. Clarithromycin D. Metronidazole E. Captopril
12. Which of the following dosage forms cannot contain suspensions?
I) Intravenous
II) Ophthalmic
III) Otic
A. I only B. III only C. I and II only D. II and III only E. All are correct

Copyright © 2000-2014 TIPS Inc. Unauthorized reproduction of this manual is prohibited. This manual is being used
during review sessions conducted by PharmacyPrep. 43-2
www.pharmacyprep.com
ANSWERS:
1. A
2. B
3. C
4. D
5. E
6. C
7. C
Tips: hydroxypropyl methyl cellulose (HPMC) and CMC increase viscosity in ophthalmic preparations
8. A
9. D
10. C

11. C
12. A

Copyright © 2000-2014 TIPS Inc. Unauthorized reproduction of this manual is prohibited. This manual is being used
during review sessions conducted by PharmacyPrep. 43-3
www.pharmacyprep.com
Copyright © 2000-2014 TIPS Inc. Unauthorized reproduction of this manual is prohibited. This manual is being used
during review sessions conducted by PharmacyPrep. 43-4
www.pharmacyprep.com Sterile Preparations

PHARMACY PREP
STERILE PREPARATIONS
1. A vertical laminar flow hood is the best choice when preparing parenteral solutions of:
A-Penicillin B-Doxycycline C-Doxorubicin D-Nicotine formulations
E-Amino glycosides
2. The following factor is the cause of most contaminations in sterile preparations:
A-Solutions B-Surface area C-Personal D-Clothing E-Equipment
3. In sterile preparation, to wipe the surface of laminar flow hood, the following disinfectant can be
used:
I)70% isopropyl alcohol
II) Chlorhexidine
III) Benzalkonium chloride
A-I only B-III only C-I and II only D-II and III only E-I, II and III
4. Sterile preparation for parenteral solutions should include?
I-Pyrogen free II-Particulate free III-Pathogen free (microorganism free)
A-I only B-III only C-I and II only D-II and III only E-I, II and III
5. Pyrogen in parenteral preparation are eliminated by?
A-Chromatography B-Heat and distillation C-Lyophylization D-Reverse osmosis E-Filtration
6. The size filters used for filtration of microorganism in sterile water preparations
A-0.22 mm B-0.33 mm C-0.44 mm D-0.55 mm E-0.66 mm
7. Which of the following dosage form is NOT a sterile preparation?
A-Ophthalmic B-Intramuscular C-nasal spray D-otic E-i.v.
8. What type of laminar flow hood is used in preparation of cytotoxic (neoplastic) products?
A-Horizontal laminar airflow hood
B-Vertical Laminar airflow hood
C-Vertical and Horizontal laminar airflow hood
D-None of the above
9. What size of filters is used in water sterilization?
A-0.22 mm B-0.33 mm C-0.11 mm D-0.44 mm E-0.55 mm
10. What sizes of HEPA filters are used in laminar airflow hood that is used for parenteral
preparations?

Copyright © 2000-2014 TIPS Inc. Unauthorized reproduction of this manual is prohibited. This manual is being used
during review sessions conducted by PharmacyPrep. 44-1
www.pharmacyprep.com Sterile Preparations
A) 0.22 mm B-0.33 mm C-0.11 mm D) 0.44 mm E) 0.55 mm
11.Which of the following dosage form have first pass effect?
A-Rectal B-Sublingual C-Topical D-Oral solution E- Buccal
12. What is appropriate way to clean sterile compounding area?
A) Sterile water and wipes and followed by 70% isopropyl alcohol
B)70% isopropyl alcohol, wipes and followed by sterile water
C)Wipes with 70% isopropyl alcohol and followed by sterile water
D)70% isopropyl alcohol
13. What is correct about hand wash before sterile preparations?
A)wash hands and forearms for at least 30 seconds with soap and water
B)Wash hand and up to shoulders for at least 30 seconds with soap and water
C)Wash hand with 70% isopropyl alcohol for at least 30 seconds
D)don the gown and gloves before washing hand
14. What type of laminar airflow hood is used in compounding pharmacies?
A)vertical B)horizontal C)both D)None
15. Chemotherapy or hazardous product preparation should be performed in?
A) vertical B)horizontal C)both D)None
16. The function of HEPA filters in laminar air flow hood is?
A) to remove particulates B)to remove contaminants C)to remove pathogens D)all of the above
17. Horizontal laminar air flow hood produce air flow to?
A)toward the preparer from the back to the front of the hood
B)towards from front to backwards
C)top down to work surface
D) down up to HEPA filter
18. HEPA filter size for laminar airflow hood?
A) 0.2 micron B)0.3 microns C)0.4 microns D)0.5microns
19. Which of the following electrolyte cause precipitation in total parenteral nutrition (TPN)?
A) KCl B)Calcium Phosphate C)Sodium Chloride D)MgSO4
20. Which drug is hazardous handle?
A) Oxybutinin B) Urespas (flavoxate) C) Finasteride D) Misoprostol
21. What number of laminar airflow hood (LAH) for chemotherapy preps is used?

Copyright © 2000-2014 TIPS Inc. Unauthorized reproduction of this manual is prohibited. This manual is being used
during review sessions conducted by PharmacyPrep. 44-2
www.pharmacyprep.com Sterile Preparations
A) Class 2B B) Class 1A C) Class 100 D) Class 1000
Tips: the LAH is designed to provide class 100 conditions it means no more than 100 particles of 0.5
micron size or larger per cubic foot of air
22. Choose wrong statement?
A) Laminar airflow hood should be disinfect by bleach, chlorhexidine, and IPA 70%, before use.
B) Blower should be turned off before operation of hood.
C) UV lamp should be turned off
D) Laminar airflow hood should be certified and accredited annually
23. Which of the following TPN can cause constipation?
A)CaCO3 B)Ca. gluconate C)Milk of magnesia D)MgSO4
Mg = must go (diarrhea)
Ca.phosphate cause precipitation in TPN solution.
24. In chemotherapy syringes, what type of pressure syringe is used?
A)negative pressure syringe B)positive pressure syringe C)both
25. Which of the following should NOT be discarded into sharp containers?
A)needle B)spike C)spatula D)syringe with needle
26. Which of the following personal protection kit (PPK) is least appropriate to reuse?
A) poly carbonate goggles B)face mask C)sterile lint free gown D)sterile nitrile gloves
27. Which of the following method of sterilization is NOT used to sterilize heat sensitive drugs?
A-Gas B-Radiation C-Filtration D-Lyophilization E)None of the above
Ans: D
28. Which of the sterilization procedures is best suited for extemporaneous preparation on a sterile
solution?
a. Steam sterilization
b. Gas sterilization
c. Filtration
d. Ionizing radiation
29. A large volume IV solution is defined as one which contains which of the following
volumes or more?
a. 10 ml
b. 100 ml
c. 500 ml
d. 1 L

Copyright © 2000-2014 TIPS Inc. Unauthorized reproduction of this manual is prohibited. This manual is being used
during review sessions conducted by PharmacyPrep. 44-3
www.pharmacyprep.com Sterile Preparations
30. The following is/are true for water for injection USP
I) Can be prepared by distillation
II) Can be prepared by reverse osmosis
III) It is not pyrogen free A) I only
B) III only
C) I and II only
D) II and III only
E) All

31. These are true with respect to sterile water for injection USP
I) This type of water is sealed in multi-dose containers
II) Sterilized within 24 hours of preparation.
III) The containers vary in size from 5ml to a maximum of 1L A) I only
B) III only
C) I and II only
D) II and III only
E) All

32. The following is/are true with respect to “Bacteriostatic water for injection USP” EXCEPT:
A) It may be packaged in multi-dose containers
B) It may be injected in large volumes
C) The bacteriostatic agent are a possible source of incompatibility if water is used for dilution.
D) It should not be packaged in vials or syringes exceeding 30ml
E) None of the above

33. The following is/are considered solubilizing agents.


I) Ethyl alcohol
II) Propylene glycol
III) Methyl alcohol A) I only
B) III only
C) I and II only
D) II and III only
E) All

34. The following is/are main method for sterilizing pharmaceutical and medical equipments, EXCEPT
A) Steam sterilization
B) Dry heat sterilization

Copyright © 2000-2014 TIPS Inc. Unauthorized reproduction of this manual is prohibited. This manual is being used
during review sessions conducted by PharmacyPrep. 44-4
www.pharmacyprep.com Sterile Preparations
C) Sterilization by reverse osmosis
D) Sterilization by filtration
E) Gaseous sterilization

35. Steam sterilization is:


I) It is a terminal method used after the drug solution has been sealed in its final containers.
0
II) It uses an elevated temperature of 121 C for 20 minutes.
III) It is not reliable method for pharmaceutical sterilization. A) I only
B) III only
C) I and II only
D) II and III only
E) All

36. The following is/are true with respect to dry heat sterilization , EXCEPT
A) It is a terminal method
B) More effective than moist heat
C) Requires higher temperature and longer time.
D) It destroys microbes by dehydrating them
0 0
E) The sterilization is carried out in ovens at a temperature of 150-170 -170 for at least 2hours.

37. Sterilization by filtration has the following properties:


I) Used for those drugs solutions that decompose by high temperature
II) Filters of pore size of 0.2microns is used.
III) To speed up the filteration process, negative pressure on the non-sterile solution should be
used. A) I only
B) III only
C) I and II only
D) II and III only
E) All

38. All the following is/are true with respect to gaseous sterilization:
I) This method is not used for commercial levels.
II) It is used for certain enzymes and antibiotic preparations.
III) The procedure is slow taking 4-16 hours for sterilization. A) I only
B) III only
C) I and II only
D) II and III only
E) All

39. All of the following are require sterile preps, except?

Copyright © 2000-2014 TIPS Inc. Unauthorized reproduction of this manual is prohibited. This manual is being used
during review sessions conducted by PharmacyPrep. 44-5
www.pharmacyprep.com Sterile Preparations
A) Inhaler B)ophthalmic drops C)IA injections D)i.v injections E)IM injection
40. Which of the following is NOT discarded in a sharp object recycle?
A) Spatula B) Syringe C)Broken glass D) Needle e) Ampoules
41. What is incorrect about hypodermic needle in sterile preparations?
A) Avoid touching hub and plunger of needle
B) Avoid touching bevel of needle
C) gauge is diameter of syringe shaft
D) Leur-lock syringes are used in cytotoxic preparation
E) All parenteral preps are have to manipulate in laminar airflow hood
ANSWERS:
1. C
2. C
3. C
4. E
5. B
6. A
7. C
8. B
9. A
10. B
11. D
12. A
13. A
14. A
15. A
16. D
17. A
18. B
19. B
20. C
21. A
22. A
23. A

Mg = must go (diarrhea)
Ca.phosphate cause precipitation in TPN solution.
24. C

Copyright © 2000-2014 TIPS Inc. Unauthorized reproduction of this manual is prohibited. This manual is being used
during review sessions conducted by PharmacyPrep. 44-6
www.pharmacyprep.com Sterile Preparations
25. C
26. D
Tips: sterile nitrile gloves are changed at the start preparing sterile area, change at start of
preparation or every spill or syringe is compromise. for long preparations change every 1/2hours.
27. D
Tips: lyophilization is not sterilization method, it is method of drying to heat sensitive
substances.
28. C
29. B
30. C
Tips: It must be pyrogen free. This differentiate it from purified water USP
31. D
Tips: It should be sealed in a single-dose containers.
32. B
Tips: This water should not be injected in large volumes because of possible toxicity of the
bacteriostatic agent.
33. C
Tips: Methyl alcohol is toxic to the body.
34. C
Tips: Other kind of sterilization is by ionizating radiation.
35. C
Tips: It is a reliable method for pharmaceutical sterilization.
36. B
Tips: Dry heat sterilization is not as effective as moist heat.
37. C
Tips: Positive pressure on the non-sterile solutions, or suction (not as reliable) on the receiving
container need to be used.
38. D
Tips: This method is used on a commercial level for sterilizing medical supplies such as packages
of disposal needles and plastic syringes and catheters.
39. A

Copyright © 2000-2014 TIPS Inc. Unauthorized reproduction of this manual is prohibited. This manual is being used
during review sessions conducted by PharmacyPrep. 44-7
www.pharmacyprep.com Sterile Preparations
40. A
41. C
Tips: gauge is diameter of needle.

Copyright © 2000-2014 TIPS Inc. Unauthorized reproduction of this manual is prohibited. This manual is being used
during review sessions conducted by PharmacyPrep. 44-8
www.pharmacyprep.com Sterile Preparations
Copyright © 2000-2014 TIPS Inc. Unauthorized reproduction of this manual is prohibited. This manual is being used
during review sessions conducted by PharmacyPrep. 44-9
www.pharmacyprep.com

PHARMACY PREP
EXTEMPORANEOUS PREPARATIONS
1. What are correct instructions should be labelled in the following preparation?
Salicylic acid 2%
Lactic acid 6 ml
Flexible collodion ad 30 ml
A. Flexible collodion is extremely flammable.
B. Tight closed container to prevent ether and alcohol.
C. Store away from fire
D. All of the above
E. None of the above
2. What are correct instructions should be labelled in the following preparation?
Iodine 2%
Sodium iodide 2.7%
Alcohol q.s. 30 mL
A. A rubber or plastic spatula is used because iodine is corrosive.
B. Can stain
C. A and B
D. None of the above
3. Which of the following is NOT a sharp hazard?
A. syringe needle B. spatula C. Diabetic blood sample prick
D. epipen E. scissors
4.
Rx
Ephedrine HCl
NaCl qs
Sodium bisulfate
What is use of sodium bisulfate in this prescription?
A. Antioxidant
B. Preservative
C. Vasoconstrictor
D. Base
E. Solvent
5. Ascorbic acid is added to iron preparations as reducing agent to maintain it in Ferrous 2+ state and
form absorbable chelate with iron Ferric 3+ state. Ascorbic acid act as?
A. oxidative agent
B. Reducing agent
C. Preservative

Copyright © 2000-2014 TIPS Inc. Unauthorized reproduction of this manual is prohibited. This manual is being used
during review sessions conducted by PharmacyPrep. 45-1
www.pharmacyprep.com
D. Chelating agent
E. Buffering agent
2+ 3+
6. How many moles of Ca and Fe respectively could EDTA chelate?
A. 1 mole and 1 mole
B. 1 mole and 2 mole
C. 2 mole and 2 mole
D. 3 mole and 3 mole
E. 3 mole and 1 mole
7. What vitamins have antioxidant activity ?
i) Vitamin C ii) Vitamin E iii) Vitamin D
A. I only B. III only C. I and II only D. II and III only E. All are correct
8. Compounding of extemporaneous mixtures include
I) Pharmacist
II) Pharmacy technicians
III) Nurse
A. I only B. III only C. I and II only D. II and III only E. All are correct
9. All the following is/are important to be put on label of an extemporaneous product, EXCEPT:
A. Strength and quantity
B. Lot number
C. Beyond use of date
D. Listing of names of inactive ingredient is not encourage
E. Special storage condition
10. When suppositories are compounded, all the following are true, EXCEPT
I) Only disposable, moulds should be used.
II) Moulds should be stored in fridge
III) Reusable moulds may be used
A. I only B. III only C. I and II only D. II and III only E. All are correct
11. When processing a prescription for a compound or for extemporaneous preparation. The least
important is:

A. Time fee for processing the prescription.

B. The expiry date.

C. Knowing the compound is covered or not.

D. How to formulate the compound.

E. None of the above.

Copyright © 2000-2014 TIPS Inc. Unauthorized reproduction of this manual is prohibited. This manual is being used
during review sessions conducted by PharmacyPrep. 45-2
www.pharmacyprep.com
ANSWERS:
1. D
2. C
3. B
4. A
Tips: Sodium bisulfate is a water-soluble antioxidant.
5. B
6. B
7. C
8. C
9. D
Tips: Listing the names of inactive ingredients on the label is encouraged.
10. C
11. A

Copyright © 2000-2014 TIPS Inc. Unauthorized reproduction of this manual is prohibited. This manual is being used
during review sessions conducted by PharmacyPrep. 45-3
www.pharmacyprep.com

PHARMACY PREP.
PHARMACEUTICAL ANALYSIS
1. By addition of a solute, solubility of non-electrolytes is decreased. This is:
A. Common ion effect
B. Disintegration
C. Salting out
D. Salting in
E. None of the above

2. Which of the following compound is known as amphiprotic solvent?


A. Methanol
B. Water
C. Acetic acid
D. Ethanol
E. All are right
+
3. Molecular ion (M ) or molecular weight is detected by:
A. Colorimetry
B. Mass spectroscopy
C. Spectrophotometry
D. Spectrometry
E. Ultraviolet detectors

4. Determination of alcohol in blood in done by:


A. HPLC
B. TLC
C. Reverse phase HPLC
D. Gas chromatography & Mass spectroscopy (GC-MS)
E. Mass spectroscopy

5. Mass spectroscopy is used to:


A. Detect traces of unknown chemical structure
B. Structure elucidation of unknown chemical
C. Traces in environmental samples
D. Drug traces in blood samples
E. All of the above

6. All of the following detector are used in High Performance Liquid Chromatography (HPLC):
EXCEPT:
A. Diode Array Detector

Copyright © 2000-2014 TIPS Inc. Unauthorized reproduction of this manual is prohibited. This manual is being used
during review sessions conducted by PharmacyPrep. 46-1
www.pharmacyprep.com
B. UV detector
C. Refractive Index detector
D. Gamma Rays
E. Radioactive detector

7. Mechanism of action of Thin Layer Chromatography (TLC) is by:


A. Gas absorption
B. Column theory
C. Capillary action
D. High pressure extraction
E. Temperature range theory

8. Lyophilization (freeze dry method) technique used in:


I- Sterilization of liquids and solids
II- Sterilization of solids only
III- To dry heat-sensitive wet powders
A. I only B. III only C. I and II only D. II and III only E. All are correct
9. Sterilization of hormone is done by:
A. Dry heat
B. Lyophilization
C. Filtration
D. Radiation
E. Autoclave

10. Correct statements regarding ethylene oxide sterilization include:


I- A gas sterilization method that combines heat and moisture
II- Has a high penetration power to sealed containers
III- It is used in dry heat sterilization
A. I only B. III only C. I and II only D. II and III only E. All are correct
11. In the assay of amphetamines in plasma, it can be extracted by:
A. Ether
B. Isopropyl alcohol
C. Tetrahydrofuran
D. Ninhydrin
E. Amides

12. Reverse phase high performance liquid chromatography (RP-HPLC)


I- Mobile phase is aqueous solution
II- The particle size of stationary phase affects resolution
III- Utilize high pressure pump to increase efficiency in the resolution
A. I only B. III only C. I and II only D. II and III only E. All are correct
13. All of the following proprieties are classified as colligative proprieties, EXCEPT:

Copyright © 2000-2014 TIPS Inc. Unauthorized reproduction of this manual is prohibited. This manual is being used
during review sessions conducted by PharmacyPrep. 46-2
www.pharmacyprep.com
A. Elevation of boiling point
B. Osmotic pressure
C. Increase in conductivity
D. Lowering of freezing point
E. Magnitude of vapor pressure

14. The colligative proprieties of a solution are related to:


A. The total number of solute particles
B. pH of the solution
C. Number of ions
D. Ratio of number of ions and number of molecules
E. Number of non-ionized molecule

15. Colligative proprieties are important in determining:


A. pH B. Tonicity C. Solubility D. Stability E. Sterility
16. What is the plate’s function in thin layer chromatography (TLC) ?
A. Filter
B. Increase rate of resolution
C. Support mobile phase
D. Make resolution
E. Support stationary phase

17. At acid/base titration, which affect the reaction?


I- Change in the pH of the preparation
II- Addition of a specific buffer will increase the reaction
III-Addition of a specific buffer will decrease the reaction
A. I only B. III only C. I and II only D. II and III only E. All are correct
18. Atoms with the same number of protons are known as:
A. I only B. III only C. I and II only D. II and III only E. All are correct
19. Instrument used to measure specific gravity by floating in a liquid:
A. Picnometer
B. Hydrometer
C. Thermometer
D. Thermostat
E. Tensiometer

20. Instrument used to measure quantity of two liquids with equal volume:
A. Picnometer
B. Hydrometer
C. Thermometer
D. Thermostat

Copyright © 2000-2014 TIPS Inc. Unauthorized reproduction of this manual is prohibited. This manual is being used
during review sessions conducted by PharmacyPrep. 46-3
www.pharmacyprep.com
E. Tensiometer

21. Solvents can be classified as:


A. Amphiprotic
B. Protophilic
C. Aprotic
D. Protogenic
E. All are correct

22. Kind of solvent where acids and bases are produced, react and dissociates producing such anion
or cation species is called:
A. Amphiprotic solvent
B. Protophilic solvent
C. Aprotic solvent
D. Protogenic solvent
E. Inert solvent

23. Correct statements regarding TITRATION include:


I- Determination of a given component in a solution by addition of a liquid reagent of known
strength until a given endpoint is reached.
II- Non-aqueous titration can be used in the assay of some materials that cannot be easily titrated in
aqueous system.
III- Titration is a chromatographic method of substance separation.
A. I only B. III only C. I and II only D. II and III only E. All are correct
24. The end point of a titration analysis is characterized by:
A. Change in color
B. Change in solubility
C. Change in pH
D. Change in dispersability
E. Change in particles aggregation
25. Considerations that should be taken in selecting a solvent for an assay analysis:
A. Nature and solubility of the sample under investigation
B. Side reactions should not occur between the sample in analysis and the solvent
C. The solvent should be pure, low toxic and inexpensive
D. Solvent should have a high dielectric constant
E. If the sample is a weak acid the solvent should also be a weak acid
26. Solvent used in the assay of levodopa and methyldopa includes:
A. Acetic acid
B. Pyridine
C. Chloroform
D. Formic acid

Copyright © 2000-2014 TIPS Inc. Unauthorized reproduction of this manual is prohibited. This manual is being used
during review sessions conducted by PharmacyPrep. 46-4
www.pharmacyprep.com
E. Dimethyl formamide
27. Dimethyl formamide is a solvent used in the assay of which of the following drugs?
A. Adrenaline
B. Sulfonamides
C. Cimetidine
D. Chlortalidone
E. Levodopa
28. Which of the following spectroscopic method is used in detection of nanogram amount of
unknown substance?
A. Microspectroscopy
B. Nanospectroscopy
C. Spectroscopy
D. Microscopy
E. None of the above
29. Gel electrophoresis is based on separation of;
A. Micro molecules
B. Electric charges
C. Protein mixture separation
D. electrolysis
30. Gel electrophoresis is used for separation of?
A. large molecules like proteins B. small organic molecules C. organic chemicals D. Chemical structure
elucidation
31. Method that used to detect in-organic impurities:
A. Flame photometry B. Mass spectroscopy C. Nuclear Magnetic Resonance D. Infra red spectroscopy
32. For reversed phase high performance liquid chromatography (RP-HPLC)?
A. Stationary phase is hydrophobic
B. Mobile phase is polar
C. Polar is separated first
D. used for lipid soluble substances
33. GLC separation depend on?
A. Molecular weight B. polarity C. hydrophilicity D. boiling point
34. Removal of impurities through semi permeable membrane is?
A. Dialysis B. Osmosis C. Diffusion D. Active transport
E. Chromatography

Copyright © 2000-2014 TIPS Inc. Unauthorized reproduction of this manual is prohibited. This manual is being used
during review sessions conducted by PharmacyPrep. 46-5
www.pharmacyprep.com
35. Solubility in organic solvent and in water is determined by?
A. GC-MS B. High performance liquid chromatography (HPLC) C. Gas chromatography (GC) D. Mass
spectroscopy E. Partition coefficient
36. ELISA is a common serological test for the presence of particular antigens or antibody, what is
ELISA stands for?
A. Enzyme Linked Immunosorbent Assay
B. Enzyme Linked Immune system Assay
C. Enzyme Linked Immune Serum Assay
D. none of the above
37. Gel electrophoresis is a technique used for the separation of
I) deoxyribonucleic acid (DNA) II-ribonucleic acid (RNA) III-protein molecules
A. I only B. III only C. I and II only D. II and III only E. All are correct
38. Which of the following is NOT in HPLC?
A. Pump B. Silica gel column C. UV detector D. Gas layer
E. Solvent
39. Gel electrophoresis is used for?
A. Detection of proteins and amino acids
B. Separation of protein and protein mixture
C. Separation of nucleic acid mixture
D. Detection of nucleic acid
E. B and C
40. Enzyme Linked Immunosorbent Assay (ELISA) is used for the diagnosis of?
A. HIV infections
B. Hepatitis infections
C. Estimating antigen and antibodies in solution
D. Detection of viral infections
E. All of the above
41. What are the types of detectors in mass spectrometry analysis used?
I-EI-MS II-FAB-MS III-CI-MS
A. I only B. III only C. I and II only D. II and III only E. All are correct
42. In high performance liquid chromatography (HPLC) what is carrier?
A. Gas B. Silica gel C. liquid solvents D. stationary phase E. Column

Copyright © 2000-2014 TIPS Inc. Unauthorized reproduction of this manual is prohibited. This manual is being used
during review sessions conducted by PharmacyPrep. 46-6
www.pharmacyprep.com
CORRECT ANSWERS
PHARMACEUTICAL ANALYSIS
1. C
Tips: Salting out is characterized when the addition of a solute in a solution cause the decrease of
solution solubility, when the addition of a solute cause the increase of solubility is called salting in.
2. E
Tips: Amphiprotic is a type of solvent where acids and bases are produced. It reacts and dissociates
producing such anion and/or cation species.
3. B
Tips: M+ molecular ion is detected by mass spectroscopy that determines the molecular weight of
the sample.
4. D
Tips: Gas chromatography is a chromatographic method used to separate volatile organic
compounds as alcohol, hydrocarbons and aerosols.
5. D
Tips: Mass spectroscopy may be used as adjunct in gas chromatography.
6. D
Tips: X-rays are not used as detectors in HPLC
7. C
Tips: TLC-Thin Layer Chromatography is a very fast and simple chromatographic method that
separates organic compounds, progress of organic reactions, and tests the purity of products by
capillary theory.
8. B
Tips: Freeze dryer is used for the conversion of wet powders to dry powders.
9. D
Tips: Radiation is the most appropriated sterilization method for hormones; the only exception is
testosterone that is normally sterilized by dry heat sterilization.
10. C
Tips: Ethylene oxide sterilization is a very effective technique that combines heat and moisture
having a very high penetration power to sealed containers.
11. A
Tips: Amphetamines can be extracted from plasma by ether.

Copyright © 2000-2014 TIPS Inc. Unauthorized reproduction of this manual is prohibited. This manual is being used
during review sessions conducted by PharmacyPrep. 46-7
www.pharmacyprep.com
12. E
Tips: HPLC- High Performance Liquid Chromatography is a method that utilizes high-pressure pumps
to increase efficiency in the separation. The mobile phase is normally an aqueous solution and the
particles size of the stationary phase definitely alters the effect of resolution.
13. C
Tips: Colligative properties in physical chemistry, depending on the number of molecules present in a
given space, rather than on their size, molecular weight or chemical constitution. The colligative
properties of solutions are osmotic pressure, boiling point elevation, freezing point depression, and
vapor pressure lowering.
14. A
Tips: Colligative properties in physical chemistry of a solution, depending on the number of
molecules present in a given space, rather than on their size, molecular weight or chemical
constitution.
15. B
Tips: 15- Colligative proprieties are important in determining the tonicity of the solution. The
colligative properties of solutions are osmotic pressure, boiling point elevation, freezing point
depression, and vapor pressure lowering
16. E
Tips: The only function of plates in TLC is support the mobile phase that moves by capillary
mechanism through the stationary phase (plates).
17. A
Tips: Buffers are added to the solutions exactly to avoid the change in the solution’s pH, therefore if
added to the solution will not affect the reaction.
18. B
Tips: In physical chemistry, isotopes are atoms with the same number of protons. Isotones with the
same number of neutrons and isobaros with the same number of mass.
19. B
Tips: Hydrometer is an instrument used to measure specific gravity by floating in a liquid:
Hydrometer (SWIM in water)
20. A
Tips: Picnometer is an instrument used to measure quantity of two liquids with equal volume

Copyright © 2000-2014 TIPS Inc. Unauthorized reproduction of this manual is prohibited. This manual is being used
during review sessions conducted by PharmacyPrep. 46-8
www.pharmacyprep.com
21. E
Tips: Solvents play an important rule in determining acid/base properties in a solution and may be
classified depending of their chemical structure and activity as Solvents can be classified as
amphiprotic, protophilic, aprotic and protogenic.
22. A
Tips: Amphiprotic solvent as water, acetic acid, methanol and ethanol are kind of solvents where
acids and bases are produced, react and dissociates producing such anion or cation species.
23.C
Tips: Titration is a determination of a given component in a solution by addition of a liquid reagent of
known strength until a given endpoint is reached. Non-aqueous titration can be used in the assay of
some materials that cannot be easily titrated in aqueous system.
24. A
Tips: The end point of a titration analysis is characterized by change in color. Commonly used
indicators include triphenyl methane group corants and azo group corants.
25. E
Tips: In order to chose the proper solvent to be used in an analysis we need to consider some
important points as the nature and solubility of the sample under investigation, and make sure that
no any side reaction between the sample in analysis and the solvent will happen. The solvent should
be pure, low toxic, inexpensive and have a high dielectric constant.
26. D
Tips: Formic acid is a solvent used in non-aqueous titration in the assay of levodopa and methyldopa
27. B
Tips: Dimethyl formamide is a solvent used in non-aqueous titration in the assay of sulfonamides,
allopurinol and acetazolamide.
28. C
Tips: mass spectroscopy methods
29. B
30. A
42. C

Copyright © 2000-2014 TIPS Inc. Unauthorized reproduction of this manual is prohibited. This manual is being used
during review sessions conducted by PharmacyPrep. 46-9
www.pharmacyprep.com
BIBLIOGRAPHIC REFERENCE
1- COMPREHENSIVE PHARMACY REVIEW – Lippincott William & Wilkins – Fourth edition
2- CPS-COMPENDIUM OF PHARMACEUTICALS AND SPECIALITIES - Canadian Pharmacist Association –
2001 edition.
th
3- MEDICAL DICTIONARY – Dorland’s illustrated – 27 edition.
4- PHARMACY PREP – Lectures series & study guide for Evaluating Examination-TIPS - 2003/2004
5- THERAPUTIC CHOICES – Canadian Pharmacist Association -Third edition
th
6- USP DI – Drug Information for the Health Care Professional–15 edition – Volume I.

Copyright © 2000-2014 TIPS Inc. Unauthorized reproduction of this manual is prohibited. This manual is being used
during review sessions conducted by PharmacyPrep. 46-10
www.pharmacyprep.com

Pharmacy PREP
ETHICAL STANDARDS
1. The ethical standard beneficence in pharmacy practice means:
A. Pharmacist act with justice
B. Pharmacist act with fairness
C. Pharmacist act with honesty and without deception
D. Pharmacist does well to patient
E. Pharmacist do prevent harm
2. The ethical standard nonmaleficence in pharmacy practice means:
A. Pharmacist act with justice
B. Pharmacist act with fairness
C. Pharmacist act with honesty and without deception
D. Pharmacist does well to patient
E. Pharmacist do prevent harm
3. The ethical standard veracity in pharmacy counselling means:
A. Pharmacist act with justice
B. Pharmacist act with fairness
C. Pharmacist act with honesty and without deception
D. Pharmacist does well to patient
E. Pharmacist do prevent harm
4. The ethical standard justice in pharmacy practice means:
A. Pharmacist act with equality
B. Pharmacist act with fairness
C. Pharmacist act with honesty and without deception
D. Pharmacist does well to patient
E. Pharmacist do prevent harm
5. Patient cannot afford to buy new medication that is not covered by his insurance plan. Pharmacist
called his doctor and asked him to change the medication into another medication covered by the
patient’s insurance plan. Which ethical principle did the pharmacist follow?
A. Beneficence
B. Veracity
C. Justice
D. Nonmaleficence
E. Fidelity

6) A pharmacist stops counseling a patient who repeatedly takes the medication incorrectly. Which
ethical principle does pharmacist violate?
A) Justice

Copyright © 2000-2014 TIPS Inc. Unauthorized reproduction of this manual is prohibited. This manual is being used
during review sessions conducted by PharmacyPrep. 47-1
www.pharmacyprep.com
B) Beneficence
C) Nonmaleficence
D) Autonomy
E) Veracity
7. Pharmacy professional ethical standards include nonmaleficence, which means:
A. Pharmacist should act with honesty and veracity
B. Pharmacist should act with fairness
C. Pharmacist should prevent harm
D. Pharmacist should place the benefit of the patient above all else.
E. Pharmacist should respect patient choice.
8. A physician phones a pharmacist asking him not to tell the patient about the side effects of a drug
because if he does, the patient will not take it. The pharmacist complies. Both the physician and
pharmacist are seeking which ethic for the patient?
A. Nonmaleficence
B. Beneficence
C. Autonomy
D. Veracity
E. Justice

9. A physician phones a pharmacist asking him not to tell the patient about the side effects of a drug
because if he does, the patient will not take it. The pharmacist complies. Both the physician and
pharmacist have violated which ethic principle?
A. Nonmaleficence
B. Beneficence
C. Autonomy
D. Veracity
E. Justice

10. A pharmacist who refuses to counsel and AIDS patient violates which ethic?
A. Nonmaleficence
B. Beneficence
C. Autonomy
D. Veracity
E. Justice

11) A father of teenage girl approach to your pharmacy. Asks if her teenage 16 yo daughter using
oral contraceptives? What ethical dilemma of pharmacist?
A. confidentiality and veracity
B. confidentiality and autonomy
C. confidentiality and beneficence
D. confidentiality and fidelity
E. confidentiality and justice

Copyright © 2000-2014 TIPS Inc. Unauthorized reproduction of this manual is prohibited. This manual is being used
during review sessions conducted by PharmacyPrep. 47-2
www.pharmacyprep.com
12. A father of teenage girl approach to your pharmacy. Asks if her teenage 16 yo daughter using oral
contraceptives? What ethical dilemma of pharmacist?
A. Veracity and confidentiality
B. Beneficence and confidentiality
C. Justice and confidentiality
D. Nonmaleficence and confidentiality
E. Autonomy and confidentiality

13. When can you break confidentiality?


A. patient who has HIV with partner
B. Patient inmate who broke the law
C. Patient who has Alzheimer's with care giver
D. All of the above
14. What is the principle of confidentiality does not break if a pharmacist disclose the following
medical conditions to the society?
A. diabetes
B. madness
C. HIV
D. high BP
E. cancer
15. In which of the following situation you can break confidentiality of patient?
A. patient is suffering from HIV
B. patient is on parole
C. suspect patient can harm himself
D. patient is in jail

16. A 30 y/o woman comes to your pharmacy to buy oral contraceptives. While conversation she
says she found antiretroviral drugs in her husband bag. If her partner has HIV. Your medication
record show patient is HIV? What is appropriate to do?
I) Tell her to take HIV tests
II) Tell her to talk his doctor
III) Tell her that she got HIV infection from her partner
A. I only
B. III only
C. I and II only
D. II and III only
E. All are correct
17. Can you release information to a prescriber if you have firsthand knowledge from the third party
or DUR about double doctoring?
A. patient information is confidential
B. let third part inform to prescriber

Copyright © 2000-2014 TIPS Inc. Unauthorized reproduction of this manual is prohibited. This manual is being used
during review sessions conducted by PharmacyPrep. 47-3
www.pharmacyprep.com
C. Yes, you can inform to prescriber because it is in circle of care
D. Ask patient first, before informing doctor
18. Pam is working in rural pharmacy. She is friend with all colleagues, including Chris. Chris had
vision problems in the past, but underwent laser surgery and now is ok. He does not work pharmacy
anymore, but Pam is still has his profile in her computer. Should she date him?
19. Patient is using Terbutaline inhaler a lot. The pharmacist failed to advise patient to see his doctor
to add corticosteroids therapy to manage his condition. What is violated?
A. Beneficence B. Nonmaleficence C. veracity D. autonomy E. Paternalism
20) A patient has diagnosed with terminal ill cancer. His doctor advised him about his serious illness.
What ethical principle is followed?
A. Beneficence
B. Nonmaleficence
C. veracity
D. autonomy
E. Paternalism
21. Which of the following is express consent?
I) given verbally
II) written
II) given electronically
A) I only B) III only C) I and II only D) II and III only E) All are correct
22. A pharmacist Joe is being restricted from practice because of history of substance abuse. Based
on what the college makes this decision?
A. Incompetent B. Incapacity C. Negligence D. Mis-conduct E. accountability
23) A returned medication is placed on shelf by the technician? what you have to as staff
pharmacist?
A) talk to technician
B) inform to manager
C) complain to college of pharmacist
D) Discuss in meeting
24) Which of the following functions cannot be delegated to pharmacy technician?
A) preparing prescription
B) Checking expiry date
C) demonstrating a device to customer
D) selecting a over the counter drug to a patient
E) Providing a disease literature to customer

Copyright © 2000-2014 TIPS Inc. Unauthorized reproduction of this manual is prohibited. This manual is being used
during review sessions conducted by PharmacyPrep. 47-4
www.pharmacyprep.com
25) Pharmacist suspects a junior technician preparing sterile IV preparations. However technician did
not followed proper sterile prep proper technique. What is appropriate to do?
A) Discard the sterile preps and redo new batch
B) Tell the tech to follow standard techniques
26. While preparing prefilled syringes, in laminar airflow hood. After packaging you find some
syringes leaking. What you have to do?
A) Call the company complain
B) Discard all the batch and document
C) Select the good ones and through the damage
D)
27. A doctor is asking a patient to sign a consent form for a particular treatment. Which of the
following applies?
A. inform consent B. Implied consent C. Express consent D. all
28) A patient comes to your pharmacy with a prescription of his wife who is not covered under his
prescription drug plan. He requests you to process her prescription under his name. He will pay cash
for the cost and later send the receipt to his insurance company to process the claim?
A) it is unethical
B. It is illegal
C. It is ok
D. He can claim from insurance
29. Which of the following is incorrect combination?
A. Autonomy = Right of self determination
B. Beneficence = doing good
C. Nonmaleficence = Preventing harm
D. Justice = unequal distribution of benefits and burden
E. Veracity = Honesty without deception
30. This principle expresses the concept that pharmacist have a duty to be honest and trustworthy in
their dealings with people.
A. Principles of Ethics
B. Code of Professional Conduct
C. Principle of veracity ("truthfulness")
D. Principle of honesty
31. A physician write prescribed for own use or family member for emergency?
A. Unethical B. Illegal C. physician can prescribe can prescribe in emergency
D. physician to refer to emergency

Copyright © 2000-2014 TIPS Inc. Unauthorized reproduction of this manual is prohibited. This manual is being used
during review sessions conducted by PharmacyPrep. 47-5
www.pharmacyprep.com
32. A very frequent customer of your pharmacy, request for 15 days supply of Lipitor 10 mg tablets.
Next few hours he has to take a flight for vacation. Currently all walk-in- clinics are closed and his
physician is not available for prescription. Customer has no prescription and no refills of Lipitor?
Pharmacist refuse to medication for 15 days. What ethical principles are broken?
A. Beneficence
B. Nonmaleficence
C. veracity
D. Justice
E. Autonomy
33. A terminally ill cancer patient. Doctor did NOT informs patient that patient is seriously ill and may
die in few weeks? What ethics violated?
A. Beneficence
B. Veracity
C. Justice
D. Nonmaleficence
E. Fidelity
34) A police officer comes to your pharmacy, wants a patient profile of medication, who caught on
street drugs and in traffic rules violation? What is appropriate action?
A. Ask police ID and patient ID and give patient profile
B. Ask police ID and if he is real police officer give patient profile
C) Refuse to any patient information to police
D) Ask police officer if he has any court letter (search warrant) or letter of authorization from patient
to release patient information.
E) Never talks to police in pharmacy
35. A lawyer calls to your pharmacy wants to get medication information of a divorced spouse
Children? What to do?
A. talk to lawyers, client and provide information
B. Refuse to provide information
C. Children information only can be shared by parents
D. none of the above
36) A phone call from emergency medical services, wants to a patient profile of patient who is found
unconscious in his apartment? What is appropriate action?
A) Ask ID of EMS and refuse to give patient information
B) Never trust any phone calls ask EMS to come personally to pharmacy.
C) Verify and give patient profile to EMS, it is in best interest of patient.
D) Ask EMS to contact to doctor of a patient to get information
E) Ask EMS to contact family member of patient.

Copyright © 2000-2014 TIPS Inc. Unauthorized reproduction of this manual is prohibited. This manual is being used
during review sessions conducted by PharmacyPrep. 47-6
www.pharmacyprep.com
37. A frequent customer, on benzodiazepine for long time, his doctor is away and there are no walk
in clinics open. He wants to get some pills advance? What you do?
A. talk to that person
B. ask patient to see other doctor to get prescription
C. advance benzodiazepine
D. Refer to pharmacist
38. In what situation pharmacist call police?
I) forged prescription
II) Break - in happened
III) narcotic theft
A. I only B. III only C. I and II only D. II and III only E. All are correct
39. A father of two adults children and wife customer of your pharmacy, wants you to print receipts
of their medications for filing tax returns.
A. Get the authorization from wife and adult children
B. Receipts only without specifying medication just total amount can be printed to father
C. Refuse to provide receipts
D. None of the above
40. A physician called to pharmacy to get patient record of medications patient investigational drugs
receipt. Patient is under the care of the doctor. Pharmaceutical company already paid to that doctor
and as technician what you have to reply?
A. need patient authorization
B. If it is not legal way to give to doctor
C. print out investigational drug receipt to the doctor
D. tell doctor to contact with pharmaceutical company
41) Who sets the prices of patented medications?
A) Patented Medicine Price Review Board (PMPRB)
B) Health Canada
C) Manufacturer
D) Pharmacist associations
42. A patient consent may be essential in all, except?
A. To collect patient information
B. To release patient information to third parties
C. To publish clinical trial results of a anticancer drug
D. To prepare prescription by the technician of your pharmacy
E. None of the above

Copyright © 2000-2014 TIPS Inc. Unauthorized reproduction of this manual is prohibited. This manual is being used
during review sessions conducted by PharmacyPrep. 47-7
www.pharmacyprep.com
ANSWERS:
1. D
2. E
3. C
4. B
5. A
6. C
7. C
8. B
9. D
10. E
11. A
12. A
13 A
14. C
15. A
16. C
17. C
Tips: information can be shared in circle of care (doc, Rxst, nurse)
19. A
20. C
21. E
Tips: Three types of consents: express, implied, informed

Copyright © 2000-2014 TIPS Inc. Unauthorized reproduction of this manual is prohibited. This manual is being used
during review sessions conducted by PharmacyPrep. 47-8
www.pharmacyprep.com
22. B
Incompetent: lack of knowledge, skills, judgment
Incapacity: substance abuse or accidents
Negligence: working lower than standard
Misconduct: unethical
Accountability: legally and morally liable when carrying out duty
Advance directives = legal will
23. A
24. C
25.
26.
27. C
28. B
29. D
30. C
31. C
32.
33. B
34. D
35. C
36. C
Tips: It is circle of care, you can share information after confirmation.
37. B
38) E
39) B
40) C

Copyright © 2000-2014 TIPS Inc. Unauthorized reproduction of this manual is prohibited. This manual is being used
during review sessions conducted by PharmacyPrep. 47-9
www.pharmacyprep.com
41. A
42. D

Copyright © 2000-2014 TIPS Inc. Unauthorized reproduction of this manual is prohibited. This manual is being used
during review sessions conducted by PharmacyPrep. 47-10
PharmacyPrep.Com Canadian Healthcare System

PHARMACY PREP
CANADIAN HEALTHCARE SYSTEM
1. All of the following are the principles of Canadian health act (CHA), EXCEPT:
A. Universality
B. Private administration
C. Accessibility
D. Portability
E. Comprehensiveness
2. A national pharmacy regulatory body NAPRA stands for:
A. National Association of Pharmaceutical Regulatory Administration.
B. National Administration of pharmacy regulatory authority
C. National association of pharmacy regulatory authorities
D. National association of pharmacy manufacturer regulatory authority
E. National association of provincial regulatory authorities
3. Who determines the patented prescription drug price?
A. Canada Research Based Pharmaceutical companies (Rx&D)
B. Generic pharmaceutical Association (CGPA)
C. Patented Medicines Prices Review Board (PMPRB)
D. Health Canada (HC)
E. National Association of Pharmacy Regulatory Authority (NAPRA)
4. Regulation or rules of Canadian pharmacies are mainly created by:
A. Provincial authorities
B. Pharmacy college authorities
C. Governmental authorities
D. National Association of Pharmacy Regulatory Authorities (NAPRA)
E. None of the above

5. What is PMPRB?
A. Patented Medicines Prices Review Board
B. Prescription Medicines Prices Review Board
C. Non-Prescription Medicines Prices Review Board
D. Prescription Medicines Patent Review Board
E. None of the above

6. Who sets the prices of over the counter drugs in pharmacy?


A. Patented Medicines Prices Review Board
B. B-Health Canada
C. C-Federal Government
D. D-Manufacturer

Copyright © 2000-2014 TIPS Inc. Unauthorized reproduction of this manual is prohibited. This manual is being used
during review sessions conducted by PharmacyPrep. 48-1
PharmacyPrep.Com Canadian Healthcare System
E. E-Pharmacy Manager/owner

7. What is not included the functions of health Canada?


A. National immunization programs B. Pandemic preparedness
C. New drug approvals D. Senior over 65 years medical coverage E. Regulating drug imports in
Canada
8. What is not included in Canadian health act?
A. Seniors over 65 years medical coverage B. Hospitalization expenses C. immigrants
D. Natives or aboriginals E. Children immunization

Copyright © 2000-2014 TIPS Inc. Unauthorized reproduction of this manual is prohibited. This manual is being used
during review sessions conducted by PharmacyPrep. 48-2
PharmacyPrep.Com Canadian Healthcare System
ANSWERS:
1. B
Tips: Canadian Health Act is single tier system that is based on public administration, not a private
administration.
2. C
3. C
4. D
5. A
6. E
7. D
8. A

Copyright © 2000-2014 TIPS Inc. Unauthorized reproduction of this manual is prohibited. This manual is being used
during review sessions conducted by PharmacyPrep. 48-3
www.PharmacyPrep.Com Pharmacy Practice Regulations

PHARMACY PREP
PHARMACY PRACTICE REGULATIONS
1. What is NAPRA stands for?
A. National association of Pharmaceutical Regulatory Authority
B. National Association of Pharmacy Regulatory Authority
C. National Administration of pharmacy Regulatory authority
D. National Administration of pharmacy Regional authority

2. What drugs require prescription:


A. schedule I
B. Schedule 2
C. Schedule 3
D. Unscheduled
E. none of the above
3. Drug schedules in Canada is controlled by the?
A. Health Canada
B. Pharmaceutical Manufacturer
C. NAPRA
D. National Drug Scheduling Advisory Committee (NDSAC)
E. Registrar of Provincial College of Pharmacy
4. Which of the following fall under NAPRA schedules?
I. Prescriptions
II. OTCs sold with pharmacist intervention
III. OTC sold without pharmacist intervention

A. I only B. III only C. I and II only D. II and III only E. All of the above
5. Tylenol # 1 is:
A. Acetaminophen300mg+ codeine8mg+ Caffeine15mg
B. Acetaminophen300mg+ codeine15mg+Caffeine15mg
C. Acetaminophen300mg+ codeine30mg+Caffeine15mg
D. Acetaminophen300mg+ codeine60mg
E. Acetaminophen 160mg+codeine 8mg+caffeine 15mg

6. A patient comes to pick up lorazepam refills, 15 days before refill time, what action is appropriate?
Patient profile also have insomnia treatment and chronic alcoholic.
A. Ask him to come just before 4 to 5 days before refill
B. Call doctor to get refill date change
C. Ask patient reason for early refill
D. Go to emergency or walk in clinic to get medication
E. Refuse refills and educate patient about compliance

Copyright © 2000 - 2014 TIPS Inc. Unauthorized reproduction of this manual is prohibited. This manual is being
used during review sessions conducted by PharmacyPrep. 49-1
www.PharmacyPrep.Com Pharmacy Practice Regulations
7. Police officer comes to pharmacy, wants a patient profile. Patient caught having illegal drugs
trafficking and found drug sample in his car trunk? As a pharmacist what is appropriate action?
A. Give patient profile to police officer
B. Ask police officer a letter of court warrant or patient authorization.
C. Refuse and inform you cannot release patient information to police
D. Police officer is law enforcement agency, thus you should release all information to police
E. Never release patient information to police
8. A family of 2 adults and two children ages 10 year old and 13 year old. Father comes to pharmacy
and wants you to print receipts all family purchases of the year for tax purpose. What is appropriate
action should be taken?
A. Print all receipt and provide to husband
B. Refuse to print receipts; it is illegal to use prescriptions purchases for taxes
C. Ask authorization from other adult family members
D. None of the above
9. A mother wants to purchase medications for her 2-year-old son on her insurance plan, in which
her son is not covered in policy. What is appropriate response?
A. It is illegal B. It is unethical C. It is appropriate
D. Pharmacist should offer beneficence E. It is nonmaleficence
10. A patient has received; wrong medication from pharmacy, and this was prescription error from
prescriber. Who is responsible?
A. Pharmacist B. Physician C. Pharmacy technician D. A and B E. A, B and C

11. A 70-year-old senior man travelling to Florida on vacation for 6 months. Gets prescription of
Lipitor for 6 months.Wants to cover on his insurance plan. But his insurance covers only for 3
months. What is appropriate action?
A. Talk to insurance company if they cover for 6 months coverage, if they agreed dispense 6 months
supply.
B. Cover 3 months supply on insurance and ask patient to pay cash for 3 months supply
C. Dispense for 3 months now and ask patient to pick 3 months supply after 3 months
D. Call doctor to change for 3 months only
E. None of the above
12. What is true about combination drug acetaminophen 300 mg + codeine 8 mg + caffeine 15 mg
A. It does NOT require prescription and can be given by pharmacist.
B. Require prescription because it contain codeine
C. it is over the counter drug
D. It is straight narcotic and given by written prescription only
E. None of the above

Copyright © 2000 - 2014 TIPS Inc. Unauthorized reproduction of this manual is prohibited. This manual is being
used during review sessions conducted by PharmacyPrep. 49-2
www.PharmacyPrep.Com Pharmacy Practice Regulations
13. All the following is/are true with respect to net worth, EXCEPT
A. It represents the residual claim of the owners of the business.
B. Liabilities and net worth must not equal assets
C. Net worth is the book valve of the claim
D. It is what appears on the balance sheet
E. None of the above

14. The overall gross profit percent depends on:


I) Sales of dispensary
II) Sales of front shop
III) Inventory

A. I only B. III only C. I and II only D. II and III only E. All of the above
15. A minimum rate of turnover is a ratio of
I) 12
II) 5
III) 3

A. I only B. III only C. I and II only D. II and III only E. All of the above
16. Net profit means
I) Gross margin minus the cost of expenses per year
II) Net income before income tax
III) Net income after income tax

A. I only B. III only C. I and II only D. II and III only E. All of the above
17. An inventory control system serves:
I) To keep an accurate record of what is in inventory
II) To determine the amount to be ordered
III) To determine what items need to be purchased.

A. I only B. III only C. I and II only D. II and III only E. All of the above
18. The goals of inventory management is
I) Maximize customer service
II) Maximize profit
III) Maximize inventory investment

A. I only B. III only C. I and II only D. II and III only E. All of the above
19. Benefits of effective inventory management include, EXCEPT
A. Placing more orders
B. Protection against shortages and stock outs
C. Increased customer loyalty by providing convenience
D. Increased sales
E. Save money

Copyright © 2000 - 2014 TIPS Inc. Unauthorized reproduction of this manual is prohibited. This manual is being
used during review sessions conducted by PharmacyPrep. 49-3
www.PharmacyPrep.Com Pharmacy Practice Regulations
20. Excess stock may result
I) Taking too much space
II) Higher risk of expired
III) Higher sales

A. I only B. III only C. I and II only D. II and III only E. All of the above
21. This method of inventory control is the most accurate of cell systems
A. Visual method
B. Wantbook method
C. Periodic method
D. Perpetual method
E. Point of sale system

22. To calculate turnover, the following values must be determined.


I) The value of goods available for sale
II) The cost of goods sold
III) The average inventory

A. I only B. III only C. I and II only D. II and III only E. All of the above
23. Disruptions in the supply of pharmaceuticals may occur from
I) Unexpected increase in utilization of the drug
II) Voluntary discontinuation or recalss of the product by the manufacturer
III) The ft

A. I only B. III only C. I and II only D. II and III only E. All of the above
24. To resolve the problem of drug shortages, you should do all, EXCEPT
I) Adhere to your regular stock saler.
II) Assess how critical the drug is for the patient
III) Exhaust every avenue to supply the patient.

A. I only B. III only C. I and II only D. II and III only E. All of the above
25. Methods of ordering include all, EXCEPT
A. By fax
B. Using an order entry device
C. Point of sale system
D. Borrowed from another pharmacy
E. Placing an order directly with a sale representative

26. When the orders are delivered the following(s) may sign the delivery manifest:
I) a receiving clerk
II) pharmacy technician
III) pharmacist

A. I only B. III only C. I and II only D. II and III only E. All of the above

Copyright © 2000 - 2014 TIPS Inc. Unauthorized reproduction of this manual is prohibited. This manual is being
used during review sessions conducted by PharmacyPrep. 49-4
www.PharmacyPrep.Com Pharmacy Practice Regulations
27. Pareto’s law shows that
I) every item require the same degree of control
II) 20% of all products stocked generate 80% of total sales
III) 15% of inventory stocked generate 15% of all sale.

A. I only B. III only C. I and II only D. II and III only E. All of the above
28. Elemental that can vary from one manufacturer/wholesaler to another include all, EXCEPT
A. Expiry date guidelines
B. Full bottle guidelines
C. Damage specifications
D. Restocking charges
E. Credit

29. The fundamental objective of space management is achieved by:


I) maximizing space utilization
II) impulse buying
III) reducing carts

A. I only B. III only C. I and II only D. II and III only E. All of the above
30. Conducting successful clinic include all, EXCEPT:
A. The topic chosen should be based on client’s needs, your interest and expertise.
B. Lifestyle counseling should be the last one to think
C. Inform patient that benefits most
D. Hold the clinic in the store
E. Use a host

31. All the following is/are true with respect to gross margin, EXCEPT
A. It is the difference between the cost of the goods and the retail selling price.
B. It is also called gross profit
C. The gross margin in the pharmacy is 10-15%
D. Sometimes it is just called margin
E. None of the above

32. The invoice cost is $ 7.00.At 30% mark-up the retail price is :
A. $9.21
B. $10
C. $25
D. $7.21
E. None of the above

33. An item cost $5.00.If the retail price is $7.5%, the gross margin percent is:
A. 40%
B. 15%

Copyright © 2000 - 2014 TIPS Inc. Unauthorized reproduction of this manual is prohibited. This manual is being
used during review sessions conducted by PharmacyPrep. 49-5
www.PharmacyPrep.Com Pharmacy Practice Regulations
C. 35.5%
D. 33.33%
E. None of the above

34. Pharmacy and Therapeutics (PT) committee is all EXCEPT;


A. Nursing staff
B. Technician
C. Director of the hospital or an administrator
D. Representative of the medical staff
E. Public med

35. The following is/are true in implementing pharmaceutical care


I) Performing pharmaceutical care is time consuming
II) Suffers from a lack of suitable reimbursement
III) It is difficult since there is a limited understanding and demand from the public.

A. I only B. III only C. I and II only D. II and III only E. All of the above
36. The Pharmaceutical and Therapeutics (PT) committee is responsible for all, EXCEPT
A. The evaluation and selection of all drugs currently in use in the hospital
B. Setting of use for all medications in the hospital
C. Setting standards with that third part
D. Evaluating drugs based on drug studies , available literature and financial considerations.
E. Education program for staffs

37. Formulary list in the hospital is the responsibility of:


I) Pharmacist
II) Physician
III) Nurse

A. I only B. III only C. I and II only D. II and III only E. All of the above
38. Non-formulary are requested by physicians for the following reasons:
I) New products that are more effective than current therapy
II) A new patient that has been admitted who is using non-formulary medication for an indication
that is not the reason for their admission into hospital.
III) Psychiatric patients may become confused or agitated if medication has been charged .

A. I only B. III only C. I and II only D. II and III only E. All of the above
39. Investigational drugs are all. EXCEPT
A. Non-formulary drugs
B. New products that have been evaluated and tested

Copyright © 2000 - 2014 TIPS Inc. Unauthorized reproduction of this manual is prohibited. This manual is being
used during review sessions conducted by PharmacyPrep. 49-6
www.PharmacyPrep.Com Pharmacy Practice Regulations
C. Forms must be submitted by physicians
D. Not yet been approved for marketing in Canada
E. None of the above

40. In order for the hospital to carry out a study for drugs, should include all, EXCEPT
A. Ensure adequate, safeguard for all integrity of the study and of the patient.
B. Has verbal policies and procedures
C. Ensure all staff comply with these protocols.
D. Patients who participate must really participate.
E. Tell the patient risks and benefits of the study

41. The following is true with respect to emergency drugs:


I) Only obtained in emergency cases
II) Obtained by Special Access Program (SAP)
III) By a form from a physician or a pharmacist

A. I only B. III only C. I and II only D. II and III only E. All of the above
42. Compassionate release drugs include all, EXCEPT
A. Provide on individual basis
B. People unable to pay the drug
C. Patients are eligible for at least one governmental plan.
D. Patients pay dispensing fee only
E. Generally supplied to the hospital pharmacies only

43. In Pareto’s law, class A products represent all, EXCEPT


A. High annual dollar volume
B. High percentage of total inventory
C. High inventory costs
D. Include TPN, IV antibiotics, radio pharmaceuticals, antineoplastic medications.
E. None of the above

44. In Pareto’s law, class B products represent all


I) Large percentage of total inventory
II) Medium inventory costs
III) High annual dollar volume

A. I only B. III only C. I and II only D. II and III only E. All of the above
45. Class C drugs in Pareto’s law include;
I) Low annual dollar volume
II) Low inventory costs
III) Low percent of total inventory

A. I only B. III only C. I and II only D. II and III only E. All of the above

Copyright © 2000 - 2014 TIPS Inc. Unauthorized reproduction of this manual is prohibited. This manual is being
used during review sessions conducted by PharmacyPrep. 49-7
www.PharmacyPrep.Com Pharmacy Practice Regulations
46. For class C items according to Pareto’s law, all the following are true, EXCEPT
A. A 30-day supply product
B. Datrium is of the products that fit in this category
C. The buyer try to increase inventory.
D. The products in this category are absolutely needed although they are expensive.
E. None of the above

47. In preparing the drug budget in hospitals who have a drug budget, the following is/are
considered, EXCEPT
A. Changes in services required (i.e. TPN/IV admixture, programs)
B. Changes in patient population
C. Increase in drug costs
D. Essential returnable drugs
E. Trends in drug usage

48. Purchase records provide all, EXCEPT


A. Change in services provided (TPN/IV admixture programs)
B. Value of drug purchases
C. Inventory turnover rate
D. Trends in drug usage
E. Re-order point

49. The inventory manager in a hospital pharmacy is:


A. Any associate
B. Personnel manager
C. Pharmacist
D. Technicians
E. None of the above

50. The criteria for supplier assessment by each institution should include:
I) Availability of acceptable returned-good policies.
II) Recent back order history
III) Accessibility of supplier representatives.

A. I only B. III only C. I and II only D. II and III only E. All of the above
51. In selecting a particular brand of a drug used in the hospital, the criteria should include:
I) Drug packaging
II) Drug labeling
III) Determination of bioavailability

A. I only B. III only C. I and II only D. II and III only E. All of the above

Copyright © 2000 - 2014 TIPS Inc. Unauthorized reproduction of this manual is prohibited. This manual is being
used during review sessions conducted by PharmacyPrep. 49-8
www.PharmacyPrep.Com Pharmacy Practice Regulations
52. Regular sources of supply for the hospital includes:
I) Wholesaler /Distributor
II) Manufacturer
III) Another hospital

A. I only B. III only C. I and II only D. II and III only E. All of the above
53. Consolidated distribution services has the following characteristics, EXCEPT
A. Pharmaceuticals can be purchased easily and cost effectively.
B. Ordering is easy using a number of options.
C. Delivery is within working hours only.
D. Combines multiple vendors on one purchase order.
E. No minimum order, processing or transportation fee.

54. The advantages of the use of prime vendor or wholesale distributor have all the following,
EXCEPT:
A. Fewer purchase order
B. Higher investment inventory
C. High stock turn over
D. Higher stock control
E. Fewer payment cheques

55. In purchasing the items , the pharmacist must exercise:


I) Professional responsibility
II) Moral responsibility
III) Legal responsibility

A. I only B. III only C. I and II only D. II and III only E. All of the above
56. The following is/are characteristics of out-dated products:
I) They are issued to or stocked on the nursing units.
II) On occasion, a manufacturer may notify the pharmacy to grant an extension on expiry date of
a product.
III) Credit for returns are arranged with the sales representative.

A. I only B. III only C. I and II only D. II and III only E. All of the above
57. All the following is/are true respect to the vault:
I) In a separate area with limited access
II) Kept locked
III) Wired into an alarm station

A. I only B. III only C. I and II only D. II and III only E. All of the above
58. Goal of technology is all, EXCEPT
A. Improve efficiency
B. Improve resource allocation
C. Improve accurate medication distribution

Copyright © 2000 - 2014 TIPS Inc. Unauthorized reproduction of this manual is prohibited. This manual is being
used during review sessions conducted by PharmacyPrep. 49-9
www.PharmacyPrep.Com Pharmacy Practice Regulations
D. Perfect inventory management

59. Which of the following methods of maintaining a formulary is more convenient and yields the
most current product information in a purchasing and inventory system?
A. bound formulary book
B. on-line formulary
C. Closed formulary
D. Open formulary

60. Which of the following drug comply with the requirements of the Narcotic Control Regulations?
A. Alprazolam
B. Tylenol # 1
C. Ketamine
D. Amphetamine
61. What is PHIPA stands for?
A. Personal Health Information and Privacy Act
B. Pharmacy Health Information and Privacy Act
C. Pharmacy Health Information and Problem Act
D. None of the above
62. Which of the following drugs prescription refill/repeats are NOT allowed?
A. Benzodiazepine
B. Methylphenidate
C. Narcotic drugs
D. Hydrochlorothiazide/Ramipril
63. A physician prescribes a specified quantity of medication and directs that the medication be
dispensed in smaller portions until the total quantity is used, is defined as?
A. Refill
B. Repeats
C. Part fill
D. Interchangeable
64. What is correct about the following prescription
Rx
Tylenol® #3 Mitte: 500 Sig: 1 q4h Dispense 100 at a time every 30 days (intervals recommended but
not a requirement)

Copyright © 2000 - 2014 TIPS Inc. Unauthorized reproduction of this manual is prohibited. This manual is being
used during review sessions conducted by PharmacyPrep. 49-10
www.PharmacyPrep.Com Pharmacy Practice Regulations
A. The part fill prescription for 100 tablets of Tylenol® #3 can be dispensed every 30 days to a final
total of 500 tablets.
B. The part fill prescription for 30 tablets of Tylenol® #3 can be dispensed every 30 days to a final
total of 500 tablets.
C. The part fill prescription for 500 tablets of Tylenol® #3 can be dispensed every 30 days to a final
total of 1500 tablets.
D. None of the above
65. What practitioners may legally cannot prescribe controlled substances
A. Dentist
B. Veterinarian
C. Physician
D. Optometrist
66) Drug schedules in Canada is controlled by the?
A-Health Canada
B-Pharmaceutical Manufacturer
C-Pharmacy examination board of Canada
D-National Drug Scheduling Advisory Committee (NDSAC)
E-Registrar of Provincial College of Pharmacy
67) A patient requests a refill for Diazepam 10mg. Upon reviewing his profile, a pharmacy reveals
that the prescription for the requested drug has been refilled 5 times. The pharmacist will:
A. Refill the prescription since it has been written for five times.
B. Asks the patient to contact doctor
C. Refuse to fill since the strength of the requested medication is commercially unavailable.
D. Refuse to refill because benzodiazepine refills are NOT allowed

Ans: A
Tips: Benzodiazepine prescription can be refilled any number of times. However it should not beyond
the one year from the date of prescription

Copyright © 2000 - 2014 TIPS Inc. Unauthorized reproduction of this manual is prohibited. This manual is being
used during review sessions conducted by PharmacyPrep. 49-11
www.PharmacyPrep.Com Pharmacy Practice Regulations
ANSWERS:
1. B
2. A
3. D
Tips: NDSAC is a national drug schedule advisory committee, which a part of NAPRA.
4. E
5. A
Tips: Tylenol#1, No prescription needed, Pharmacist intervention required
Schedule II drug.
6. C
7. B
8. C
9. A
10. D

11. A
12. A
Tips: Straight narcotic (single opioid alone: morphine, codeine): written rx only. No refills. (Tylenol 4)
Verbal narcotic or narcotic preps: 1 opioid + 2 non opioids drugs combination: Tylenol 2
(acetaminophen 300 mg+15mg codeine + 15 mg caffeine), Tylenol # 3.
Exempted narcotics or OTC narcotics: (1 opioid (8 mg)+ 2 non opioid); Tylenol 1 (codeine
8mg+Acetaminophen 300 mg + 15 mg caffeine), codeine 19.8 mg/30 ml+ 2 non opioid
No prescription necessary, schedule 2
13. B
Tips: Liabilities and net worth must equal assets.
14. C
15. B
The optimal rate is 12

Copyright © 2000 - 2014 TIPS Inc. Unauthorized reproduction of this manual is prohibited. This manual is being
used during review sessions conducted by PharmacyPrep. 49-12
www.PharmacyPrep.Com Pharmacy Practice Regulations
16. C
17. E
18. D
Tips: Should minimize inventory investment.
19. A
Save money by placing fewer orders.
20. C
21. D
Tips: Perpetual is the most effective means of inventory control. This type of system records
sales as they occur, calculating increases in inventory as items are purchased or returned and
subtracting items as sold.
22. E
Tips: Orders may be sent by tax or by phone or put manually on the electronic system (i.e. not
automatically ordered)
23. C
24. D
25. D
Tips: Emergency supplies only may be purchased or borrowed from another pharmacy.
26. E
27. D
28. B
Tips: Part bottle guidelines
29. E
30. B
Lifestyle counseling is a good place to start since diet and exercise apply to most disease states.
31. C
Tips: It varies from 15-40%

Copyright © 2000 - 2014 TIPS Inc. Unauthorized reproduction of this manual is prohibited. This manual is being
used during review sessions conducted by PharmacyPrep. 49-13
www.PharmacyPrep.Com Pharmacy Practice Regulations
32. B
Tips: 100%-30%=70% i.e. 70/10=0.7
Thus 7/.7=70/7=$10
33. D
Tips: (7.5-5)=(2.5x100) / 7.5=33.33%
34. E
35. E
36. C
37. A
Tips: It is pharmacy’s responsibility to carry the drugs listed in the formulary and physicians and
residents. Check the formulary to determine which drugs are available.
38. E
39. A
Tips: Investigational drugs are not marketed since they are not approved.
40. B
Tips: The study should have written policies and procedures.
41. D
Tips: The requesting physician must fill out the forms or a pharmacist on behalf of the physician.
42. C
Tips: These people are not eligible for any governmental programs or benefits that would help
subsidize the product.
43. B
Tips: It includes small percentage of total inventory.
44. C
Tips: It includes medium annual dollar volume.
45. C
Tips: It includes high percent of total inventory.
46. C

Copyright © 2000 - 2014 TIPS Inc. Unauthorized reproduction of this manual is prohibited. This manual is being
used during review sessions conducted by PharmacyPrep. 49-14
www.PharmacyPrep.Com Pharmacy Practice Regulations
Tips: The buyer tries to reduce inventory, if possible and would have a 30-day supply on hand.
47. D
Tips: Hidden costs like essential non-returnable drugs which are very costly and required in high
quantities, and has short dating and cannot be returned for credit like Datrium.
48. A
49. D
Tips: A pharmacy technician usually is the inventory manager.
50. E
51. E
52. C
Tips: For emergency supplies, a hospital may purchase drugs from another hospital or a nearby
community pharmacy.
53. C
Tips: Delivery is within 24 hours.
54. B
Tips: It has a lower investment inventory.
55. E
56. D
Tips: Out dated drugs are never issued or stocked on nursing units.
57. D
58. E
Tips: Technology is a useful tool to enhance the goal of inventory management, but not perfect.
59. B
Tips: An on-line computerized formulary is generally more current than printed formularies and can
be searched more rapidly.
60. C

Copyright © 2000 - 2014 TIPS Inc. Unauthorized reproduction of this manual is prohibited. This manual is being
used during review sessions conducted by PharmacyPrep. 49-15
www.PharmacyPrep.Com Pharmacy Practice Regulations
61. A
62. C
63. C
64. A
65. D
66) D
Tips: NDSAC is a national drug schedule advisory committee, which a part of NAPRA.

Copyright © 2000 - 2014 TIPS Inc. Unauthorized reproduction of this manual is prohibited. This manual is being
used during review sessions conducted by PharmacyPrep. 49-16
www.pharmacyprep.com

PHARMACY PREP
PHARMACY PRACTICE MANAGEMENT
1. Pharmacy ABC has sales of prescription medication for the year 2007 to 2008. In order to find out
profit of pharmacy ABC, which of the following financial statement may be useful:
A. Balance sheet
B. Retain earning statement
C. Income statement
D. Bank statement
E. Personal statement
2. All of the following include as part of income statement, EXCEPT:
A. Sales
B. Profit
C. Cost of goods sold
D. Account payable
E. Gross margin
3. Financial statement that is total income minus expenses is:
A. Income statement B. Profit statement C. Retained statement D. Net income E. Income sheet
5. A financial statement that is used to determine the total value of Rxs drug sales for a pharmacy
during the course of a year is known as:
A. Income statement B. Profit statement C. Retained statement D. Net income E. Balance sheet
6. Delegation includes:
I) Delegating a project to technician
II) Supervising technician in project
III) Monitoring completion of project
A. I only B. III only C. I and II only D. II and III only E. I, II, III
7. Negligence may be described as:
A. Refusing to fill prescription for control drug substance
B. Refusing to fill narcotic drug prescription
C. Unable to meet the standard of the average pharmacist
D. Refuse to disclose patient information to unidentified person
E. Avoiding a patient who is difficult to handle and argumentative

Copyright © 2000-2014 TIPS Inc. Unauthorized reproduction of this manual is prohibited. This manual is being used
during review sessions conducted by PharmacyPrep. 51-1
www.pharmacyprep.com
8. The manager of the pharmacy made a list of work that should be done by his employees in order
of priority, this is called
A. Delegation B. Job analysis C. Job description D. Negligence
E. None of the above
9. How can you provide the best counseling to your patient?
I- The pharmacy assistant gives an information sheet to the patient
II- The pharmacist explain how to use the drug
III- The pharmacist discuss with patient side effects, therapy and listening the expectations of patient
A. I only B. III only C. I and II only D. II and III only E. I, II, and III
10. A senior pharmacy technician has a worked in a community pharmacy for 10 years. He is well
trained and welcomes new learning opportunities. His manager now wanting to delegate a new
technical project to him, should:
I) Discuss suggested steps to accomplish the project
II) Negotiate a time schedule for completion of the project
III) Provide support and follow up as required
A. I only B. III only C. I and II only D. II and III only E. I, II, and III
11. Which of the following is the most important component of business plan:
A. Location analysis B. Market area analysis C. Site analysis D. Product analysis E-None of the above
12. What is POS inventory management:
A. Point of science B. Point of society C. Point of sale D. Point of submit
E. All of the above
13. Computer software and hardware program very useful in pharmacies that combines and track all
the business information as product information, cash register, inventory, stocks and so on.
A. Pharmacare system B. Health Watch system C. Point of sale (POS)
D. Universal Pharmacy system E. Business update computer system
14. What is DIN:
A. Drug Identification Notice B. Notice of compliance C. Drug identification number D. Drug
information number E. Drug Identity Notice
15. Which of the following is not a type of ownership?
A. Incorporation

B. Sole proprietorship
C. Leasing
D. Franchise
E. Cooperative businesses

Copyright © 2000-2014 TIPS Inc. Unauthorized reproduction of this manual is prohibited. This manual is being used
during review sessions conducted by PharmacyPrep. 51-2
www.pharmacyprep.com
16. Which of the following is not a type of pharmacy?
A. Banner pharmacy
B. Franchise pharmacy
C. Central fill pharmacy
D. Mail order pharmacy
E. Rental pharmacy license
17. Total sales of prescription drugs are $210,000 and cost of good sold is 180,000. What is gross
profit?
A. 71% B. 14% C. 50% D. 20% E. 25%
18. Net income is?
A. Total income from sales
B. Gross income + total expenses
C. Gross income - total expenses
D. Gross income + total expenses + current assets
E. Gross income + total expenses + current assets + Fixed assets

19. Which of the following is not a form of business


A. Incorporation
B. sole proprietorship
C. leasing
D. Franchise
20. Which of the following is the most important in pharmacy location analysis:
A. Trading area
B. Market area analysis
C. Site consideration
D. Region
E. Geographical area

21. Which of the following is the most important in pharmacy site consideration:
A. Sales per square feet
B. Convenience and distance
C. Parking space
D. Physical characteristics of space
E. Use of ratios

22. What is the test marketing in pharmacy:


A. through new papers
B. through community papers
C. through TV
D. Door to door flyers
E. Distribution of flyer in mall

Copyright © 2000-2014 TIPS Inc. Unauthorized reproduction of this manual is prohibited. This manual is being used
during review sessions conducted by PharmacyPrep. 51-3
www.pharmacyprep.com
23. Pharmacy ABC have sales of prescription medication for the year 2004 to 2005. In order to find
out profit of pharmacy ABC, which of the following financial statement may be useful:
I-Balance sheet
II-Retain earning statement
III-income statement or profit & loss statement
A. I only B. III only C. I and II only D. II and III only E. I, II, and III
24. All of the following include as part of income statement, EXCEPT:
A. sales B. profit C. cost of goods sold D. account payable E. gross margin
25. Which of the following is not covered in total assets:
I-Pharmacy furniture
II-Account receivable
III-Account payable
A. I only B. III only C. I and II only D. II and III only E. I, II, and III
26. The style of management in which owner of pharmacy store emphasizes the development of
detailed policies and written procedure for employees to observe at all times is referred as:
A. bureaucraticwork procedure are described in details policies and written procedure.
B. autocraticImplementing personal opinion without consultations.
C. assertive
D. consultativewith consultation
E. benevolentgenerous and kind
27. The inventory turnover rate is a measure of a pharmacy’s
A. liquidity B. solvency C. Efficacy D. Profitability
28) The cost of goods sold is $150,000, at start of year $25,000 at the end of year 50,000 the
inventory capital. What is the turnover rate?
A) 4 B) 3 C) 2 D) 1 E) 5
29. The previous turnover rate was 6 and this year cost of good sold is $150,000, at start of year
$25,000 at the end of year 50,000 the inventory capital. What is the difference of turn over rate this
year from previous year?
A. 4 B) 3 C) 2 D. 1 E. 5
30. What is the normal turnover rate of retail pharmacy?
A. 3 B. 7 C. 5 D. 1

Copyright © 2000-2014 TIPS Inc. Unauthorized reproduction of this manual is prohibited. This manual is being used
during review sessions conducted by PharmacyPrep. 51-4
www.pharmacyprep.com
31. Turnover rate (TOR) is indicator of:
A. Profit B. Efficiency C. Loss D. Inventory status
E. Merchandising problems
32. In a market there is only one seller and there are many buyers. This competition described as?
A. Monopoly B. Monopolistic competition C. Oligopoly
D. Oligopolistic competition
33. All of the following are essential of business, except?
A. Promotion B. Place C. Product D. Price E. Transportation
34. If you open a pharmacy in multicultural area, what you should do increase your sales?
A. you have signs of multicultural
B. Hire technician from same cultures
C. Develop literature with different language
D. Offer help in different languages
35. Which of the following structure of pharmacies is difficult open and operate?
A. Sole ownership B. Franchise C. Banner pharmacy D. Partnership
36. If pharmacy manager for 16 year he is doing all the job by himself, without taking any idea and
skills from other, what is that called?
A. Beaurocratic B-Autocratic C-Independent D-Assertive E-Submissive
ABC pharmacy has following figures appearing on its balance sheet.
Cost of goods sold $500,000
Beginning inventory $200,000
Ending inventory $220,000
Total current assets $150,000
Total fixed assets $40,000
Total liabilities $75,000
37. Inventory turnover rate for ABC pharmacy is
A. 4.3
B. 2.3
C. 3.5
D. 6.0
E. 8.0

38. Net worth of ABC pharmacy is


A. $195,000
B. $225,000

Copyright © 2000-2014 TIPS Inc. Unauthorized reproduction of this manual is prohibited. This manual is being used
during review sessions conducted by PharmacyPrep. 51-5
www.pharmacyprep.com
C. $115,000
D. $75,000
E. $110,000

39. Liquidity generally expresses a pharmacy’s abilities to meet its


A. Assets
B. Current liabilities
C. Inventory
D. Prepaid expense
E. Desired inventory levels

40. All of the following are important fundamental principles of marketing EXCEPT:
A. Product
B. Price
C. Profession
D. Promotion
E. Place

41. What is NOT business structure in pharmacy?


A. Sole proprietorship B. Corporation C. Leasing a pharmacy D. Partnership E. Limited
42. Drug identification number is given to each drug that is sold in Canada. This is assigned by the?
A. Pharmacy Manager
B. Manufacturer
C. Health Canada
D. Pharmacy owner
E. None of the above
43. Which is true about franchise, except?
A. has only one owner
B. has many owners
C. same name
D. same standard operating procedure (SOP)
44. When pharmacy manager describes the position in pharmacy. The qualifications required and
the duties required in the position are called?
A. Job analysis
B. Job posting
C. Position allocation
D. Job description
E. Job finding

Copyright © 2000-2014 TIPS Inc. Unauthorized reproduction of this manual is prohibited. This manual is being used
during review sessions conducted by PharmacyPrep. 51-6
www.pharmacyprep.com
45. Inventory efficiency is measured by?
A. Net sales
B. Liquidity
C. Solvency
D. Turnover rate
E. Gross margin
46. A person brings prescription for 6 months of medical supplies, before he travels. However, his
insurance covers only 3 months. What is appropriate?
A. Insurance may cover for 3 months, rest a customer should pay
B. Call insurance, to verify if customer is covered for vacation supply
C. Do not dispense for 6 months
D. None of the above
47. A retail store general manager, what should we do all except?
A. reading resume
B. interviewing
C. Selecting candidate
D. placing advertisement
E. responsible for narcotics
48. Canada Health Acts includes all, EXCEPT
A. Accessibility
B. Affordability
C. Universality of population covered
D. Public administration on non-profit basis
E. Comprehensiveness of insured service covered

49. Types of business ownership include:


I) Sole ( single) proprietorship
II) Partnership
III) Corporations

A. I only B. III only C. I and II only D. II and III only E. I, II, and III
50. Pharmacy formats include:
A. Independent
B. Banner
C. Franchise
D. Food stores and mass merchandisers
E. All of the above

51. Banner pharmacies include all, EXCEPT


I) Independent pharmacies with a central office
II) They used a recognized name

Copyright © 2000-2014 TIPS Inc. Unauthorized reproduction of this manual is prohibited. This manual is being used
during review sessions conducted by PharmacyPrep. 51-7
www.pharmacyprep.com
III) Do not participate in central buying and marketing

A. I only B. III only C. I and II only D. II and III only E. I, II, and III
52. The characteristics of a chain pharmacies include all, EXCEPT
A. A chain is not necessarily a member of the Canadian Association of chain drug stores.
B. An individual or corporation must own at least 3 stores to be considered a chain.
C. Head office directs all marketing and merchandising.
D. They employ pharmacy managers who are salaried employees of head office.
E. None of the above

53. A business plan requires all the following information, EXCEPT


A. Demographics
B. Type of business
C. Location
D. Organizational structure
E. 10 years plan

54. All the following is/are true with respect to the balance sheet, EXCEPT
A. It is prepared at the end of each fiscal year
B. It is governed by assets, liabilities, and networth
C. Not all business must have a balance sheet
D. It shows the financial position at a given date
E. None of the above

55. All the following is/are true with respect to net worth.
I) It is the arithmetical difference between the assets and liabilities.
II) It is also called owner’s equity
III) The assets may be tangible or intangible

A. I only B. III only C. I and II only D. II and III only E. I, II, III
56. All the following(s) are true with respect to fixed assets
A. they are tangible and used for operating business
B. goods that are purchased for resale
C. operation expenses
D. cash in account
E. sale products in pharmacy
57. What is average turnover rate of hospital pharmacy inventory?
A. 2 B. 5 C. 1 D. 9 E. 15

Copyright © 2000-2014 TIPS Inc. Unauthorized reproduction of this manual is prohibited. This manual is being used
during review sessions conducted by PharmacyPrep. 51-8
www.pharmacyprep.com
58. Total sales is = $530,000; Cost of goods sold = $350,000
What % of gross margin ?
A. 50% B. 30% C. 20% D. 100% E. none
59. Cost of goods sold is $2,000,000; Start of year inventory capital $400,000; End of year inventory
capital is $600,000. What is turnover rate?
A. 9 B. 6 C. 4 D. 10 E. 5

60. All the following are the basic principles for the Canada Health Act except:

A. Accessibility

B. Portability of benefits

C. Public administration on profit basis

D. Comprehensiveness of insured services covered

61. The principle federal acts legislating pharmacy are:

A. The Controlled Drugs & Substances Act.

B. The Food and Drug Act.

C. The Personal Health Information Protection Act

D. Excise Act.

62. Secure e-script delivery system require consideration:

A. Transaction integrity

B. Data integrity

C. Secure routing

D. All of the above

63. The most expensive & fastest growing sector of pharmacy technology is by filling Rx by:

A. Pharmacist

B. Technicians

C. Other associates

D. Robotics
64. Cash consists of:

A. Money bills and coins

B. Cheques

C. Credit card

Copyright © 2000-2014 TIPS Inc. Unauthorized reproduction of this manual is prohibited. This manual is being used
during review sessions conducted by PharmacyPrep. 51-9
www.pharmacyprep.com

D. All of the above

65. All the following is/are correct with respect to Flow except:

A. Flow refers to the rate of movement or change in assets during an operating period.

B. A cash flow statement is usually prepared annually.

C. It is important for budgeting.

D. It is important for purchasing & for borrowing money.

66. A business requires all except:

A. Demographics

B. Location

C. Market Analysis

D. 1 year plan

67. A balance sheet has all the following characteristics except:

A. Done at the end of each month.

B. Every business must have it.

C. It reflects a financial position at a given date.

D. It shows the assts & liabilities of the business.

68. All the following are true for net worth except:

A. It is the assets plus the liabilities.

B. It is the amount owners have invested in the business.

C. It is the capital and any earnings which have been retained and not distributed to owners.

D. It is the arithmetical difference between the assets and the liabilities.

69. Liabilities may include:


A. Account payable & Accrued

B. Employee’s deductions payable

C. Retail sales tax

D. All of the above.

70. The following applies for inventory turnover rate except:

A. It is used for financial analysis.

B. Cost of goods sold divided by the average inventory at cost.

Copyright © 2000-2014 TIPS Inc. Unauthorized reproduction of this manual is prohibited. This manual is being used
during review sessions conducted by PharmacyPrep. 51-10
www.pharmacyprep.com

C. This ratio will increase or decrease in direct proportion with purchase volume.

D. A ration of 3 is considered a minimum rate of turnover.

71. An inventory control of a pharmacy includes the following:

A. Serves to keep an accurate record of what is in inventory.

B. Serves to determine what items need to be purchased.

C. Serves to determine the amount to be ordered.

D. All of the above.

72. A special mouthwash sells for 29.99. The mark up is 50%. Our cost is:

A. $10

B. $15

C. $20

D. $5

73. In the previous question if we buy 12 bottles, the manufacturer will give us one bottle free. How
much do we save on each bottle:

A. 1.25

B. 2.50

C. 0.75

D. 0.50

74. In the previous question if we sell all 13 bottles for $29.99. What is our gross margin profit?

A. $16.24

B. $20.00

C. $13.75

D. None of the above.


75. A case of Gravol costs 49.55 the pharmacy has a choice between a 1-5% discount from the
wholesaler or a free bottle from the sales representative. Which provides us with the best price?

A. Wholesaler

B. Representative

76. A supplier offers your pharmacy a discount in the form of free goods. By purchasing ten dozen
bottles of Tylenol extra Strength, you will receive one dozen additional

Copyright © 2000-2014 TIPS Inc. Unauthorized reproduction of this manual is prohibited. This manual is being used
during review sessions conducted by PharmacyPrep. 51-11
www.pharmacyprep.com
bottles free. If the regular unit cost is $2.99, what is the actual unit cost paid by the pharmacy?

A. 4.71

B. 2.71

C. 4.98

D. None of the above

77. The main difference between hospital pharmacy inventory and community pharmacy inventory
is:

A. Turnover rate in hospital pharmacy is much higher than the community pharmacy.

B. Hospital pharmacy more staff employees than the community pharmacy.

C. The pharmacy area of hospital pharmacy is larger than the community pharmacy.

D. No sales records in the hospital pharmacy.

78. The following is true with respect to hospital formulary:

A. Approved by the physicians working in the hospital.

B. Approved only by the manager of the pharmacy.

C. Approved by P&T committee.

D. Approved by physicians and nursing staff.

79. The following applies to non-formulary drugs

A. Special forms must be filled out.

B. Submitting a formulary addition request.

C. Patient brings their own medication for the nurse to administer.

D. All of the above.

80. Non-formulary items are requested in the hospital by doctors for the following reasons, except:

A. A new product is released in the market that is proven to be more effective than the
current therapy.

B. New patients admitted that use non-formulary drug.

C. For cancer patients.

D. For psychiatric patients.


81. When a team is considering preparing a budget for a hospital pharmacy. All is/are true except:

Copyright © 2000-2014 TIPS Inc. Unauthorized reproduction of this manual is prohibited. This manual is being used
during review sessions conducted by PharmacyPrep. 51-12
www.pharmacyprep.com
A. Changes in services (HIV clinic, Diabetic clinic)

B. Changes in demographics of area.

C. Trends in drug use.

D. More advertisements.

82. The following items is /are considered hard to be predicted in a hospital pharmacy.

A. Timely fashion requirement for drugs & supplies.

B. Highest quality products at lowest available price.

C. Lowest financial investment.

D. All laws & regulations of federal & provincial are followed.

83. All the following is/are true with respect to the storage of medications except:
o
A. Cold not to exceed 10 C
o
B. Freezer (-10 to -20 C)
o
C. Cool (8-15 C)
o
D. Warm (30-4 C)

84. The following is true with respect to storage of narcotics except:

A. Must segregate stock from regular stock.

B. Must be kept in separate area under lock and key with limited access.

C. Must keep perpetual inventory of narcotics.

D. Can be easily audited.

85. All the following is/are true with automation except:

A. Improve efficiencies.

B. Increase costs.

C. Improve resource allocation.

D. Improve accurate medication distribution, and safe medication use.

86. Protection of medications involve all except:

A. Protect from excessive heat.


B. Protect from freezing.

C. Protect from wind.

D. Protect from wet place.

87. All the following is true with respect to benefits of effective inventory management except:

A. Saved money

B. Saved staffing

Copyright © 2000-2014 TIPS Inc. Unauthorized reproduction of this manual is prohibited. This manual is being used
during review sessions conducted by PharmacyPrep. 51-13
www.pharmacyprep.com
C. More expired date drug checked

D. Less stock outs

88. The following is an obstacle to the full implementation of bar code technology:

A. Staff of pharmacy

B. Schedule 2 products

C. Injection vials & unit dose packages

D. Unscheduled medications

89. The following is true:

A. Pharmacies generally buy most of their prescription medications from one major supply.

B. Pharmacies generally buy most of their prescription medication from two or more major
supplies.

C. Pharmacies, generally buy most of their prescription medications from manufacturer.

D. Most pharmacies place order every week.

90. Disruptions in the supply of pharmaceuticals may occur from all except:

A. Disaster conditions

B. Recalls

C. Outbreaks

D. Increase in population

91. You are a technician in a busy pharmacy and you have drug shortages. To resolve the problem
you should tell the pharmacist:

A. Exhaust every avenue to supply the medication.

B. Select an alternative medication, if required.

C. Determine whether the patient can tolerate the new drug.

D. All of the above.

92. Methods of ordering medications should go in all these ways except:

A. Via computer

B. By telephone
C. By (POS) point of sale

D. Borrowed from another pharmacy.

93. A product that is sent from the wholesaler to the pharmacy & you found out that the item
received is damaged or shipped in error or recalled. You should do an action like:

Copyright © 2000-2014 TIPS Inc. Unauthorized reproduction of this manual is prohibited. This manual is being used
during review sessions conducted by PharmacyPrep. 51-14
www.pharmacyprep.com
A. Inform the customer service at weekend.

B. Must contact customer service within 2 business days.

C. Inform the customer service with the invoice number.

D. B&C

94. All the following apply to return of narcotic inventory except:

A. Prior return authorization must be obtained from the narcotics department.

B. Must be packaged separately from other items.

C. Maybe returned to any branch of wholesaler.

D. Should have narcotic label.

95. All the following applies to merchandising of medications except:

A. Promoting items at attractive prices.

B. Targeting customers.

C. Surveying the competition.

D. Purchasing expensive items.

96. The basic objective of space management is to contribute to the achievement of highest possible
sales volume at the lowest possible cost. This can be achieved by all except:

A. Minimizing space utilization.

B. Encouraging the customer to buy additional merchandise.

C. Reducing costs.

D. Elimination of losses of merchandise through pilferage, theft and breakage.

97. Shrinkage refers to:

A. Any decrease in the inventory due to sales.

B. Any decrease in the inventory due to transfer to another pharmacy.

C. Override of a sale.

D. None of the above.

98. What is the retail price at 33.3% mark up when the invoice is $12?
A. $12

B. $9

C. $18

D. $6

99. If the cost of an item is 6.00 and the desired gross margin is 40%. What is the retail price?

Copyright © 2000-2014 TIPS Inc. Unauthorized reproduction of this manual is prohibited. This manual is being used
during review sessions conducted by PharmacyPrep. 51-15
www.pharmacyprep.com
A. 15

B. 20

C. 30

D. 10

100) An OTC item sells for $15.90, with a mark up at 33.3%, the cost is:
A) $10.7
B) $15.7
C) $20
D) $11.9
101. The cost of an item is $12 and the retail price is $16. The mark up on cost is?

A. 66.7

B. 33.3

C. 50

D. None of the above.

102. Investigational drugs have the following characteristics:

A. Only for research work.

B. Approved by Health Canada

C. Not yet evaluated and tested.

D. None of the above.

103. In preparing the drug budget in hospitals, the following(s) should be considered.

A. Changes in patient population

B. Trends in drug usage

C. Changes in services provided

D. All of the above

104. The following is the characteristic of perpetual inventory record in the hospital.

A. It could be maintained for all medications.

B. Purchases are added to the daily total and issues to the floors are subtracted.

C. Once a month, the actual quantity is counted.

D. All of the above.


105. Consolidated Distribution Service (CPDN/RCDP) has the following characteristics except:

A. Delivery is within 3 days.

B. Combines multiple vendors on one purchase order.

Copyright © 2000-2014 TIPS Inc. Unauthorized reproduction of this manual is prohibited. This manual is being used
during review sessions conducted by PharmacyPrep. 51-16
www.pharmacyprep.com
C. No long term commitment required.

D. Fax and mail orders are processed as they are received.

106. Purchase record in the hospital pharmacy provide all the following except:

A. Inventory turnover rate

B. Re-order point

C. Value of drug sales

D. Annual usage of individual drug items

107. Pre-authorized resalable products should have all the following characteristics except:

A. Products must have an expiry date of 3 months.

B. Refrigerated and injectable products will not be accepted as pre-authorized resalable.

C. Invoice number and expiry date must be indicated.

D. Handling charges are approximately 10% for Rx/Narcotics, 15% for OTC.

108. Liabilities must include:

A. Accounts payable

B. Loans payable

C. Accrued expenses

D. All of the above

109. In preparing the drug budget in hospitals who have a drug budget, the following is/are
considered, EXCEPT
A. Changes in services required (i.e. TPN/IV admixture, programs)
B. Changes in patient population
C. Increase in drug costs
D. Essential returnable drugs
E. Trends in drug usage

110. Purchase records provide all, EXCEPT


A. Change in services provided (TPN/IV admixture programs)
B. Value of drug purchases
C. Inventory turnover rate
D. Trends in drug usage
E. Re-order point
111. The inventory manager in a hospital pharmacy is:
A. Any associate
B. Personnel manager

Copyright © 2000-2014 TIPS Inc. Unauthorized reproduction of this manual is prohibited. This manual is being used
during review sessions conducted by PharmacyPrep. 51-17
www.pharmacyprep.com
C. Pharmacist
D. Technicians
E. None of the above
112. The criteria for supplier assessment by each institution should include:
I. Availability of acceptable returned-good policies.
II. Recent back order history
III. Accessibility of supplier representatives.
A. I only B. III only C. I and II only D. II and III only E. I, II, and III
113. In selecting a particular brand of a drug used in the hospital, the criteria should include:
I. Drug packaging
II. Drug labeling
III. Determination of bioavailability
A. I only B. III only C. I and II only D. II and III only E. I, II, and III
114. Regular sources of supply for the hospital include:
I. Wholesaler /Distributor
II. Manufacturer
III. Another hospital
A. I only B. III only C. I and II only D. II and III only E. I, II, and III
115. Consolidated distribution services has the following characteristics, EXCEPT
A. Pharmaceuticals can be purchased easily and cost effectively.
B. Ordering is easy using a number of options.
C. Delivery is within working hours only.
D. Combines multiple vendors on one purchase order.
E. No minimum order, processing or transportation fee.
116. The advantages of the use of prime vendor or wholesale distributor has all the following,
EXCEPT:
A. Fewer purchase order
B. Higher investment inventory
C. High stock turn over
D. Higher stock control
E. Fewer payment cheques
117. In purchasing the items , the pharmacist must exercise:
I. Professional responsibility
II. Moral responsibility
III. Legal responsibility
A. I only B. III only C. I and II only D. II and III only E. I, II, and III

Copyright © 2000-2014 TIPS Inc. Unauthorized reproduction of this manual is prohibited. This manual is being used
during review sessions conducted by PharmacyPrep. 51-18
www.pharmacyprep.com
118. If pharmacy has good turnover rate. Which of the following is correct in Perito's law?
A. 80 % - 20%
B. 50% - 50%
C. 35% - 65%
119. If your pharmacy have lots products on the floor and shelf. What is correct about Turnover
rate?
A. Increased TOR
B. Decreased TOR
C. No change in TOR
120. A staff pharmacist fired from job. After losing his job he reports to police about fraud activities
of pharmacy owner. What is this called?
A. Whistle blower
B. Smoking gun
C. Reporter
D. Announcer
121. A senior and regular customer of your pharmacy. Each time pays a specific amount for
pharmacy before pick up a prescription?
A. coinsurance
B. deductible
C. copayment
D. loss and profit
E. direct payment
122) Cost of goods for a drug is $10 and mark up is 30%. What is retail price? Prescription fee is $10.
What is the total amount customer pay?
A) $13 B) $10 C) $23 D) $22 E) $33

Copyright © 2000-2014 TIPS Inc. Unauthorized reproduction of this manual is prohibited. This manual is being used
during review sessions conducted by PharmacyPrep. 51-19
www.pharmacyprep.com
ANSWERS:
1. C
Tips. Income statement consist information related to sale and profit.
2. D
3. D
Tips: Net income is the financial statement that is total income minus expenses.
5. A
Tips: Income statement is the financial statement used to determine the total value of RXs drug sales
for a pharmacy during the course of a year.
6. E
7. C
8. C
Tips: Job description is a description of all the work that should be done by the employers in order of
priority. The manager normally does it.
9. D
Tips: The pharmacist should always interact with the patient providing clear verbal information such
as explaining how to take the medication, side effects and therapy, and completing the concealing
with extra writing or sheet information.
10. E
Tips: Delegation of a job should include the suggested steps to accomplish the project; a negotiation
of time schedule for completion of the project and the person doing the delegation should provide
support and follow up as required.
11. B
12. C
13. C

Copyright © 2000-2014 TIPS Inc. Unauthorized reproduction of this manual is prohibited. This manual is being used
during review sessions conducted by PharmacyPrep. 51-20
www.pharmacyprep.com
Tips: POS-Point of sale is a computer software and hardware program very useful in pharmacies that
combines and track all the business information such as product information, cash register,
inventory, stocks and so on.
14. C
15. C
Tips: Leasing (Renting) is not a form of enterprise.
16. E
17. B
Tips: [Sales - COGS/Sales] x 100
COGS = Cost of goods sold
Total sales $210,00 is 100%
Cost of goods sold is 180,000 is 85.5%
100%-85.5% = 14.5%
18. C
19. C
Tips: Leasing (Renting) is not a form of enterprise
20. B
21. E
Tips: use of ratios is frequently used rule of thumb in site considerations, this include sales per
square feet, number of prescription per patient, and average price for prescription.
22. D
23. B
24. All of the following include as part of income statement, EXCEPT:
A. sales B. profit C. cost of goods sold D. account payable E. gross margin
Ans: D
25. B
26. A
27. C
28. A

Copyright © 2000-2014 TIPS Inc. Unauthorized reproduction of this manual is prohibited. This manual is being used
during review sessions conducted by PharmacyPrep. 51-21
www.pharmacyprep.com
Tips: Turnover rate = COGS/average inventory
Average inventory = (beginning inventory + closing inventory)/2
$150000/[$50,000 + $25000]/2 = 4
29. C
Tips: Average inventory = (beginning inventory + closing inventory)/2
$150000/[$50,000 + $25000]/2 = 4
this year TOR is 4, so previous year was 6.
30. C
31. B
32. A
33. E
Tips: there are 4 Ps; Place, Product, Price and promotion
34. C
35. A
36. C
37. B
Average inventory = $200,000+$220,000/2 = $210,000
Turnover rate = 500,000/210,000 =2.3
38. C
Tips: Net worth = Total assets – total liabilities
($150,000+$40,000) - $75,000 = $115,000
39. B
40. C
Tips: There are 4 “P” of marketing = Product, Price, Promotion and place.
41. C
42. C
43. A
44. D

Copyright © 2000-2014 TIPS Inc. Unauthorized reproduction of this manual is prohibited. This manual is being used
during review sessions conducted by PharmacyPrep. 51-22
www.pharmacyprep.com
45. D
46. B
47. E
48. B
49. E
50. E
51. C
Tips: They participate in central buying, marketing and professional programs.
52. B
Tips: An individual or corporation must own 5 or more stores to be considered a chain.
53. E
Tips: 3-5 year plan is required.
54. C
Tips: At year end, every business must have a balance sheet prepared.
55. E
56. C
57. D
58. B
Tips: [$53- $35/$53] x 100 = 30%
59. C
Tips: 2000,000/500,000 = 4
60. C

61. C

62. D

Copyright © 2000-2014 TIPS Inc. Unauthorized reproduction of this manual is prohibited. This manual is being used
during review sessions conducted by PharmacyPrep. 51-23
www.pharmacyprep.com

63. D

Tips: Automated filling systems are probably the most expensive and fastest growing sector of
pharmacy technology.

64. D

65. B

Tips: A cash flow statement is usually prepared monthly.

66. D

Tips: It requires (3-5) years plan.

67. A

Tips: Done at year end.

68. A

69. D

70. C

Tips: This ratio will increase or decrease in direct proportion with sales volume.

71. D

72. B

Solution:

29.99= cost = 14.99 = $15.00

0.50

0
73.
A 180-15= $163

1 163 = $13.75
5
x 12
1
2 15.00-13.75 = $1.25 it is the saving on
each bottle.
=

$
1
8
74. A

29.99-13.75= $16.24

Copyright © 2000-2014 TIPS Inc. Unauthorized reproduction of this manual is prohibited. This manual is being used during
review sessions conducted by PharmacyPrep. 51-24
www.pharmacyprep.com

75. B

Solution:

(49.55 /25)= $1.98

(one is free from the representative)

From wholesaler

(49.55 /24) = 2.06x0.015 = 0.03

2.06-0.03= $2.03

76. B

Solution:

10x12=120 bottles

120x2.99= $358.8

358.8/(12+12) = (358.8/132) = $2.71

77. A

Tips: The turnover in the hospital pharmacy is (8-12) versus (4-5) in community pharmacy.

78. C

79. D

80. C

81. D

82. C

Tips: There should be an appropriate financial investment and not the lowest financial investment.

83. A
o
Tips: Cold means not to exceed 8 C

84. A

Tips: Storage of toxic & carcinogenic must be a segregate stock from regular stock.

85. B

Copyright © 2000-2014 TIPS Inc. Unauthorized reproduction of this manual is prohibited. This manual is being used during
review sessions conducted by PharmacyPrep. 51-25
www.pharmacyprep.com
Tips: Automation decreases costs.

86. C

87. C

Tips: Reduced waste due to expired drugs.

88. C

Tips: The lack of bar codes on all injection vials and unit dose packages possess a challenge.

89. A

90. D

91. D

92. D

Tips: Emergency supplies may be purchased or borrowed from another pharmacy.

93. D

94. C

Tips: Narcotics must be returned to point of origin.

95. D

Tips: The purchase should be for those items that meet customer needs.

96. A

Tips: Maximizing space utilization in which “dead” areas are eliminated.

97. C

98. C

Solution:
(412/.0667)= $18
Tips: When the retail price is $18.0 (100%), the cost is $12.00 (67%) and desired gross margin is 33.3%

Copyright © 2000-2014 TIPS Inc. Unauthorized reproduction of this manual is prohibited. This manual is being used during
review sessions conducted by PharmacyPrep. 51-26
www.pharmacyprep.com
99. A

6/0.4 = $15
100. A

15.90 = cost = cost

Mark up 0.067

Cost= 10.65

101. B

Solution:

MU/C = [(16-12)/ 12] x100= 33.33%

102. D

Tips: They are new products being evaluated and tested but which have not been approved for marketing in
Canada. Forms must be submitted by the requesting physician.

103. D

104. B

105. A

Tips: Delivery is within 24 hrs.

106. D

Tips: Value of drug purchases.

107. A

Tips: Products must have an expiry date of 9months or greater.

108. D

109. D
Tips: Hidden costs like essential non-returnable drugs which are very costly and required in high quantities,
and has short dating and cannot be returned for credit like Datrium.
110. A

Copyright © 2000-2014 TIPS Inc. Unauthorized reproduction of this manual is prohibited. This manual is being used during
review sessions conducted by PharmacyPrep. 51-27
www.pharmacyprep.com
111. D
Tips: A pharmacy technician usually is the inventory manager.
112. E
113. E
114. C
Tips: For emergency supplies, a hospital may purchase drugs from another hospital or a nearby community
pharmacy.
115. C
Tips: Delivery is within 24 hours.
116. B
Tips: It has a lower investment inventory.
117. E
118. A
119. B
120. A
121. C
122: C
$10 x1.3 = $13
$13 + $10 = $23

Copyright © 2000-2014 TIPS Inc. Unauthorized reproduction of this manual is prohibited. This manual is being used during
review sessions conducted by PharmacyPrep. 51-28
PHARMACY PREP
DRUG DISCOVERY AND DEVELOPMENT
1. What is correct about phase I clinical trials?
A. Healthy human volunteers are tested
B. Disease human volunteers are tested
C. Larger population than Phase II is tested
D. Drugs are approved for sale after phase III clinical trials.
2. All of the following are true about Phase III human clinical trial, EXCEPT?
A. Healthy human volunteers are tested
B. Disease human volunteers are tested
C. Larger population than Phase II is tested
D. Drugs are approved for sale after phase III clinical trials.
3. Medications that are prescribed and sold over the counter in Canada are approved by?
A. Canadian Health Act
B. NAPRA
C. Health Canada
D. Federal Government of Canada
E. Provincial governments
4. Healthy volunteers are tested in?
A. Pre clinical studies
B. Phase I clinical studies
C. Phase II clinical studies
D. Phase III clinical studies
E. Phase IV clinical studies
5. Which of the following is decisive phase in new drug approval
A. Pre clinical studies
B. Phase I clinical studies
C. Phase II clinical studies
D. Phase III clinical studies
E. Phase IV clinical studies
6. Who approves and authorizes the sale of medications in Canada?
A. Therapeutic Product Directorate of Health Canada
B. Food & Drug and Administration
C. Pharmacy manager
D. Pharmaceutical Manufacturer
E. National association of pharmacy regulatory authority (NAPRA)

Copyright © 2000-2014 TIPS Inc. Unauthorized reproduction of this manual is prohibited. This manual is being used during
review sessions conducted by PharmacyPrep. 54-1
7. Who sets prescription drug prices in Canada?
A. Health Canada
B. Patented medication review board (PMPRB)
C. Pharmacy manager
D. Provincial college of pharmacy
E. Pharmaceutical Manufacturer
8. Who sets OTC drug prices?
A. Health Canada
B. Patented medicine review board (PMPRB)
C. Pharmacy manager/owner
D. Provincial college of pharmacy
E. Pharmaceutical Manufacturer
ANSWERS:
1. A
2. A
3. C
4. B
5. D
6. A
7. B
8. C

Copyright © 2000-2014 TIPS Inc. Unauthorized reproduction of this manual is prohibited. This manual is being used during
review sessions conducted by PharmacyPrep. 54-2
Pharmacyprep.com

PHARMACY PREP
EPIDEMIOLOGY AND CLINICAL STUDIES
1. Which of the following studies are the most credible?
A. Single blind B. Double blind C. Meta analysis D. Case reports
E. Case series
2. In controlled clinical trial, treatment should be allocated by
A. Randomization B. Day of week C. Date of birth D. Hospital number
3. Choose the incorrect statement about the p (probability) value?
A. The p (probability) value is the chance of a type I error.
B. If a p value is equal to/or lower than 0.05, it is unlikely that a type I error has been made; that is, a type I
error will be made 5 times or less out of 100.
C. A p value equal to/or less than 0.05 is generally considered to be statistically significant; lower p values
(e.g., p < 0.01) may be required for statistical significance in studies with large sample sizes.
D. The p (probability) value is the chance of a type II error.

4. Cohort is as study design, which is?


A. Common characteristic of a group B. Past study comparison
C. Current study comparison D. A study model that are compared with past records
E. None of the above
5. Odd ratio is used to calculate outcome for?
A. Retrospective studies B. Prospective studies C. Cross sectional studies
D. Cohort studies E. Meta analysis
5. Randomization
A. Specific criteria are used to assign any patient to particular group.
B. No specific criteria are used to assign any patient to particular group.
C. In clinical trial ensuring compliance with government regulations
D. System or procedures designed to ensure that study is being performed
6. Clinical trials should be conducted in accordance with?
A. Federal regulations
B. Provincial regulations
C. Clinical practice guidelines
D. Ethical principle
E. Manufacturer guidelines

Copyright © 2000-2014 TIPS Inc. Unauthorized reproduction of this manual is prohibited. This manual is being used during
review sessions conducted by PharmacyPrep. 55-1
Pharmacyprep.com
7. Which of the following is an example of non interventional trials or observational trials like a comparison
of two patient with COPD, one smokes and the other non smoker?
A. cohort study B. Clinical trials C. interventional trials D. all of the above
8. The opposite of blind study is an?
A. cohort study B. retrospective study C. open label study D. double blind study E.triple blind study
9. Which of the following phase of clinical trial purpose is to gather information to evaluate overall benefit
risk relationship?
A. phase 1 B. Phase 2 C. Phase 3 D. phase 4 E. phase 5
10. Which of the following phase of clinical trial purpose is post marketing surveillance?
A. phase 1 B. Phase 2 C. Phase 3 D. phase 4 E. phase 5
11. Which of the following phase of clinical trial purpose is safety short time side effects?
A. phase 1 B. Phase 2 C. Phase 3 D. phase 4 E. phase 5
12. Which of the following is the highest level of evidence?
A. Expert opinion
B. Case series
C. Case control or historical control
D. Non randomized observational trials
E. Randomized control trials
13. When considering a blinding study designs, all the following factors should be considered in constructing
that design, Except:
A. Closed label B. Open-label C. Combination D. Single blind
E. Double blind
14. The Wilcoxon signed-rank test is a?
A. non-parametric statistical hypothesis test used when comparing two related samples or repeated
measurements on a single sample to assess whether their population means differ
B. parametric statatistical hypothesis test used when comparing more than two related samples
C. non-parametric statistical hypothesis test used when comparing more than two related samples
D. non-parametric statistical hypothesis test used when comparing single sample
E. none of the above

Copyright © 2000-2014 TIPS Inc. Unauthorized reproduction of this manual is prohibited. This manual is being used during
review sessions conducted by PharmacyPrep. 55-2
Pharmacyprep.com
15. Which of the following is the strongest (the most credible) clinical trial design?
A. randomized experiments B. cohort C.case control D.Case series E.Case report
16. In a perspective study comparing the effectiveness of two chemotherapeutic treatments for cervical
cancer causes were selected from one group of patient who had annual pap smears for at least five years
prior to their initial diagnosis and from another groups of patients who had no history of prior pap smears
screening. The selection cases from both groups in this study may result in:
Bias = systemic error = Distortion of a test measurement
A. Interview bias—Because of blinding of interviewers response may be influenced, known as interview bias.
B. Recall bias-Differentials in the memory capabilities.
C. Lead time bias---The selection of cases from both of these groups introduces a form of non random error
known as lead time bias
D. Confounding---effects because of dependent and independent variables.
E. Berksons bias (admission rate bias)—Distortion in risk ratios occur as result of different hospital
admissions.
F. All of the above
17. What is not included in "intent-to-treat analysis"?
A. Data from patients originally assigned
B. Data from patient participated from entire treatment
C. Data from patient discontinued treatment
D. Data from dropped out patient for non therapeutic reason
E. None of the above

Copyright © 2000-2014 TIPS Inc. Unauthorized reproduction of this manual is prohibited. This manual is being used during
review sessions conducted by PharmacyPrep. 55-3
Pharmacyprep.com
ANSWERS:
1. C
2. A
3. D
4. A
5. A
5. B
6. D
7. A
8. C
9. C
10. D
11. B
12. E
Tips: the sequence of lowest level of evidence to highest level is expert opinion --> case series --> case
control --> non randomized observational trials --> randomized control trials.
13. A
Tips: There is no closed label design.
14. A
15. A
Tips: Sequence of strength is A>B>C>D>E
16. F
17. E

Copyright © 2000-2014 TIPS Inc. Unauthorized reproduction of this manual is prohibited. This manual is being used during
review sessions conducted by PharmacyPrep. 55-4
Pharmacyprep.com
Example: Suppose 100 patient are enrolled in a study, and only 50 complete, if remainder are dropped out
non therapy reasons. If drug is efficacious in 40 patients of the 50, the efficacy with intent to treat analysis
method is 40% (i.e. 40 of 100)

Copyright © 2000-2014 TIPS Inc. Unauthorized reproduction of this manual is prohibited. This manual is being used during
review sessions conducted by PharmacyPrep. 55-5
Pharmacyprep.com
Copyright © 2000-2014 TIPS Inc. Unauthorized reproduction of this manual is prohibited. This manual is being used during
review sessions conducted by PharmacyPrep. 55-6
www.pharmacyprep.com

PHARMACY PREP.
BIOSTATISTICS CALCULATIONS
The next four questions have as reference the following information:
In a chromatographic analysis of a drug we found the following results
2, 4, 8, 5, 7, 4 and standard deviation 5.
1. What is the mean in this analysis?
A. 4 B. 5 C. 30 D. 15 E. 6
2. What is the median in this analysis?
A. 2 B. 5 C. 4.5 D. 7 E. 8
3. What is the mode in this analysis?
A. 2 B. 4 C. 5 D. 7 E. 8
4. What is the Coefficient of variance?
A. 5 B. 10 C. 25 D. 50 E. 100
5. Type I error, also called alpha-error give what kind of results?
A. False-positive B. False-negative C. Positive false D. False-false
E. Positive-positive
6. Correct statements regarding biostatics error may include:
I- Type I error also known as α-error can admit a variance error of 1 to 5%
II- Type II error also known as β-error is a negative-false type of error because it shows a very small
difference-delta error (99,99998)
III- In type I error the data shows no difference but there is difference, it is false-positive
A. I only B. III only C. I and II only D. II and III only E. I,II, III
7. Five subjects given a single intravenous dose of a drug have the following elimination half-life: 3, 7, 2, 3
and 5hr. The mean half-life is:
A. 3hr B. 2hr C. 5.4hr D. 4hr E. 4.5hr
8. In a pharmacoepidemiologic study, a group of patients receiving a drug and WHO share a common
characteristic or conditioning such as age or diagnostic:
A. Cluster B. Segment C. Strata D. Control E. Cohort
9. In statistic a parametric test can be used to analyze data provided that:

Copyright © 2000-2014 TIPS Inc. Unauthorized reproduction of this manual is prohibited. This manual is being used during
review sessions conducted by PharmacyPrep. 56-1
www.pharmacyprep.com
I- There is homogeneity of variance among the groups
II- The data is normally distributed
III- That the level of significance (p) is not less than 0.05
A. I only B. III only C. I and II only D. II and III only E. All are correct
10. An increase in the incidence of diseases, conditions or other health related events in a defined human
population that is clearly in excess of that which was expected during a specific time is called:
A. Epidemics
B. Endemics
C. Pandemics
D. Epidemiologic predisposition
E. All are correct
11. Endemic diseases are conditions or health related behaviors that are constantly present in human
population. Example of endemic disease may include:
A. SARS B. AIDS C. Flu D. Influenza E. Cancer
12. Correct statement regarding primary prevention may include:
A. Early detection and treatment of the disease
B. Involve the palliative treatment and rehabilitation therapy
C. Reduction in the incidence of disease through immunization, education and others
D. The diagnostic of disease
E. Treatment of disease
13. In biostatistic the difference between the highest and lowest data observation is called:
A. Mean B. Mode C. Median D. Range E. Standard deviation
14. In a biostatistics analysis, the normal distribution calculations found to have the mean, media and mode
with the same value. This is called:
A. Skewed distribution
B. Symmetrical distribution
C. Measures of dispersion
D. Measure of central tendency
E. Standard deviation
15. In a biostatistic test of a drug, the normal distribution found was equal to 68%. We may assume that the
standard deviation from the mean value is:
A. 1 B. 2 C. 3 D. 4 E. 5
16. In biostatistics, which of the following is not a statistical test?
A. Student’s test
B. Chi-square test

Copyright © 2000-2014 TIPS Inc. Unauthorized reproduction of this manual is prohibited. This manual is being used during
review sessions conducted by PharmacyPrep. 56-2
www.pharmacyprep.com
C. Null hypothesis
D. Paired-t test
E. F-distribution
17. Statistical test where two treatments are applied to a single group of experimental unit is called:
A. Student’s test
B. Paired-t test
C. F-distribution
D. Chi-square test
E. Measures of central tendency
18. Correct statements regarding BIAS include:
I- They are experimental errors
II- They can be classified in selection bias, information bias and confounding bias
III- Selection bias happens when the study results are distorted by the selection process
A. I only B. III only C. I and II only D. II and III only E. All are correct
19. Experimental error resulted from poor data collection or inaccurate measurement is called:
A. Selection bias
B. Information bias
C. Confounding bias
D. Type II error
E. Type I error
20. In a clinical trial for development of a new drug, phase II procedures include:
A. Animal experiments
B. Pharmacokinetic test in a health adult
C. Safety and efficacy tests done in a large population that have the disease or condition for what the drug is
developed
D. Test the new drug with comparison of standard drug
E. Studies to compare the drug with its competitor
21. What is NOT a statistical variable?
A. drug strength
B. tablets size
C. number of bottles
D. blood pressure
E. number of patients
22. With the results of the coefficient of elasticity from question 21, we can assume that the revenue will:
A. Be the same as before

Copyright © 2000-2014 TIPS Inc. Unauthorized reproduction of this manual is prohibited. This manual is being used during
review sessions conducted by PharmacyPrep. 56-3
www.pharmacyprep.com
B. Be increased
C. Be decreased
D. Have neither increase nor decrease
E. Have loss
23. Distribution and determinants of disease frequency in human studies is
A. Epidemiology
B. Biostatistics
C. Publisher
D. Socio economics
24. Difference between means of two samples can statistically analysed by?
A. T-tests
B. Analysis of variance
C. Chi square test
D. Correlation
E. Multiple regression
25. Differences between means of more than two samples can statistically analysed by.
A. T-tests
B. Analysis of variance:
C. Chi square test
D. Correlation
E. Multiple regression
26. Differences between frequencies in a sample can statistically analysed by?
A. T-tests
B. Analysis of variance:
C. Chi square test
D. Correlation
E. Multiple regression
27. Which of the following is NOT correct about statistical tests?
A. p-value is a component of statistical tests
B. Null hypothesis is a component of statistical tests
C. Alternate hypothesis is a component of statistical tests
D. Confidence interval is a component of statistical tests
E. Clinical significance is a component of statistical tests

Copyright © 2000-2014 TIPS Inc. Unauthorized reproduction of this manual is prohibited. This manual is being used during
review sessions conducted by PharmacyPrep. 56-4
www.pharmacyprep.com
CORRECT ANSWERS:
1. B 2+4+8+5+7+4 = 5
6
2. C 2, 4, 4, 5, 7, 8
3. B 2, 4, 4, 5, 7, 8
4. E Coef. of Variance = Standard Dev. x 100 = (5/5) x 100 = 100
Mean
5. A
Tips: Type I error is used in biostatistics analysis and classified as “False-Positive” test because it can accept a
variance error in the test of 1 to 5%, however this difference does not mean complete error of the test.
6. C
Tips: Type I error, please read answer 5. Type II error (β-error) is a “Positive-False” test because the variance
of error is very small (Delta error), so the data shows difference but there is no difference.
7. D (3+7+2+3+5)/ 5 = 4
8. E
Tips: In pharmacoepidemiology, cohort is a study design of a group of people who are either of the same age
or share some other common characteristics
9. E
10. A
Tips: Epidemics are an increase in the incidence of diseases, conditions or other health related events in a
defined human population that is clearly in excess of that which was expected during a specific time is called.
Example: Flu
11. B
Tips: Endemics diseases are conditions or health related behaviours that are constantly present in a human
population. Example: AIDS
12. C
Tips: Primary prevention is a reduction in incidence of disease through immunization, sanitation, education
or other means of eliminating pathogenic contamination in the human environment.
13. D
Tips: Range is the difference between the highest and lowest data observation.

Copyright © 2000-2014 TIPS Inc. Unauthorized reproduction of this manual is prohibited. This manual is being used during
review sessions conducted by PharmacyPrep. 56-5
www.pharmacyprep.com
Example: 2, 4, 6, 8, 10. The range is 10-2= 8
14. B
Tips: Symmetrical distribution happens when the mean, median and mode are the same valour in a given
data.
15. A
Tips: Number of standard deviation from the mean data characterizing the normal population: 68% = 1, 95%
= 2, 99% = 3, 99.99% = 4
16. C
Tips: Null hypothesis is not a statistical test it is a hypothesis test that states no difference between the
hypothesis and the reality (test done).
17. B
Tips: Paired-t test is a type of test where two treatments are applied to a single group of experimental unit.
Example: The bioavailability test of generic drug to its reference standard drug in a group of people.
18. E
Tips: Bias are experimental errors normally classified in selection bias also called sampling, information bias
also called misclassification and confounding bias.
19. B
Tips: Information bias also called misclassification is a type of experimental error happening when the study
results become distorted by poor data collection or inaccurate measurement.
20. C
Tips: In a phase II clinical trial the drugs is tested for safety and efficacy in a large population that have the
disease or conditions for what the drug is developed.
21. D
22. B
23. A
24. A
25. B
26. C
27. B

Copyright © 2000-2014 TIPS Inc. Unauthorized reproduction of this manual is prohibited. This manual is being used during
review sessions conducted by PharmacyPrep. 56-6
www.pharmacyprep.com
BIBLIOGRAPHIC REFERENCE
1- Pharmacy PREP Evaluating Exam REVIEW 2014
2- CPS-COMPENDIUM OF PHARMACEUTICALS AND SPECIALITIES - Canadian Pharmacist Association – 2001
edition.
th
3- MEDICAL DICTIONARY – Dorland’s illustrated – 27 edition.
4- THERAPEUTIC CHOICES – Canadian Pharmacist Association -7th edition
th
5- USP DI – Drug Information for the Health Care Professional–15 edition – Volume I.

Copyright © 2000-2014 TIPS Inc. Unauthorized reproduction of this manual is prohibited. This manual is being used during
review sessions conducted by PharmacyPrep. 56-7
www.pharmacyprep.com
Copyright © 2000-2014 TIPS Inc. Unauthorized reproduction of this manual is prohibited. This manual is being used during
review sessions conducted by PharmacyPrep. 56-8
www.Pharmacyprep.com

PHARMACY PREP
PRESCRIPTION PROCESSING AND DISPENSING
1. What is a prescription?
A. A written order by the doctor to pharmacist
B. A written order by the doctor to patient
C. A written order by the doctor to public
D. A written order by the pharmacist
E. A sales invoice from the pharmacy
2. What is not present on prescription label?
A. Prescriber initial B. Drug name C. Expiry date D. Patient name E. Drug strength
3. What is correct about after meals
A. bc B. ac C. pc D. hs E. qd
4. The Latin abbreviation for: “Take one tablet four time daily after meals and at bedtime” is:
A. İ QID ac & pc B. İ QID pc & hs C. İİ BID pc & hs D. İİİ OD ac & pc
E. İ QID ac & hs
5. What is the prescription:
A. A written order by a doctor to pharmacist
B. A written order by a doctor to pharmacist and patient
C. A written order by a doctor to patient
D. A written order by a doctor to patient dispensed by the pharmacy.
E. All of the above
6. What is po means?
A. Take through mouth B. Take crush & chew tablets C. Take under and on tongue D. Oral liquids only E.
drugs no first pass metabolism
7. What is FP?
A. for oral B. for pain C. for family physician D. for pneumonia
8. A patient presents to a pharmacy with a new Rx for 60 tablets levothyroxin 0.15 mg to be taken OD. On
checking the patient’s history the pharmacist notices that the patient filled a similar Rx for 180 tablets
levothyroxin 0.15 mg with the same directions 4 weeks ago. What course of action should be the pharmacist
follow?
A. Fill the prescription
B. Check with the doctor before dispensing

Copyright © 2000-2014 TIPS Inc. Unauthorized reproduction of this manual is prohibited. This manual is being used during
review sessions conducted by PharmacyPrep. 59-1
www.Pharmacyprep.com
C. Talk to the patient about this
D. Give only 1-month supply
E. Refuse to fill the prescription
9. Nitroglycerin 0.4 mg #100: i SL prn CP. MR q5min x 2, your label directions?
A. Dissolve under the tongue one tablet as needed for chest pain. May repeat every 5 minutes if pain
persists, to a maximum of 2 additional doses.
B. Take nitroglycerin 0.4mg SL as needed and may repeat every 5 minutes if pain persist, to maximum of 2
doses
C. Take nitroglycerin 0.4mg SL as needed and may repeat every 5 minutes if pain persist, to minimum of 2
doses
D. Give nitroglycerin 0.4mg SL as needed and may repeat every 5 minutes if pain persist, to maximum of 2
doses
10. How many 4 mg tab are needed? Weight of the child = 44 lb
Rx
Ondansetron HCl 0.15mg/kg/tsp
cherry syrup qs 60 ml
A. 4 tabs B. 6 tabs C. 9 tabs D. 12 tabs E. 10 tabs
11. If miss one pill of oral contraceptive, what do you recommend?
A. Take two pill next day B. Take no pill next day C. Take three pill next day D. Take two pill next 2 days
12. 1 po q4-6h prn pain means:
A. Take 1 tablet every 4 to 6 hours as needed for pain.
B. Take 4-6 tablets every 1 hour as needed for pain.
C.Take 1 tablet every 4 to 6 hours with milk
D.Take 1 tablet every 4 to 6 hours with juice
E.Take 1 tablet every 4 to 6 hours every other day
13. A prescription sign as indicates “1 gtt ou qid x 10”. It can be interpreted as:
A. 1 drop to right eye four times a day for 10 days
B. Apply 1 tube to affected area four times a day for 10 days
C. Instill 1 drop into both eyes four times a day for 10 days
D. 1 drop to left eye four times a day for 10 days

14. What is ac means:


A. After meals B. Before meals C. With meals D. Take empty stomach E. Take at bedtime
15. Prednisolone each tablet containing 5 mg. Start 35 mg and then taper by 5 mg every day. How many
tablets are needed?
A. 20 tab B. 56 tab C. 14 tab D. 28 tab E. 8 tab

Copyright © 2000-2014 TIPS Inc. Unauthorized reproduction of this manual is prohibited. This manual is being used during
review sessions conducted by PharmacyPrep. 59-2
www.Pharmacyprep.com
16. Prednisolone each tablet containing 5 mg. Start 35 mg and then taper by 5 mg every 2 days. How many
tablets are needed?
A. 20 tab B. 56 tab C. 14 tab D. 28 tab E. 8 tab
17. Instill in Both eyes?
A. OU B. os C. od D. au E. qd
18. Instill in both ears?
A. au B. as C. ad D. ou E. a.u
19. A prescription for combined oral contraceptives:
Rx
COC
Day 16 – 25 for 3 months
How many pills should dispense:
A. 60 B. 30 C. 90 D. 120 E. 3 month's supply
20. Prepare in ex. aqua.
A. with water B. in water C. out of water D. without water
21. A customer of your pharmacy brings a prescription Alprazolam 1 mg SL for 30 days. The next day the
same person brings the same prescription from other doctor, What to do?
A. Refuse to fill the second prescription
B. Contact the doctor and inform about prescription from other doctor
C. Contact the insurance company and inform about repeat prescription
D. Contact police and inform
E. Talk to patient first
22. According to ISMP guidelines, which of the following is the least confusing abbreviation?
A. q4h B. per os C. q.d D. ud E. mg
23. To minimize the misinterpretation of "HCT" what is better option to do?
A. write bar code of drug
B. write DIN of drug
C. write complete spelling for drug names
D. Do not use this abbreviations
E. If misunderstood contact doctor
24. "Nitro" dip is misinterpreted as?
A. nitroglycerin infusion
B. nitroglycerin SL tab
C. sodium nitroprusside infusion

Copyright © 2000-2014 TIPS Inc. Unauthorized reproduction of this manual is prohibited. This manual is being used during
review sessions conducted by PharmacyPrep. 59-3
www.Pharmacyprep.com
D. Nitroglycerin transdermal patch
E. Nitroglycerin SL spray
25. A prescription of regular insulin 10U daily can be misinterpreted as?
A. inject regular insulin daily 10 U
B. inject intermediate insulin daily 10 U
C. inject regular insulin daily 100 U
D. inject regular insulin 1 U
E. inject regular insulin 1000 U
26) "HCT" is abbreviated for?
A) Hydrochlorothiazide
B) Hydrocortisone
C) Hydrocarbons
D) High cholesterol
E) CT scan
27) "HCT" is misinterpreted as?
A) Hydrochlorothiazide
B) Hydrocortisone
C) Hydrocarbons
D) High cholesterol
E) CT scan

Copyright © 2000-2014 TIPS Inc. Unauthorized reproduction of this manual is prohibited. This manual is being used during
review sessions conducted by PharmacyPrep. 59-4
www.Pharmacyprep.com
ANSWERS:
1. A
2. C
3. C
Tips: pc = after meals, ac = before meals, cc = with meals.
4. B
5. A
6. A
7. B
8. C
9. A
10. C
wt = 44 / 2.2 = 20 kg
the dose = 0.15 x 20x = 3 mg/tsp
but 60 ml/5ml = 12 doses
Total dose = 3 x 12 = 36 mg
Tab needed = 36/4 = 9 tabs
11. A
Tips: If miss one pill take two pills next day.
12. A
13. C
14. B
15. D
Tips: 7+6+5+4+3+2+1+0 = 28 tablets
16. B
Tips: 7+7+6+6+5+5+4+4+3+3+2+2+1+1 = 56
17. A

Copyright © 2000-2014 TIPS Inc. Unauthorized reproduction of this manual is prohibited. This manual is being used during
review sessions conducted by PharmacyPrep. 59-5
www.Pharmacyprep.com
Tips: ou = both eyes, os = left eye, od = right eye
18. A
Tips: au = both ears, as = left ear, ad = right ear
19. B
20. A
Tips: just prepared in water extemporaneous
21. E
Tips: only control drug part I (CNS stimulants) require interval specified by the doctor.
22. A
23. C
24. C
25. C
26. A
27) B

Copyright © 2000-2014 TIPS Inc. Unauthorized reproduction of this manual is prohibited. This manual is being used during
review sessions conducted by PharmacyPrep. 59-6
www.pharmacyprep.com Adverse Drug Reactions

PHARMACY PREP
ADVERSE DRUG REACTIONS
1. Which of the following drugs give severe rebound hypertension?
A. Prazosin B. propranolol C. clonidine D. captopril E. enalapril
2. All of the drug cause orthostatic hypotension, except:
A. propranolol B. Prazosin C. captopril D. Doxazosin E. enalapril
3. Neuroleptic malignant syndrome side effects is not associated with?
A. haloperidol B. Levodopa C. clozapine D. Chlorpromazine
E. Risperidone
4. Nitroglycerine patch comes off during shower, what is suitable action?
A. refer to the physician
B. put another patch
C. reapply the same patch
D. use glue and reapply the same patch
E. None of the above
5. Which of the following drug should not be taken if it is not effective with first dose?
A. Nitro glycerine spray
B. Nitro-glycerine SL
C. Beta blockers
D. Nifedipine
E. Sumatriptan
6. Drugs that gives bronchospasm?
A. Salbutamol B. acebutolol C. Salmeterol D. Formoterol
E. Ipratropium
7. Drugs that alter serum TSH levels?
A. Lithium
B. Amiodarone
C. Levothyroxine
D. Synthroid
E. All of the above
8. All of the following drugs can cause weight gain except?
A. Fluoxetine
B. Paroxetine
C. Risperidone

61-1 Copyright © 2000 - 2014 TIPS Inc. Unauthorized reproduction of this manual is prohibited. This manual is being used
during review sessions conducted by PharmacyPrep.
www.pharmacyprep.com Adverse Drug Reactions
D. Quetiapine
E. topiramate
9. Which of the following drugs priapism side effects, patient should report to doctor?
A. Trazodone
B. Sildenafil
C. fluoxetine
D. Alprostadil
E. All of the above
10. All of the following drugs can give pulmonary fibrosis, except?
A. Bromocriptine
B. Cisapride
C. Amiodarone
D. Ropinirole
E. Methotrexate
11. Headache is side effects of?
A. Amlodipine B. Sumatriptan C. Acetaminophen D. Sildenafil
E. Fluoxetine
12. Venous pooling is associated with?
A. Statins B. Triptans C. Sodium nitroprusside D. Verapamil
E. Beta blockers
13. Black stool and tongue can cause by?
A. Loperamide
B. Bismuth sub-salicylates
C. Ciprofloxacin
D. Tetracycline
E. Erythromycin
14. The rare but serious side effect of clindamycin is?
A. Constipation
B. Diarrhea
C. Stomach upset
D. Nausea and vomiting
E. Bloating
15. The most common side effect of erythromycin?
A. Constipation
B. Diarrhea
C. GI discomfort
D. Nausea and vomiting

61-2 Copyright © 2000 - 2014 TIPS Inc. Unauthorized reproduction of this manual is prohibited. This manual is being used
during review sessions conducted by PharmacyPrep.
www.pharmacyprep.com Adverse Drug Reactions
E. Bloating
16. Phenytoin is associated with?
A. Gingival hyperplasia
B. Diarrhea
C. GI irritation
D. Nausea and vomiting
E. Bloating
17. Neutropenia is side effect of?
A. Clopidogrel
B. ASA
C. Ticlopidine
D. Warfarin
E. LMWH
18. Angioedema is associated with?
A. ACE I
B. ARBs
C. CCBs
D. A and B
E. Beta blockers
19. Lactic acidosis is associated with?
A. metformin
B. Chlorpropamide
C. Gliclazide
D. Roziglitazone
E. Acarbose
20. Torse de pointes is associated with?
A. Amiodarone
B. quinidine
C. Procainamide
D. all of the above
21. What is most common side effects of SSRIs?
A. Nausea and vomiting
B. Sexual dysfunction
C. Sedation
D. GI upset
E. weight gain
22. Bupropion side effects include?

61-3 Copyright © 2000 - 2014 TIPS Inc. Unauthorized reproduction of this manual is prohibited. This manual is being used
during review sessions conducted by PharmacyPrep.
www.pharmacyprep.com Adverse Drug Reactions
A. bulimia nervosa
B. anorexia nervosa
C. sexual dysfunction
D. weight gain
E. A and B
23. A 50 year old female currently she is on atorvastatin, and atenolol and Tylenol # 1 for arthritic pains. She
visit pharmacy complaining of diarrhea, which of the following medication may associated to her problem:
A. Atenolol
B. Codeine in Tylenol
C. Tylenol
D. Atorvastatin
E. All of the above
24. All of the following drugs may require WBC or CBC monitoring, except?
A. clozapine B. ticlopidine C. levothyroxine D. propylthiouracil E. methimazole
Ans: C
25. All of the following drugs give constipation side effect, except?
A. Atropine B. Ipratropium C. Tiotropium D. Scopolamine E. Donepezil
26. A 45 yo person using ramipril 10 mg daily for the treatment of hypertension. However, hypertension was
not controlled, and now doctor have added prazosin 10 mg daily. What is the pharmacist concern?
A. potassium levels
B. low blood pressure
C. side effects of prazosin
D. drug interactions
E. patient age
27. A 72 yo person on atenolol and ASA 325mg, clopidogrel 75 mg, therapy. Recently diagnosed with
hypothyroidism. Doctor has recommend low dose, levothyroxine 12.5 mcg daily dose. Why is the low dose
given?
I)cardiovascular disease II)age III)drug interactions with atenolol, ASA and clopidogrel
A. I only B. III only C. I and II D. II and III E.I,II, III
28. A customer of your pharmacy using sumatriptan for migraine headache. However, this person headache
is not relieved after using sumatriptan. What is appropriate to do?
A. double the dose of sumatriptan
B. wait for 2 hour and try again sumatriptan
C. add other triptan with sumatriptan
D. Decrease dose and gradually increase the dose
E. Do not use sumatriptan

61-4 Copyright © 2000 - 2014 TIPS Inc. Unauthorized reproduction of this manual is prohibited. This manual is being used
during review sessions conducted by PharmacyPrep.
www.pharmacyprep.com Adverse Drug Reactions
29. A 65 yr old Parkinson's patient is on levodopa/carbidopa therapy. Experiencing nausea and vomiting.
What is incorrect?
A. treat nausea vomiting using dimenhydrinate
B. treat nausea vomiting using metoclopramide
C. Recommend vitamin B6 supplement
D Recommend Diclectin (vitamin B6 + doxylamine)
E. Recommend ondansetron a serotonin antagonist
30. A 72 year old regular customer of your pharmacy brings a new prescription of levothyroxine 12.5 mcg
daily. Recently patient is discharged from hospital after MI. Dr prescribed low dose of levothyroxine
because?
I) Myocardial infarction
II) Age 72 yr
III) Regular customer of your pharmacy
A. I only B. III only C. I and II D. II and III E.I,II, III
31. A 55 yo man using warfarin 5 mg daily to treat deep vein thrombosis. Recently doctor prescribed
metoprolol 50 mg bid to treat hypertension. What is the pharmacist concern?
A. high dose of warfarin
B. deep vein thrombosis at age of 55 y
C. Metoporolol is a beta blocker
D. Not a concern
E. INR monitoring is required

61-5 Copyright © 2000 - 2014 TIPS Inc. Unauthorized reproduction of this manual is prohibited. This manual is being used
during review sessions conducted by PharmacyPrep.
www.pharmacyprep.com Adverse Drug Reactions
ANSWERS:
1. C
2. A
3. B
4. C
5. E
6. B
7. E
8. E
9. B
10. D
11. A
12. C
13. B
14. B
15. C
16. A
17. C
18. D
19. A
20. D

61-6 Copyright © 2000 - 2014 TIPS Inc. Unauthorized reproduction of this manual is prohibited. This manual is being used
during review sessions conducted by PharmacyPrep.
www.pharmacyprep.com Adverse Drug Reactions
21. A
22. E
23. D
24. C
25. E
Tips: all anticholinergic drug gives constipation side effect. However donepezil is anticholinesterase (agonist)
drug.
26. B
27. C
Tips: low dose levothyroxine is used in seniors and cardiovascular disease patient. However no interactions
with atenolol, ASA and clopidogrel
28. E
Tips: do not use triptans if no relief with first dose.
29. B
30. C
31. C
Tips:

61-7 Copyright © 2000 - 2014 TIPS Inc. Unauthorized reproduction of this manual is prohibited. This manual is being used
during review sessions conducted by PharmacyPrep.
www.pharmacyprep.com Adverse Drug Reactions
61-8 Copyright © 2000 - 2014 TIPS Inc. Unauthorized reproduction of this manual is prohibited. This manual is being used
during review sessions conducted by PharmacyPrep.
PharmacyPrep.Com Drug Interactions

PHARMACY PREP
DRUG & DRUG INTERACTIONS
1. A 55 year old male patient receiving warfarin for deep vein thrombosis, he get the prescription of
carbamazepine; which of the following drug interaction possible:
I-Carbamazepine decrease metabolism of warfarin
II-Carbamazepine is has not interaction with warfarin
III-Carbamazepine increase metabolism of warfarin
A. I only B. III only C. I and II only D. II and III only E. All of the above
2. A 20 year old female patient currently on fluoxetine for depressive disorder, she get the prescription of
Tylenol # 3 for sever headache. Which of the following drug interaction are possible:
A. Fluoxetine metabolism increases
B. Codeine metabolism increases
C. Fluoxetine metabolism decrease
D. Codeine metabolism decrease
E. No change and there is no drug interactions
3. A 40-year old female is regular customer of your pharmacy, her current medical history include lovastatin.
She brings prescription for her recent condition, Which of the following medication is safe for her medical
profile:
A. Erythromycin
B. Ketoconazole
C. Grapefruit juice
D. Niacin
E. Hydrochlorothiazide
4. Enzyme complex not working as a drug-metabolizing enzyme may include:
A. Cytocrome P-450
B. Cytocrome P450 3A4
C. Alcohol Dehidrogenase
D. UDP Glucoronyl Transferase
E. Cytocrome oxidase.
5. What is incorrect about tetracycline antibiotics:
I-Take with water and milk
II-Take with food
III- Must take empty stomach
A. I only B. III only C. I and II only D. II and III only E. All of the above

Copyright © 2000-2014 TIPS Inc. Unauthorized reproduction of this manual is prohibited. This manual is being used during
review sessions conducted by PharmacyPrep. 62-1
PharmacyPrep.Com Drug Interactions
6. Tetracyclin binds with?
A. Mono valent ions
B. Bi valent ions
C. Tri valent ions
D. B and C only
E. All of the above
7. Ciprofloxacin should be taken with:
A. Avoid taking with full glass of milk
B. Take with or after food
C. Take empty stomach
D. Take with or without food
E. A and D
8) A patient using calcium supplements 500 mg tid. Now gets the prescription of ciprofloxacin 500 mg bid for
3 days. What is appropriate method of administration?
A) Take Ca supplements and ciprofloxacin together morning and evening.
B) Take Ca supplements before 2 hour and after 4 hours of ciprofloxacin.
C) Stop taking Ca supplements for 3 days and continues ciprofloxacin
D) Stop taking ciprofloxacin and continue Ca supplement daily.
E) Call the doctor, change to levofloxacin.
9. Which of the following statement is incorrect about alcohol and drug interactions?
A. Alcohol and Metronidazole can give disulfiram reactions
B. Alcohol and metformin can give lactic acidosis
C. alcohol and chlorpropamide an antidiabetic drug can give lactic acidosis
D. alcohol and benzodiazepine can give sedation
10. A patient using Sildenafil 50mg should not take nitrates, because?
A. It can cause hypotension
B. It can cause hypertension
C. It can cause angina
D. It can cause congestive heart failure
E. None of the above
11. All of the following agent can cause urine discoloration, EXCEPT;
A. Metronidazole
B. Rifampin
C. Sulfasalazine
D. pyrivinium pamoate
E. tetracycline

Copyright © 2000-2014 TIPS Inc. Unauthorized reproduction of this manual is prohibited. This manual is being used during
review sessions conducted by PharmacyPrep. 62-2
PharmacyPrep.Com Drug Interactions
12. Iron supplement should be separated 2 to 4 hrs from all, except?
A. Thyroxin B. Tetracycline C. Cholestyramine D. Atorvastatin E. None
13. What type of mechanism for drug interactions of warfarin and antibiotics?
A. Altering GI flora
B. altering prothrombin time
C. increasing bleeding time
D. Increasing warfarin metabolism
14. A patient has migraine 3 to 4 times a months. Currently also diagnosed with benign prostatic hyperplasia.
Doctor has prescribed prazosin. Medical profile also includes constipation. What drug is recommendation are
suitable for migraine prophylaxis?
A. Amitriptyline
B.Verapamil
C. Propranolol
D. Sumatriptan
E. Nortriptyline
15. What over the counter drugs should be avoided with MAOIs?
A. Pseudoephedrine
B. Xylometazoline
C. Dextromethorphan
D. Food containing Tyramine
E. All of the above
16. Calcium supplement should separated 2 to 4 hrs from all, except?
A. Ciprofloxacin
B. Tetracycline
C. Alendronate
D. Thyroxin
E. Acetyl salicylic acid
17. All of the following are pharmacodynamic reactions, except?
A. Release of 5-ASA from sulfasalazine
B. Diuretic and insulin actions
C. Sedative with alcohol gives sedation
D. None of the above
18. A 60-year-old women-taking calcium supplement 500 mg tid. Recently doctor prescribed ciprofloxacin
500 mg tid for 3 days. What is appropriate recommendation?
A. Take calcium supplement with 2 hours apart from ciprofloxacin
B. Skip calcium supplement during ciprofloxacin therapy
C. Skip ciprofloxacin and just take calcium supplements

Copyright © 2000-2014 TIPS Inc. Unauthorized reproduction of this manual is prohibited. This manual is being used during
review sessions conducted by PharmacyPrep. 62-3
PharmacyPrep.Com Drug Interactions
D. Recommend other antibiotics such as ofloxacin
E. Calcium and ciprofloxacin can be combined
19. Tetracycline binds with?
I-antacids
II-bi valent ions
III-tri valent ions
A. I only B. III only C. I and II only D. II and III only E. All of the above
20. A patient using atorvastatin 80 mg daily, also wants to use daily 1 glass of grapefruit juice, what is
pharmacist concern?
A. grapefruit juice induce CYP3A4
B. grapefruit juice inhibit CYP3A4
C. grapefruit juice is substrate for CYP3A4
D. grapefruit juice is essential for statin metabolism
E. grapefruit juice may decrease cholesterol
21. A patient using sildenafil 50 mg, should be cautious taking high dose of take the following
antihypertensive drugs
A. Amlodipine B. Propranolol C. Hydrochlorothiazide D. Prazosin
E. Ramipril
22. Which of the following is incorrect?
A. Insulin plus physical exercise increase risk of hypoglycemia
B. Insulin plus alcohol intake increase risk of hypoglycemia
C. Insulin plus glyburide increase risk of hypoglycemia
D. Insulin plus alcohol increase risk of lactic acidosis
E. Insulin overdose can cause sweating, confusion and palpitation
23. A 60 year old women taking calcium supplement bid. Recently doctor prescribed alendronate 70 mg/wk.
What is appropriate recommendation?
A. Take together calcium supplements and alendronate 70 mg once weekly
B. Do not take calcium supplements during the therapy of alendronate
C. Separate calcium supplements 2 to 4 hours from the dose of alendronate
D. Do not use alendronate while taking calcium supplement therapy
24. A 56 yo female using levothyroxine 25 mcg daily and metformin 500 bid. She comes to buy Calcium
supplements to prevent osteoporosis. What is true statement?
A. levothyroxine, calcium supplement should be separated for 4 hours so take levothyroxine empty stomach
first thing in the morning.
B. avoid taking calcium supplements because she is using levothyroxine
C. Separate Ca supplement 4 hour from levothyroxine
D. take Ca supplement in morning and take levothyroxine in evening
E. Never take levothyroxine, metformin and calcium supplement together

Copyright © 2000-2014 TIPS Inc. Unauthorized reproduction of this manual is prohibited. This manual is being used during
review sessions conducted by PharmacyPrep. 62-4
PharmacyPrep.Com Drug Interactions
25. A 56 yo female using metformin 500 bid, and ciprofloxacin 500 mg tid for 3days. She comes to buy
calcium supplements 1500 mg tid to prevent osteoporosis. What is true statement?
A. recommend to separate Ca supplement 4 hour after ciprofloxacin
B. recommend to take ciprofloxacin tid, metformin bid, and Ca supplement tid as doctor prescribe
C. Recommend to start Ca supplement after she finish taking Ciprofloxacin
D. Never combine ciprofloxacin and Ca supplements
E. All of the above
26. A customer of your pharmacy enquiring that can she use herbal therapy like grapefruit juice to treat her
borderline high cholesterol, She using atorvastatin for cholesterol therapy. what is appropriate to do?
A. talk to your doctor
B. definitely she can use grapefruit juice
C. She should start taking statins and avoid grapefruit juice
D. She should never take grapefruit juice for high cholesterol
E. Give her some information to read about high cholesterol therapy
27. A customer of your pharmacy using allopurinol for the past 10 yrs for hyperurecimia and currently
diagnosed for cancer. A doctor want to initiate azathioprine chemotherapy. What is appropriate to do?
A. decrease dose of azathioprine
B. decrease dose of allopurinol
C. avoid allopurinol during chemotherapy
D. avoid azathioprine chemotherapy
E. do not change, any medications
28. A customer of your pharmacy profile shows nitroglycerin SL spray prn. He brings prescription of sildenafil
50 mg prn. What to do?
A. Do not dispense sildenafil 50 mg because it can cause severe hypotension with nitroglycerin SL
B. Dispense low dose sildenafil 25 mg
C. Dispense sildenafil 50 mg and counsel customer not to combine with nitroglycerin
D. dispense sildenafil 50 mg and counsel not to use nitroglycerin SL in case of chest pain
E. Talk to doctor and inform doctor about patient profile nitroglycerin SL
29. A customer of your pharmacy wants to use St. Johns wart to improve mood. Currently he is on
paroxetine for major depression. What is your opinion?
A. ok to use B. do not use St. Johns wart during paroxetine therapy C. refer to doctor D. give him some
literature of general health E. none of the above

Copyright © 2000-2014 TIPS Inc. Unauthorized reproduction of this manual is prohibited. This manual is being used during
review sessions conducted by PharmacyPrep. 62-5
PharmacyPrep.Com Drug Interactions
30. Which of the following is the least drug interaction with levothyroxine?
A. Iron supplements
B. Calcium supplements
C. Magnesium supplements
D. Aluminum supplements
E. Dairy products
31. A regular customer, takes a glass of grapefruit juice daily. If medication profile have amiodarone,
atorvastatin, hydrochlorothiazide, amlodipine and ASA 81 mg daily. Which of the following drug is the least
concern?
A. atorvastatin
B. amlodipine
C. hydrochlorothiazide
D. amiodarone
32. Warfarin and carbamazepine drug interactions are categorized as?
A. metabolism inhibitors
B. Pharmacodynamic interaction
C. enzyme inducer
D. Pharmacokinetic interaction
E. Receptor & receptor interactions
33. A regular customer of your pharmacy. Brings a new prescription of sildenafil 50 mg. Which of the
following drugs in patient profile is the least concern?
A. Nitroglycerin SL spray
B. Nitroglycerin SL tablets
C. Nitroglycerin transdermal patch
D. Sodium nitroprusside
E. Warfarin 5 mg tablets
34. If the above patient profile have nitrates, what is the pharmacist concern?
A. priapism
B. double vision
C. headache
D. hypotension
E. mood changes
35. A patient medical profile include, metformin, gliclazide, rosuvastatin, and ASA 81 mg. Occasionally
patient uses omega 3, 6,9 and multivitamin supplements. Patient is chronic alcoholic, renal disease, and liver
diseases. Recently patient discharged from hospital after congestive heart failure. What is the least concern
of pharmacist with respect to metformin?
A. chronic alcoholism
B. renal disease

Copyright © 2000-2014 TIPS Inc. Unauthorized reproduction of this manual is prohibited. This manual is being used during
review sessions conducted by PharmacyPrep. 62-6
PharmacyPrep.Com Drug Interactions
C. disulfuram like reactions
D. congestive heart failure
E. lactic acidosis
36. A patient lifestyle include high proteins diet. Which of the following is the most concern for the
pharmacist?
A. theophylline
B. levodopa
C. renal disease
D. orlistat
E. liver diseases

Copyright © 2000-2014 TIPS Inc. Unauthorized reproduction of this manual is prohibited. This manual is being used during
review sessions conducted by PharmacyPrep. 62-7
PharmacyPrep.Com Drug Interactions
ANSWERS:
1. B
Tips: Carbamazepine is an inducer of CYP2C9, and the same enzyme is substrate of warfarin. Therefore
Carbamazepine increase metabolism of warfarin
2. D
Tips: Fluoxetine inhibits CYP 2D6 and this same enzyme is a substrate for codeine and therefore decrease
codeine metabolism
3. E
4. C
5. C
6. D
Tips: Tetracycline's bind with bi and tri valent ions such as calcium and aluminum ions.
7. E
8. C
9. C
10. A
11. E
12. D
13. A
14. C
15. E
16. E
17. A
18. B
19. E

Copyright © 2000-2014 TIPS Inc. Unauthorized reproduction of this manual is prohibited. This manual is being used during
review sessions conducted by PharmacyPrep. 62-8
PharmacyPrep.Com Drug Interactions
20. B
Tips: Grape fruit juice is potent inhibitor of the most common drug metabolizing enzyme CYP3A4
21. D
22. D
23. C
24. A
25. C
26. C
27. A
Tips: azathioprine is metabolized by xanthine oxidase, thus reduce the dose of azathioprine to 2/3 or 3/4
28. E
29. B
30. E
31. C
32. C
Tips: carbamazepine induce CYP 2C9 and 1A2, these enzymes are substrate of warfarin
33. E
34. D
35. C
36. C

Copyright © 2000-2014 TIPS Inc. Unauthorized reproduction of this manual is prohibited. This manual is being used during
review sessions conducted by PharmacyPrep. 62-9
PharmacyPrep.Com Drug Interactions
Copyright © 2000-2014 TIPS Inc. Unauthorized reproduction of this manual is prohibited. This manual is being used during
review sessions conducted by PharmacyPrep. 62-10
PharmacyPrep.Com Therapeutic Drug Monitoring

PHARMACY PREP
THERAPEUTIC DRUG MONITORING
1-Which antipsychotic drugs monitored for WBC or CBC?
A-Olanzapine
B-Clozapine
C-Risperidone
D-Haloperidol
E-Quetiapine
2) What is NOT monitored for patient using risperidone?
A) White blood cells
B) diabetes
C) blood pressure
D) Parkinson’s symptoms
E) Weight gain
3) Patient using statins should be monitored for?
A-LFT and CK-MM
B-LFT only
C-CK-MM only
D-Renal function test
E-LDL only
4) CK-MM indicates?
A-Creatinin kinase in skeletal muscles
B-Creatinin kinase in brain
C-Creatinin kinase in Heart
D-Creatinin kinase in liver
E-Creatinin kinase in renal elimination
5) A patient taking acetaminophen, what liver enzyme may be elevated?
A-ALT B-AST C-LFT D-ALP E-LDH
6) A patient taking atorvastatin, what liver enzymes may be elevated?
A-ALT and AST
B-LDH and ALT
C-LDH only
D-ALT only
E-AST only
7) Patient using warfarin should be monitored for?

Copyright © 2000-2014 TIPS Inc. Unauthorized reproduction of this manual is prohibited. This manual is being used during
review sessions conducted by PharmacyPrep. 63-1
PharmacyPrep.Com Therapeutic Drug Monitoring
A-Prothrombin time (PT)
B-INR
C-aPTT
D-A and B
E-All of the above
8) Hypothyroidism patient should be monitored
A-Decrease TSH
B-Elevated TSH
C-Elevated FT3
D-Decreased FT4
E-TBG
9) A 50-year-old male a regular customer of your pharmacy, his current medications include:
Hydrochlorthiazide, Furosemide 50 mg, and Digoxin 0.125 mcg
Which of the following he should be monitored? I-Monitor K levels
II-Serum creatinin
III-Blood pressure
a) I only
b) III only
c) I and II only
d) II and III only
e) All are correct
26) Which drugs patient may complain sandy and gritty feeling in eye? The pharmacist recommends patent
to consult doctor get eye exam
I) amiodarone
II) hydroxychloroquine
III) Quinidine
a) I only
b) III only
c) I and II only
d) II and III only
e) All are correct
11) A 52 yo women using Synthroid 75 mcg daily dose. Recently her blood test done it showed her serum
TSH are less than 0.4 mU/L. What is the appropriate action to do?
A) Increase dose of Synthroid to 100 mcg
B) Double the dose Synthroid
C) Do not change the dose of Synthroid
D) Decrease dose of Synthroid
E)Recommend to take empty stomach

Copyright © 2000-2014 TIPS Inc. Unauthorized reproduction of this manual is prohibited. This manual is being used during
review sessions conducted by PharmacyPrep. 63-2
PharmacyPrep.Com Therapeutic Drug Monitoring
12) A 68 yo male using warfarin 4 mg daily and currently his INR is 2 to 3. Recently he cut his finger slightly
with nail clipper. and had slightly bruise on his face. What is appropriate action do?
A) Refer him to emergency
B) Ask him to discontinue warfarin and contact doctor immediately
C)give him first aid like band aid his finger nail.
D)Recommend vitamin K supplements
E)Recommend to increase dose of warfarin
13) A 45 yo man using rosuvastatin 20 mg daily and metformin 500 mg bid, gliclazide SL 30 mg daily. A doctor
recommends monitoring of lab test for metformin?
I)Renal Function Test
II) Creatinine clearance
III) Liver Function Test
A) I only B) III only C)I and II D)II and III E)I, II, III
14) A 45 yo man using rosuvastatin 20 mg daily and metformin 500 mg bid, gliclazide SL 30 mg daily. A doctor
recommends regular monitoring of lab test for rosuvastatin?
A)CK-mm B)LFT C)Renal function test D) B.P E)ECG
15) Procainamide is type of?
A)ester type of local anesthetic
B)amide type of local anesthetic
C)it is gas general anesthetic
D)It is antiarrhythmic drug
E)It is potassium channel blocker
16) A 65 yo man using digoxin, hydrochlorothiazide, furosemide and ramipril for congestive heart failure for
the past one year. Which of the following laboratory test require monitoring?
+ + 2+ 2+
A)Na B)Cl C)K D)Ca E)Mg
17)A 55 yo customer of your pharmacy. Currently on hydrochlorothiazide 75 mg daily, metformin 500 mg tid,
gliclazide 30 mg daily, and allopurinol 100 mg. Which of the following is not monitored?
A)uric acid levels B)glucose test C)Blood pressure D)ECG E)CrCl
+
18) Which of the following diuretic may require Ca2 monitoring?
A)hydrochlorothiazide B)spironolactone C)furosemide D)Acetazolamide E)Mannitol
19) A 45 yo man using rosuvastatin 20 mg daily and metformin 500 mg bid, glicalizide SL 30 mg daily. A
doctor recommends regular monitoring of lab test for rosuvastatin?
A) LFT B) CK-mm C) CrCl D)glucose E)Renal function test
Ans: A

Copyright © 2000-2014 TIPS Inc. Unauthorized reproduction of this manual is prohibited. This manual is being used during
review sessions conducted by PharmacyPrep. 63-3
PharmacyPrep.Com Therapeutic Drug Monitoring
20) Which of the following lithium serum levels gives lithium toxic symptoms?
A) > 1.5 mEq/L
B) < 1.5 mEq/L
C) > 2.0 mEq/L
D) > 3.0 mEq/L
E) > 4.0 mEq/L
21) A 45 yo man recently diagnosed with serum TSH 10 mU/L. However normal range of serum TSH is 0.5 to
5 mU/L. This patient should be initiated?
A) hyperthyroidism therapy
B) hypothyroidisms therapy
C) Methimazole
D) Propylthiouracil
E) Radioactive therapy
22) A patient is using furosemide to treat congestive heart failure. Which of the following electrolytes loss is
NOT monitored?
A) Potassium
B) Calcium
C) Magnesium
D) Sodium
E) Aluminum
23) Which of the following does NOT required monitoring, serum TSH?
A) amiodarone
B) Prednisone
C) Levothyroxine
D) methyl prednisone
E) infliximab
24) A 70 yo patient is using hydrochlorothiazide 50 mg daily to treat hypertension. Which of the following is
NOT monitored?
A) uric acid
B) blood glucose
C) cholesterol (lipids)
D) hypocalcemia
E) hypokalemia
25)A 29 yo patient using lithium carbonate to treat manic depression. What is NOT monitored?
A) serum TSH
B) CBC
C) blood glucose levels

Copyright © 2000-2014 TIPS Inc. Unauthorized reproduction of this manual is prohibited. This manual is being used during
review sessions conducted by PharmacyPrep. 63-4
PharmacyPrep.Com Therapeutic Drug Monitoring
D) serum level of lithium
E) eye exam
ANSWERS:
1) B
Tips: Clozapine associated with side effect of agranulocytosis, thus it should be monitored for WBC or CBC.
2) A
3) A
4) A
5) A
6) A
7) D
8) B
9) C
10) E
Tips: Amiodarone, hydroxychloroquine and quinidien may conreal deposits.
11) D
Tips: TSH are less than 0.4 mU/L is indicator of overdose of Synthroid, thus decrease dose.
12) C
13) E
14) B
15) B
16) C

Copyright © 2000-2014 TIPS Inc. Unauthorized reproduction of this manual is prohibited. This manual is being used during
review sessions conducted by PharmacyPrep. 63-5
PharmacyPrep.Com Therapeutic Drug Monitoring
Tips: hydrochlorothiazide and furosemide can cause hypokalemia, this can lead to digitalis toxicity.
17) D
18) C
Tips: furosemide can cause hypocalcemia (loop loose calcium)
19) A
20) C
21) B
22) E
23) E
24) D
Tips: thiazides cause hypercalcemia or hyperGLUC. Whereas loop diuretics cause hypocalcemia
25) E

Copyright © 2000-2014 TIPS Inc. Unauthorized reproduction of this manual is prohibited. This manual is being used during
review sessions conducted by PharmacyPrep. 63-6
www.pharmacyprep.com

PHARMACY PREP
DRUGS IN SPECIAL POPULATIONS
1-What are the physicochemical characteristic of drug influence the rate and extent of drug passage into the
breast milk?
A-Molecular weight of drug
B-Degree of ionization
C-Plasma protein binding
D-Lipid solubility
E-All of the above
2-Which of the following drug decrease serum Prolactin levels:
A-L-dopa
B-Ergot alkaloids
C-Bromocriptine
D- All of the above
3-Which of the following drug increase serum prolactin levels:
A-Metoclopramide B-Haloperidol C-Domperidone D-Methyldopa
E-Phenothiazine F-Theophylline G-amphetamine H-All of the above
4-Which of the following drug readily enters into breast milk?
A-Narcotics
B-Antidepressants
C-Antipsychotics
D-Anticholinergic
E-all of the above
5- Critical period of teratogenicity in pregnancy is which of the following period?
a) Last trimester
b) Second trimester
c) Last month
nd th
d) 2 to 8 week
st
e) 1 week
6-A 20 year pregnant female, diagnosed with urinary tract infections and she is allergic to penicillins, which
of the following drug should be avoided in this patient:
A. Tetracycline
B. Amoxicillin
C. Cephalosporin's
D. Vancomycin
E. Clarithromycin

Copyright © 2000-2014 TIPS Inc. Unauthorized reproduction of this manual is prohibited. This manual is being used during
review sessions conducted by PharmacyPrep. 64-1
www.pharmacyprep.com
7. Volume of distribution in neonates is affected by many age-dependant factors including
I. The degree of protein binding
II. The sizes of various body compartments
III. The presence of various endogenous substances

a) I only
b) III only
c) I & II only
d) II & III only
e) All of the above
8-All of the following factors are decreased in seniors, except:
A-total body water
B-first pass metabolism
C-renal function
D-body mass index
E-gastric pH
9-The most teratogenic period in pregnancy?
A- 2 to 8 weeks
B- 8 weeks to birth
C- 8 weeks to 9 weeks
D- 1 to 8 days
E- first 8 days

10) A pregnant woman comes to your pharmacy, and wants to know, which of the following vitamin
supplements can be toxic in pregnancy?
A)Vitamin B6 B)Vitamin D C)Vitamin C D)Vitamin A
11) A man approaches to your pharmacy for his wife because she is ill, not eating food and not talking for the
past two day. She is pregnant and have severe nausea vomiting. She has vomited several time in last two
days. What to do?
A) give dimenhydrinate B) refer doctor get Diclectin prescription C) refer to emergency d) recommend self
care
E) give oral rehydration solution
12) A pregnant women has mild to moderate nausea and vomiting. What is the correct self care?
A) minimize meals and take liquid food
B) take large meals and reduce the time of meals
C) take small and frequent meals
D) increase activity at the time of nausea and vomiting
E) none

Copyright © 2000-2014 TIPS Inc. Unauthorized reproduction of this manual is prohibited. This manual is being used during
review sessions conducted by PharmacyPrep. 64-2
www.pharmacyprep.com
13) Correct statement/s regarding the general considerations in drug therapy in pediatric patients may
include
A. Drug therapy monitoring is not essential
B. Pharmacokinetic parameters in children changes as they mature from birth to adolescence
C. Neonates are relatively chlorhydric
D. Neonates are fast metabolizers
E. Neonates have high concentration of cytochrome
14. In elderly population, which of the following are pharmacokinetic changes may occur?
A. increase in renal clearance
B. Decrease in renal clearance
C. Increase in volume distribution
D. Anticholinergic side effects have negligible affect
E. Benzodiazepine is used to decrease the risk falls
15) All of the following are TRUE about prescribing medication for elderly patients EXCEPT?
a) Lower doses may be used in elderly
b) Absorption of drugs may be decreased
c) Efficiency of drug elimination is likely to be increased in old age
d) Elderly patients are particularly susceptible to adverse drug reactions
e) None of the above

16) Which of the following increases in seniors with age?


A-Drug dosage
B-Adipose tissue
C-Renal functions
D-Body mass
E-Drug elimination
17) A pregnant woman and drinks alcohol, milk, fluids, take vitamins and minerals. What make it hard for
you to treat from constipation, all EXCEPT:
a. She is pregnant
b. She is taking iron
c. She drinks milk
d. She drinks alcohol

18) A pregnant woman and drinks alcohol, milk, fluids, take vitamins and minerals. What make it hard for
you to treat from constipation, all EXCEPT:
a. She is pregnant

Copyright © 2000-2014 TIPS Inc. Unauthorized reproduction of this manual is prohibited. This manual is being used during
review sessions conducted by PharmacyPrep. 64-3
www.pharmacyprep.com
b. She is taking iron
c. She drinks milk
d. She drinks alcohol
ANSWERS:
1- E
2- E
3- All of the above
Tips: All antipsychotic drug increase prolactin levels.
4- E
5- D
6- B

Copyright © 2000-2014 TIPS Inc. Unauthorized reproduction of this manual is prohibited. This manual is being used during
review sessions conducted by PharmacyPrep. 64-4
www.pharmacyprep.com
7. E
8- E
9- A
10) D
11) B
12) C
13) B
Tips: High hydrochloric acid levels are chlorhydric. Low hydrochloric acid levels are Achlorhydric. Relatively
neonates are achlorhydric and Relatively elderly are chlorhydric
14) B
15)C
16)B
17)D
18) D

Copyright © 2000-2014 TIPS Inc. Unauthorized reproduction of this manual is prohibited. This manual is being used during
review sessions conducted by PharmacyPrep. 64-5
Pharmacyprep.com

PHARMACY PREP
PHARMACY COMMUNICATION SKILLS
1) Which of the following is the most effective communication skill?
A. Verbal skills and Nonverbal listening skills
B. Verbal skills only
C. Non verbal skills
D. Counseling skills
E. Patient education skills
2-A pharmacist is required to give knowledge of drugs and supply the patient with enough pieces of
information to overcome the patient ignorance. This is done by:
a) Written information only
b) Verbal information only
c) Verbal and written information
d) Magazines and folders information
e) Video information
3-The most effective communication skills include?
A-Written only
B-Verbal and written
C-Verbal and non-verbal
D-Verbal only
E-Listening only
4-All of the following communication distractions, except:
A-Telephone
B-Language
C-Cell phone
D-Music
E-Television
5) A patient who does not speak English and you don't understand his language. How do you resolve this
problem?
A-Find a interpreter or translator
B-Patient problem do not worry
C-Pharmacist problem learn his language
D-Both pharmacist and technician problem
E) Use non verbal techniques
6-A 90-year-old senior patient cannot hear, how to counsel?
A-Speak louder
B-Counsel in quite area of pharmacy

Copyright © 2000-2014 TIPS Inc. Unauthorized reproduction of this manual is prohibited. This manual is being used during
review sessions conducted by PharmacyPrep. 66-1
Pharmacyprep.com
C-Give recorded tape of counseling
D-speak vivid and closer to customer
E-Don’t speak to customer
7) As a pharmacist you should have speaking skills. All the following are true except:
A) Speak in a monotonous voice pitch.

B) Speak with precise language.

C) Write when verbal is not sufficient.

D) Greet the patient.

8) All the non verbal skills to use during telephone conversations except:
A) Give your full attention to phone call.

B) Put the patient on hold if you are busy.

C) Be prepared.

D) Smile

9) In non verbal communication you should:


A) Use appropriate body language.

B) Look down repeatedly.

C) Tilt to one side.

D) Have the barriers between you and the patient like at desk or counter.

10) A patient coming to take OTC cough syrup and pharmacist noticed that patient took same medication
yesterday, what you should do?
a. Tell her no more medication available
b. Ask her what medical conditions she needs the medication
c. Give her the name of the addiction group
d. Call the police

11) If you open a pharmacy in multicultural area, what will you do increase sales?
A) Display signs of different culture
B) Hire staff of different cultures
C) Offer only one culture service
D) Refuse all the customer who disagree with opinion
E) Offer services in different languages
12) What is the best marketing in pharmacy?
A) Advertise in national TV
B) Advertise in national news paper
C) Advertise door to door in neighborhood
D) Advertise in mall
13) What are the characteristics of culturally competent pharmacist?
Copyright © 2000-2014 TIPS Inc. Unauthorized reproduction of this manual is prohibited. This manual is being used during
review sessions conducted by PharmacyPrep. 66-2
Pharmacyprep.com
A) Understanding other cultures and respecting sensitive issues.
B) Offer services to all customers of different belief system
C) Offer privacy and confidentiality to customers of their personal information
D) All of the above
14) Customer of your pharmacy with kids. Parents discipline their kids but they are beating and spanking?
What to do?
A) Talk to parents and kids
B) Call police and inform about it
C) Call children society to protect kids
D) Ignore and let them go
15) Adult women come to your pharmacy to buy analgesics for her wounds. She seems to physically and
sexually abused what to do?
A) Call police
B) Ask her to go emergence
C) Refer to doctor
D) Give her police phone number to contact
E) Ask her to contact sexually abused support groups
16) Which of the following is NOT an environmental barrier?
A) Background noise
B) Telephone rings
C) Height of pharmacy counter
D) Dealing with a depressed patient
E) Loud music

Copyright © 2000-2014 TIPS Inc. Unauthorized reproduction of this manual is prohibited. This manual is being used during
review sessions conducted by PharmacyPrep. 66-3
Pharmacyprep.com
ANSWERS:
1) A
2- C
3- B
4- B
Tips: The language can be barrier. However it is not distraction.
5. A
6- D
7. A
Tips: It is boring to speak in a monotonous language

8. B
9. A
10) B
11) E
12) C
13) D
14) A
15) E
16) D

Copyright © 2000-2014 TIPS Inc. Unauthorized reproduction of this manual is prohibited. This manual is being used during
review sessions conducted by PharmacyPrep. 66-4
PharmacyPrep.Com Strategies to Prevent errors and safe dispensing

PHARMACY PREP
MEDICATION ERRORS
1. Verbal prescriptions of drug "names sound like" drugs, can cause medication errors. What should manager
do to decrease the error EXCEPT:
A. Put them away from each other
B. Put label on vial (call for error)
C. Put in computer system alarm to work on dispensing
D. Technician ask patient for what disease being treated for
E. Make two persons double check before dispensing

2-Busy pharmacies a long line up in 15 customers, a person came he is insisting and fighting with pharmacist,
do dispense him first?
A-ignore that customer
B-dispense him first
C-don’t dispense
D-just talk to him and explain there are other before you.
E-Call police
3) A very angry customer comes to your pharmacy and complains that you have dispensed wrong
medication. What is initial appropriate action?
I-offer private counselling area
II-check or verify medication
III-if there is dispensing error, apologize and correct it
A) I only B) III only C) I and II D) II and III E) I, II, III
4) In case of recall products, what is first thing to do?
A-Call all patient still using it and inform about recall
B-Inform the manager follow procedure for recall product
C-Post a sign in the pharmacy regarding recall product.
D-Inform doctor to stop dispensing this product in the future
E-Discard all product in waste container
5) Angry person walks in your pharmacy and complains that you have dispensed wrong medications? What is
appropriate first step in resolving this problem?
A-Apologise patient for dispensing error
B-Calm down patient
C-Acknowledge his anger, ask and verify what is error
D-Ignore and let him vent his anger
E-Acknowledge his anger and refer to doctor

Copyright © 2000-2014 TIPS Inc. Unauthorized reproduction of this manual is prohibited. This manual is being used during
review sessions conducted by PharmacyPrep. 67-1
PharmacyPrep.Com Strategies to Prevent errors and safe dispensing
6) To minimize error in look-alike drug names tall man (mixed case) letters is used to help draw attention?
A. To the dissimilarities in their names
B. To similarities in their names
C. To change name to new name
D. To prescribe different drug
E. To dispense different drug than prescribed
7) Which of the following drug can be mistaken for prednisone prescription?
A. Premarin
B. Prednisolone
C. Methylprednisolone
D. Methyltestosterone
E. Dexamethasone
8) Angry customer comes to your pharmacy. Says dispensing error occurred, which of the following first step
should be done?
A) Apologize customer
B) Calm down and verify prescription error
C) Calm down angry customer
D) Apologize and return new drug
E) Refuse to accept any error
9) Which of the following should first checked by the pharmacist to minimize the dispensing error?
A) Patient name B) Doctor name C) Drug identification number D) Date of birth E) Drug name
10) A patient comes to your pharmacy and complains that he got 10 less tablets of oxycontin for his
prescription. What is the better strategy to minimize these errors.
A) always count tablets in front of patient
B) double or triple count in pharmacy before dispense
C) if some patient complains just give extra tablets
D) narcotics cannot be dispensed extra tablets
E) Check inventory and reassure patient we double check
11) Which of the following is the best method of minimizing calculations errors in preparations?
A) Choose a person has multiple years of experience
B) Choose a inexperience a new pharmacy graduate
C) Ask a pharmacist or technician to double check the calculation before preparation
D)The calculations should be performed by calculators
E) None of the above

Copyright © 2000-2014 TIPS Inc. Unauthorized reproduction of this manual is prohibited. This manual is being used during
review sessions conducted by PharmacyPrep. 67-2
PharmacyPrep.Com Strategies to Prevent errors and safe dispensing
12) Which of the following medication error would not risk harm?
A)Wrong drug B) Wrong instruction C)minor spelling mistakes D)inappropriate auxiliary labels E) expired
medications
13) A pharmacy dispensing error dispensed Pradox (dabigatran) instead of Plavix (clopidogrel). If patient
used. Which of the following problem expected?
A) Increase risk myocardial infarction
B) Increase risk of platelet aggregation
C) Increase risk of bleeding
D) Increase risk of blood clot formation
14) If patient used Lasix (furosemide) instead of Losec (omeprazole). What problem is expected?
A) hyperglycemia B) hyperthyroidism C)hypotension D)Hypercalcemia E)None
15) A pharmacy technician prepared a prescription of Pradox instead of Plavix. What is the appropriate
strategies to prevent this error.
I) Advise technician to change to Plavix (clopidogrel)
II) Educate technician on drug name sound similar
III) Read label to patient while dispensing
A) I only B) III only C) I and II D) II and III E) I, II, III
16) A pharmacy intern receive a phone call from doctor for a new prescription. Dr want to prescribe Losec.
However pharmacy intern documented as Lasix. What is appropriate to minimize this error?
A) Only pharmacist should take phone call for a new prescription
B) Pharmacy intern should require more training on sound like name drugs
C) Ask doctor to give generic name for the sound like name drugs
D) Ask doctor to fax prescription for sound like name drugs
E) Ask doctor to written prescription for all new prescriptions
17) Which of the following is NOT dangerous drug abbreviation?
A) od
B) qid
C) daily
D) IU
E) qd
18) What is recommended to write instead of "IU"
A) iu
B) units
C) international units
D) IU
E) I.units

Copyright © 2000-2014 TIPS Inc. Unauthorized reproduction of this manual is prohibited. This manual is being used during
review sessions conducted by PharmacyPrep. 67-3
PharmacyPrep.Com Strategies to Prevent errors and safe dispensing
19) What is the best recommendation to write prednisone and prednisolone to avoid errors.
A) prednisone and prednisolone
B) predniSONE and predniSOLONE
C) predNISone and predNISOLone
D) PREDNISONE and prednisolone
E) predsnisone and PREDNISOLONE
20) What is correct tallman letters for diphenhydramine and dimenhydrinate
A) dimenHYDRINATE and diphenHYDRAMINE
B) diMENhydrinate and diPHENhydramine
C) DIMENHYDRINATE and diphenhydramine
D) dimenhydrinate and DIPHENHYDRAMINE
21) Which of the following abbreviation least likely cause error?
A) q.d B) QD C) qid D) q6h E) I.U
22) What agencies in Canada are associated in reporting medication incidents?
A) Institute of safe medication practices (ISMP)
B) Health Canada’s “Med effect” program
C) Canadian Coalition for Medication Incident Reporting
D) All of the above
Ans. D
23) Which of the following can contribute to medication incidents?
A) Poor lighting
B) Poor handwriting
C) Lack of patient education
D) Clutter in work area
E) All of the above

Copyright © 2000-2014 TIPS Inc. Unauthorized reproduction of this manual is prohibited. This manual is being used during
review sessions conducted by PharmacyPrep. 67-4
PharmacyPrep.Com Strategies to Prevent errors and safe dispensing
ANSWERS:
1. D
2- D
3) E
4) B
5) C
6) A
Tips: look-alike drug names tall man (mixed case) letters is used to help draw attention to the dissimilarities
in their names.
7) B
Tips: Prednisone can be mistaken for prednisolone
8) B
9) A
10) B
11) C
12) C
13) C
Tips: Pradox is a new anticoagulant dabigatran act as factor Xa inhibitor. However Plavix is clopidogrel an
antiplatelets drugs. So increase risk of bleeding
14) C
Tips: Lasix is furosemide can cause hypotension. Losec is omeprazole.
Immodium (Loperamide) and Motilium (domperidone)
Dimenhydrinate and diphenhydramine
Cyclosporine and Cyclophosphomide
Proscar and Prozac
Lamisil (terbinafine) and Lamictal (lamotrigine)
15) E

Copyright © 2000-2014 TIPS Inc. Unauthorized reproduction of this manual is prohibited. This manual is being used during
review sessions conducted by PharmacyPrep. 67-5
PharmacyPrep.Com Strategies to Prevent errors and safe dispensing
16) C
17) C
18) B
19) C
Tips: Tallman letter can be used to avoid error
20) B
21) D
22) D
23)E

Copyright © 2000-2014 TIPS Inc. Unauthorized reproduction of this manual is prohibited. This manual is being used during
review sessions conducted by PharmacyPrep. 67-6
PharmacyPrep.Com Pharmaceutical Storage Conditions

PHARMACY PREP
PHARMACEUTICAL STORAGE CONDITIONS
1- You received a new drug in your pharmacy and the manufacture instructions tell you to keep this drug in a
cool place. Which temperature should you keep this drug?
a) <0°C b) 2 to 8°C c) 8 to 15°C d) 15 to 30°C e) >30°C
2-All of the following antibiotics extemporaneous suspensions are stored at room temperature, EXCEPT
A-Metronidazole B-Clarithromycin C-Azithromycin D-Clindamycin
E-Cotrimoxazole
3) Insulin is stored at?
A-Room temperature
B-Refrigerator
C-Freezer
D-Deep freezer
E-Cool temperature
4) All vaccines should be stored in?
A-Room temperature
B-Refrigerator
C-Freezer
D-Deep freezer
E-Cool temperature
5) What is the optimal temperature for vaccine fridge is?
A. 5°C B. 0°C C.1°C D.2°C E. 4°C
6) Which of the following drugs are in cold chain?
A. Inhalers
B. Insulin
C. Vaccine
D. Azithromycin suspension
E. Digoxin

Copyright © 2000-2014 TIPS Inc. Unauthorized reproduction of this manual is prohibited. This manual is being used during
review sessions conducted by PharmacyPrep. 68-1
PharmacyPrep.Com Pharmaceutical Storage Conditions
ANSWERS:
1- C
2- A
3- B
4- B
5. A
6. C

Copyright © 2000-2014 TIPS Inc. Unauthorized reproduction of this manual is prohibited. This manual is being used during
review sessions conducted by PharmacyPrep. 68-2
www.pharmacyprep.com OTC medications dermatological conditions

PHARMACY PREP
OTC DRUGS FOR DERMATOLOGICAL CONDITIONS
1-Incorrect statement about poison ivy treatment?
A-Hydrocortisone 0.5%
B-Calamine lotion
C-Antihistamine
D-Antibiotics
E-Oatmeal cream
2-Incorrect pharmacotherapy for psoriasis treatment?
A-Hydrocortisone 1%
B-Coal tar
C-Tazarotene
D- Antibiotics
E-UVA light + coal tar
Burns
3) What is incorrect about burns?
st
A) 1 degree is superficial burns, that may cause by sunburn
nd
B) 2 degree is superficial and partial thickness burns that may cause by hot water, oil spills and splashes
rd
C) 3 degree is the severe burns that can give lathery skin.
D) If it burn put burn part in cold water
E) If it burn put burn part in warm water
4-Incorrect statement about sunburn?
A-peak time of sunburn is 10:00 am to 4:00pm
B-High altitude has low sunburn
C-cloud do not protect sunburn
D-beach area may have more sunburn
E-Apply sunscreen before get into sun
5-What is incorrect about sunburn treatment?
A-Use cool baths or wet compresses with tap water or saline
B-Take plenty of fluids
C-Topical vitamin E applied two minutes after UV exposure may decrease erythema and edema
D-Acetaminophen can relieve pain
E-Avoid NSAIDs for sunburn pain

Copyright © 2000-2014 TIPS Inc. Unauthorized reproduction of this manual is prohibited. This manual is being used during
review sessions conducted by PharmacyPrep. 69-1
www.pharmacyprep.com OTC medications dermatological conditions
Head lice
6) What is incorrect about head lice treatment?
A-Permethrin 1% cream rinse apply to hair and scalp (1/2 to 1 bottle) and leave it on for 10 min then rinse.
B-Permethrin may repeat after 7 days if live head lice is observed.
C-May temporarily exacerbate itching and pruritus in head.
D-May gives burning/stinging, tingling numbness in scalp
E-After permethrin if it gives itching and pruritus refer to doctor
7) What is incorrect about head lice?
A-All close contact need not to treated if one person has head lice
B-Head lice can fly and transmit to other family member
C-Long hair or short hair both can have head lice
D-Hygiene is a not criteria for head lice
E-Nits can hatch head lice in 7 to 10 days
8-Contact dermatitis and itching :
I-oatmeal cream
II-calamine lotion or zinc oxide
III-local anesthetics (procaine)
A-I only
B-III only
C-I and II only
D-II and III only
E-All of the above
9) Complicated diaper rash (diaper rash more than 3 days):
I) barrier bases (zinc oxide, petrolatum)
II-Antifungal agents (Nystatin)
III-Anti-inflammatory agents (0.5% hydrocortisone)
A-I only
B-III only
C-I and II only
D-II and III only
E-All of the above
10-What is incorrect in treatment of diaper rash?
A-Keep diaper area dry
B-clean diaper area with alcohol swabs
C-Apply petrolatum on diaper area
D-Apply 0.5% hydrocortisone

Copyright © 2000-2014 TIPS Inc. Unauthorized reproduction of this manual is prohibited. This manual is being used during
review sessions conducted by PharmacyPrep. 69-2
www.pharmacyprep.com OTC medications dermatological conditions
11-What does sun protection factor (SPF) 15 mean?
A-if it takes 10 minutes to get sunburn, in SPF 15, it would take 15x10 min
B-If it takes 10 minutes to get sunburn in SPF 15, it would take 15x100 min
C- If it takes 10 minutes to get sunburn in SPF 15, it would take 150x100 min
D- If it takes 10 minutes to get sunburn in SPF 15, it would take 150x1000 min
E-If it takes 10 minutes to get sunburn in SPF 15, it would take 1500x100 min
12) All of the following are used for dandruff, except?
A-Ketoconazole
B-Coal tar
C-Zinc pyrethione
D-Hydrocortisone
E-Selenium sulfide
13-Which of the following is NOT a pharmacotherapy for acne?
A-Oral contraceptives
B-Corticosteroids
C-Tetracycline
D-Retinoid
E-A and B
14-Which of the following is pharmacotherapy for acne in pregnancy?
A-Oral contraceptives
B-Erythromycin
C-Tetracycline
D-Retinoid
E-Minocycline
15) A 20 year old women treated with topical treatment for acne, however, it not treated, which of the
following is recommended?
A-Benzoyl peroxide
B-Tretinoin
C-Isotretinoin
D-Salicylic acid
E-None
16) Which of the following skin condition have symptoms of red scaly patches?
A) Psoriasis B) Dermatitis C) Acne D) Rosacea E) Urticaria
17) Rosacea have?
A) Postules
B) Nodules
C) Pimples
D) Erythema
E) Red scaly patches

Copyright © 2000-2014 TIPS Inc. Unauthorized reproduction of this manual is prohibited. This manual is being used during
review sessions conducted by PharmacyPrep. 69-3
www.pharmacyprep.com OTC medications dermatological conditions
18) Which of the following used to treat acne vulgaris?
I) retinol
II) cis/trans retinoic acid
III) Vitamin E
A) I only B) III only C) I and II only D) II and III E)all
19) What is NOT a symptoms of acne?
A) White head (comedon)
B) Black head (comedon)
C) Pimple
D) Postule
E) Red scaly patches

Copyright © 2000-2014 TIPS Inc. Unauthorized reproduction of this manual is prohibited. This manual is being used during
review sessions conducted by PharmacyPrep. 69-4
www.pharmacyprep.com OTC medications dermatological conditions
ANSWERS:
1- D
2- D
Burns
3) E
4- B
5- E
Head lice
6- E
7) B
8- C
Tips: Astringents such as calamine lotion, aluminum acetate, zinc oxide
9- E
10- B
11- A
Tips: snow, high altitude, and beach, mountains can give more sun burn.
12) D
13) B
14- B
15) C
16) A
17) D
18) C
19) E
Tips: Red scaly patches are symptoms of psoriasis,

Copyright © 2000-2014 TIPS Inc. Unauthorized reproduction of this manual is prohibited. This manual is being used during
review sessions conducted by PharmacyPrep. 69-5
www.pharmacyprep.com OTC medications dermatological conditions
Copyright © 2000-2014 TIPS Inc. Unauthorized reproduction of this manual is prohibited. This manual is being used during
review sessions conducted by PharmacyPrep. 69-6
PHARMACY PREP
OTC AND PRESCRIPTION DRUGS FOR OPHTHALMIC, EAR AND MOUTH DISORDERS
1. What is an optimal range of pH for ophthalmic products
A. 2-3 B. 4-5 C. 5-6 D. 6-8 E. 8-10
2. Ophthalmic agents contraindicated in glaucoma patients include which of the following substances:
A. antioxidants
B. decongestants
C. emollients
D. antipruritics
E. All of the above
3. Which of the following lenses can be worn continuously for 6 days:
A. Soft lenses
B. Extended wear soft lenses
C. Frequent replacement soft lenses
D. Hard lenses
E. Soft and hard lenses
4. Image formed by lens received by:
A. iris B. retina C. sclera D. conjunctiva E. pupil
5. What is correct statement about treatment in eruption cysts in infants
A. In general cysts rupture spontaneously
B. Use teeters to rupture cysts
C. Manually cysts can be removed
D. None of the above
6. Which of the following treatment is not used to treat cold sores?
A. Apply ice
B. Zinc oxide (ZnO)
C. Benzocaine 7.5% gel
D. Abreva
E. Acyclovir
7. What is treatment of cold sores in chicken pox?
I-Calamine lotion
II-Oatmeal bath (micellar colloidal bath)
III-antibiotics
A. I only B. III only C. I and II only D. II and III only E. All of the above

Copyright © 2000-2014 TIPS Inc. Unauthorized reproduction of this manual is prohibited. This manual is being used during
review sessions conducted by PharmacyPrep. 70-1
8. Which of the following is the incorrect action of sympathomimetics?
I- Increase of IOP
II- Increase the drainage of H2O humor from the eye
III- Decrease the H2O humor in the eye
A. I only B. III only C. I and II only D. II and III only E. All of the above
9. Which of the following ophthalmic conditions required referral to doctor?
A. Red eye B. Pain in eye C. Blurred vision D. Dry eye E. B and C
10) Which of the following ophthalmic conditions require referral to doctor?
I) Red eye or pink eye
II) Pain in eyes
III) Blurred vision
A. I only B. III only C. I and II only D. II and III only E. All of the above
11. A customer of your pharmacy using metformin 500 mg bid and glyburide qd. Complains to have gritty
and sandy feeling in eye. What is appropriate to do?
A. Refer to doctor because of diabetic condition
B. Refer to doctor because sandy and gritty feeling in eye is conjunctivitis
C. Refer to doctor because glyburide side effect is dry eye
D. Refer to doctor because metformin side effect is dry eye
E. Refer to doctor because these are side effects of anticholinergic drugs
12. Which of the following can be used for earwax removal?
I) carbamide peroxide
II) soft mineral oil
III) Burrow's solution (aluminum acetate)
A. I only B. III only C. I and II only D. II and III only E. All of the above
13. Otitis externa (swimmers ear), is treated by?
A. amoxicillin high dose
B. amoxicillin + clavulanate
C. Aluminum acetate (Burrow's solution)
D. Cephalosporin's
E. All of the above
14. A 60 yo customer of your pharmacy medical profile include; metformin, glyburide, amiodarone,
rosuvastatin, ASA 81 mg. Recently doctor prescribed methotrexate to treat psoriasis. Patient reports sandy
and gritty eyes. Pharmacist direct patient to see doctor. Which of the following drug concern prompted
pharmacist refer to doctor?
A. methotrexate
B. metformin
C. Glyburide

Copyright © 2000-2014 TIPS Inc. Unauthorized reproduction of this manual is prohibited. This manual is being used during
review sessions conducted by PharmacyPrep. 70-2
D. amiodarone
E. rosuvastatin
ANSWERS:
1. D
2. B
3. B

Copyright © 2000-2014 TIPS Inc. Unauthorized reproduction of this manual is prohibited. This manual is being used during
review sessions conducted by PharmacyPrep. 70-3
4. B
5. A
6. B
7. C
Tips: Calamine lotion (ZnO+FeO) and oatmeal bath and oral antihistamine can be helpful in treatment of cold
sores.
8. D
Tips: Sympathetic agonist (sympathomimetics) increases IOP. Open angle glaucoma (OAG) is due to increase
in IOP and increase in H2O humor of eye or could be due decrease in drainage of H2O humor in the eye. For
the treatment of OAG sympathetic blockers such as Timolol are effectively used to decrease IOP and H 2O
humor.
9. E
10. D
11. A
Tips: Diabetic retinopathy symptoms require referral to doctor
12. C
13. C
14. D
Tips: amiodarone gives ocular deposits SEs.

Copyright © 2000-2014 TIPS Inc. Unauthorized reproduction of this manual is prohibited. This manual is being used during
review sessions conducted by PharmacyPrep. 70-4
PharmacyPrep.Com OTC Antihistamine's, Decongestants and Antitussives

PHARMACY PREP
OTC ANTIHISTAMINE, DECONGESTANTS AND ANTITUSSIVES
1) Which of the following is NOT an environmental measure to prevent allergic rhinitis:
A-Avoidance of window or attic fans
B-Avoid outdoor activities during pollen season
C-Shower or bath following outdoor activity to avoid contamination of bedding
D-Use air conditioning
E-Open windows over night to get fresh air.
2-Sympathomimetics decongestant acts as?
A-Vasoconstrictor
B-Vasodilator
C-Congestion
D-Anti-inflammatory actions
E-None of the above
3-Sympathomimetics decongestant can cause, all of the following complications, except?
A- Open angle glaucoma
B- hypertension
C- diabetics
D-Can cause rebound congestion in prolong use
E-Can cause vasodilatation
4) All of the following drugs gives bronchodilators, except
A-short acting beta2 agonist
B-long acting beta2 agonist
C-Propranolol
D-Salbutamol
E-Salmeterol
5-Opioids may be used as all the following agents, EXCEPT:
a) Antitussive b) Analgesic c) Antidiarrheal d) Anti-inflammatory
e) Pre-anesthetic
6-Alpha 1 agonist, phenylephrine has all of the following side effect EXCEPT:
A-vasoconstriction B-Headache C) orthostatic hypotension D-Arrhythmias
E-Mydriasis
7- Fexofenadine HCI is classified as:
A-Histamine H2-Receptor Antagonist B- Histamine H1-Receptor Antagonist
C-Proton Pump inhibitor (PPI) D-Serotonin reuptake inhibitor (SSRI)
E-5-HT1 Receptor Agonist

Copyright © 2000-2014 TIPS Inc. Unauthorized reproduction of this manual is prohibited. This manual is being used during
review sessions conducted by PharmacyPrep. 71-1
PharmacyPrep.Com OTC Antihistamine's, Decongestants and Antitussives
8-A mother comes to your pharmacy with concern of her child 2 yr old having fever and pharyngitis,
currently using amoxicillin. What is appropriate recommendation?
A-recommend ASA
B-recommend oral ibuprofen
C-Recommend acetaminophen suppositories
D-Recommend cold sponges
E-do not recommend any medications
9) A customer of your pharmacy travelling to London, Ontario, wants to buy some medications for nausea
and vomiting in motion sickness. What is appropriate recommendation?
A) Dimenhydrinate B)Dextromethorphan C) Scopolamine D)Ginger root E) Meclazine
10) A customer of your pharmacy using MAO Inhibitors for major depression. Now looking for nasal
congestion medication. What is recommended?
A) pseudoephedrine B) ephedrine C) codeine D) dextromethorphan E) saline nasal drops
11. A regular customer, purchased dextromethorphan 2 times in last 20 days. Pharmacist talk to patient and
checked patient profile and referred to doctor. Which of the following medication in patient profile
prompted pharmacist?
A) Glyburide B) Metformin C) Rosuvastatin D) Ramipril E) All of the above
12. A 50 year old school bus driver brings prescription of alprazolam and clonazepam to treat anxiety. He is
frequent customer of pharmacy to purchase diphenhydramine and dimenhydrinate. Pharmacist informed to
doctor patient use of benzodiazepines and antihistamine. Dr reported to employer of patient. What ethical
principle were applied?
A) Nonmaleficence
B) Beneficence
C) Autonomy
D) Veracity
E) Justice

Copyright © 2000-2014 TIPS Inc. Unauthorized reproduction of this manual is prohibited. This manual is being used during
review sessions conducted by PharmacyPrep. 71-2
PharmacyPrep.Com OTC Antihistamine's, Decongestants and Antitussives
ANSWERS
1) E
2- A
3- E
4) C
5- D
6- C
Tips: Hypertension is common side effect phenylephrine.
7- B
8- B
9) A
10) E
Tips: with MAO I for nasal congestion avoid using pseudoephedrine, ephedrine can cause hypertension crisis
For cough avoid dextromethorphan cause serotonin syndrome
11. D
Tips: ACE Inhibitors may induce cough. This can be managed by changing therapy to ARBs.
12. A

Copyright © 2000-2014 TIPS Inc. Unauthorized reproduction of this manual is prohibited. This manual is being used during
review sessions conducted by PharmacyPrep. 71-3
PharmacyPrep.Com OTC Antihistamine's, Decongestants and Antitussives
Copyright © 2000-2014 TIPS Inc. Unauthorized reproduction of this manual is prohibited. This manual is being used during
review sessions conducted by PharmacyPrep. 71-4
PharmacyPrep.Com OTC drugs for headache, sports injuries, pressure ulcers, and back pain

PHARMACY PREP
OTC DRUGS FOR HEADACHE, SPORTS INJURIES, PRESSURE ULCERS, AND LOW BACK PAIN
1. Correct statement/s about ASA may include
I. Low dose are used in heart problem
II. Less potent than naproxen as an analgesic
III. More potent than acetaminophen as analgesic

a) I only b) III only c) I and II only d) II and III only e) All of the above
2-What is antidote of acetyl salicylic acid (ASA)?
A-N-acetylcysteine B-Flumazenil C-Naloxone D-Naltrexone E-None of the above
3-Anti-inflammatory action of ASA is due to?
A-Irreversible inhibition of platelet aggregation
B-Prevention of prostaglandin synthesis
C-Inhibition of COX-I receptors
D-Inhibition of Cox-1 and Cox-II receptors
E-Inhibition of Cox-II receptors
4. Dyspnea means
a) Painful muscle spasms b) Pain in the heart c)Pain in extremities d)Painful breathing

e) Painful menstruation
5. Over dosage of acetaminophen in patients would affect
I) Liver functions ii) Prothrombin time iii) Renal functions
a) I only b) III only c) I and II only d) II and III only e) All of the above
6-Which of the following is not a side effect of an opioids analgesic for example morphine:
a) C.V depression
b) Constipation
c) Sedation
d) CNS depression
e) Dilatation of eye pupil
7-A sports person has ankle sprain while playing ice hockey, what is inappropriate recommendation?
A-Rest to injured part
B-Apply ice
C-Warm compress
D-Elevate injured area
E-Recommend OTC NSAIDs

Copyright © 2000-2014 TIPS Inc. Unauthorized reproduction of this manual is prohibited. This manual is being used during
review sessions conducted by PharmacyPrep 73-1
PharmacyPrep.Com OTC drugs for headache, sports injuries, pressure ulcers, and back pain
8) Mechanism of action of sumatriptan is?
A) 5HT1b/d agonist
B) 5HT1b/d antagonist
C) 5HT3 antagonist
D) 5HT3 agonist
E) 5HT2 agonist
9) What is treatment for tension headache?
A) Triptans B-Ergot alkaloids C-Propranolol D-Acetyl salicylic acid
E) Acetaminophen
10) All of the following can trigger migraine, except?
A) Smell
B) Food like chocolates
C) Light
D) Emotional stress
E) Physical activity

11) A 45 yo man diagnosed with benign prostatic hyperplasia, and migraine prophylaxis. Which of the
following is the appropriate therapy for migraine prophylaxis?
A) Propranolol
B) Nortriptyline
C) Amitriptyline
D) Sumatriptan

E) Zolmitriptan
12) A customer of your pharmacy using sumatriptan for migraine attacks. What is correct statement?
A)after taking sumatriptan migraine headache does not relieve than double the dose of sumatriptan
B)after taking sumatriptan migraine headache does not relieve than decrease the dose of sumatriptan
C)after taking sumatriptan migraine headache does not relieve than do not use sumatriptan
13) A 12 month old child swallowed acetaminophen 10 ml syrup. A panic mother contact pharmacy. What is
the initial symptoms of acetaminophen overdose?
A) Nausea B) Vomiting C) Stomach upset D) Diarrhea E) Hepatotoxicity

Copyright © 2000-2014 TIPS Inc. Unauthorized reproduction of this manual is prohibited. This manual is being used during
review sessions conducted by PharmacyPrep 73-2
PharmacyPrep.Com OTC drugs for headache, sports injuries, pressure ulcers, and back pain
ANSWERS:
1. C
2- E
3- B
4. D
5. C
6- E
7- C
Tips: R-I-C-E is recommended approach for sports injuries.
R= rest to injured part, I = apply ice, C = cold compress, E = elevated injured part.
8) A
9) E
10) D
11) A
12) C
13) B

Copyright © 2000-2014 TIPS Inc. Unauthorized reproduction of this manual is prohibited. This manual is being used during
review sessions conducted by PharmacyPrep 73-3
PharmacyPrep.Com OTC drugs for headache, sports injuries, pressure ulcers, and back pain
Copyright © 2000-2014 TIPS Inc. Unauthorized reproduction of this manual is prohibited. This manual is being used during
review sessions conducted by PharmacyPrep 73-4
www.PharmacyPrep.Com Asthma and COPD

PHARMACY PREP
ASTHMA AND COPD
1) Asthma acute exacerbation can be triggered by all of the following EXCEPT:
A. warm and dry weather
B. stressful emotional time
C. bacterial and viral pneumonia
D. withdrawal of asthma medications
2) Emphysema is characterized by:
A. Permanent enlargement of small sacs in lungs
B. Symptoms are scanty sputum
C. Shortness of breath.
D. All of the above
3) Which of the following inhaler require rinsing mouth after use.
A. Servent (salmetrol)
B. Salbutamol
C. Atrovent (ipratropium)
D. Pulmicort (budesonide)
E. Theophylline
4) A 3 yr child was newly diagnosed with asthma. All of the following questions are important to ask his
mother EXCEPT:
A) Any cardiovascular problems?
B) Does other family member have asthma?
C) How often symptoms come?
D) Old asthma medication he used before?
E) How many times wake up in night due to symptoms
5) A 45 yo patient with emphysema and pain, and is not controlled, the doctor switched him to morphine
and his daughter came to pharmacy, you will call the doctor because:
a) You will ask to change the medication
b) To tell the dose of medication because its too much
c) To ask for written prescription of morphine
d) Her concern about constipation
6. What is true about Ipratropium?
I. Reversible competitive of anticholinergic
II. Synergistic with Theophylline in asthma
III. Quaternary ammonium compound
a. I only

Copyright © 2000-2014 TIPS Inc. Unauthorized reproduction of this manual is strictly prohibited and it is illegal to reproduce
without permission. This manual is being used during review sessions conducted by PharmacyPrep. 74-1
www.PharmacyPrep.Com Asthma and COPD
b. III only
c. I and III only
d. II and III only
e. I, II, and III only

7) Which of the following is incorrect statement of leukotriene receptors inhibitors?


A) used to control the dose of steroids
B) Indicated to treat congestion in children over 2 yr
C) Treatment of choice for beta blockers and Aspirin induced asthma
D) Patient used leukotriene receptors inhibitors require increase requirement of steroids.
E) Montelukast is used in children over 2 yr age
8) A 40 yr old asthma patient brings a new prescription of montelukast bid. Currently medication profile
include salbutamol prn, Symbicort (budesonide and formoterol) 1 puff bid. What is the action to do?
A) ask patient to stop using Symbicort
B) ask patient to stop using salbutamol prn
C) dispense montelukast and counsel how to use it
D) Contact doctor to change Symbicort to Advair
E) Contact doctor to change to Zafirlukast
9) A COPD patient brings a prescription of doxycycline daily to treat pneumonia. Which of the following is the
correct statement?
A) ask patient if he is taken flu vaccine
B) ask patient if his close contacts have taken flu vaccine
C) ask patient if his close contacts have taken flu and pneumonia vaccine
D) ask patient if he had taken flu and pneumonia vaccine
E) ask patient if he has taken pneumonia vaccine only
10. What is inappropriate for acute asthma?
A) beta2 agonist inhaler B) iv hydrocortisone C) iv epinephrine D) inhaler anticholinergic
11. A 55 yo male admitted in emergence with asthma exacerbations. What is the preferred drug of choice?
A) corticosteroids B) salbutamol nebulizer C) antihistamine D) theophylline
12. Which of the following asthma device at home helps patient to determine when to go emergence?
A) Peak flow meter measures peak expiratory flow rate (PEFR)
B) Forced expiratory volume in one second (FEV1)
C) Forced vital capacity (FVC)
D) Total lung capacity

Copyright © 2000-2014 TIPS Inc. Unauthorized reproduction of this manual is strictly prohibited and it is illegal to reproduce
without permission. This manual is being used during review sessions conducted by PharmacyPrep. 74-2
www.PharmacyPrep.Com Asthma and COPD
13. Which of the following is NOT a side effect of oral prednisone?
A) weight gain
B) acute pancreatitis
C) Hypertension
D) viral infections
E) Steven Johnson syndrome
14) The following medications are used in asthma emergencies:
A. Salbutamol
B. Ipratropium
C. Salmeterol
D. All of the above
15) Theophylline serum levels are monitored in neonates and infants because
A) They are poorly absorbed in systemic circulations
B) Drug has narrow therapeutic index
C) Slowest rate of clearance
D) Fastest rate of clearance
E) It is not used in neonates and infants
16) A patient is suffering from COPD and asthma. He is on theophylline, ranitidine and now doctor prescribed
ciprofloxacin which of the following is correct?
A) ranitidine have drug interaction with theophylline
B)Ciprofloxacin have drug interaction with theophylline
C)Ranitidine and ciprofloxacin both have drug interactions with theophylline
D) None of the above
17) Which of the following is NOT a side effect of theophylline?
A-Insomnia
B-Sedation
C-Appetite suppressant
D-None of the above
18) In emergency treatment of asthma, the preferred first line treatment is:
A. theophylline
B. salbutamol
C. cromolyn sodium
D. Zafirlukast
19. Nosocomial infection is
A. Nasal infection
B. Mixed bacterial infection
C. Mixed viral infection

Copyright © 2000-2014 TIPS Inc. Unauthorized reproduction of this manual is strictly prohibited and it is illegal to reproduce
without permission. This manual is being used during review sessions conducted by PharmacyPrep. 74-3
www.PharmacyPrep.Com Asthma and COPD
D. Infections acquired in a hospital or other health care settings
20. Drugs that Acts as long acting B2 adrenergic agonist:
A. Salbutamol
B. Metaproterenol
C. Terbutaline
D. Formoterol
E. Prednisone
21) Which drug is not use in asthma?
A. B2 adrenergic antagonist
B. Methylxanthine
C. Leukotriene antagonist
D. Corticosteroid
22) All of the following are side effects of Salbutamol except:
A. Tremors
B. Palpitation
C. Restlessness
D) Oral thrush
23) Therapeutic use of inhaled corticosteroids:
A. Prevent Nocturnal asthma
B. Prevent exercise induced asthma (EIA)
C. To treat uncontrolled asthma
D. All of the above
24) Which drug decreases effect of Theophylline?
A. Ciprofloxacin
B. Allopurinol
C. Pulmonary edema
D. Carbamazepine
25) Drug of choice for Aspirin induced asthma:
A. Salbutamol
B. Zafirlukast
C. Montelukast
D. Fluticasone
26) Which anticholinergic drugs is used in asthma?
A. Ipratropium bromide
B. Scopolamine
C. Pilocarpine
D. Atropine

Copyright © 2000-2014 TIPS Inc. Unauthorized reproduction of this manual is strictly prohibited and it is illegal to reproduce
without permission. This manual is being used during review sessions conducted by PharmacyPrep. 74-4
www.PharmacyPrep.Com Asthma and COPD
27) Fluticasone drugs are use to:
A. Control inflammation in asthma and allergic rhinitis.
B. Control asthma only
C. Treat Aspirin induced asthma
D. Relaxes the muscles surrounding the bronchioles.
28. Combivent available dosage forms is/are:
A. Nebulizers
B. MDI
C. A & B
D. Capsule
29. Risk factors that causes COPD except:
A. Second hand smoke
B. Family history
C. Asthma
D. Age
30. High risk factor for COPD
A. Asthma
B. Smoking
C. Second hand smoke
D. Air pollution
31. Function as mast cell inhibitor:
A. Cromolyn sodium
B. Budesonide
C. Zafirlukast
D. Beclomethasone
32) Onset of action for short acting drugs is?
A. 10-15 minutes
B. 10-20 seconds
C. 10-15 seconds
D. 10-20 minutes
33) What to do to reduce corticosteroids side effects?
A. Stop corticosteroid
B. Rinse mouth and gargle after using the drug
C. Chew a gum
D. Brush your teeth every after using the drug

Copyright © 2000-2014 TIPS Inc. Unauthorized reproduction of this manual is strictly prohibited and it is illegal to reproduce
without permission. This manual is being used during review sessions conducted by PharmacyPrep. 74-5
www.PharmacyPrep.Com Asthma and COPD
34) Drugs are not affected by catecholamine O-methyl transferase (COMT) enzymes except:
A. Salmeterol
B. Salbutamol
C. Metoproterenol
D. Terbutaline
35) Function as mast cell stabilizers:
A. Zafirlukast
B. Ipratropium
C. Prednisone
D. Nedocromil
36) Rx: Vanceril (Beclomethasone) inhaler: dispense # 1
Sig: ii puffs inhaled bid. In filling the prescription, the technician should:
A. Inform the patient that this is a corticosteroid that has many serious side effects.
B. Include the following specific directions on the label: “Inhale 2 puffs into each nostril 3x a day”
C. Provide the patient the package insert included with the inhaler
D. Tell the patient to use this inhaler only when he really needs it.
37) Which of the following cause bronchospasm?
A. Ipratropium B. Metoprolol C. Albuterol D. Zafirlukast
38) A 45 yo patient with emphysema and pain, and is not controlled, the doctor switched him to morphine
and his daughter came to pharmacy, you will call the doctor because:
a) You will ask to change the medication
b) To tell the dose of medication because its too much
c) To ask for written prescription of morphine
d) Her concern about constipation
39) What is true about Ipratropium?
I. Reversible competitive of anticholinergic
II. Synergistic with Theophylline in asthma
III. Quaternary ammonium compound
a) I only
b) III only
c) I and III only

Copyright © 2000-2014 TIPS Inc. Unauthorized reproduction of this manual is strictly prohibited and it is illegal to reproduce
without permission. This manual is being used during review sessions conducted by PharmacyPrep. 74-6
www.PharmacyPrep.Com Asthma and COPD
d) II and III only
e) I, II, and III

40) Which of the following is incorrect statement of leukotriene receptors inhibitors?


A) used to control the dose of steroids
B) Indicated to treat congestion in children over 2 yr
C) Treatment of choice for beta blockers and Aspirin induced asthma
D) Patient used leukotriene receptors inhibitors require increase requirement of steroids.
E) Montelukast is used in children over 2 yr age
41) A 40 yr old asthma patient brings a new prescription of montelukast bid. Currently medication profile
include salbutamol prn, Symbicort (budesonide and formoterol) 1 puff bid. What is the action to do?
A) ask patient to stop using Symbicort
B) ask patient to stop using salbutamol prn
C) dispense montelukast and counsel how to use it
D) Contact doctor to change Symbicort to Advair
E) Contact doctor to change to Zafirlukast
42) A COPD patient brings a prescription of doxycycline daily to treat pneumonia. Which of the following is
the correct statement?
A) ask patient if he is taken flu vaccine
B) ask patient if his close contacts have taken flu vaccine
C) ask patient if his close contacts have taken flu and pneumonia vaccine
D) ask patient if he had taken flu and pneumonia vaccine
E) ask patient if he has taken pneumonia vaccine only
43) What is inappropriate for acute asthma?
A) beta2 agonist inhaler B) iv hydrocortisone C) iv epinephrine D) inhaler anticholinergic
44) A 55 yo male admitted in emergence with asthma exacerbations. What is the preferred drug of choice?
A) corticosteroids B) salbutamol nebulizer C) antihistamine D) theophylline
45) Which of the following asthma device at home helps patient to determine when to go emergence?
A) Peak expiratory flow rate (PEFR)
B) Forced expiratory volume in one second (FEV1)
C) Forced vital capacity (FVC)
D) Total lung capacity
46) Which of the following is NOT a side effect of oral prednisone?

Copyright © 2000-2014 TIPS Inc. Unauthorized reproduction of this manual is strictly prohibited and it is illegal to reproduce
without permission. This manual is being used during review sessions conducted by PharmacyPrep. 74-7
www.PharmacyPrep.Com Asthma and COPD
A) weight gain B) acute pancreatitis C) Hypertension
D) viral infections E) Steven Johnson syndrome
ANSWERS:
1) A
2) D
3) B
4) A
5) D
6. E
Tips: Theophylline is added id doses not response to anticholinergic)
7) D
8) C
9) D
10. C
11. B
Tips: In an emergence situation, the most rapidly acting agent is used first. The choice of route of
administration depends on severity of attack.
12. A
13. E
14. A
15. C
Tips: neonates and infants have slowest rate of clearance
16. C
Tips: Theophylline is substrate of CYP1A2, and ciprofloxacin is the inhibitor of CYP1A2
17. B

Copyright © 2000-2014 TIPS Inc. Unauthorized reproduction of this manual is strictly prohibited and it is illegal to reproduce
without permission. This manual is being used during review sessions conducted by PharmacyPrep. 74-8
www.PharmacyPrep.Com Asthma and COPD
18. B
19. D
20. D
21. A
22. D
23. D
24. D
25. B
26.A
27. A
28. C
29. D
30. B
31. A
32. D
33. B
34. A
Tips: Short acting is not affected by COMT enzyme because this is activated by non COMT enzyme.
35.C
36. C
37. B
Tips: Asthma is characterized by increased responsiveness of trachea and bronchi to various stimuli and
narrowing of airways. It is associated with shortness of breath, chest

Copyright © 2000-2014 TIPS Inc. Unauthorized reproduction of this manual is strictly prohibited and it is illegal to reproduce
without permission. This manual is being used during review sessions conducted by PharmacyPrep. 74-9
www.PharmacyPrep.Com Asthma and COPD
tightness, wheezing and coughing. The Beta-2 receptors stimulation generally dilates the bronchial smooth
muscles and helps in controlling asthma.
38. D
39. E
Tips: Theophylline is added id doses not response to anticholinergic)
40. D
41. C
42. D
43. C
44. B
Tips: In an emergence situation, the most rapidly acting agent is used first. The choice of route of
administration depends on severity of attack.
45. A
46. E

Copyright © 2000-2014 TIPS Inc. Unauthorized reproduction of this manual is strictly prohibited and it is illegal to reproduce
without permission. This manual is being used during review sessions conducted by PharmacyPrep. 74-10
PharmacyPrep.Com Smoking Cessation

PHARMACY PREP
SMOKING CESSATION
1-What is incorrect statement about smoking cessation?
A-Person using nicotine replacement therapy, should stop smoking same time as start NRT
B-Person using bupropion can set quit date in 1 to 2 week of after start of treatment
C-A person using nicotine patches can smoke but reduce number of cigarettes
D-A person with hypertension, body weight less than 45 Kg should start with low dose of NRT i.e. step 2
patch
E-Patches can be applied on while taking shower
2) Nicotine patches are stored in?
A-Refrigerator B-Room temperature C-Cold temperature D-Freezer E-Any temperature
3) What is incorrect about nicotine patches?
A-Stored in room temperature
B-Avoid sun light exposure
C-Hygroscopic, thus open from pack when you ready to use
D) Does not need to protect from light
E-Can be applied on arms and thighs
4) What is incorrect about bupropion?
A-Reduce the seizure threshold
B- Avoid in bulimia nervosa
C-Avoid in anorexia nervosa
D-Avoid in sexual dysfunction
E-It has low weight gain side effects
5-Which of the following is least likely caused by bupropion?
A-anorexia
B-weight gain and sexual dysfunction
C-↓ seizure threshold
D-hypertension
E-gingivitis
6) All of the following can be used in smoking cessation, EXCEPT:
A) Fluoxetine
B. bupropion
C. Nicotine transdermal patch
D. Nicotine inhaler
E. Varenicline

Copyright © 2000-2014 TIPS Inc. Unauthorized reproduction of this manual is prohibited. This manual is being used during
review sessions conducted by PharmacyPrep 75-1
PharmacyPrep.Com Smoking Cessation
ANSWERS:
1- C
2) B
3) D
4) D
Tips: bupropion has least sexual dysfunction among antidepressants.
5- B
6. A

Copyright © 2000-2014 TIPS Inc. Unauthorized reproduction of this manual is prohibited. This manual is being used during
review sessions conducted by PharmacyPrep 75-2
PharmacyPrep.Com Insomnia

PHARMACY PREP
INSOMNIA
1) What is incorrect recommendations for persons complaining insomnia?
A-make a regular sleep schedule for 7 days/wk
B-Over the counter diphenhydramine for 3 days may be helpful
C-It is essential to exercise before bedtime
D-Avoid heavy meals before bedtime
E-Avoid long day nap can give good sleep in night
2-What is incorrect about zopiclone?
A-A non benzodiazepine act on benzodiazepine receptor
B-Associated with sever hangover effect
C-It indicated for short time insomnia
D-It is short acting
E-It has low dependency
3-All of the following are long acting benzodiazepines except?
A-Diazepam
B-Clonazepam
C-Flurazepam
D-Temazepam
E-Chlorozepate
4) Zopiclone is?
A-Non-benzodiazepine act on BZ1 receptors
B-Benzodiazepine act on BZ1 receptors
C-Non-benzodiazepine act on BZ2 receptors
D-Non-benzodiazepine act on BZ1 and BZ2 receptors
E-Benzodiazepine act on BZ2 receptors
5) A 75 yo senior is receiving treatment of anxiety, paroxetine daily, clonazepam 5 mg bid, oxazepam 10 mg
tid. Which of the following is the important to monitor?
A) Insomnia
B) Anxiety
C) Disorientation and falls
D) Confusion
E) Depression

6) Which of the following drug has NOT taste change side effects?
A) zopiclone B) captopril C) metformin D) tetracycline E)metronidazole
7) Which of the following drug has faster rebound insomnia side effects?

Copyright © 2000-2014 TIPS Inc. Unauthorized reproduction of this manual is prohibited. This manual is being used during
review sessions conducted by PharmacyPrep 76-1
PharmacyPrep.Com Insomnia
A) Triazolam B) Diazepam C) Oxazepam D) lorazepam E) Alprazolam
8) What benzodiazepines are preferable to treat the type of insomnia that is associated with difficulty in
falling in sleep?
A) Short acting B) Intermediate acting C) Long acting D) intermediate and long acting E) short and long acting
9) MK is 50 year patient comes to pick up prescription of lorazepam 0.5 mg tab. Which of the following
statement is relevant?
A. Taking alcohol concomitant with lorazepam can cause severe sedation thereby avoid.
B) Take benzodiazepine with food
C) avoid taking with grapefruit juice
D) All of the above
ANSWERS:
1- C
Tips: avoid exercise at least 2 to 3 hours before bedtime
2- B
3- D
4) A
5) C
Tips: Lethargy = fatigue; Hangover = disorientation; Sedation = sleepy; Somnolence = sleepy
6) D
7) A
8. A
9) A

Copyright © 2000-2014 TIPS Inc. Unauthorized reproduction of this manual is prohibited. This manual is being used during
review sessions conducted by PharmacyPrep 76-2
PharmacyPrep.Com Eating disorders and weight loss treatments

PHARMACY PREP
EATING DISORDER AND WEIGHT LOSS TREATMENTS
1. What is incorrect about anorexia nervosa?
I-It is characterized by deliberate loss of weight
II-refusal to maintain normal body weight, fear of weight gain and amenorrhea
III-Binge eating is common in anorexia nervosa
A-I only B-III only C-I and II only D-II and III only E-All of the above
2-You notice Ms A’s, a young girl of normal weight, in the pharmacy purchasing laxatives. You take the time
to talk to her about her laxative use, and find that she has been binge eating and using laxatives about three
times per week for the last six months.
Based on this information, which of the following statements is most likely to be correct?
A. She may have bulimia nervosa of the non-purging type since she is not self-inducing vomiting
B. She may have bulimia nervosa of purging type since she uses laxatives
C. She does not have bulimia nervosa, since she is of normal weight
D. She does not have bulimia nervosa, since her bingeing episodes are not frequent enough

3-What is overweight body mass index (BMI)


A. 18.5 to 24.9
B. 25.0 to 29.9
C. >30
D. <30
E. All of the above

4-What is obese body mass index (BMI)


A. 18.5 to 24.9
B. 25.0 to 29.9
C. >30
D. <30
E. All of the above

5-All of the following drugs are used to treat weight loss except?
A-Meridia B) Orlistat C-Bupropion D-Topiramate E-Mirtazapine
6-Which of the following antidepressant should be avoided in bulimia an anorexia nervosa?
A-Venlafaxine B-Fluoxetine C-Bupropion D-Mirtazapine
E-Paroxetine
7) What is body mass index (BMI)
2
A) Weight in kg/height in meter

Copyright © 2000-2014 TIPS Inc. Unauthorized reproduction of this manual is prohibited. This manual is
being used during review sessions conducted by PharmacyPrep 77-1
PharmacyPrep.Com Eating disorders and weight loss treatments
B)Weight in kg/height in meters
2
C) height in meter / weight in kg
D)Weight in grams/ height in m
E) none of the above
8-What is normal body mass index (BMI)
A- 18.5 to 24.9
B- 25.0 to 29.9
C- >30
D- <30
E- All of the above

EATING DISORDERS
CORRECT ANSWERS
1. B
2- B
3- B
2 2
Tips: BMI = weight/height = kg/m
4- C
2
Tips: = kg/m
5- E
6- C
7) A
2 2
Tips: BMI = weight/height = kg/m
8- A

Copyright © 2000-2014 TIPS Inc. Unauthorized reproduction of this manual is prohibited. This manual is being used during
review sessions conducted by PharmacyPrep 77-2
Pharmacyprep.com

PHARMACY PREP
GASTROINTESTINAL DISORDERS
1) Correct statements regarding the characteristics of hernias may include:
I-It is a protrusion of a loop of an organ or tissue normally through the GI area
II-Hernia hiatus is a herniation of stomach through the esophageal hiatus of diaphragm
III-Strangulated hernia is an incarcerated hernia that is so tightly constricted as to compromise the blood
supply of the contents of the hernia sac.
a) I only
b) III only
c) I and II only
d) II and III only
e) All are correct
2- Misoprostol is used as mucosal protectants of GI and may be classified as:
a) PGE1 analog
b) H2 inhibitor
c) Proton pump inhibitor
d) NSAID
e) Antimuscarinic
3- Drugs are mainly metabolized in the GIT by many different reactions. The most common drug reaction in
the GIT may include:
a) Oxidation
b) Reduction
c) Hydrolysis
d) Dealkylation
e) Ionization
4- ASA-Acetyl salicylic acid is one of the most used worldwide analgesics. Which of the following reactions
metabolizes it in the gastrointestinal tract and liver?
a) Oxidation
b) Hydrolysis
c) Reduction
d) Dealkylation
e) Deamination
5- Accumulation of fluids in the peritoneal cavity is normally known as:
a) Pannus
b) Starvation
c) Edema
d) Ascitis
e) Cholecystitis

Copyright © 2000-2014 TIPS Inc. Unauthorized reproduction of this manual is prohibited. This manual is being used during
review sessions conducted by PharmacyPrep. 78-1
Pharmacyprep.com
6- Examples of the used of Karaya gum in ostomy patients may include:
a) Decrease odor from colostomy
b) Act as germicidal agent
c) Liquefy the stomal effluent
d) Protect skin from stomal effluent
e) Soften the stomal effluent
7- All of the following are examples of gastric acid stimulators, EXCEPT:
a) Gastrin
b) Acetylcholine
c) Somatotropin
d) G-Cell
e) None of the above
8- Gastroesophageal reflux (GERD) is mainly characterized by:
a) Diaphragm wall inflammation
b) Diaphragm perforation
c) Reflux of the stomach content into the esophagus
d) Hyperacidity of stomach
e) All are correct
9) One of the most common conditions associated with acute hemorrhagic gastritis may include:
a) Chronic alcoholism
b) H. pyloric bacterial infection
c) Carcinoma of stomach
d) Carcinoma of colon
e) Traveler’s diarrhea
10) Antacids are normally used to equilibrate the stomach pH and protect the stomach from the effect of
acids. Which of the following antacids has a cathartic side effect?
a) Calcium carbonate
b) Dihydroxyaluminum sodium carbonate
c) Magnesium hydroxide
d) Calcium sulfate
e) Aluminum silicate
11- Elongated proliferations in the small intestine are called:
I- Villi
II- Fontanelle
III- Hernias
a) I only b) III only c) I and II only d) II and III only e) All are correct

Copyright © 2000-2014 TIPS Inc. Unauthorized reproduction of this manual is prohibited. This manual is being used during
review sessions conducted by PharmacyPrep. 78-2
Pharmacyprep.com
12- Metoclopramide is a parasympathetic agent that may have which of the following characteristics?
I- Cholinomimetic agent that stimulates motility of the stomach
II- Used in treatment of diabetes gastro paralysis and GERD
III- Used centrally as antiemetic during cancer chemotherapy
a) I only
b) III only
c) I and II only
d) II and III only
e) All are correct
13- All of the following are correct statements regarding bile, EXCEPT:
a) It is secreted by the liver
b) It is poured into the small intestine via the bile ducts
c) It is constituted of bile acids, cholesterol, phospholipid, bilirrubin and electrolytes
d) Bile is acid due to its bicarbonate content
e) It is also responsible for the alkalinization of intestinal content
14) Proton Pump Inhibitors are irreversible direct inhibitors of K pump. Correct statements regarding these
drugs may include:
I- Used in gastric and duodenal ulcers
II- Omeprazole and lansoprazole are examples of proton pump inhibitors
III- They decrease stomach acid secretion
a) I only
b) III only
c) I and II only
d) II and III only
e) All are correct
15- Zollinger-Ellison disease is a grave stomach disease. Drug of choice in the treatment of Zollinger-Ellison
diseases may include:
a) Ranitidine
b) Famotidine
c) Loratidine
d) Omeprazol
e) Antacids
16- Correct statements regarding the administration of antacids may include:
I- Taken 1hr before meals
II- Taken 3hrs after meals
III- Taken 2hrs before bedtime
a) I only b) III only c) I and II only d) II and III only e) All are correct

Copyright © 2000-2014 TIPS Inc. Unauthorized reproduction of this manual is prohibited. This manual is being used during
review sessions conducted by PharmacyPrep. 78-3
Pharmacyprep.com
17- All of the following enzymes are found in saliva, EXCEPT:
a) Amylase
b) Ptyalin
c) Gastrin
d) Lysoenzyme
e) All are correct
18- Which of the following is/are examples of salivary glands?
I- Parotid
II- Submandibular
III- Sublingual
a) I only
b) III only
c) I and II only
d) II and III only
e) All are correct
19- The digestive process is done in many different steps. The first step of digestion may be taken in:
a) Mouth
b) Esophagus
c) Stomach
d) Small intestine
e) Large intestine
20- Wrong statement regarding stomach secretions may include:
a) Mucus is present in stomach as lubricant and protect surface from acid
b) Acids breakdown food, kill bacterias and convert pepsin to pepsinogen
c) Pepsinogen is also called “chief cell”
d) Gastrin is the one that inhibit acid secretion
e) The intrinsic factor in stomach is secreted from parietal cell
21- The small intestine is classified in three parts, the anatomical order division of small intestine is known
as:
a) Jejunum, duodenum and ileum
b) Ileum, jejunum and duodenum
c) Duodenun, jejunun and ileum
d) Duodenun, ileum and jejunun
e) Jejunun, ileum and duodenun
22- Conditions that may be the main cause of duodenal ulcers include:
a) High motility of stomach
b) Bacterial migration
c) Squirting of acid stomach content into the duodenal wall

Copyright © 2000-2014 TIPS Inc. Unauthorized reproduction of this manual is prohibited. This manual is being used during
review sessions conducted by PharmacyPrep. 78-4
Pharmacyprep.com
d) Reflux of stomach content into the esophagus
e) Hyperacidity of stomach
23- The pancreas is constituted of endocrine and exocrine glands. What is the function of endocrine gland of
pancreas?
a) Production of enzymes
b) Production of insulin
c) Production of proteins
d) Production of glycogen
e) Production of bile
24- Colon is the last part of the digestive system and may be anatomical divided in:
a) Ascending colon
b) Transverse colon
c) Descending colon
d) Sigmoid colon and rectum
e) All are correct
25- H2 is involved in gastric acid and pepsin secretion. All of the following are examples of H2 inhibitors,
EXCEPT:
a) Ranitidine
b) Nizatidine
c) Cimetidine
d) Omeprazole
e) Famotidine
26- H2 inhibitors are used in the treatment of gastric hyper secretion and GI ulcers. One of the most potent H2
inhibitors may include:
a) Famotidine
b) Cimetidine
c) Ranitidine
d) Nizatidine
e) Promethazine
27- Example(s) of conditions that may be treated with H2 inhibitors may include:
a) Gastric hyper secretion
b) Duodenal ulcers
c) Zollinger-Ellison syndrome
d) Pain associated with GERD
e) All are correct
28- Proton pump inhibitors used in gastrointestinal complications are irreversible inhibitors of:
+
a) Na pump

Copyright © 2000-2014 TIPS Inc. Unauthorized reproduction of this manual is prohibited. This manual is being used during
review sessions conducted by PharmacyPrep. 78-5
Pharmacyprep.com
+ +
b) Na /K pump
c) K+ pump
-
d) Cl pump
2+ +
e) Mg /K pump
29- The most common side effect during treatment with proton pump inhibitors may include:
I- Headache
II- Diarrhea
III- GIT pain
a) I only
b) III only
c) I and II only
d) II and III only
e) All are correct
30- Ulcers are complications of gastrointestinal tract. The risk of having ulcers is bigger in people having
which of the following conditions:
a) Smokers
b) Alcoholics
c) NSAID long-term treatment
d) Corticosteroids long-term treatment
e) All are correct
31- Examples of effective treatment of ulcers may include:
I- Antacids
II- H2 inibitors
III- Proton pump inhibitors
a) I only
b) III only
c) I and II only
d) II and III only
e) All are correct
32- Sucralfate is used in gastrointestinal complications and may act as:
a) Antacid
b) H2 inibitor
c) Mucosal protectant
d) Anti-secretary prostaglandin analog
e) Proton pump inhibitor
33- Antacids are commonly used to neutralize gastric acid pH. Example of best antacids constitution may
include which of the following combinations?
a) Magnesium hydroxide only

Copyright © 2000-2014 TIPS Inc. Unauthorized reproduction of this manual is prohibited. This manual is being used during
review sessions conducted by PharmacyPrep. 78-6
Pharmacyprep.com
b) Aluminum hydroxide only
c) Calcium carbonate only
d) Magnesium and aluminum hydroxide
e) Calcium and aluminum hydroxide
34- Antacids neutralize the gastrointestinal acid pH by which of the following mechanism of action?
a) Reduce the concentration and total load of acid in the gastric content by increasing gastric pH.
+ +
b) Inhibit (H , K )-ATPase enzyme system at the secretory surface of the gastric parietal cell
c) Inhibit H2 neurotransmitters
d) Inhibit gastrin secretion
e) Inhibit the conversion of pepsin to pepsinogen
35- Diarrhea is a common side affect seen with which of the following antacids?
a) Aluminun containing antacid
b) Magnesium containing antacid
c) Calcium containing antacid
d) Magnesium and aluminum containing antacid
e) Calcium and aluminum containing antacid
36- Constipation and/or diarrhea may be side effects of antacids therapy. Which of the following antacids
may have constipation as side effect?
a) Aluminun containing antacid
b) Magnesium containing antacid
c) Calcium and magnesium containing antacid
d) Magnesium and aluminum containing antacid
e) Calcium and aluminum containing antacid
37- Correct statements regarding the advantages of taken antacids mixtures of aluminum and magnesium
may include:
I- Provides more sustained action
II- Permit a lower dose of each compound
III- The constipation effect of aluminum may counter the diarrhea caused by magnesium
a) I only
b) III only
c) I and II only
d) II and III only
e) All are correct
38- Laxative is normally contra-indicated during pregnancy. The only agent with laxative effects that may be
used during pregnancy include:
a) Senna

Copyright © 2000-2014 TIPS Inc. Unauthorized reproduction of this manual is prohibited. This manual is being used during
review sessions conducted by PharmacyPrep. 78-7
Pharmacyprep.com
b) Magnesium hydroxide
c) Psyllium
d) Docusate sodium
e) Bisacodyl
39- Examples of stimulant laxative may include all of the following, EXCEPT:
a) Senna
b) Cascara
c) Bisacodyl
d) Lactulose
e) Castor oil
40- Common or regular diarrhea may best be treated by the administration of:
I- Loperamide
II- Aluminum hydroxide
III- Bismuth subsalicylate
a) I only
b) III only
c) I and II only
d) II and III only
e) All are correct
41- Antidiarrheal agent that appears to be bactericidal to H. pylori being useful in the treatment of duodenal
ulcers and gastric ulcers associated with H. pylori:
a) Bismuth subsalicylate
b) Loperamide
c) Docusate sodium
d) Psyllium
e) Milk of magnesium
42- Drug(s) used for the short-term treatment of gastro esophageal reflux and peptic ulcer diseases may
include which of the following?
I- H2 receptors antagonists
II- Proton pump inhibitors
III- Corticosteroids
a) I only
b) III only
c) I and II only
d) II and III only
e) All are correct
43- One of the most frequent side effect seen with SULCRAFATE therapy may include:
a) Diarrhea
b) Constipation

Copyright © 2000-2014 TIPS Inc. Unauthorized reproduction of this manual is prohibited. This manual is being used during
review sessions conducted by PharmacyPrep. 78-8
Pharmacyprep.com
c) Edema
d) Cramps
e) Vomiting
44- Metoclopramide can act as effective agent in the management of many different conditions. Some
characteristics of metoclopramide include:
I- It increases the rate of gastric emptying
II- It has both peripheral and central effect
III- It can induce Parkinson’s disease
a) I only
b) III only
c) I and II only
d) II and III only
e) All are correct
45- Drug of choice in treating NSAIDs-induced ulceration may include:
a) Ranitidine
b) Omeprazole
c) Misoprostol
d) Sulcrafate
e) Aluminium hydroxide
46- A 5-Aminosalicylate active metabolite drug used in the treatment of Crohn’s disease:
a) Lansoprazole
b) Famotidine
c) Misoprostol
d) Sulfasalazine
e) Omeprazole
47- Active metabolite of sulphasalazine may include:
a) Acid salicylic acid
b) Salicilyc acid
c) Salicylates
d) Aminosalicylate
e) 5-Aminosalicylate
48- Sulfasalazine, a 5-Aminosalicylate active metabolite drug suffer its metabolism mainly where?
a) Colon
b) Stomach
c) Small intestine
d) Duodenum
e) Esophagus

Copyright © 2000-2014 TIPS Inc. Unauthorized reproduction of this manual is prohibited. This manual is being used during
review sessions conducted by PharmacyPrep. 78-9
Pharmacyprep.com
49) Lipase inhibitor used to treat obesity may include:
a) Diethylpropiom
b) Orlistat
c) Benzphetamine
d) Mazindol
e) Phenylpropanolamine
50- Which of the following drugs are available in a transdermal form for the prevention of motion sickness?
a) Metoclopramide
b) Granisetron
c) Ondasetron
d) Scopolamine
e) Diphenhydramine
51- Simvastatin used in hypercholesteramia is characterized by which of the following mechanism of action?
a) Inhibiting xanthine oxidase
b) Inhibiting HMG-CoA reductase
c) Inhibit acetylcholinesterase
d) Acting as a bile sequestrant
e) Interfering with fat absorption from GI
52- Lactulose is an osmotic laxative agent that may be used to treat:
I- Constipation
II- Portal-systemic encephalopathy
III- Renal tubular necrosis
a) I only
b) III only
c) I and II only
d) II and III only
e) All are correct
53- The primary function of simethicone used in antacid products is to act as:
a) Suspending agent
b) Adsorbent
c) Buffer
d) Anti-flatulent
e) Flavoring agent
54) Chronic alcoholism and smoking is risk factors of?
I) GERD II) Gastritis III) Crohn's disease
A) I only B) III only C) I and II D) II and III E) All

Copyright © 2000-2014 TIPS Inc. Unauthorized reproduction of this manual is prohibited. This manual is being used during
review sessions conducted by PharmacyPrep. 78-10
Pharmacyprep.com
56) What is correct about sucralfate?
A) taken after meals B) taken empty stomach C) Taken with antacids is more effective D) It prevents heart
burn E) It is taken with proton pump inhibitors
57) Which of the following is NOT used for lifelong therapy for ulcerative colitis?
A) Methotrexate B) azathioprine C) NSAIDS
58) A 34 yo male diagnosed with chronic gastritis and peptic ulcer disease. What organism are possible
cause?
A) E. coli
B) Helicobacter pilori
C) Giardia lamblia
D) Compylobacter jejuni
59) What drugs when administered with antacids, decrease the therapeutic efficacy?
I) Proton pump inhibitors (PPIs)
II) H2RA
III) Sucralfate
A) I only B) III only C) I and II D) II and III E) I, II, III
60) A customer walks into your pharmacy and wants something for fast relief of heart burn symptoms?
A) ranitidine 150 mg
B) cimetidine
C) Mg antacids
D) Omeprazole
E) sucralfate

Copyright © 2000-2014 TIPS Inc. Unauthorized reproduction of this manual is prohibited. This manual is being used during
review sessions conducted by PharmacyPrep. 78-11
Pharmacyprep.com
GASTROINTESTINAL DISORDERS
CORRECT ANSWERS
1- E
Comments: Hernias is a protrusion of a loop of an organ or tissue normally through the GI area. Hernia hiatus
is a herniation of stomach through the esophageal hiatus of diaphragm. Strangulated hernia is an
incarcerated hernia that is so tightly constricted as to compromise the blood supply of the contents of the
hernia sac.
2- A
Comments: Misoprostol is a prostaglandin analogue that increases bicarbonate and mucin release and
reduces acid secretion.
3- C
Comments: The most common drug reaction in the GIT is hydrolysis because most of the drugs have an ester
and/or amide group attached to their molecular formula and/or react with water contests in GIT.
4- B
Comments: Salicylate compounds are largely hydrolyzed in the GIT, liver and blood to salicylate, which is
further metabolized primarily in the liver.
5- D
Comments: Ascitis is an effusion and accumulation of serous fluid in the abdominal cavity; called also
abdominal or peritoneal dropsy.
6- D
Comments: Karaya gum is used as skin adhesives and protective skin barriers in the fitting and care of
ostomy, colostomy appliances and in other conditions involving an artificial stoma
7- C
Comments: Somatotropin is a hormone growth with no effect on gastric acid secretion.
8- C
Comments: Gastro-esophageal Reflux disease is a chronic, pathologic, potentially life-threatening disease
manifested by reflux of the stomach and duodenal contents into the esophagus, which is principally
characterized by heartburn and regurgitation
9- A
Comments: Acute hemorrhagic gastritis is an erosive inflammation of the stomach with bleeding normally
seen with chronic alcoholism.
10- C

Copyright © 2000-2014 TIPS Inc. Unauthorized reproduction of this manual is prohibited. This manual is being used during
review sessions conducted by PharmacyPrep. 78-12
Pharmacyprep.com
Comments: Magnesium salts used in antacids cause diarrhea. It also may cause systemic effect as
hypermagnesemia.
11- A
Comments: Villi and microvilli are found in the small intestine and are formed of capillaries that works in the
absorption of digestive food to rich the venous and capillaries drainage of the gut.
12- E
Comments: Metoclopramide can be classified as dopaminergic blocking agent, gastrointestinal emptying
adjunct, peristaltic stimulant and antiemetic agent. It can be used in cancer chemotherapy avoiding nausea
and vomiting as well in diabetes gastroparesis.
13- D
Comments: Bile is a fluid secreted from the liver and poured into the small intestine via the bile ducts. It
constituted of conjugated bile acids, cholesterol, phospholipid, bilirubin diglucuronide and electrolytes. Bile
is also responsible for the alkalinization in the intestinal content due to its bicarbonate content.
14- E
Comments: Proton-pump inhibitors are the ultimate mediator of gastric acid secretion inhibition; they are
specific inhibitors of H+, K+ ATPase enzyme system at the secretory surface of the gastric parietal cells. The
most widely used proton-pump inhibitors are omeprazole and lansoprazole.
15- D
Comments: Omeprazole is a proton-pump inhibitor is the drug of choice in treating Zollinger-Ellison disease
and reflux disease.
16- E
Comments: Antacids should be taken 1 hour before meals, 3 hours after meals and at bedtime
17- C
Comments: Gastrin, also known as G-cell is present in the stomach and stimulates acid secretion.
18- E
Comments: Salivary glands are composed of 3 pairs of glands: Parotid, submandibular and sublingual gland.
19- A

Copyright © 2000-2014 TIPS Inc. Unauthorized reproduction of this manual is prohibited. This manual is being used during
review sessions conducted by PharmacyPrep. 78-13
Pharmacyprep.com
Comments: The first step of digestion is done in the mouth by the salivary enzymes, amylase, ptyalin and
lysozyme.
20- D
Comments: Gastrin is responsible for the stimulation of gastric acid secretion
21- C
Comments: The small intestine is anatomically divided in Duodenum, Jejunum and ileum.
22- C
Comments: Duodenal ulcers also known as peptic ulcers are mainly characterized by squirting of acid
stomach into the duodenal wall.
23- B
Comments: The pancreas has two different glands. 1-Exocrine, responsible for the production of enzymes
which go to the duodenum via the pancreatic duct and 2- Endocrine gland, responsible for the production of
insulin, the blood sugar regulator.
24- E
Comments: The colon is anatomically divided into: Ascending colon, transverse colon, descending colon,
sigmoid colon and rectum.
25- D
Comments: Omeprazole is a proton-pump inhibitor.
26- A
Comments: Famotidine is the most potent H2 blocker. It reduces the gastric acid in 94% up to 10 hours.
27- E
Comments: H2 blockers can be used to treat gastric hypersecretion, duodenal ulcers, Zollinger-Ellison disease
and reduce pain associated with gastroesophageal reflux disease.
28- C
Comments: Proton-pump inhibitors are irreversible direct inhibitor of potassium pump.
29- E
Comments: The most frequent side effect seen in the treatment with proton-pump inhibitors are: Abdominal
pain or colic, diarrhea or constipation, unusual tiredness, muscle pain, headaches, dizziness and skin rashes

Copyright © 2000-2014 TIPS Inc. Unauthorized reproduction of this manual is prohibited. This manual is being used during
review sessions conducted by PharmacyPrep. 78-14
Pharmacyprep.com
30- E
Comments: Long-term treatment with NSAIDs and corticosteroids definitely increase the risk of ulcers
disease because these drugs cause many different GI side effects as pain, cramps, acidity of stomach and
other GI motility effects. Ulcers are also increased in alcoholism and in smokers due to irritation and
destruction of gastric mucosa. Other agents that may increase the incidence of ulcers are coffee, orange
juice, fatty and spice food and emotional stress.
31- E
Comments: The basic pharmacological choices in the treatment of ulcers include H2-antagonists, proton-
pump inhibitors, misoprostol and sucralfate. Ulcers associated with H. pylori can be treated by triple therapy
with omeprazole, clarithromycin and amoxicillin.
32- C
Comments: Sucralfate acts as protective coating of the gastric mucosa, it is particularly used in ulcerated
areas.
33- D
Comments: The mixtures of aluminum and magnesium are the most widely used combination of antacids,
because they compensate the side effects of each other.
34- A
Comments: Antacids act neutralizing gastric acid, which increases the pH of refluxed gastric contents and
lower esophageal sphincter pressure.
35- B
Comments: Diarrhea is the most common side effect seen with the administration of magnesium hydroxide
antacid. Aluminum and calcium carbonate cause constipation.
36- A
Comments: Constipation is the most common side effect seen with the administration of aluminum
containing antacid. Magnesium is an antacid with cathartic side effect.
37- E
Comments: Magnesium and aluminum mixtures are commonly used together because they provide more
even sustained action than used as single agent and permit lower dose of each compound. The constipation
effect of aluminum may counter the diarrhea caused by magnesium.
38- C
Comments: Therapeutically safe in pregnancy, psyllium is a bulk-forming agent used as laxative, it contains
plant matter that absorbs water and softens the stool.

Copyright © 2000-2014 TIPS Inc. Unauthorized reproduction of this manual is prohibited. This manual is being used during
review sessions conducted by PharmacyPrep. 78-15
Pharmacyprep.com
39- D
Comments: Lactulose is an osmotic laxative.
40- E
Comments: Diarrhea caused by infections, toxins and drugs are mostly treated with specific agents, however
the regular diarrhea normally caused for stress or stomach indisposition is best treated with loperamide,
bismuth subsalicylate or aluminum hydroxide.
41- A
Comments: Duodenal and gastric ulcers are often caused by H. pylori. The treatment objective is eradication
of the bacterium with combination of antibiotics and h2-antagonists, however bismuth subsalicylate is also
used because it appears to be bactericidal to H. pylori in high doses concentration.
42- C
Comments: Corticosteroids are neither used in the treatment of GERD nor in the treatment of peptic ulcers.
They normally induce or make worsen the ulcer condition.
43- B
Comments: Sucralfate is used as protective coating of the gastro intestinal mucosa and have constipation as
its main side effect.
44- E
Comments: Metoclopramide can be used as gastrointestinal emptying adjunct, peristaltic stimulant,
antiemetic agent and dopaminergic blocking agent, which may induce Parkinson’s disease due to the
blockage of dopamine.
45- C
Comments: Misoprostol is a prostaglandin analogue that increases bicarbonate and mucin release and
reduces acid secretion. It is used to treat NSAID-induced ulceration.
46- D
Comments: Sulfasalazine is an 5-Aminosalicylate (5-ASA) active metabolite agent used as the drug of choice
in the treatment of crohn’s disease.
47- E
Comments: The pharmacologic action of sulfasalazine is seen through its active metabolite 5-Aminosalicylate
(5-ASA).
48- A

Copyright © 2000-2014 TIPS Inc. Unauthorized reproduction of this manual is prohibited. This manual is being used during
review sessions conducted by PharmacyPrep. 78-16
Pharmacyprep.com
Comments: Sulfasalazine is metabolized in the colon by resident bacteria into 5-ASA and sulfa pyridine.
Sulfapyridine is absorbed while the 5-ASA remains in the colon.
49- B
Comments: Orlistat is a lipase inhibitor being used to treat obesity, it binds to pancreatic and gastric lipase
and inactivate the enzyme reducing the dietary absorption of fat by about 30%.
50- D
Comments: Scopolamine is the only available transdermal drug used to treat motion sickness.
51- B
Comments: Simvastatin is a statin (HMG-CoA reductase inhibitor) used as antihyperlipidemic agent.
52- C
Comments: Lactulose is an osmotic laxative that can be used to treat constipation and portal systemic
encephalopathy as well.
53- D
Comments: Simethicone is mainly used in antacids preparations as antiflatulant agent (anti-gas).
54) E
56) B
Tips: Sucralfate is taken empty stomach to have better action. Because it make barrier on to ulcer and
prevent acid contact. Avoid taking with food, antacids, H2RA, and PPIs.
57) B
58) B
59) E
60) E

Copyright © 2000-2014 TIPS Inc. Unauthorized reproduction of this manual is prohibited. This manual is being used during
review sessions conducted by PharmacyPrep. 78-17
Pharmacyprep.com
BIBLIOGRAPHIC REFERENCE
1- PHARMACY PREP – Evaluating Exam Review Book 2014
2- CPS-COMPENDIUM OF PHARMACEUTICALS AND SPECIALITIES - Canadian Pharmacist Association – 2014
edition.
th
3- MEDICAL DICTIONARY – Dorland’s illustrated – 27 edition.
4- COMPENDIUM OF THERAPUTIC CHOICES – Canadian Pharmacist Association SEVENTH edition
th
6- USP DI – Drug Information for the Health Care Professional–15 edition – Volume I.

Copyright © 2000-2014 TIPS Inc. Unauthorized reproduction of this manual is prohibited. This manual is being used during
review sessions conducted by PharmacyPrep. 78-18
Pharmacyprep.com Diabetes Mellitus

PHARMACY PREP
DIABETES
1)What is incorrect about insulin's?
A-insulin lispro is rapid acting
B-regular insulin's is available as iv and sc
C-NPH is intermediate acting insulin is available as iv and sc
D-Glargine is long acting insulin should not physically mix with other insulin
E-All insulin preps should be stored in refrigerator
2. What is/are NOT symptoms of hypoglycemia
A-sweating B-hunger C-polyuria D-fainting E-fatigue
3-Hypoglycemia symptoms, except?
A-palpitation
B-tremor or shaking
C-fails to think
D-sweating
E-polyphagea
4-Which of the following drug associated hypoglycemia treated with glucose rather than sucrose:
I-Metformin
II-Insulin
III-Acarbose
A-I only
B-III only
C-I and II only
D-II and III only
E-All of the above
5-A 17-year-old type 1 diabetic patient is currently on diabetic medications. All of the following antidiabetic
drugs are not used in type I diabetes EXCEPT:
A-Chlorpropamide
B-Glyburide
C-Repaglinide
D-Nateglinide
E-Pioglitazone
6-Which of the following antidiabetic drug is gender selective:
A-Metformin
B-Chlorpropamide
C-Glyburide
D-Rosiglitazone E-All of the above

Copyright © 2000-2014 TIPS Inc. Unauthorized reproduction of this manual is prohibited. This manual is being used during
review sessions conducted by PharmacyPrep. 79-1
Pharmacyprep.com Diabetes Mellitus
7) Diabetic patient with hypercholesterolemia has high cholesterol, high LDL, normal TG. What would be the
drug of choice?
a) Lovastatin
b) Colestipol
c) Gemfibrozil
d) Niacin
e) Metformin

8)Which of the following antidiabetic drug effect on lipid levels (increase HDL, LDL and decrease TG).
A-Metformin B-chlorpropamide C-Pioglitazone D-Repaglinide
E-Insulin
9) Which of the following is the intestinal lipase inhibitor:
A-Acarbose B-Metformin C-Pioglitazone D-Orlistat E-Repaglinide
10-Chlorpropamide is not used with ethanol because
A-It can induce hypoglycemia
B-Hypoglycemia occur
C-Alcohol intolerance occur
D-Disulfiram like reaction
E-All of the above
11-Which antidiabetic agent decrease mortality rate?
A-Metformin b-Chlorpropamide C-Acarbose D-Rosiglitazone
E-Metiglinides
12-What is incorrect statement about metformin?
A) May cause lactic acidosis a rare side effect
B-Decrease gluconeogenesis
C-Increase glucose uptake
D-Avoid in sulfa allergy
E-May cause anorexia
13-What is correct about sulfonylureas?
A-All sulfonylureas have sulfa allergy
nd
B-Gliclazide is long acting 2 generation drug used single daily dose
nd
C-Glyburide is long acting 2 generation drug used single daily dose
st
D-Chlorpropamide is 1 generation drug has short half life.
E-A and B are correct statements
14-Which of the following antidiabetic drug may be taken regardless of meals:
A-Metformin
B-Chlorpropamide

Copyright © 2000-2014 TIPS Inc. Unauthorized reproduction of this manual is prohibited. This manual is being used during
review sessions conducted by PharmacyPrep. 79-2
Pharmacyprep.com Diabetes Mellitus
C-Glyburide
D-Rosiglitazone
E-Acarbose
15) An obese patient with type II diabetic, a drug of choice is?
A-Metformin B-Chlorpropamide C-Glyburide D-Insulin
16) A doctor prescribed metformin 1000 mg tid as initial treatment, what is anticipated most common side
effect?
A-Diarrhea B-lactic acidosis C-liver failure D-Congestive heart failure E-Renal failure
17) The risk of lactic acidosis, associated with metformin, is enhanced by?
I-Congestive heart failure II-Renal disease
III-Alcohol intake
A-I only B-III only
C-I and II only
D-II and III only
E-I, II, III
18) A obese patient has CHF, renal disease and drinks alcohol. Recently diagnosed with type II DM, what is
drug of choice?
A-Metformin
B-Chlorpropamide
C-Rosiglitasone
D-Acarbose
E-Glyburide
19) HbA1c is test shows blood sugar levels for?
A-the past 3 months
B- the past 6 months
C- the past 3 years
D- the past 3 days
E- the past 3 weeks
20) Sitagliptin (Januvia) act by?
A) Inhibiting DPP4 enzyme
B) Inhibiting insulin
C) Inhibiting glucagon
D) Increasing incretin levels
E) Analog of incretin

Copyright © 2000-2014 TIPS Inc. Unauthorized reproduction of this manual is prohibited. This manual is being used during
review sessions conducted by PharmacyPrep. 79-3
Pharmacyprep.com Diabetes Mellitus
21) What antidiabetic drugs should skip if you skip meals?
A-Metformin B-Chlorpropamide C-Rosiglitazone D-Acarbose
E-Glyburide
22) Oral antidiabetic drugs that does not give hypoglycemia's?
A-chlorpropamide
B-Acarbose
C-insulin
D-rosiglitazone
23) A type II dm patient using Acarbose, experiencing hypoglycemia should be treated with?
A-a candy
B-sucrose
C-juice with sugar
D-Glucose
24) In which of the following cases insulin should not be started?
A- high potassium levels
B- low potassium levels
C- high sodium levels
D- low sodium levels

E-urine ketone <160 mg/dL


25) A patient with hyperglycemic in the evening and hypoglycemic in the morning and currently on regular
insulin and NPH insulin, he should receive:
I) Increase NPH insulin in the morning
II) Decrease NPH insulin in the evening
III) Increase regular insulin levels in the morning
A. I only B. III only C. I and II only D. II and III only E) I, II, III
26) A patient is in emergence with diabetic ketoacidosis. Which of the following insulin is used to treat
diabetic ketoacidosis (DKA) ?
A) Rapid lispro
B) Regular
C) NPH
D) Glargine
E) Detemir

Copyright © 2000-2014 TIPS Inc. Unauthorized reproduction of this manual is prohibited. This manual is being used during
review sessions conducted by PharmacyPrep. 79-4
Pharmacyprep.com Diabetes Mellitus
27. Diabetic patient have increased risk following infections, except?
A) skin and foot infections
B) Cellulitis
C) Gangrene
D) Urinary tract infection
E) Pneumonia
28. A type I DM patient on insulin regimen. Mistakenly take double the dose of insulin at bed time. What
symptoms are/is NOT expected?
A) confusion B) sweating C) heart burn D) palpitation E) Tremors
29. What ketones are formed in blood sample of diabetic ketoacidosis patient?
I) beta hydroxybutyric acid
II) acetone
III) glucose
A) I only B) III only C) I and II D) II and III E) I, II, III

Copyright © 2000-2014 TIPS Inc. Unauthorized reproduction of this manual is prohibited. This manual is being used during
review sessions conducted by PharmacyPrep. 79-5
Pharmacyprep.com Diabetes Mellitus
ANSWERS:
1) C
2. C
Tips; sweating, hunger, dizziness, fainting, fatigue are symptoms of hypoglycemia.
Polyurea, polydipsea, polyphagea, neuropathic pains, and blurred vision are symptoms hyperglycemia.
3- E
4- B
Tips: Acarbose impair the digestion of sucrose. Hypoglycemia in-patient taking acarbose should be treated
with glucose rather than sucrose.
5- E
6- D
Tips: increase risk of pregnancy if inadequate contraception is used. Resume ovulation in previously
anovulatory women
7) A
8) C
9) D
Tips: Orlistat (Xenical) act as intestinal lipase inhibitor and prevents absorption of fat soluble vitamins ADEK.
10- D
11- A
12- D
Tips: Metformin is a biguanide, it does not have sulfa group, and thus it can be used in sulfa allergy patients.
13- E
14- D
Tips: Rosiglitazone can be taken with or without meals, where as metformin. Chlorpropamide, glyburide
taken with food, and acarbose is taken with first bite of meals.

Copyright © 2000-2014 TIPS Inc. Unauthorized reproduction of this manual is prohibited. This manual is being used during
review sessions conducted by PharmacyPrep. 79-6
Pharmacyprep.com Diabetes Mellitus
15) A
16) A
17) E
18) D
Metformin is CI in renal, hepatic, diseases. Chlorpropamide dose adjustment required in renal impairment
rosiglitazone contraindicated in congestive heart failure CHF.
19) A
20) A
21) D
22) B
23) D
24) B
25) C
26) B
27. E
28. C
29. D

Copyright © 2000-2014 TIPS Inc. Unauthorized reproduction of this manual is prohibited. This manual is being used during
review sessions conducted by PharmacyPrep. 79-7
Pharmacyprep.com Diabetes Mellitus
Copyright © 2000-2014 TIPS Inc. Unauthorized reproduction of this manual is prohibited. This manual is being used during
review sessions conducted by PharmacyPrep. 79-8
Pharmacyprep.com

PHARMACY PREP
THYROID DISORDERS
1-Select incorrect combinations in the following pairs:
A-Antithyroid drugsthionamides (PTU, methimazole)
B-Antithyroid drugsLugol's solution
C-Lugol’s solution5%I +10% KI
D-HypothyroidismThyroxin (T4) and liothyrosine (T3)
E-HypothyroidismGraves disease
F-HypothyroidismHoshimoto thyroiditis
2-All of the following are the symptoms of hypothyroidism EXCEPT:
A-Weight gain B-Fatigue C-Impaired memory D-Constipation
E-Heat intolerance
3-Conversion of T4 to T3 is catalyzed by:
A-Tetraiodinase enzyme
B-Deiodinase enzyme
C-Triiodinase enzyme
D-Hydrogenase enzyme
E-Iodinase enzyme
4) Which of the following laboratory monitoring indicated hypothyroidism:
I) Elevated TSH levels
II) No change in TSH levels
III) Decreased TSH levels
a) I only
b) III only
c) I and II only
d) II and III only
e) I, II, III
5-All of the following are the symptoms of hyperthyroidism, EXCEPT:
A-weight gain B-heat intolerance C-palpitation D-diarrhea
E-anxiety
6-Which of the following condition is hyperthyroidism
I-Hoshimoto thyroiditis
II-Myxedema
III-Graves disease
a) I only b) III only c) I and II only d) II and III only
e) I, II, and III

Copyright © 2000-2014 TIPS Inc. Unauthorized reproduction of this manual is prohibited. This manual is being used during
review sessions conducted by PharmacyPrep. 80-1
Pharmacyprep.com
7-Lugol solution contain:
A-Iodine
B-KI 10%+Iodine 5%
C-KI
D-Radioactive iodine
E-Sodium iodide
8) A patient using Lugol's solution should be counseled:
I) Lugol's solution may cause stains
II-Use topical only
III-Used to treat hypothyroidism patients
a) I only
b) III only
c) I and II only
d) II and III only
e) I, II, III
9-Drug of choice in pregnancy for hyperthyroidism
A-Lugol's solution
B-Propylthiouracil
C-Methimazole
D-Levothyroxine
E-Liothyronine
10-Patient receiving levothyroxine (L-T4) (Synthroid) should monitor levels of:
A-INR B-T4 C-TSH D-PT E-fT3
11- A woman with Grave’s disease. Doctor prescribed methimazole. All of the following statements are true,
EXCEPT:
a) This drug inhibits thyroid hormone
b) Onset of action is after 2 weeks synthesis
c) Do not stop the drug if there is a rash
d) Call the doctor if there are symptoms of infection

12- Instructions for Synthroid (levothyroxin) tablet.


I. Should not be taken with Iron supplements
II. Should not be taken within 4 hours of Ibuprofen

III. Should not be cut


a) I only
b) III only
c) I and II only
d) II and III only
e) I, II, III

Copyright © 2000-2014 TIPS Inc. Unauthorized reproduction of this manual is prohibited. This manual is being used during
review sessions conducted by PharmacyPrep. 80-2
Pharmacyprep.com
13- Symptoms of overdose of levotyroxin all, EXCEPT:
a) Cold tolerance
b) Tachycardia
c) Diarrhea
d) Weight loss

14-What is the most common side effects of propylthiouracil?


A-weight loss
B-weight gain
C-agranulocytosis
D-platelet aggregation
E-diarrhea
15-Hypothyroidism symptoms except?
A-diarrhea B-weight gain C-bradycardia D-hair loss
E-puffy face
16-Elevated TSH indicate?
A-Hyperthyroidism B-Hypothyroidism C-Euthyroidism
D-Hyperglycemia E-Graves disease
17-Hashimoto disease is?
A-Hyperthyroidism B-Hypothyroidism C-Euthyroidism
D-Hyperglycemia E-Graves disease
18-Absorption of levothyroxine is reduced by?
A-Al and Ca antacids
B-Calcium supplements
C-Iron supplements
D-Excessive Soya fiber
E-All of the above
19-What is effect of cotrimoxazole on warfarin
A-decrease INR by inhibiting GI flora
B-Increase INR by inhibiting GI flora
C-does not alter INR
D-Increase the risk of thromboembolism
20) A 55 yo women using levothyroxine 75 mcg to treat hypothyroidism. which of the following lab test are
used to monitor?
I) Serum TSH II) FT4 III)TT3
A)I only B) III only C) I and II D) II and III E) I, II, III

Copyright © 2000-2014 TIPS Inc. Unauthorized reproduction of this manual is prohibited. This manual is being used during
review sessions conducted by PharmacyPrep. 80-3
Pharmacyprep.com
21) A 55 yo women using levothyroxine 75 mcg to treat hypothyroidism. She is experiencing sweating, heat
sensitive and diarrhea. What is the best action to do?
A)Increase dose of levothyroxine
B)Decrease dose of levothyroxine
C)Change treatment to other hormone
D)Double the dose of levothyroxine
22) What is true about levothyroxine?
A) take in the morning with breakfast
B) take in evening with supper
C) take in evening empty stomach
D) take in the morning empty stomach
E) take with full glass of milk
23) All of the following drugs are used to treat thyrotoxicosis, (Graves disease) except?
A) propylthiouracil
B) methimazole
C) Lugol's solution
D) Propranolol
E) Levothyroxine
24) All of the following are correct in hypothyroidism, except?
A) Total T4 decrease
B) Free T4 decrease
C) Total T3 decrease
D) Serum TSH decrease
E) Free Thyroxin index decrease
25) Which of the following hormone is secreted from hypothalamus?
A) Thyroid stimulating hormone
B) Luteinizing hormone
C) Follicular stimulating hormone
D) Prolactin
E) Oxytosin
26) Which of the following stimulate calcitonin production from thyroid gland?
A) Hypocalcemia
B) Hypercalcemia
C) Hypokalemia
D) Hyperkalemia
E) Hypothyroidism

Copyright © 2000-2014 TIPS Inc. Unauthorized reproduction of this manual is prohibited. This manual is being used during
review sessions conducted by PharmacyPrep. 80-4
Pharmacyprep.com
27) A customer of your pharmacy uses insulin to treat diabetes. she is active and decided to jogging 5 km
daily. What is correct suggestion?
A) Take insulin injection before exercise
B) Take insulin injection after exercise
C) Take sandwich before exercise
D) Take sandwich after exercise
E) Stop taking insulin injections
28) Which of the following is NOT a hypothyroidism symptoms?
A) hypertension B) weight gain C) sensitivity to cold D) diarrhea E) dry skin
29) What is incorrect about hypothyroidism?
A) Patient using thyroid hormone therapy
B) Serum TSH is elevated more than 5 mU/L
C) TT3 is increased
D) Patient is tiered and fatigue
E) Patient have often dry skin
30) A patient is on insulin therapy. Which of the following is NOT correct about hypoglycemia?
A) Patient may be sweating
B) Patient blood glucose in lower than 4 mmol/L (fasting)
C) Patient have palpitation
D) Patient have confusion
E) Patient should increase insulin dose
31) An asthma patient excessive dose of prednisone. Which of the following can cause?
A) Graves disease
B) Hoshimoto disease
C) Addison disease
D) Cushing syndrome
E) Weight loss
32. A 49 yo women is using treatment of Synthroid 75 mcg daily for hypothyroidism. She complains
palpitation, weight loss and sensitivity to heat. What is appropriate?
A) refer doctor to increase dose of Synthroid 100 mcg
B) refer to doctor to decrease dose of Synthroid
C) hypothyroid symptoms so should refer to doctor
D) She may have some new problem so refer to doctor
E) None

Copyright © 2000-2014 TIPS Inc. Unauthorized reproduction of this manual is prohibited. This manual is being used during
review sessions conducted by PharmacyPrep. 80-5
Pharmacyprep.com
33. The same above patient current laboratory test indicate?
I) Increase serum TSH
II) Increase thyroxine (T4)
III) Increase triiodothyronin (T3)
A) I only B) III only C) I and II D) II and III E) I, II, III
34. Graves disease is?
A-Hyperthyroidism B-Hypothyroidism C-Euthyroidism D-Hyperglycemia
E-Hashimoto disease
35. Which of the following is the least likely decrease absorption of levothyroxine
A) Al antacids
B) Mg antacids
C) Diary products
D) Iron supplements
E) Low fiber diet
36. If you get prescription of thyroxin120 mcg daily as initial dose for senior patient. What symptoms may
expected?
A) hyperthyroidism
B) hypothyroidism
C) Parkinson's disease
D) Schizophrenic symptoms
E) Serotonin syndrome

Copyright © 2000-2014 TIPS Inc. Unauthorized reproduction of this manual is prohibited. This manual is being used during
review sessions conducted by PharmacyPrep. 80-6
Pharmacyprep.com
CORRECT ANSWERS
1- E
Tips: Graves disease is hyperthyroidism
2- E
3- B
4- A
5- A
Tips: in hyperthyroidism, there is weight loss
6- B
7- B
8- A
Tips: Lugol's solution is used as oral drops, Lugol's solution is indicated in hyperthyroidism patients.
9- B
10- C
11- C
12- A
Synthroid should not be taken with aluminum hydroxide, cholestyramine resins, colestipol, ferrous sulfate,
sod. Polystyrene sulfonate, soybean flour (infant formula) due to decreased absorption of levothyroxin from
GI tract.
May be given to children by crushing the tab. and suspending them in a small amount of water, breast milk
or non-soybean based formula. Do not store the suspension for any period of time.
13- A
14- C
15- A
16-B
Tips: elevated serum TSH > 5 mU/L indicate hypothyroidism.
17- B

Copyright © 2000-2014 TIPS Inc. Unauthorized reproduction of this manual is prohibited. This manual is being used during
review sessions conducted by PharmacyPrep. 80-7
Pharmacyprep.com
18- E
19- B
20) E
Tips: increase serum TSH and decrease Free T4 and TT3
21) B
22) D
23) E
24) D
Tips: FT4, TT4, TT3 and FTI decrease in hypothyroidism, only serum TSH increase. For hyperthyroidism,
exactly opposite changes.
25) E
26) B
27) C
28) D
29) C
30) E
31) D
32. B
Tips: patient experiencing overdose or hyperthyroidism symptoms, may be due overdose of levothyroxine.
So refer to doctor to decrease dose of levothyroxine
33. D
Tips: In hyperthyroidism increase T4 and T3.
34. A
35. E
Tips: Food and formula containing large amount of soybean, fiber, or iron should not be used for
administration of levothyroxine. However low fiber food does not interact with thyroxin.
36. A

Copyright © 2000-2014 TIPS Inc. Unauthorized reproduction of this manual is prohibited. This manual is being used during
review sessions conducted by PharmacyPrep. 80-8
PHARMACY PEP
CONTRACEPTION
1-Contraceptive methods that protect STDs such as, gonorrhea, syphilis, chlamydia, and AIDS?
A-Oral contraceptive B-Condoms C-Contraceptive sponges D-Vaginal cream
E-Plan B
2-A 18 year old women walks into pharmacy, wants to buy emergency contraception Plan-B. Pharmacist on
duty believes do not allow dispensing contraceptive, and refuse to give oral contraceptives. What ethical
principle is violated?
A-beneficence
B-Non maleficence
C-Autonomy
D-veracity
E-Justice
F-Paternalism
3) What is the most common side effects plan B?
A-abdominal pain
B-Constipation
C-Nausea and vomiting
D-vaginal bleeding
E-Deep vein thrombosis
4)If one contraceptive pill missed, recommend?
A-take two pills next day
B-take 3 pills next day
C-take one pill next day
D-use alternative method of protection for next week
E-Refer to doctor
5) A customer of your pharmacy she asking for your opinion to choose a contraceptive method, that she
does not want to use daily and convenience and also near future she has plan to start family. What is the
good contraceptive you recommend?
A)OCP B) Condoms C)Evra path D)Nuvaring E) contraceptive injections
6) Which of the following questions is appropriate to ask a customer who wants to buy plan B?
I)when was the last period II) are you pregnant? III)how long ago the unprotected intercourse
A) I only B) III only C) I and II only D) II and III only E)I, II, III

Copyright © 2000-2014 TIPS Inc. Unauthorized reproduction of this manual is prohibited. This manual is being used during
review sessions conducted by PharmacyPrep. 81-1
7) Which of the following contraceptive methods prevents gonorrhea and chlamydea infections?
A) Transdermal contraceptives B) Intrauterine device C) Vaginal cream D) Subdermal progesterone implants
E) None of the above
8) Many women who take OCs are poorly informed about the proper use of these medications. Which of the
following is the first effective methods of providing information?
A) The patient first should be given package insert of OCs
B) Auxiliary labels on OCs
C) Verbal information describing the way medication works
D) Patient should be told that this do not protect STIs
E) Patient using these products should have knowledge of this product from internet
9) What is correct about transdermal contraceptive Evra patch?
I) The patch should be applied to the abdomen, buttocks at the beginning of menstrual cycle
II) The patch should be applied to the upper torso, and upper arm at the beginning of menstrual cycle
III) The patch should be applied to the chest, and behind the ear at the beginning of menstrual cycle
A) I only B) III only C) I and II D) II and III E) I, II, III
ANSWERS:
1- B
2- C
3) C
4) A
5) D
6) E
7) E
Tips: STI are only prevented by condoms contraceptive methods
8) A
9) C

Copyright © 2000-2014 TIPS Inc. Unauthorized reproduction of this manual is prohibited. This manual is being used during
review sessions conducted by PharmacyPrep. 81-2
Copyright © 2000-2014 TIPS Inc. Unauthorized reproduction of this manual is prohibited. This manual is being used during
review sessions conducted by PharmacyPrep. 81-3
Pharmacyprep.com

PHARMACY PREP
GENITOURINARY CONDITIONS
1) Toxic shock syndrome is caused by:
A-E. coli
B-S. aureus
C-Chlamydia
D-Gonorrhea
E-Non gonococcal
2-Toxic shock syndrome may have risk with the use of:
I-Tampons use
II-Diaphragm contraceptive
III-Condoms as contraceptive
A-I only
B-III only
C-I and II only
D-II and III only
E-All of the above
3-Finastride is alpha reductase inhibitor, indicated in:
I-Benign prostate hyperplasia
II-Alopecia
III-Increase sexual function
a) I only
b) III only
c) I and II only
d) II and III only
e) All are correct
4-All of the following are the benign prostate hyperplasia, (BPH) symptoms, EXCEPT:
A-Nocturia
B-Frequent urination
C-Jet urination
D-Urinary retention
E-Post void dripping
5-Correct statement about pre menstrual symptoms (PMS) include:
I-All women have PMS
II-Luteal phase of cycle have PMS
III-vitamins, and estrogens deficiency may cause PMS
a) I only
b) III only

Copyright © 2000-2014 TIPS Inc. Unauthorized reproduction of this manual is prohibited. This manual is being used during
review sessions conducted by PharmacyPrep. 82-1
Pharmacyprep.com
c) I and II only
d) II and III only
e) All are correct
9. Urinary incontinence is NOT aggravated by:
A-Hydrochlorthiazide
B-Furosemide
C-Diphenhydramine
D-Acetazolamide
E-None of the above
10-Patient profile includes sildenafil 50 mg and brings a prescription of erythromycin for 5 days upper
respiratory infections, what is appropriate advice?
A-call doctor to decrease dose of sildenafil 25 mg while using erythromycin
B-Stop using sildenafil while using erythromycin
C-Call doctor to change other antibiotic
D-all of the above
11) A 50 year old lady will begin to take Hormone Replacement Therapy. All of the following questions will
help achieved appropriateness of her medication EXCEPT:
A) Did she have any children?
B) When did you last monitor your lipids?
C) Hysterectomy
D) When is the last menstruation period?
12. A patient is being treated with UTI, what is important to treat?
I. Dysuria and burning
II. Vaginal candidiasis
III. Fever and flare ups
a. I only
b. III only
c. I and III only
d. II and III only
e. I, II, and III

13) If a patient is not treated for asymptomatic sexually transmitted infections. What is correct?
A) symptoms are obvious after few days
B) can cause pelvic pain and dysmenorrhea
C) Can cause pelvic inflammatory disease and infertility
D) can cause endometrial cancer
E) can results in cervical cancer

Copyright © 2000-2014 TIPS Inc. Unauthorized reproduction of this manual is prohibited. This manual is being used during
review sessions conducted by PharmacyPrep. 82-2
Pharmacyprep.com
14) What is correct about premenstrual symptoms?
I) all women have premenstrual symptoms
II) pre menstrual symptoms occur during luteal phase
III) a natural product prime rose oil is used
A) I only
B) III only
C) I and II only
D) II and III only
E) I, II and III
15) What is incorrect about toxic shock syndrome?
A) caused by infections of S. aureus
B) can cause by tampon use
C) Can cause by condom use
D) can cause by candida infections
E) Can cause cervical cap contraceptives
16) Which of following condition avoid using sildenafil?
A) patients experienced priapism
B) patient have reported visual disturbances
C) patient using nitrates
D) patient have diplopia
E) children or women with hypertension
17) A customer of your pharmacy presents with symptoms of vaginal discharge, yellow and fishy odor.
A) Recommend over the counter antifungal drugs
B) Refer to doctor is appropriate because this bacterial infection
C) Recommend self care
D) None of the above
18) What is the mechanism action of finasteride?
A-Blockade of intestinal lipase
B-Blockade the conversion of estrogen to dihydroestrogen
C-Blockade of testosterone to dihydrotestosterone
D-Blockade of sebaceous glands
E-Blockade of hair growth
19) What period of menstrual cycle pre-menstrual symptoms (PMS) occur?
A-Follicluar phase
B-Ovulation phase
C-Immediately after ovulation
D-Luteal phase
E-During menstruation

Copyright © 2000-2014 TIPS Inc. Unauthorized reproduction of this manual is prohibited. This manual is being used during
review sessions conducted by PharmacyPrep. 82-3
Pharmacyprep.com
20) Which of the following methods of contraception can prevent sexually transmitted diseases
A-oral contraceptives
B-reservoir type contraceptive
C-Intrauterine devices
D-condoms
E-contraceptive sponges

Copyright © 2000-2014 TIPS Inc. Unauthorized reproduction of this manual is prohibited. This manual is being used during
review sessions conducted by PharmacyPrep. 82-4
Pharmacyprep.com
CORRECT ANSWERS
Genitourinary Conditions
1- B
2- C
3- C
4- C
5- D
9. C
Tips: diphenhydramine belongs to the ethanolamine class of antihistamines gives anticholinergic side effects
such as urinary retention, and induce sedation. Hydrochlorothiazide, furosemide and acetazolamide are
diuretics increase urinary frequency.
10- A
11) A
12. E
13) C
14) D
15) C
16) C
17) B
Tips: Color discharge and fishy odor is indicator of bacterial infection, thus refer to doctor.
18) C
19) D
20) D

Copyright © 2000-2014 TIPS Inc. Unauthorized reproduction of this manual is prohibited. This manual is being used during
review sessions conducted by PharmacyPrep. 82-5
Pharmacyprep.com

PHARMACY PREP.
BONES AND JOINT COMPLICATIONS
1- Osteoporosis is a disease characterized by loss of bone mass and may be aggravated by:
I-Corticosteroids
II-Menopausa
III-Grave’s disease
a) I only
b) III only
c) I and II only
d) II and III only
e) All are correct
2- A patients with allergy to Sulphonamides (i.e.: Sulphametoxazole) can take all of the following
medications, EXCEPT:
a) Penicillin
b) Lidocaine
c) Celecoxib
d) Clindamycin
e) Tetracycline
3- All of the following are examples of Disease Modifying Antirheumatic Agents (DMARD), EXCEPT
a) Methothrexate
b) Sulfasalazine
c) Organic Gold Compounds
d) Sulfinpyrazone
e) Penicillamine
4- Calcitonin is mainly classified as hormone liberated by thyroid gland. What stimulate the release of
calcitonin?
a) Hypercalcemia
b) Hypocalcemia
c) Hypophosphatemia
d) Hyperphosphatemia
e) High concentration of Vit. D
5- Which of the following statements is/are right regarding calcitonin
I- It secretion is normally stimulated during feeding, particularly of high calcium meals.
II- Prevents hypercalcemia by inhibiting the transport of Ca into extra cellular fluid and by increasing the
renal clearance of calcium

Copyright © 2000-2014 TIPS Inc. Unauthorized reproduction of this manual is prohibited. This manual is being used during
review sessions conducted by PharmacyPrep. 83-1
Pharmacyprep.com
III- It is a vitamin D derivative
a) I only
b) III only
c) I and II only
d) II and III only
e) I, II and III
6- The level of which drug should be adjusted during treatment with allopurinol
I) Mercapturic acid
II) Theophylline
III) Anticoagulants
a) I only
b) III only
c) I and II only
d) II and III only
e) All are correct
7) Uricosuric agents are used in gout treatment by inhibiting the uric acid re-absorption in the proximal
tubules, the best example of a uricosuric agent is:
a) Allopurinol
b) Sulphinpyrazone
c) Indomethacin
d) Penicillamine
e) Methotrexate
8- An oral chelating agent used in the treatment of rheumatoid arthritis is:
a) Allopurinol
b) Sulphinpyrazone
c) Indomethacin
d) Penicillamine
e) Methotrexate
9) In the treatment of rheumatoid arthritis the salicylates:
a) Stop and often reverse the progressive joint involvement
b) Specifically reverse the cause of the disease
c) Provide only analgesic effect
d) Are effective because they are uricosuric agents
e) Are more effective when given with allopurinol
10- Importance of Vitamin D in osteoporosis:
I- Increase the absorption of calcium from the small intestine
II- The best vitamin D analog to treat osteoporosis is calcitriol
III- Antacids containing aluminum and magnesium may interfere with proper activity of vitamin D.

Copyright © 2000-2014 TIPS Inc. Unauthorized reproduction of this manual is prohibited. This manual is being used during
review sessions conducted by PharmacyPrep. 83-2
Pharmacyprep.com
a) I only
b) III only
c) I and II only
d) II and III only
e) All are correct
11- Drug that inhibits xanthine oxidase and affect the metabolism of 6-mercaptopurine:
a) Sulfinpyrazone
b) Allopurinol
c) Penicillamine
d) Purinethol
e) Celecoxib
12- Pannus is an inflammatory exudates characterized by:
I- Synovial cell proliferation inside of a joint
II- Normally occurs in patients with rheumatoid arthritis
III- An immunologic complication
a) I only
b) III only
c) I and II only
d) II and III only
e) All are correct
13- Intra articular means:
a) In the joint space
b) In the tissues space
c) In the spinal fluid
d) In the spinal column
e) In the dermis
14- Colchicines, a widely used drug in gout treatment, can give false results of:
a) Glucose
b) Uric acids
c) Urates
d) Ketones
e) RBCs
15- Colchicine is used as anti-inflammatory agent in the acute treatment of:
a) Osteoporosis
b) Rheumatoid arthritis
c) Osteomyelites
d) Gout
e) Pressure atrophy

Copyright © 2000-2014 TIPS Inc. Unauthorized reproduction of this manual is prohibited. This manual is being used during
review sessions conducted by PharmacyPrep. 83-3
Pharmacyprep.com
16- The use of indomethacin in gout treatment:
a) Inhibit the acute gout arthritis inflammation by inhibition of prostaglandin formation
b) Inhibits tubulin synthesis
c) Accelerate renal excretion of uric acid
d) Inhibit uric acid synthesis
e) Inhibit the reabsorption of uric acid by the kidney
17- Allopurinol, a xanthine oxidase inhibitor, is a derivative of:
a) Uric acid
b) Purine
c) Pyridine
d) Pteridine
e) Urate salts
18- Goals of gout treatment may include:
I- Reduce inflammation during acute attacks
II- Accelerate renal excretion of uric acid
III- Reduce the conversion of purine to uric acid
a) I only
b) III only
c) I and II only
d) II and III only
e) All are correct
19- Characteristics of GOUT, a group of disorders of purine and pyrimidine metabolism may include:
I- Hereditary metabolite disease marked by inflammation of the joints
II- The joint inflammation is caused by precipitation of uric acid crystals
III- It is a type osteoporosis
a) I only
b) III only
c) I and II only
d) II and III only
e) All are correct
20- Allopurinol, xanthine oxidase inhibitor, used in gout treatment may produce all the following side effects,
EXCEPT:
a) Increase in protrombin time
b) Hypersensitivity rashes
c) GIT intolerance
d) Peripheral neuritis
e) Necrotizing vasculitis

Copyright © 2000-2014 TIPS Inc. Unauthorized reproduction of this manual is prohibited. This manual is being used during
review sessions conducted by PharmacyPrep. 83-4
Pharmacyprep.com
21- Use of estrogen in osteoporosis:
I- Estrogen enhance calcium retention and retard bone loss
II- Estrogen is useful in treat osteoporosis associated to menopause
III- Estrogen is not effective at increasing bone mass that has already been lost.
a) I only
b) III only
c) I and II only
d) II and III only
e) All are correct
22- Osteoporosis is mainly characterized by:
a) Loss of joint movements
b) Loss of bone mass
c) Imbalance of uric acid
d) Hormonal imbalance
e) All are correct
23- Antimalarial drug used in the treatment of rheumatoid arthritis as Disease-Modifying Antirheumatic Drug
(DMARD)
a) Chloroquine
b) Quinine
c) minoxidil
d) Hydroxycloroquine
e) Primaquine
24- Which is the correct start dose of methothrexate in the treatment of rheumatoid arthritis?
a) 7.5 mg once a week
b) 15 mg once a week
c) 7.5 mg daily
d) 15 mg daily
e) 25 mg once a week
25- Correct administration of Penicillamine in the treatment of rheumatois arthritis include:
a) With meals
b) In emptying stomach
c) With plenty of water
d) With orange juice
e) Before bedtime
26- Which of the following agents is the best choice of treatment for an asthma patient with rheumatoid
arthritis who is considered to have aspirin sensitivity?
a) Ibuprofen

Copyright © 2000-2014 TIPS Inc. Unauthorized reproduction of this manual is prohibited. This manual is being used during
review sessions conducted by PharmacyPrep. 83-5
Pharmacyprep.com
b) Acetaminophen
c) Gold therapy
d) Azathioprine
e) Cyclophosphamide
27- Diflunisal is most likely to be prescribed for the treatment of:
a) Wilson’s disease
b) Rheumatoid arthritis
c) Cysteinuria
d) Psoriasis
e) Hansen’s disease
28-Azathioprine and allopurinol drug interactions are due to?
A-Azathioprine inhibits xanthine oxidase
B-Allopurinol inhibits xanthine oxidase
C-Azathioprine is metabolized to mercaptopurine by xanthine oxidase
D-Allopurinol is metabolized to mercaptopurine by xanthine oxidase
E- B and C are correct
29- In the treatment of rheumatoid arthritis the salicylates:
a) Stop and often reverse the progressive joint involvement
b) Specifically reverse the cause of the disease
c) Provide only analgesic effect
d) Are effective because they are uricosuric agents
e) Are more effective when given with allopurinol

Copyright © 2000-2014 TIPS Inc. Unauthorized reproduction of this manual is prohibited. This manual is being used during
review sessions conducted by PharmacyPrep. 83-6
Pharmacyprep.com
CORRECT ANSWERS
1- E
Comments: Osteoporosis is a disease characterized by the loss of bone mass. Substances or conditions that
alter the metabolism of bones may predispose or aggravated osteoporosis. Example: Corticosteroids,
menopause and Grave’s disease.
2- C
Comments: There is a cross-sensitivity drug interaction between sulphonamides and celecoxib
3- D
Comments: Sulphynpyrazone is an uricosuric acid agent used in the treatment of gout.
4- A
Comments: Calcitonin is a hormone elaborated by the thyroid gland in response to hypercalcemia.
5- C
Comments: Calcitonin is not a vitamin D derivative. It is a hormone elaborated by the thyroid gland in
response to hypercalcemia, lowering plasma calcium and phosphate levels.
6- E
Comments: Allopurinol may inhibit enzymatic metabolism of anticoagulants, leading to potentiation of the
anticoagulant effect. Mercapturic acid dosage should be reduced with concomitant use with allopurinol
because it induced inhibition of xanthine oxidase decreasing metabolism of mercapturic acid and
potentiating therapeutic and toxic effects. Allopurinol decrease theophyllin clearance resulting in increased
serum theophyllin concentrations
7- B
Comments: Allopurinol and indomethacin are used in gout treatment however not as uricosuric agent but as
xanthine oxidase inhibitor and NSAID respectively. Methotrexate is used to treat rheumatoid arthritis as
DMRA agent.
8- D
Comments: Allopurinol and sulphynphyrazone are only used in gout treatment. Indomethacin can be used in
rheumatoid arthritis however as NSAID and Methotrexate is used as DMRA agent.
9- C

Copyright © 2000-2014 TIPS Inc. Unauthorized reproduction of this manual is prohibited. This manual is being used during
review sessions conducted by PharmacyPrep. 83-7
Pharmacyprep.com
Comments: Salicylates in the treatment of rheumatoid arthritis provides only analgesic effect by reducing
prostaglandin synthesis.
10- E
Comments: Osteoporosis is characterized by the loss of bone mass. Vitamin D supplements have been used
in the treatment of osteoporosis because vitamin D increases the absorption of calcium that is essential to
reduce bone loss. Antacids interact with vitamin D in many ways, magnesium containing antacids may lead
to hypermagnesemia and aluminum containing antacids have its levels increased leading to aluminum bone
toxicity when currently used with vitamin D.
11- B
Comments: Allopurinol inhibit xanthine oxidase enzyme therefore the conversion of 6-mercaptopurine to
uric acid inhibiting the uric acid synthesis-formation
12- C
Comments: Pannus is an inflammatory exudates overlying the lining layer of synovial cells on the inside of a
joint, usually occurring in patients with rheumatoid arthritis or related articular rheumatisms, and sometimes
resulting in fibrous ankylosis of the joints.
13- A
Comments: Intra-articular is normally referred to a kind of injection given within a joint.
14- E
Comments: Colchicine may cause false-positive test results of red blood cells-RBC and hemoglobin in urine. It
may also cause decrease in platelet count.
15- D
Comments: Cochicine is only used in gout treatment. It has no effect on uric acid therefore it is used just in
the acute treatment of gout by inhibiting the migration of leukocytes, phagocytes and leukotrienes B4 to the
affected joint.
16- A
Comments: Indomethacin is a potent NSAID that inhibits the acute gout arthritis inflammation by inhibition
of prostaglandin formation and inhibition of crystals phagocytes by macrophages.
17- B
Comments: Allopurinol is a purine derivative working as xanthine oxidase enzyme inhibitor responsible for
the conversion of 6-mercaptopurine to uric acid.
18- E

Copyright © 2000-2014 TIPS Inc. Unauthorized reproduction of this manual is prohibited. This manual is being used during
review sessions conducted by PharmacyPrep. 83-8
Pharmacyprep.com
Comments: The gout disease is normally controlled by three different mechanisms: 1- Control the
inflammatory response by inhibition of leukocytes, macrophages and leukotrienes migration to the affected
joint. 2- Increase the renal clearance of uric acid by inhibiting the reabsorption of uric acid by the kidney and
3- Reduce the conversion of 6-mercaptopurine to uric acid by inhibition of the enzyme xanthine oxidase.
19- C
Comments: Gout is a group of disorders of purine and pyrimidine metabolism. It is manifested by
hyperuricemia, recurrent characteristic arthritis induced by deposit of crystals in and around the joint of the
extremities, which may lead to destruction and severe crippling of joints.
20- A
Comments: Main side effects of allopurinol therapy include: Steven-Johnson’s hypersensitivity rashes
reactions, gastro-intestinal intolerance, peripheral neuritis and necrotizing vasculitis.
21- E
Comments: Estrogen is widely used to prevent bone loss after menopause. Estrogen can enhance calcium
retention and retard bone loss but does not increase bone mass that has already been lost.
22- B
Comments: Osteoporosis is mainly characterized by the loss of bone mass. It is the most common of the
metabolic bone diseases and is an important cause of morbidity in elderly.
23- D
Comments: Cloroquine and primaquine are normally used together only in the treatment and prevention of
malaria. Quinine and hydroxycloroquine are antimalarial agents and alternatively used in prophylaxis and
treatment of muscle cramps leg and rheumatoid arthritis respectively.
25- B
Comments: In the treatment of rheumatoid arthritis penicillamine should be administrated in an empty
stomach. Improvement of the condition may take up to 2 to 3 months of therapy
26- C
Comments: Gold therapy would be the best choice for this particular patient because the other drugs may
have cross-sensitivity with aspirin and/or induce brochospasm.

Copyright © 2000-2014 TIPS Inc. Unauthorized reproduction of this manual is prohibited. This manual is being used during
review sessions conducted by PharmacyPrep. 83-9
Pharmacyprep.com
27- B
Comments: Diflunisal is a NSAID used for both acute and chronic anti-inflammatory treatment of rheumatoid
arthritis.
28-E
29-C
Tips: Salicylates in the treatment of rheumatoid arthritis provides only analgesic effect by reducing
prostaglandin synthesis.
BIBLIOGRAPHIC REFERENCE
1- PHARMACY PREP – Evaluating Exam Review Book 2014
2- CPS-COMPENDIUM OF PHARMACEUTICALS AND SPECIALITIES - Canadian Pharmacist Association – 2014
edition.
th
3- MEDICAL DICTIONARY – Dorland’s illustrated – 27 edition.
4- COMPENDIUM OF THERAPUTIC CHOICES – Canadian Pharmacist Association -Third edition
th
5- USP DI – Drug Information for the Health Care Professional–15 edition – Volume I.

Copyright © 2000-2014 TIPS Inc. Unauthorized reproduction of this manual is prohibited. This manual is being used during
review sessions conducted by PharmacyPrep. 83-10
PharmacyPrep.Com Osteoporosis

PHARMACY PREP
OSTEOPOROSIS
1) These two components in bone are responsible for the hardness and pliability of bone: a) osteoclasts &
collagen b) mineralized salts & osteocytes c) mineralized salts & collagen d) collagen & elastic fibres e)
collagen & mesenchyme
2) A fracture in the shaft of a long bone would be a break in the: a) epiphysis b) metaphysis c) diaphysis d)
epiphyseal plate e) mesenchyme
3) Osteoclast are bone cells cause?
A. Bone formation
B. Bone resorption or bone cell death
C. Bone mineral density
D. Bone remodelling
E. Bone receptor modulator
4) Chondroblasts produce: a) basement membranes b) bone matrix c) cartilage matrix d) mesothelium e)
endothelium
5) All of the following are the risk factor for osteoporosis, except?
A. Older age
B. Inadequate intake of calcium and vitamin D
C. Post menopausal women
D. over weight
E. oral contraceptive
6) The cell type that is responsible for maintaining bone matrix once it has formed is: a) osteoclasts b)
chondrocytes c) osteocytes

Copyright © 2000-2014 TIPS Inc. Unauthorized reproduction of this manual is prohibited. This manual is being used during
review sessions conducted by PharmacyPrep. 84-1
PharmacyPrep.Com Osteoporosis
d) fibroblasts e) osteoblasts
7) Soft connective tissue membranes between the cranial bones at birth are: a) an indication of microcephaly
b) frontal sinuses c) epiphyseal plates d) cribriform plates e) fontanelles
8) Endochondral and intramembranous are two mechanisms of: a) bone remodeling b) embryonic skeletal
ossification c) controlling blood calcium levels d) cartilage synthesis
9) The two pairs of bones that make up the hard palate are the right and left: a) zygomatic and temporal b)
palatine and maxillae c) maxillae and zygomatic d) maxillae and mandible
10) The two bones that make up the posterior nasal septum are the: a) nasal and lachrymal b) inferior nasal
choncae and vomer c) vomer and ethmoid d) ethmoid and sphenoid
11) Which of the following drugs are pyrophosphates?
A. Raloxifine
B. Alendronate
C. Colchicine
D. Cisplatin
E. Calcitrol
12)All of the following are the risk factors for osteoporosis, except?
A-Race
B-Family history
C-Obesity
D-Smoking
E-Excessive coffee intake
13- Importance of Vitamin D in osteoporosis:

Copyright © 2000-2014 TIPS Inc. Unauthorized reproduction of this manual is prohibited. This manual is being used during
review sessions conducted by PharmacyPrep. 84-2
PharmacyPrep.Com Osteoporosis
I- Increase the absorption of calcium from the small intestine
II- The best vitamin D analog to treat osteoporosis is calcitriol
III- Antacids containing aluminum and magnesium may interfere with proper activity of vitamin D.
a) I only
b) III only
c) I and II only
d) II and III only
e) All are correct
14. All of the following are side effect of a systemic corticosteroids prednisone: EXCEPT:
A-Weight gain
B-Glucose intolerance
C-Osteoporosis
D-Cataract
E-reduce infection
15) Osteoporosis caused by the deficiency of the following:
I. Inadequate intake of vitamin D
II. Inadequate intake of calcium
III. Estrogen deficiency
a) I only
b) III only
c) I and II only
d) II and III only
e) All are correct
16-What calcium supplement has the most elemental calcium?
A-Calcium citrate B-natural calcium C-Calcium carbonate D-dairy products E-Calcium gluconate
17) The type of calcium supplement produced from oyster’s shells is?
A-Calcium citrate
B-natural calcium
C-Calcium carbonate
D-dairy calcium
E-Calcium gluconate
18-What is calcium supplement daily dose in adults?
A-1 g B- 500 mg C-800 mg D-100 mg E- 2 g
19-What is incorrect about alendronate?
A-Take first thing in the morning empty stomach with 1 full glass of water
B-Contraindicated renal diseases

Copyright © 2000-2014 TIPS Inc. Unauthorized reproduction of this manual is prohibited. This manual is being used during
review sessions conducted by PharmacyPrep. 84-3
PharmacyPrep.Com Osteoporosis
C-70 mg/wk dose have equivalent effect as daily 10 mg dose
D-Monitor hypercalcemia before initiating treatment
E-Do not lie down for 30 min after oral dose
20-A patient using alendronate 70 mg/wk and calcium supplement, what is incorrect?
A-Avoid calcium supplement while using alendronate therapy
B-It is important to take calcium supplement while using bisphosphonates therapy
C-separate alendronate and calcium supplement at least 2 hour before and 4 after.
D-It is also essential to take vitamin D supplements
E-It is recommended to take daily 800 mg IU vitamin D
21- To monitor alendronate safety and efficacy all are needed EXCEPT:
a) Hypercalcemia
b) Bone pain and swelling
c) Ca and Vitamin D intake
d) BMD
e) Esophagitis

22- What is correct about osteoporosis?


A) Inadequate Ca and vitamin D can cause osteoporosis
B) Alendronate can be used once month
C) Zolendronoic acid injection is used daily
D) Recommend daily vitamin D 2000 IU in over age 65yo
E) Recommend swimming a good exercise to increase bone mineral density
23- Which of the following increase risk of osteoporosis?
A) History of oral contraceptive use
B) Smoking
C) Physical activity like weight bearing exercises like stair climbing, walking, and jogging.
D) Increase in dietary soy intake
E) Increase intake of broccoli
24- A 35 yo women get the prescription of 50,000/wk unit of vitamin D. What to do?
A) talk to doctor and dispense
B) this could extremely high dose and may cause toxicity
C) reduce dose to 500 units and dispense
D) May be prescription error just dispense 500 units
E) It is outrageous to get prescription of 50, 000 units
25- What is not risk factor of osteoporosis?
a)estrogen deficiency B) androgen deficiency C) race D)age E)protein diet
26- Which of the following is the least recommended exercises in osteoporosis?
A) rope jumping B) basket ball playing C) horse riding D) jogging E) Dancing

Copyright © 2000-2014 TIPS Inc. Unauthorized reproduction of this manual is prohibited. This manual is being used during
review sessions conducted by PharmacyPrep. 84-4
PharmacyPrep.Com Osteoporosis
27- Which of the following is not a cranial suture: a) epiphyseal b) lambdoidal c) coronal d) sagittal e)
squamous
28. Which of the following is least likely risk factor for osteoporosis?
A) Inadequate vitamin D and Ca supplement intake
B) Age
C) family history of osteoporosis
D) Prednisone therapy for over 3 months
E) alcohol intake
29. What is the most likely risk for osteoporosis?
A) smoking
B) alcohol
C) coffee
D) tea
E) none

Copyright © 2000-2014 TIPS Inc. Unauthorized reproduction of this manual is prohibited. This manual is being used during
review sessions conducted by PharmacyPrep. 84-5
PharmacyPrep.Com Osteoporosis
ANSWERS:
1) C
2) C
3) B
4) C
5) D
6) C
7) E
8) B
9) B
10) C
11) B
Tips. Bisphophonates like alendronate are pyrophosphates
12) C
Tips: obesity or overweight is not a risk factor of osteoporosis. Race, family history, smoking, excessive coffee
intake, deficiency of estrogen, and sedentary lifestyle are some of the risk factors.
13- E
Tips: Osteoporosis is characterized by the loss of bone mass. Vitamin D supplements have been used in the
treatment of osteoporosis because vitamin D increases the absorption of calcium that is essential to reduce
bone loss. Antacids interact with vitamin D in many ways, magnesium containing antacids may lead to
hypermagnesemia and aluminum containing antacids have its levels increased leading to aluminum bone
toxicity when currently used with vitamin D.
14. E
Tips: Corticosteroids increase the risk of infections.
15- E

Copyright © 2000-2014 TIPS Inc. Unauthorized reproduction of this manual is prohibited. This manual is being used during
review sessions conducted by PharmacyPrep. 84-6
PharmacyPrep.Com Osteoporosis
16- C
17- C
18- A
19- D
20- A
21- A
22- A
23- B
24- A
25- E
Tips: protein diet like dietary are plant derived phytoestrogen present in soy proteins
26- C
27- A
28. E
29. C

Copyright © 2000-2014 TIPS Inc. Unauthorized reproduction of this manual is prohibited. This manual is being used during
review sessions conducted by PharmacyPrep. 84-7
PharmacyPrep.Com Osteoporosis
Copyright © 2000-2014 TIPS Inc. Unauthorized reproduction of this manual is prohibited. This manual is being used during
review sessions conducted by PharmacyPrep. 84-8
PharmacyPrep.com Hypertension

PHARMACY PREP
HYPERTENSION
1) According to Joint National Committee (JNC-7) standards. In measuring blood pressure of someone whose
systolic blood pressure in the past had 150 mm Hg. Now if you like to check blood pressure. You would have
to inflate the bulb to?
A-170 mm Hg
B-150 mm Hg
C-200 mm Hg
D-120 mm Hg
E-210 mm Hg
2. Bradycardia associated with which of the following drug?
A-Nifedipine
B-Amlodipine
C-Verapamil
D-Nitroglycerin
E-All of the above
3-A 57-year-old patient is regular customer of your pharmacy. His current medication include
hydrochlorothiazide. He complains the following side effects. Which of the following is not due to his
medication:
A-Gout
B-Erectile dysfunction
C-Hyperglycemia
D-Hypokalemia
E-Hypocalcemia
4-A hypertensive patient used sildenafil 20 hours before, which of the following is contraindicated in this
patient:
A-Nitroglycerin transdermal
B-Isosorbide dinitrate
C-iv nitroglycerin
D-Nitroglycerin sublingual
E-All of the above
5-When spironolactone is used in-patient with cardiac heart failure, it works through which of the following
mechanism?
A-Aldosterone antagonist
B-Negative inotropic effect
C-Angiotensin II Inhibitors
D-Positive inotropic drugs E-Positive chronotropic effect

Copyright © 2000-2014 TIPS Inc. Unauthorized reproduction of this manual is prohibited. This manual is being used during
review sessions conducted by PharmacyPrep. 85-1
PharmacyPrep.com Hypertension
6)A diabetic patient suffering from hypertension is having enalapril, however he is complaining from severe
dry cough that we can associated as a side effect of the enalapril. Which would be the best choice for
changing this medication?
a) Ramipril
b) Losartan
c) Amlodipine
d) Indapamide
e) Captopril
7) What is incorrect about captopril?
A-It contain sulfhydril group
B-It is long acting taken once a day
C-It is short acting taken 2 to 3 times a day
D-It active drug of ACE Inhibitors
E- It causes hyperkalemia
8-A 45-year-old male diabetic patient blood pressure is 150/90, what action considered appropriate?
A-Refer to emergency
B-Refer to physician
C-Uncontrolled diabetes
D-uncontrolled hypertension
E-It is normal blood pressure in 40 year of age patient
9) Which of the following is a selective b-blocker with ISA?
A- Atenolol B-Propranolol C-Pindolol D-Acebutolol E) Timolol
10) Which of the following suitable for treatment of hypertensive crisis?
I-Na nitroprusside
II-Diazoxide
III-Hydrochlorothiazide
A) I only B) III only C) I and II only D) II and III only E) all
11) What is true for ACE inhibitors?
A-Contraindicated in CHF
B-Decrease bradykinin level
C- ARBs can be alternatives
D-Used safely in pregnancy
E) Cause hypokalemia
12. What is correct in counselling for hypertension patient?
I) recommend to check blood pressure at home frequently
II) carryout medication review
III) Advise patient on salt intake and lifestyle modification
A) I only B) III only C) I, and II D) II and III E) I, II, III

Copyright © 2000-2014 TIPS Inc. Unauthorized reproduction of this manual is prohibited. This manual is being used during
review sessions conducted by PharmacyPrep. 85-2
PharmacyPrep.com Hypertension
ANSWERS:
1- A
2. D
Tips: Dihydropyridine type of calcium channel blockers have reflex tachycardia, where as non-
dihydropyridine type of calcium channel blockers such as verapamil and diltiazem have bradycardia.
3- E
4- E
5- A
6- B
7) B
8- B
Tips: diabetics with hypertension and renal disease with hypertension should be treated for hypertension at
blood pressure over 130/85
9) D
10) C
11) D
12. E

Copyright © 2000-2014 TIPS Inc. Unauthorized reproduction of this manual is prohibited. This manual is being used during
review sessions conducted by PharmacyPrep. 85-3
PharmacyPrep.Com Coronary Artery Diseases

PHARMACY PREP
CORONARY ARTERY DISEASES
1) Which one of the following factors is most directly attributed to angina attacks?
A. Emotional Stress
B. Heavy smoking
C. Myocardial ischemia
D. Obesity
E. None of the above

2) Purple toe syndrome side effect is produced by:


A-Amiodarone
B-Reynaud phenomenon
C-Warfarin
D-Rifampin
E-Verapamil
3. Which medication CAN NOT BE used for prinzmetal angina (vasospastic angina)?
A. Amlodipine
B. Metoprolol
C. Acetyl salicylic acid
D. Verapamil
E. Diltiazem

4. Which effect of nitroglycerin is most likely responsible for its therapeutic action in angina pectoris?
a. Dilation of coronary artery
b. Relaxation of peripheral vascular smooth muscle
c. Decrease myocardial after load
d. A and B
e. A and C
5. Nitroglycerin dilates the coronary arteries in angina pectoris by
a) Decreasing the heart rate reflex
b) Increasing the metabolic work of the myocardium
c) Direct action on smooth muscle in the vessel walls
d) Increasing the effective refractory period in the atrium
e) Blocking beta-adrenergic receptors

6. Which one of the following conditions is not associated with verapamil?


A. Inhibition of calcium ion influx
B. Negative inotropic effect
C. Coronary vasodilatation effect

Copyright © 2000-2014 TIPS Inc. Unauthorized reproduction of this manual is prohibited. This manual is being used during
review sessions conducted by PharmacyPrep. 86-1
PharmacyPrep.Com Coronary Artery Diseases
D. Peripheral vasodilatation effect
E. None of the above

7-Which of the following anticholesterol medication may increase his blood sugar levels:
A-Atorvastatin
B-Niacin
C-Simvastatin
D-Fluvastatin
E-All of the above
8-Administration of which of the following drugs is most likely to PROLONG clotting time?
A. Morphine
B. Vitamin K
C. A barbiturate
D. Acetaminophen
E. Acetyl salicylic acid

9. Which of the following drugs acts by inhibiting renal reabsorption of sodium?


a) Urea
b) Chlorothiazide
c) Theophylline
d) Digitalis glycosides
e) Procainamide

10) All of the following are low molecular weight heparins (LMWH), except?
A-Enoxaparin
B-Dalteparin
C-Tinzaparin
D- Nadroparin
E-Heparin
11) What is not true about low molecular weight heparins
(LMWH)?
A-It act by inhibiting factor Xa
B-It is long acting than heparin
C-It does not require monitoring
D-It is contraindicated in pregnancy
E-It is used as once daily SC injection
12) Unstable/NSTEMI drug of choice?
A-Anticoagulants
B-LMWH
C-Heparin
D-Alteplase
E-Propranolol

Copyright © 2000-2014 TIPS Inc. Unauthorized reproduction of this manual is prohibited. This manual is being used during
review sessions conducted by PharmacyPrep. 86-2
PharmacyPrep.Com Coronary Artery Diseases
13) What is the drug of choice for ST segment elevated myocardial infarction (STEMI)?
A-Anticoagulants B-LMWH C-Heparin D-Alteplase E-Propranolol
14) What is the drug of choice for stable angina?
A-Anticoagulants
B-LMWH
C-Heparin
D-Alteplase
E-beta blockers
15)Which of the following drug may cause bradycardia?
A-Nifedipine
B-Nitroglycerine
C-Amlodipine
D-Verapamil
E-Clonidine
16) Which of the following drug should be taken with food?
A-Fluvastatin B-Lovastatin C-Simvastatin D-Atorvastatin E-Rosuvastatin
17) What statin can be taken anytime of the day?
A-Fluvastatin B-Lovastatin C-Simvastatin D-Atorvastatin E-Rosuvastatin
18) What statin do not require dosage adjustment in renal disease patient?
A-Fluvastatin B-Lovastatin C-Simvastatin D-Atorvastatin
E-Rosuvastatin
19) Which of the following is NOT used for antiplatelets action?
A-ASA B-Ibuprofen C-Naproxen D-Pyroxicam E-Acetaminophen
20) What antiplatelets drugs is used for prophylactic for angina pectoris?
A-dipyridamol b-metoprolol c-nifedipine D-all of the above
21. Which of the following is NOT associated with plaque formation?
A) Coronary artery disease
B) Angina
C) Stroke
D) Reynaud's phenonmenon
E) intermittent claudication
22. Which of the following is recommended to minimize the warfarin dispensing error?
I) dose of warfarin should be indicated in milligrams
II) check the color of tablets
III) Recommend lifestyle modification

Copyright © 2000-2014 TIPS Inc. Unauthorized reproduction of this manual is prohibited. This manual is being used during
review sessions conducted by PharmacyPrep. 86-3
PharmacyPrep.Com Coronary Artery Diseases
23. Exercise induced angina, which is relieved by rest nitroglycerine or both referred as?
A) prinzmetal angina B) stable angina C) unstable angina
D) Myocardial infarction
24. Myocardial oxygen demand is increased by all the following except?
A)exercise B) Beta blockers C) cold air D) smoking
25. A patient with angina receiving metoprolol and diltiazem should be monitored for?
I) increased heart rate
II) decreased cardiac output
III) decreased heart rate
A-I only B-III only C-I and II only D-II and III only E) I, II, III
26) A 35 year old craftsman is on nitroglycerin pills, which of the following statements should discuss with
him:
A) It causes an elevation of intracellular cGMP
B) It can cause postural hypotension, take medication while sitting.
C) Store medication away from light, and keep in tightly metal capped bottle.
D. B&C
27) Which of the following is/are the ischemic heart diseases?
A. MI
B. Angina
C. Arrhythmias
D. A&B
28) Aspirin should be used cautiously in patient receiving heparin because aspirin:
A. inhibit vitamin k absorption
B. has antithrombin activity
C. inhibit metabolism of heparin
D. inhibit platelet aggregation
29) Nitroglycerin dilates the coronary arteries in angina pectoris by
A. decreasing the heart rate reflex
B. increasing the metabolic work of the myocardium
C. direct action on smooth muscle in the vessel walls
D. increasing the effective refractory period in the atrium
Answer: C
30) In angina pectoris, the MOST rapid and dependable relief will be provided by
A. oral glyceryl trinitrate
B. oral isosorbide dinitrate
C. oral erythrityl tetranitrate
D. sublingual glyceryl trinitrate

Copyright © 2000-2014 TIPS Inc. Unauthorized reproduction of this manual is prohibited. This manual is being used during
review sessions conducted by PharmacyPrep. 86-4
PharmacyPrep.Com Coronary Artery Diseases
31) Nitroglycerin when taken together with Sildenafil will cause:
A. Hypertension
B. Hypotension
C. Headache
D. Hyperglycemia
32) Aspirin acts as:
A. irreversible antiplatelet drug
B. reversible antiplatelet drug
C. antiarrhythmic
D. anticoagulant
33) What are correct statement about Storage of Nitroglycerin:
A. Amber color glass, hygroscopic
B. Amber glass
C. Plastic container
D. Any container can be use
34) What drug shouldn’t be given to post MI?
A. ASA
B. Ace inhibitors
C. CCB and beta blockers
D. All of the above
35) Which drug causes bradycardia?
A. Diltiazem
B. Nifedipine
C. Nicardipine
D. Felodipine
36) This drugs cause reflex tachycardia except:
A. Amlodipine
B. Nifedipine
C. Nicardipine
D. Felodipine
37) Action of nitrates can be described as?
A. vasodilation
B. vasoconstriction
C. bronchoconstriction
D. Bronchodilatation

Copyright © 2000-2014 TIPS Inc. Unauthorized reproduction of this manual is prohibited. This manual is being used during
review sessions conducted by PharmacyPrep. 86-5
PharmacyPrep.Com Coronary Artery Diseases
38) An imbalance between oxygen supply and oxygen demand in cardiac muscle may produce a condition
known as:
A. CHF
B. Heartburn
C. MI
D. Angina pectoris
39) Drugs may be used in acute myocardial infarction: except
A) Beta blocker
B) ACE inhibitor
C) Calcium channel blockers
D) Nitrates
E) Digoxin
40) What dose does Aspirin acts as antiplatelet?
A. 80 mg
B. 365 mg
C. 500 mg
D. 1 g
41) Drugs used in angina pectoris except:
A. Nitrates
B. CCB
C. Cardiac glycoside
D. Beta blockers
42) What is the most common form of Ischemic heart disease:
A. Myocardial infarction
B. Atrial fibrillation
C. Angina pectoris
D. Ventricular arrhythmia

Copyright © 2000-2014 TIPS Inc. Unauthorized reproduction of this manual is prohibited. This manual is being used during
review sessions conducted by PharmacyPrep. 86-6
PharmacyPrep.Com Coronary Artery Diseases
CORRECT ANSWERS
Coronary artery diseases
1. C
2- C
3. B
Tips: TC page 275, avoid beta blockers in prinzmetal angina, beta blockers cause vasoconstriction
(vasospasm)
4. D
5. C
6. E
Tips: Verapamil act as coronary vasodilator by inhibiting calcium ion influx in coronary smooth muscles, and
it does negative inotropic effects.
7- B
8- E
9. B
10) B
11) D
12. A
13) D
14) E
15) D
16) B
Tips: Lovastatin should be always administered with food to increase Bioavailability, otherwise it can
decrease 30% Bioavailability.
17) D
Tips: Atorvastatin can be taken anytime of the day, because has long half life.
18) D
Tips: The best agent for renal disease patient is atorvastatin, because it has minimal renal elimination, thus
do not require dose adjustments. Simvastatin and lovastatin require dose adjustment in severe renal
diseases however it can be used in minor and moderate renal diseases
19) E
20) A
21. D
22. C

Copyright © 2000-2014 TIPS Inc. Unauthorized reproduction of this manual is prohibited. This manual is being used during
review sessions conducted by PharmacyPrep. 86-7
PharmacyPrep.Com Coronary Artery Diseases
23. B
24. B
25. B
26. D
Tips: Nitroglycerin increases synthesis of intracellular cGMP. However, mechanism of action may not be
discussed in counselling.
27. D
28. D
Tips: a drug that increases metabolism of the anticoagulant will lower the steady state plasma concentration
both free and bound form, where as one that displaces anticoagulant will increase plasma level of free form
only, until elimination of drug as again lower it to steady state levels.
29. C
30. D
31. B
32. A
33. A
34. D
35. A
Tips: Diltiazem and Verapamil side effect is bradycardia
36. A
37. A
39. E
38. D
40. A
41. C
42. C

Copyright © 2000-2014 TIPS Inc. Unauthorized reproduction of this manual is prohibited. This manual is being used during
review sessions conducted by PharmacyPrep. 86-8
www.pharmacyprep.com

PHARMACY PREP
STROKE

1) All of the following are initial symptoms of stroke include,


Except?
A-headache
B-blurred vision
C-paralysis
D-seizure
E-dizziness
2-Which of the followings is a condition ticlopidine is contraindicated for?
A- Neutropenia
B- Heart diseases
C- Pulmonary embolism
D- GERD
E- Arthralgia

3) What is the drug of choice to treat transient ischemic attack (TIA)


A) Acetyl salicylic acid
B) Ticlopidine
C) Warfarin
D) Heparin
E) Ramipril

4) Which of the following factor that does NOT predispose stroke?


A) Age
B) Diabetes
C) Ethnicity
D) Hypertension
E) chronic liver diseases

87-1 Copyright © 2000-2014 TIPS Inc. Unauthorized reproduction of this manual is strictly prohibited and it is ille-gal to
reproduce without permission. This manual is being used during review sessions conducted by Pharma-cyPrep.
www.pharmacyprep.com
CORRECT ANSWERS
STROKE
1- D
Tips: Stroke symptoms: Headache, dizziness, blurred vision, confusion, incoherent speech
2- A
3- A
4- E

87-2 Copyright © 2000-2014 TIPS Inc. Unauthorized reproduction of this manual is strictly prohibited and it is ille-gal to
reproduce without permission. This manual is being used during review sessions conducted by Pharma-cyPrep.
PharmacyPrep.com Congestive Heart Failure (CHF)

PHARMACY PREP
CONGESTIVE HEART FAILURE
1) Initial symptoms of heart failure include?
A) Cough
B) Dyspnea
C) Fatigue
D) Bradycardia
E) Pulmonary edema
2) Which of the following drugs is MOST useful in treating or preventing angina pectoris?
a) Digitalis
b) Quinidine
c) Propranolol
d) Procainamide
e) Pentobarbital

3. Which of the following ions augments the inotropic effect of a digitalis?


a) Sodium
b) Lithium
c) Calcium
d) Chloride
e) Magnesium

4. Which of the following diuretics is appropriate for a patient for a patient who is suffering from congestive
heart failure and tends to show digitalis toxicity?
a) Furosemide
b) Triamterene
c) Ethacrynic acid
d) Chlorothiazide
e) Hydrochlorothiazide

5-About digoxin, which of the following statement are true:


I-+ve chronotropic effects
II-+ve inotropic effects
III-Vagomimetic effects
A-I only B-III only C-I and II only D-II and III only E-All of the above
6. In the treatment of congestive heart failure, digitalis glycosides generally DECREASE all of the following
EXCEPT
a) Edema
b) Urine flow

Copyright © 2000-2014 TIPS Inc. Unauthorized reproduction of this manual is prohibited. This manual is being used during
review sessions conducted by PharmacyPrep. 88-1
PharmacyPrep.com Congestive Heart Failure (CHF)
c) Heart size
d) Heart rate
e) Residual diastolic volume

7. Symptoms of digitalis toxicity include all of the following EXCEPT


a) extra systoles
b) nausea and vomiting
c) yellow-green vision
d) A-V conduction block
e) Decreased PR interval

8. What is NOT a symptoms of congestive heart failure?


A) weight gain B) ankle edema C) dyspnea D) polyurea E)fatigue
9. A patient using hydrochlorothiazide 50 mg daily. All of the following conditions require precautions,
except?
A) gout
B) renal disease
C) diabetes
D) CHF
E) electrolyte imbalance
10) What is the drug of choice combination to treat heart failure?
A) Diltiazem and verapamil
B) Nitroglycerine and ACEi
C) ACEi and Beta blockers
D) ACEi and Digoxin
E) ACEi and Angiotensin receptor blockers
Ans. C
11) In CHF patient unable to tolerate ACEI. Doctors recommend isosorbide dinitrate (ISDN) (ISDN) and
hydralazine combination. What is correct?
I) ISDN have properties of venous dilating (preload)
II) Hydralazine has arterial dilating effect (after load)
III) ISDN and hydralazine have both preload and after load effect
A-I only B-III only C-I and II only D-II and III only E-All of the above
12. A 58 yo hypertensive man is currently being treated with ramipril, furosemide, digoxin, carvedilol,
nifedipine, ASA and nitroglycerin. He is admitted with diagnosis of HF. Which of the following drug most
likely to be discontinued in this patient.
A) ramipril B) carvedilol C) nifedipine D) ASA E) digoxin

Copyright © 2000-2014 TIPS Inc. Unauthorized reproduction of this manual is prohibited. This manual is being used during
review sessions conducted by PharmacyPrep. 88-2
PharmacyPrep.com Congestive Heart Failure (CHF)
13) Acute management of digitalis toxicity?
I) cholestyramine resin
II) Fab fragment antibody
III) Potassium administration
A-I only B-III only C-I and II only D-II and III only E-I, II, III

Copyright © 2000-2014 TIPS Inc. Unauthorized reproduction of this manual is prohibited. This manual is being used during
review sessions conducted by PharmacyPrep. 88-3
PharmacyPrep.com Congestive Heart Failure (CHF)
CORRECT ANSWERS
Congestive Heart Failure
1. B
2. C
3. C
4. B
5- D
+ +
Tips: Digoxin blocks Na /K ATpase, thus it causes +ve inotropic effect by increase free calcium ions, where as
it decreases heart rate, which is -ve chronotropic effect. It parasympathetic action cause vagomimetic
effects.
6. B
Tips: digoxin is indicated to treat CHF, its effects such as – ve chronotropic effects decrease heart rate, it also
decrease edema, heart size, however, it increases urine flow.
7. E
Tips: Digoxin prolongs PR interval.
8. D
9. D
Tips: thiazides cause HyperGLUC.
10. C
11. C
12. C
Tips: CCBs potential to produce -ve inotropic effect thus nifedipine is cautiously used in HF patients.
Amlodipine and felodipine has been studied however have no significant benefit in congestive heart failure.
Amlodipine has been less likely to cause a worsening in nonischemic HF.
13. E
Tips: Cholestyramine is also used to treat wash out of leflunomide, and chronic liver diseases like cholestasis.

Copyright © 2000-2014 TIPS Inc. Unauthorized reproduction of this manual is prohibited. This manual is being used during
review sessions conducted by PharmacyPrep. 88-4
PharmacyPrep.Com Arrhythmias

PHARMACY PREP
ARRHYTHMIAS
1-Which of the following are the most common cardiovascular side effect of beta blockers:
A-Arrhythmias
B-Bradycardia
C-Sexual impairment
D-Bronchoconstriction
E-Drowsiness
2-Beta blockers are indicated in hypertension, because it act by:
A-↓ cardiac output and ↓renin secretion
B-↓ heart rate and contractility as result ↓ O2 consumption
C-↓ AV conduction velocity
D-Decrease mortality
E -↑ cardiac output and ↓ renin secretion
3-All of the following are neuromuscular blockers EXCEPT:
A-Succinylcholine B-Pancuronium C-Atracurium D-Nicotine
E-Mivcurium
4) Absolute refractory period is?
A-Phase I to Phase 3 B)phase 0 to phase 1 C)phase 1 to phase 2 D)phase 2 to 3 E)none of the above
5) Relative refractory period
A-Phase 3 B)phase 0 C)phase 1 D)phase 2 E) phase 4
6) Digoxin is contraindicated in?
A-Congestive heart failure
B-Atrial arrhythmias
C-Ventricular arrhythmias
D-Atrial tachycardia
E-None of the above
7) What is incorrect antiarrhythmic drugs classification
+
A-Class 1a drugs are Na channel blocker speeds phase 0 depolarization
+
B-Class 1b drugs are Na channel blocker shortens phase 3 repolarization
+
C-Class 1c Na channel blocker significantly slow phase 0 depolarization
D-Class II beta blockers decrease phase 4 depolarization
+
E-Class III K channel blockers prolong phase 3 repolarization
2+
F-Class IV Ca channel shortens action potential
Ans. A
Tips: Class 1a N+ channel blocker slow phase 0 depolarizaion.

Copyright © 2000-2014 TIPS Inc. Unauthorized reproduction of this manual is prohibited. This manual is being used during
review sessions conducted by PharmacyPrep. 89-1
PharmacyPrep.Com Arrhythmias
8) Which of the following is NOT a monitoring for patient using amiodarone?
A) liver function tests B) eye exam C)chest x-ray D) complete blood count E) serum TSH
9) Which of the following drugs can cause retinopathy?
I) amiodarone II) hydrochloroquine III) Glyburide
A) I only B) III only C) I and II D) II and III E) I,II,III
10) Atrial fibrillation and flutter is risk factor for?
I) Transient ischemic attack II) stroke III)coronary artery disease
A) I only B) III only C) I and II D) II and III E) I,II,III
11) Atrial fibrillation and atrial flutter are?
A) Ventricular tachycardia
B) Supra ventricular tachycardia
C) Ventricular bradycardia
D) Supra ventricular bradycardia
E) Paroxysmal arrhythmias
12. A patient is receiving class I antiarrhythmic agents chronic basis complains fatigue, fever, joint pain,
suggestive of lupus syndrome. The patient is most likely receiving?
A) amiodarone B) lidocaine C) ciprofloxacin D) procainamide E) Flecainide
13. Drugs that from class III agents that gives Torse de pointes type of ventricular tachycardia?
A) Amiodarone B) quinidine C) Diltiazem D) Lidocaine
14. What drugs increase refractoriness in AV node, thus these drugs are used in acute treatment of atrial
fibrillations?
A) beta blockers B) verapamil C) Diltiazem D)Propranolol E) all
15) Quinidine is used to treat
A. hypertension
B. angina pectoris
C. atrial fibrillation
D. ventricular fibrillation
Ans: C
16) The most important drug in managing atrial flutter and fibrillation:
A. Digitalis
B. Amlodipine
C. Doxazosin
D. Quinine
Ans: A

Copyright © 2000-2014 TIPS Inc. Unauthorized reproduction of this manual is prohibited. This manual is being used during
review sessions conducted by PharmacyPrep. 89-2
PharmacyPrep.Com Arrhythmias
17) The drug of choice for emergency IV therapy for arrhythmias is:
A. Lidocaine
B. Sodium bicarbonate
C. Dopamine
D. Adrenalin
Ans: A
18) It describes the electrical activity of the five phases:
A. Torsades de pointes
B. Myocardial Action Potential Curve
C. Depolarization
D. Repolarization
Ans: B
19) All are side effects of amiodarone except:
A. Photophobia
B. Pigmentation
C. Psychosis
D. Pneumonitis
Ans: C
Tips: Common side effects of amiodarone are 4P’s (Pigmentation, pneumonitis, pulmonary, photosensitivity)
20) Which drug causes QT prolongation (torse des pointes)?
A. Class I antiarrhythmic drugs
B. ClassII antiarrhythmic drugs
C. Class III antiarrhythmic drugs
D. Class IV antiarrhythmic drugs
Ans:C
Tips: Drugs and diseases that can lead to QT prolongation are Phenothiazines , haloperidol, Class III
antiarrhythmic drugs and patients with renal failure.
21. Atrial fibrillation. Which of wave appears frequently?
A) P wave
B) QRS wave
C) QT wave
D) T wave
E) U wave

Ans. A

Copyright © 2000-2014 TIPS Inc. Unauthorized reproduction of this manual is prohibited. This manual is being used during
review sessions conducted by PharmacyPrep. 89-3
PharmacyPrep.Com Arrhythmias
ANSWERS
1- B
Tips: Beta blocker side effects:
CVSBradycardia, AV blockade, CHF
CNSsedation and sleep alterations
Other impotence, exacerbation of asthma,
2- A
Tips: Beta blockers indications:
Hypertension↓ cardiac output and ↓renin secretion
Angina↓ heart rate and contractility as result ↓ O2 consumption
Supraventricular Tachycardia (SVT)↓ AV conduction velocity
MIDecrease mortality
3- D
Tips: Nicotine is ganglionic blockers
4) A
5) A
6) C
7) A
8) D
Tips: amiodarone monitored serum TSH to check hypo or hyperthyroidism symptoms. Eye exam is done to
check corneal deposits. Chest X-ray is used to check pulmonary toxicities like pneumonitis. No hematological
side effects of amiodarone thus no CBC monitoring required.
9) C
10) C
11) B
12. D
Tips: Drug that causes systemic lupus erythromatus are " HIPPP MCQ"
Hydralazine, isoniazid, procainamide, penicillamine, penicillin's m.... Carbamazepine, quinidine
13) A

Copyright © 2000-2014 TIPS Inc. Unauthorized reproduction of this manual is prohibited. This manual is being used during
review sessions conducted by PharmacyPrep. 89-4
PharmacyPrep.Com Arrhythmias
Tips: marked QT prolongation cause Torse de pointes a type VT. Drugs that cause TDP: amiodarone,
disopyrmide, dofetilide, ibutilide, procainamide, quinidine, sotolol. antibiotics (erythro, levofloxacin,
moxifloxacin) indapamide, paroxetine, sumatriptan, fluoxetine.
14. E
Tips: Drugs increase refractoriness in AV nodes, thus control the ventricular rate in AF. (cause bradycardia)
15. C
16. A
17. A
18. B
19. C
20. C
21. A

Copyright © 2000-2014 TIPS Inc. Unauthorized reproduction of this manual is prohibited. This manual is being used during
review sessions conducted by PharmacyPrep. 89-5
PharmacyPrep.Com Arrhythmias
Copyright © 2000-2014 TIPS Inc. Unauthorized reproduction of this manual is prohibited. This manual is being used during
review sessions conducted by PharmacyPrep. 89-6
PharmacyPrep.Com Peripheral Vascular Diseases

PHARMACY PREP
PERIPHERAL VASCULAR DISEASES
1) Aneurysm is ?
A) bulging of arteries B) a blood clot c) a moving blot clot d)permanent enlargement of alveoli E) None of the
above
2) Thrombus is?
A) bulging of arteries B) a blood clot c) a moving blot clot d)permanent enlargement of alveoli E) None of the
above
3) Embolus is ?
A) bulging of arteries B) a blood clot c) a moving blot clot d)permanent enlargement of alveoli E) None of the
above
4) Symptoms of Reynaud phenomenon?
a) fingers spasm b) extremities turn to pale, or blue C)often in elderly D)all of the above
5) Symptoms of Intermittent claudication?
a) cold feet, B)pulses are absent, c) exercise induced cramping pain d)all of the above
6) Symptoms of Stroke?
A)headache B)confusion C)dizziness D)blurred vision E)all of the above
7) What are the symptoms of deep vein thrombosis?
A) Leg pain
B) swollen legs
C) tenderness
D) Calf pain
E) all of the above
Ans: e
8) What is treatment of deep vein thrombosis?
A) warfarin, b) heparin, c) LMWH D) thrombolytic E)All of the above
9) Drug of choice against the prophylaxis of transient ischemic attack?
A)ASA 81 mg chewable B)ASA 81 mg enteric coated C)Alteplase injection D)Warfarin E)heparin
10. Which of the following monitoring parameter is essential for patient who currently taking Low molecular
weight heparin:
A-International normalized ratio (INR)

Copyright © 2000-2014 TIPS Inc. Unauthorized reproduction of this manual is prohibited. This manual is being used during
review sessions conducted by PharmacyPrep. 90-1
PharmacyPrep.Com Peripheral Vascular Diseases
B-Prothrombin time
C-Heparin assay
D-Sensitive TSH
E-provide predictable response, thus no monitoring required
11. Which of the following are the monitoring parameters of warfarin therapy?
I Complete blood count (CBC)
II PT (prothrombin time)
III INR (international normalization ratio)
a. I only
b. III only
c. I and II
d. II and III
e. I, II and III

12-Which of the following drugs increase international normalized ratio (INR)?


A-Clotrimazole
B-Clindamycin
C-Azithromycin
D-Penicillins
13) Which of the of the following can be used in pregnancy?
A-Captopril
B-Warfarin
C-Atorvastatin
D-Heparin
E-Rosuvastatin
14) Which of the following is NOT a low molecular weight heparins (LMWH)?
A-Dalteparin B-Heparin C-Nadoparin D-Enoxaparin E-Tinazeparin
15) What is incorrect about Reynaud's phenomenon?
A- Reynaud's phenomenon in which arteries of fingers become spastic (vasospastic).
B-Reynaud's phenomenon may result from atherosclerosis, connective tissue disease, ingestion of ergot
alkaloids, or frequent use of vibrating tools.
C- Reynaud's phenomenon may result from peripheral artriel plaques formation
D) Reynaud's phenomenon symptoms fingers spasm, extremities turn to pale, or blue.
E- Reynaud's phenomenon non-pharmacological is minimizing cold exposure, and use warm gloves
16)Which of the following the most important drug is used to prevent peripheral vascular diseases?
A)Beta blocker B)ACEI C) warfarin D)digoxin E)ASA

Copyright © 2000-2014 TIPS Inc. Unauthorized reproduction of this manual is prohibited. This manual is being used during
review sessions conducted by PharmacyPrep. 90-2
PharmacyPrep.Com Peripheral Vascular Diseases
17) Which of the following the most important drug is used to treat peripheral vascular diseases?
A)Beta blocker B)ACEI C) warfarin D)digoxin E)ASA
18) Which of the following drug is least likely used to treat peripheral vascular diseases?
A)Beta blocker B)ACEI C) warfarin D)digoxin E)ASA
19) Which of the following is incorrect statement?
A)Reynaud phenomenon symptoms occurs in limb extremities
B)Calcium channel blockers are the drug of choice
C)Arterial plaques are the risk factor for Raynaud phenomenon
D)Reynaud phenomenon can be trigger cold exposure
E)Reynaud phenomenon is may occur in old age
20) Which of the following drugs have been withdrawn due to risk of blood clots in peripheral vascular
system?
A) Enoxaparin B) Alteplase C) Yazmin D) Alesse E) Dain 25
21) Which of the following drugs least likely used in DVT patients?
A) warfarin B) Dabigatran C) LMWH D) heparin E) cardio selective beta blockers
22) All are risk factors of deep vein thrombosis (DVT), except?
A) pregnancy
B) cancer and chemotherapy
C) Progestin only OCP
D) diabetes
E) mobility
23) A 64 yo man weighing 115 kg, and tall 60 inches, presents to his physician after long international flight
complaining of pain, swelling of his right lower leg. The patient had knee surgery a month before he
travelled. His medication profile include, CHF (ejection fraction < 15%), remission from lymphoma, MI. His
father, mother, and sister are all diseased from stroke, pulmonary embolism. Given his profile. Patient most
likely suffering from?
A) deep vein thrombosis
B) coronary artery disease
C) congestive heart failure
D) diabetes
24) Which of the following is NOT risk factors of deep vein thrombosis (DVT)?
A) Knee surgery B) over weight C) long flight D) family history E) gender
25) Patient is hospitalized. Doctors initiated heparin iv infusion. What is incorrect about heparin

Copyright © 2000-2014 TIPS Inc. Unauthorized reproduction of this manual is prohibited. This manual is being used during
review sessions conducted by PharmacyPrep. 90-3
PharmacyPrep.Com Peripheral Vascular Diseases
A) heparin dose is based on body weight
B) loading dose is 80 units/kg, and maintenance dose is 15 to 25 units/kg/hr.
C) Heparin is slower onset than LMWH
D) LMWH have longer half life
E) LMWH have predictable response thus not monitored
26) Over dose of heparin is treated by?
I) Stop heparin infusion
II) Protamine sulfate
III) oral vitamin K
a) I only
b) III only
c) I and II
d) II and III
e) I, II, III

27) Patient is receiving warfarin after discharged from hospital. He is receiving cotrimoxazole to treat UTI.
One may expect to see the INR (2 - 3)?
I) Increase II) No change III) Decrease
a) I only
b) III only
c) I and II
d) II and III
e) I, II, III

28. What anticoagulant is used in emergency as iv dosage form?


A) Heparin
B) Low molecular weight heparins (LMWH)
C) Warfarin
D) Dabigatran
Ans-A
29. The mechanism of action of aspirin that results in greater effect as an antiplatelet drug is because of:
A. Selective COX I inhibitor
B. Selective COX II inhibitor
C. COX I and COX II inhibitor
D. Acetylate cyclooxygenase
Ans: D
Tips: Aspirin is the only drug that irreversibly inhibits the enzyme. This action permits a longer duration of
action causing platelet aggregation and acetylation of cyclooxygenase.
30) Which of the following anticoagulant available as iv and sc injections
A. Heparin

Copyright © 2000-2014 TIPS Inc. Unauthorized reproduction of this manual is prohibited. This manual is being used during
review sessions conducted by PharmacyPrep. 90-4
PharmacyPrep.Com Peripheral Vascular Diseases
B. LMWH
C. Warfarin
D. A&B
Ans: A
31) What should be monitored in heparin treatment?
A. INR (International Normalised Ratio)
B. Heparin assay
C. aPTT
D. All of the above
Ans-A
32) Which of the following tests are used for antithrombotic therapy:
A. INR
B. aPTT
C. Heparin assay
D. B&C
Ans-D
Tips: heparin and LMWH may be appropriately monitored by aPTT and heparin assay. LMWH cannot be
reliably monitored by aPTT and therefore heparin assay can be evaluated.
33) Which of the following can be used in treatment of deep vein thrombosis (DVT):
A) warfarin
B. streptokinase
C. Heparin
D. All of the above
Ans-D
Tips: warfarin, heparin, thrombolytic agent may be used in DVT.
34) Which of the following medication can be used in pulmonary embolism and deep vein thrombosis:
A. Heparin
B. Warfarin
C. Streptokinase
D. A&B
Tips: Heparin: PE, DVT; warfarin PE, DVT, MI, streptokinase, DVT
35) In order to measure effectiveness of warfarin therapy, the following monitoring tests are used:
A. INR, must be 2-3
B. Prothrombin test (PT)
C. aPTT
D. A&B
Ans-D

Copyright © 2000-2014 TIPS Inc. Unauthorized reproduction of this manual is prohibited. This manual is being used during
review sessions conducted by PharmacyPrep. 90-5
PharmacyPrep.Com Peripheral Vascular Diseases
36) At what dose Aspirin has anticoagulant property
A. 80 mg
B. 325mg
C. 500 mg
D. 800 mg
Ans-A
37) Effects of warfarin include all except:
A. Decreasing the formation of blood clot
B. Nose bleeds
C. Red-orange urine
D. Ringing in the ears
Ans: D
Tips: Ringing in the ear is associated with ASA, salicylates, quinine, and quinidine, but not warfarin.
38) All drugs will decrease INR of warfarin except:
A. Rifampin
B. Barbiturate
C. Green leafy vegetable
D. Metronidazole
Ans: D
Tips: Metronidazole increases INR
39) Warfarin site of action is:
A. In vivo
B. In vitro
C. In vivo and in vitro
D. Neither
Ans: A
Tips: Warfarin site of action is in vivo and heparin is either in vivo or in vitro
40) Which drug is not a LMW heparin?
A. Enoxaparin
B. Nadroparin
C. Heparin
D. Dalteparin
Ans: C
41) What is the safest anticoagulant in pregnancy?
A. Dalteparin
B. Warfarin
C. Heparinoids
D. Heparin

Copyright © 2000-2014 TIPS Inc. Unauthorized reproduction of this manual is prohibited. This manual is being used during
review sessions conducted by PharmacyPrep. 90-6
PharmacyPrep.Com Peripheral Vascular Diseases
Ans: D
42) Antidote for heparin toxicity:
A. Vitamin K
B. Protamine sulphate
C. BAL
D. Atropine
Ans: B
43) All of the following increases INR except:
A. ASA and NSAID
B. Chronic alcohol
C. Cotrimoxazole
D. Acute alcohol
Ans: B
Tips: Chronic alcohol decreases INR while acute alcohol increases INR.
44) Which of the following vitamins is needed for proper blood clotting?
A. Vitamin A
B. Vitamin D
C. Vitamin E
D. Vitamin K
Ans: D
45) Which of the following OTC drugs is contraindicated for a patient taking warfarin?
A. Diphenhydramine
B. Loratidine
C. Psyllium
D. ASA
Ans: D
46) All of the following drugs should be carefully prescribed with aspirin EXCEPT:
A. Enoxaprine B. Coumadin C. Heparin D. Metoclopropamide
Ans: D
Tips: Aspirin is a blood thinning agent indicated for prevention of heart stroke. It should be carefully
prescribed with other blood thinning agents because of risks of bleeding. Lovenox (Enoxaprine), Coumadin
(Warfarin), Heparin, Plavix (Clopidrogel), Ticlid (Ticlopidine), Depakene (Valproic acid), Persantine
(Dipyridamole), Mandol (Cefamandole), Cefotan (Cefotetan), Cefobid (Cefoperazone) and Moxam
(Moxalactam) are agents that increase bleeding tendency in patients and should be carefully prescribed with
other blood thinning agents.

Copyright © 2000-2014 TIPS Inc. Unauthorized reproduction of this manual is prohibited. This manual is being used during
review sessions conducted by PharmacyPrep. 90-7
PharmacyPrep.Com Peripheral Vascular Diseases
47) An overdose of Coumadin can be treated by administering:
A. Vitamin K B. Acetylcysteine C. Mesna D. Protamine sulfate
Ans: A
Tips: The overdose of Coumadin can be treated by administration of Vitamin K1 (Mephyton).

Copyright © 2000-2014 TIPS Inc. Unauthorized reproduction of this manual is prohibited. This manual is being used during
review sessions conducted by PharmacyPrep. 90-8
PharmacyPrep.Com Peripheral Vascular Diseases
CORRECT ANSWERS
Peripheral Vascular Diseases
1) A
2) B
3) C
4) D
5) D
6) E
7) D
8) E
9) A
10. E
Tips: INR and PT is monitored for warfarin, aPTT for heparin, where as for LMWH these tests are not effective
11. D
12- A
13) D
14) B
15) C
16) C
Tips: Anticoagulants like warfarin is commonly used to treat peripheral embolic disorders.
17) C
18) A
Tips: beta blocker cause vascular constriction thus it is contraindicated in peripheral vascular diseases

Copyright © 2000-2014 TIPS Inc. Unauthorized reproduction of this manual is prohibited. This manual is being used during
review sessions conducted by PharmacyPrep. 90-9
PharmacyPrep.Com Peripheral Vascular Diseases
19) C
20) C
21) E
22) E
Tips: mobility or physical activity lower the risk of deep vein thrombosis
23) A
24) E
Tips: RF for above patient, except gender. other RFs include age > 40 yr, cancer, CHF, MI, and obesity.
25) C
Tips: Heparin iv infusion is faster onset than LMWH.
26) E
27) A
Tips: antibiotics potentiate anticoagulant effect : cotrimoxazole, erythromycin, metronidazole, (other drugs:
omeprazole, fluconazole, ketoconazole, cimetidine, amiodarone, quinidine, vitamin E (large doses), acute
phenytoin, thyroxin, acute alcohol also potentiate.
Reduce the clearance of warfarin, displacement from warfarin protein binding, decrease intestinal flora
Drugs that inhibit anticoagulant effect: carbamazepine, penicillin, rifampin, cholestyramine, barbiturates,
high vitamin K, chronic alcohol use
28) A
29) Ans: D
Tips: Aspirin is the only drug that irreversibly inhibits the enzyme. This action permits a longer duration of
action causing platelet aggregation and acetylation of cyclooxygenase.
30) A
31) A
32) Ans-D
Tips: heparin and LMWH may be appropriately monitored by aPTT and heparin assay. LMWH cannot be
reliably monitored by aPTT and therefore heparin assay can be evaluated.
33) Ans-D

Copyright © 2000-2014 TIPS Inc. Unauthorized reproduction of this manual is prohibited. This manual is being used during
review sessions conducted by PharmacyPrep. 90-10
PharmacyPrep.Com Peripheral Vascular Diseases
Tips: warfarin, heparin, thrombolytic agent may be used in DVT.
34.
Tips: Heparin: PE, DVT; warfarin PE, DVT, MI, streptokinase, DVT
35) D
36) A
37) D Tips: Ringing in the ear is associated with ASA, salicylates, quinine, and quinidine, but not warfarin.
38) D
Tips: Metronidazole increases INR
39) A
Tips: Warfarin site of action is in vivo and heparin is either in vivo or in vitro
40) C
41) D
42) B
43) Ans: B
Tips: Chronic alcohol decreases INR while acute alcohol increases INR.
44) D
45) D
46) Ans: D
Tips: Aspirin is a blood thinning agent indicated for prevention of heart stroke. It should be carefully
prescribed with other blood thinning agents because of risks of bleeding. Lovenox (Enoxaprine), Coumadin
(Warfarin), Heparin, Plavix (Clopidrogel), Ticlid (Ticlopidine), Depakene (Valproic acid), Persantine
(Dipyridamole), Mandol (Cefamandole), Cefotan (Cefotetan), Cefobid (Cefoperazone) and Moxam
(Moxalactam) are agents that increase bleeding tendency in patients and should be carefully prescribed with
other blood thinning agents.
47) A
Tips: The overdose of Coumadin can be treated by administration of Vitamin K1 (Mephyton).

Copyright © 2000-2014 TIPS Inc. Unauthorized reproduction of this manual is prohibited. This manual is being used during
review sessions conducted by PharmacyPrep. 90-11
Pharmacy Prep
ANTICOAGULANTS
1-Which of the following clotting factor is NOT inhibited by warfarin?
A-2 B-7 C-9 D-10 E-12
2. A drug is said to reach "steady state" when 5 half-lives have elapsed. In the case of warfarin
(a blood thinner), however, the steady state is dependent not upon the drug's half-life, but upon
the half-life of the clotting factors it inhibits. The clotting factors inhibited by warfarin have the
following half-lives:
Factor II - 50 hours
Factor VII - 6 hours
Factor IX - 24 hours
Factor X - 36 hours
Based on this information, how long will it take a patient to reach maximal anticoagulation?
A- 5 days B-10 days C- 15 days D- 20 days E-48 hours
3-The hematocrit (HCT) measures the
A. Total number of blood cells per volume of blood
B. Weight of red blood cells per volume of blood
C. Number of red blood cells per volume of blood
D. Weight of hemoglobin per volume of blood
E. Percentage of red blood cells per volume of blood

4) A patient is using injections of Dalteparin to treat deep vein thrombosis what is monitored?
A) aPTT B) antifactor Xa C) Prothrombin time D) international normalized ratio E) Heparin assay
5) Which of the following oral anticoagulant approved to treat atrial fibrillation?
A) heparin B) dalteparin C)Tinzaparin D)Dabigatran E)all of the above
6) What is incorrect statement?
A) LMWH therapy are effective in preventing DVT
B) Heparin therapy require close monitoring than LMWH
C) Heparin is weight based intravenous therapy is used in acute DVT
D) Warfarin produce rapid anticoagulation than LMWH
E) Long term heparin therapy can cause osteoporosis side effect
7) What is incorrect statement?
A) LMWH have longer duration of action than heparin
B) LMWH once daily SC administration

Copyright © 2000-2014 TIPS Inc. Unauthorized reproduction of this manual is prohibited. This manual is being used during
review sessions conducted by PharmacyPrep. 91-1
C) LMWH do not require routine monitoring
D) Heparin has rapid anticoagulant effect
E) aPTT reflects changes in extrinsic pathway of the coagulation cascade
ANSWERS:
1- E
2. B
Tips: “maximal” anticoagulation will be reached when all clotting factors are inhibited. This will take
approximately 10 days, since 50hrs (the half-life of the clotting factor taking the longest time to reach
steady) x 5 = 250 hours, which is 10.4 days.
3- E
Tips: Whole blood treated with anticoagulant is centrifuged in a calibrated hematocrit tube. The volume
ratio of the packed red blood cells to total blood volume is determined. The hematocrit is normally 39 to 49
for men and 33 to 43 for women. The hematocrit value provides some indication of both the number and
size of the red blood cells present in an individual
4) B
5) D
Tips: Dabigatran an oral anticoagulants recently approved to treat atrial fibrillations. It is factor Xa inhibitor.
(competitive direct thrombin inhibitor).
6) D
7) E

Copyright © 2000-2014 TIPS Inc. Unauthorized reproduction of this manual is prohibited. This manual is being used during
review sessions conducted by PharmacyPrep. 91-2
PharmacyPrep.Com Anxiety Disorders

PHARMACY PREP
ANXIETY DISORDERS
1) Ms X is a 32-year old woman, who comes into your pharmacy to seek help on finding a “good” lotion for
her hands. You noticed that her hands are extremely chapped and raw. In order to make the best product
suggestion to her, you ask her more questions to determine the cause. After some discussion, she admits to
you that she washes her hands repeatedly throughout the day, sometimes for 20 minutes at a time. When
asked why, she says she feels she must.
Which of the following terms most likely describes her repeated washing?
a. An obsession
b. A compulsion
c. An addiction
d. A phobia

2-What is the drug of choice for generalized anxiety disorder (GAD)?


A-Haloperidol B-Paroxetine C-Amitriptyline D-Phenytoin E-Benzodiazepine
3) Paroxetine is used to treatment of?
I-Social anxiety disorder
II-Major depression
III-Manic depression
A-I only B-III only C-I and II only D-II and III only E-All are correct

Copyright © 2000-2014 TIPS Inc. Unauthorized reproduction of this manual is prohibited. This manual is being used during
review sessions conducted by PharmacyPrep. 92-1
PharmacyPrep.Com Anxiety Disorders
CORRECT ANSWERS
Anxiety
1) B
Tips: Obsession = repetitive imagination and compulsion = repetitive behavior
2- B
3) C

Copyright © 2000-2014 TIPS Inc. Unauthorized reproduction of this manual is prohibited. This manual is being used during
review sessions conducted by PharmacyPrep. 92-2
PharmacyPrep.Com Anxiety Disorders
Copyright © 2000-2014 TIPS Inc. Unauthorized reproduction of this manual is prohibited. This manual is being used during
review sessions conducted by PharmacyPrep. 92-3
PharmacyPrep.com Depression

PharmacyPrep.com Depression

PHARMACY PREP
DEPRESSION
1-All of the following drugs are categorized as selective serotonin receptor inhibitor (SSRI’s) except:
A-Fluvoxamine B-Fluoxetine C-Sertraline D-Citalopram E-Mirtazapine
2) Which of the following drug is categorized a selective serotonin reuptake inhibitor (SSRI):
A-Bupropion B-Citalopram C-Venlafaxine D-Nortriptyline
E-Mirtazapine
3) Which of the following drug is categorized as a serotonin norepinephrine reuptake inhibitor (SNRI)?
A-Bupropion B-Citalopram C-Venlafaxine D-Nortriptyline
E-Mirtazapine
4) Which of the following is a reversible non selective MAO Inhibitor
A-Bupropion
B-Selegiline
C-Venlafaxine
D-Nortriptyline
E-Moclobemide
Tips: E
Reversible non-selective MAOi = Moclobemide
Irreversible non-selective MAOi = phenelzine and tranycypromide
Irreversible Type – B MAOi = Selegiline
5) All of the following factors that increase lithium concentrations and toxicity, EXCEPT
A-Dehydration
B-Sodium loss
C-Excessive caffeine intake
D-Haloperidol
E-ACE inhibitors
23) Which of the following is NOT side effects of SSRIs?
A) GI bleeding B) Insomnia C) Nausea and vomiting D) Sexual dysfunction E) Weight loss
7) Examples of serotonin norepinephrine reuptake inhibitors (SNRIs) include
I)Venlafaxine II) Duloxetine III) Sibutramine
A) I only b) III only c) I and II only d) II and III only e) I, II, and III

Copyright © 2000-2014 TIPS Inc. Unauthorized reproduction of this manual is prohibited. This manual is being used during
review sessions conducted by PharmacyPrep. 93-1
PharmacyPrep.com Depression
8) Which of the following drug can be stopped without tapering
A-Fluoxetine B-Fluvoxamine C-Sertraline D-Citalopram E-Paroxetine
9) All of the following drugs require waiting period for 2 weeks, before begin other antidepressants, except;
A-Fluvoxamine B-Sertraline C-Citalopram D-Fluoxetine E-Venlafaxine
10) Which of the following most appropriate dose range of lithium in treatment of acute mania?
I-0.6 – 0.8 mEq/L II-0.2-0.8 mEq/L III-0.8-1.5mEq/L
A-I only B-III only C-I and II only D-II and III only E-I,II,III
11) Which of the following are inappropriate combinations therapy?
I) phenelzine and pseudoephedrine II) phenelzine and amitriptyline III) phenelzine and milk
A-I only B-III only C-I and II only D-II and III only E-I,II,III
12) Moclobemide is?
A) Irreversible MAOI type A
B) Reversible MAOI type A
C) irreversible MAOI type A and B
D) Reversible MAOI type A and B
Ans. D
13) Which of the following antidepressants increase risk of bleeding when combined with NSAIDS?
A) Amitriptyline
B) Paroxetine
C) Nortriptyline
D) Phenelzine
E) Bupropion
14) What is incorrect statement?
A) SSRI have advantage of daily single dose
B) SSRI have more safety than TCAs
C) SSRI more cost effective
D) SSRI have lower SEs than TCAs
E) SSRI have lower DIs than TCAs
15) A 30 yr old women presents with depression symptoms such as
low mood, increased appetite, weight gain, somnolence and suicidal thoughts. If this symptoms are first
episode. What antidepressants are most appropriate?
A) Mirtazapine B) Sertraline C) Bupropion D) Venlafaxine E) Duloxetine

Copyright © 2000-2014 TIPS Inc. Unauthorized reproduction of this manual is prohibited. This manual is being used during
review sessions conducted by PharmacyPrep. 93-2
PharmacyPrep.com Depression
16) What drug is least preferred?
A) Mirtazapine B) Sertraline C) Bupropion D) Venlafaxine E) Duloxetine
17) What is sertraline monitored?
A) dyskinesia B) blood pressure C) fluid retention D) improve quality of sleep E) serum levels
18) A patient with major depression, doctor initiated fluoxetine. However patient did not tolerated and there
was no improvement. Doctor initiated phenelzine, a week later. After patient started therapy for phenelzine,
4 days later the patient presented with fever, high blood pressure, diarrhea, and confusion. What is the most
likely cause?
A) Neuroleptic malignant syndrome
B) Hypertension crisis
C) Serotonin syndrome
D) Serotonin withdrawal syndrome
19) A patient with bizarre dress, inappropriate make up, hair. Complains her delusions are bothering and
inability focus on work. She also feeling speechless, and cannot sleep sometimes for 2 to 3 days. Which of
the most likely cause?
A) Seizures B) Cognitive behavioral problem C) Mania D) Major depression
20) What is the drug of choice to treat mania?
A) Mirtazapine B) Sertraline C) Bupropion D) Venlafaxine E) Lithium
21) What serum level lithium give toxicity symptoms?
A) Lithium serum levels > 2 mEq/L
B) Lithium serum levels < 1.5 mEq/L
C) Lithium serum levels < 1 mEq/L
D) Lithium serum levels <0.5 mEq/L
22) Which of the following SSRI may cause least discontinued symptoms
A) Mirtazapine B) Sertraline C) Bupropion D) Venlafaxine E) Fluoxetine
23) What is true about lithium toxicity?
+
I-Dehydration and diarrhea may increase Li serum concentration
II-Lithium toxicity symptoms include, seizures change in heart rate, fluid retention
III-Lithium toxicity can occur if serum level over 1.5 mEq/L
A) I only b) III only c)I and II only d)II and III only e)I, II, and III
24) A regular customer of your pharmacy using phenelzine to treat major depression. Patient wants to
pseudoephedrine for congestion. What appropriate action of pharmacy technician.
A) Do not recommend because there is drug interaction with phenelzine
B) Do not recommend because there is contraindication
C) Do not recommend because pseudoephedrine is regulated product
D) Do not recommend because pseudoephedrine is NOT used for nasal congestion

Copyright © 2000-2014 TIPS Inc. Unauthorized reproduction of this manual is prohibited. This manual is being used during
review sessions conducted by PharmacyPrep. 93-3
PharmacyPrep.com Depression
25) Lithium carbonate is used to treat? Which of the following drugs has its primary use in the treatment of
the manic phase of depressive psychosis?
A. manic depression
B. Insomnia
C. Weight loss
D. Treatment of addiction

26) Several of the most commonly prescribed drugs are used for depression. Which of the following is not an
antidepressant?
A. Fluoxetine
B. Sertraline
C. Amitriptyline
D. Diazepam

27) An auxiliary label for SSRI anti-depressants, except


a) Take at bedtime
b) Avoid alcohol
c) Dizziness
d) Take morning

28) Auxiliary label for Amitriptyline include, except


a) Avoid sunlight
b) Dizziness
c) Avoid alcohol
d) Take morning

29) Which of the following medication related factors that may results in non adherence or stop taking
antidepressant.
A) Denial of condition
B) delayed therapeutic effect
C) Lack of symptom
D) Lack of knowledge

Copyright © 2000-2014 TIPS Inc. Unauthorized reproduction of this manual is prohibited. This manual is being used during
review sessions conducted by PharmacyPrep. 93-4
PharmacyPrep.com Depression
CORRECT ANSWERS
Depression
1) E
Tips: Fluvoxamine, fluoxetine, sertraline, Citalopram, escitalopram and paroxetine are SSRIs, Mirtazapine is
alpha2 blocker.
2) B
3) C
4) E
5) D
Tips: Lithium concentration may be increased by dehydration, sodium loss by diuretics and ACE inhibitors,
where as SSRI may cause serotonergic syndrome. Thereby drug that increase Li concentration and SSRI
require precaution when combined together. Caffeine act like diuretics and cause loss of sodium ion.
6) E
Tips: SSRI can cause GI bleeding, and insomnia, TCA may cause sedation. However weight gain is SE
7) E
8) A
Tips: Fluoxetine has long half-life, therefore it does not require tapering where as other SSRI have short half-
life require tapering.
9. D
Tips: Fluoxetine has long half life require 5 weeks washout period, where as all other SSRIs require 2 weeks
washout period
10) B
Tips: acute maniac treatment lithium serum levels should be 0.8 mEq/L to 1.5 mEq/L
11) C
12. D
Tips: Moclobemide is reversible inhibitor of MAO (RIMA)
13) B
14) E

Copyright © 2000-2014 TIPS Inc. Unauthorized reproduction of this manual is prohibited. This manual is being used during
review sessions conducted by PharmacyPrep. 93-5
PharmacyPrep.com Depression
15) B
16) A
17) B
Tips: Mirtazapine (nonadrenergic/serotonergic antidepressant) common SEs are increased weight gain,
somnolence. TCA like amitriptyline common SEs are weight gain, anticholinergic SEs.
18) C
Withdrawal symptoms Tips: Serotonin symptoms
Flu like fever, chills, Diarrhea, Fever, ANS
dizziness, light headed, Shivering, change in
N &V, sleep BP, and & N, V
disturbances
Myalgia, lethargy, Tremors, seizure, Neuromuscular
sensory disturbances myoclonus, hyper Dysfunction
reflexes
Not present Agitation, Confusion, Cognitive
hypomania
PharmacyPrep.Com Antipsychotics

PHARMACY PREP
ANTIPSYCHOTICS
1) What is the mechanism of clozapine?
A-Inhibit Dopamine receptors only
B-Inhibit D2, more inhibition of 5HT2, and little effects on alpha, and H1 and M receptors
C-Inhibit 5HT receptors only
D-Increase dopamine action
E-None of the above
2-What antipsychotics should be monitored for white blood cells (WBC)?
A-Olanzapine B-Risperidone C-quetiapine D-clozapine E-Haloperidol
3-What is antipsychotic are used for hallucinations, delusions and agitation.
A-clozapine
B-haloperidol
C-risperidone
D-olanzapine
E-quetiapine
4-The most potent D2 inhibitor anti-psychotics is?
a-clozapine
b-haloperidol
c-risperidone
D-olanzapine
5-The most common Extrapyramidal symptoms associated with?
a-clozapine
b-haloperidol
c-Risperidone
D-Olanzapine
6-Antipsychotic side effects, except
A-EPS
B-Galactorrhea
C-Hyperprolactenemia
D-orthostatic hypotension
E-depression
7-Antipsychotics that preferred in Parkinson's patients?
A-quetiapine B-clozapine C-haloperidol
D-carbamazepine

Copyright © 2000-2014 TIPS Inc. Unauthorized reproduction of this manual is prohibited. This manual is being used during
review sessions conducted by PharmacyPrep. 94-1
PharmacyPrep.Com Antipsychotics
8-A senior of 82 year age in senior home, violent and fighting and hitting spouse what is drug of choice?
A-Clozapine
B-Quetiapine
C-Risperidone
D-Haloperidol
9-Drugs that give neuroleptic malignant syndrome, except
A-phenothiazine
B-haloperidol
C-Fluphenazine
D-Chlorpromazine
E-Bromocriptine
10-All of the following drug can give serotonin syndrome except?
A-Sebritamine (Meridia)
B-fluoxetine
C-Venlafaxine
D-Phenelzine
E-Risperidone
11-All drugs have withdrawal syndrome except?
A-lorazepam B-alcohol C-nicotine D-morphine E-Omeprazole
12) Second generation antipsychotic drugs primarily act on dopamine and serotonin are mainly indicated for:
A) Positive schizophrenia symptoms
B) Negative schizophrenia symptoms
C) Positive and negative symptoms
D) Depression and schizophrenia
E) Seizures and depression
13-What antipsychotics that is used to treat +ve and –ve schizophrenia symptoms
A-Clozapine
B-Haloperidol
C-chlorpromazine
D-Fluphenazine
E-Domperidone
14) Secondary cause of Parkinson’s include?
A-Amantadine B-Phenothaizine
C-Triazolam D-Hydrochlorthiazide
E-Nifedipine

Copyright © 2000-2014 TIPS Inc. Unauthorized reproduction of this manual is prohibited. This manual is being used during
review sessions conducted by PharmacyPrep. 94-2
PharmacyPrep.Com Antipsychotics
15) Drug induced parkinsonism (Neuroleptic induced) in 70 to 80% patients, gives extra pyramidal symptoms
of?
A-Stiffness
B-Short step & akinesia
C-Ramped handwriting & tremor
D-inappropriate posture of neck, trunk, and limbs
E- All of the above
16) Antipsychotic drugs may cause? I) Steven Johnson syndrome
II) Serotonin syndrome
III) Neuroleptic malignant syndrome
A-I only
B-III only
C-I and II only
D-II and III only
E-All of the above
17) Which of the following receptors are not blocked by antipsychotic drugs?
A) alpha B) dopamine C)serotonin D) histamine E)Acetylcholine
18) Which of the following is not monitored in patient using antipsychotic drug, risperidone?
A) WBC or CBC B) weight gain C)blood pressure D)cholesterol E) rigidity
19) What consideration require in patient using antipsychotic drugs?
I) side effects profile of drugs
II) age of patient
III) dosage form of drugs
A) I only B) III only C) I and II D) II and III E) All
20) A 60 year old hypertensive patient using risperidone to treat agitation, what is not a concern?
A) compliance B) high blood pressure C) high cholesterol D) age E) Parkinson's like symptoms
21) A 22 yo man diagnosed as schizophrenic. What is treatment of choice for first episode aggression,
combative, fighter of schizophrenia? Patient is weight 210 lbs, and height is 60 cm.
A) Risperidone
B) Clozapine
C) Thiothixene
D) Chlorpromazine
E) Haloperidol

Copyright © 2000-2014 TIPS Inc. Unauthorized reproduction of this manual is prohibited. This manual is being used during
review sessions conducted by PharmacyPrep. 94-3
PharmacyPrep.Com Antipsychotics
22) A patient is on haloperidol for the past 3 weeks presents with tremor, muscle stiffness, and shuffling gait.
This most likely what type SEs of haloperidol?
A) Neuroleptic malignant syndrome
B) Extra pyramidal symptoms
C) Serotonin syndrome
D) Schizophrenia flare ups
23) Which of the following is NOT a extra pyramidal symptoms?
A) Akathisia (motor restlessness)
B) Pseudo parkinsonism (stiffness, rigidity, tremors)
C) Acute dystonia (sudden muscle spasm occurs within 24-48 hrs)
D) Tardive dyskinesia (late EPS various abnormal movements after months of therapy)
E) Seizures
24) A patient receiving haloperidol to treat schizophrenia. Presents with hyperthermia (fever), sweating,
tachycardia, muscle rigidity. Which of the following most likely cause?
A) Neuroleptic malignant syndrome
B) Serotonin syndrome
C) withdrawal symptoms
D) Extra pyramidal symptoms
25) A 29 yo women with schizophrenia (agitated, and depressed) is on olanzapine for the past 6 mo.
Currently she gained several pounds of weight. Which of the following preferred drug if doctor wants to
switch?
A)Risperidone B) Clozapine C) Decrease dose of olanzapine D) Quetiapine
26) Which of the following drug is NOT used treatment in the management of schizophrenia?
A. Clozapine and Risperidone
B. Haloperidol and Clozapine
C. Olanzapine and Quetiapine
D. Levodopa/Carbidopa

27) All of the following common issues with psychosis patient? Except
A) cognitive functions decrease
B) non-compliance increase
C) High occurrence of discontinuation of treatment
D) Addiction potential
28) Phenothiazines are used to
A. produce muscle relaxation
B. alter psychotic behaviour
C. suppress coughing
D. produce analgesia

Copyright © 2000-2014 TIPS Inc. Unauthorized reproduction of this manual is prohibited. This manual is being used during
review sessions conducted by PharmacyPrep. 94-4
PharmacyPrep.Com Antipsychotics
29) Which of the following is an IRREVERSIBLE side effect resulting from long-term administration of
phenothiazine antipsychotics?
A. Sedation
B. Xerostomia
C. Infertility
D. Tardive dyskinesia

nd
30) All of the following are 2 generation antipsychotics except:
A. Phenothiazine

B. Quetiapine

C. Clozapine

D. Risperidone

st
31) Which of the following is not the side effect of 1 generation antipsychotic drugs?
A. High sedation

B. High sexual dysfunction

C. High weight gain

D. High tardive dyskinesia

32) Which drug causes lupus like syndrome?


A. Olanzapine

B. Haloperidol

C. Trifluoroperazine

D. Chlorpromazine

Copyright © 2000-2014 TIPS Inc. Unauthorized reproduction of this manual is prohibited. This manual is being used during
review sessions conducted by PharmacyPrep. 94-5
PharmacyPrep.Com Antipsychotics
CORRECT ANSWERS
Psychosis and antipsychotics
1- B
Tips: Mechanism of clozapine is inhibitor of D2, D4, 5HT, H1, M and α1 receptors
2- D
3- B
Tips: Hallucinations and delusions are positive schizophrenia symptoms thus drug of choice is haloperidol.
4- B
5- B
st
Tips: Highest EPS are associated with 1 generation antipsychotic drugs haloperidol
nd
Least extra pyramidal symptoms 2 generation risperidone and quetiapine.
6- E
7- A
Tips: choose antipsychotics that have least extra pyramidal symptoms 2 generation risperidone and
nd

quetiapine.
8- B
nd
Tips: Drug of choice for acute agitation in seniors is Quetiapine. 2 gen (clozapine, olanzapine) increase risk of
lipids and diabetes, EXCEPT: Risperidone
9- E
10- E
11) E
12. C
Tips: the second generation antipsychotics i.e. olanzapine, risperidone, clozapine and quetiapine are mainly
st
indicated for the treatment of negative schizophrenia symptoms where as 1 generation antipsychotics i.e.
haloperidol, chlorpromazine, phenothiazines are mainly indicated for the treatment of positive schizophrenia
symptoms. Clozapine is indicated for +ve and –ve symptoms.
13. A
st nd
Tips: 1 generation 4 to 8 weeks no response, change to 2 generation. Severe case of psychosis
(schizophrenia) or bipolar disorder. For 2 episode, continue for 2 to 5 yrs.

Copyright © 2000-2014 TIPS Inc. Unauthorized reproduction of this manual is prohibited. This manual is being used during
review sessions conducted by PharmacyPrep. 94-6
PharmacyPrep.Com Antipsychotics
14) B
15) E
16) B
17) E
18) A
19) E
20) B
21) E
22) B
23) E
Tips: Extra pyramidal symptoms (EPS) Parkinson's (TRAP), Akathisia, Dystonia and Tardive dyskinesia
24)
Neuroleptic malignant Tips: NMS develops quickly
symptoms (NMS) and symptoms stay for 5 to 10
days. Require referral to
emergence. Extrapyramidal
symptoms (EPS)
Fever Parkinson's (TRAP)
Rigidity Akathisia
tachycardia Dystonia
Fluctuation BP Tardive dyskinesia
WBC, CK
PharmacyPrep,Com Dementia

PHARMACY PREP
DEMENTIA
1) Alzheimer is a progressive disease characterized by relentless and loss of mental function due to lack of:
A. Thiamine
B. Dopamine
C. Acetylcholine
D. Norepinephrine
E. Epinephrine

2. All of the following are symptoms of Alzheimer’s dementia, EXCEPT:


A. Slurred speech
B. Reduced cognitive functions
C. Fatigue
D. Reduced sexual function
E. Incoherent speech

3) All of the following are symptoms of Alzheimer's dementia, except?


A) loosing items
B) impaired perception of time
C) Hallucinations and dilutions
D) Depression
E) able to execute functions
4. Risk factors of Alzheimer’s dementia
A. High cholesterol
B. Smoking
C. Use of alcohol
D. Obesity
E. All of the above

5) All of the following are risk factors of Alzheimer's dementia, except?


A) Age B) Family history C) race D) coronary artery diseases E) sedentary life style
6) What is incorrect about Alzheimer’s dementia?
A)Alzheimer's dementia occurs due to ↓ Ach
B)the drug of choice against Alzheimer's dementia is donepezil
C)Delirium is acute agitation, lack of judgement and thinking.
D)Amnesia is short loss of memory
E) Old age is NOT a risk factors for Alzheimer's dementia

Copyright © 2000-2014 TIPS Inc. Unauthorized reproduction of this manual is prohibited. This manual is being used during
review sessions conducted by PharmacyPrep. 95-1
PharmacyPrep,Com Dementia
7. Drug of choice for dementia is?
A. Galantamine
B. Donepezil
C. Rivastigmine
D. Tacrine

8. Which of the following is classified as non selective reversible acetyl cholinesterase inhibitor?
A. Galantamine
B. Tacrine
C. Rivastigmine
D. A and B

9) What is/are the common side effects of Donepezil, Rivastigmine, and Galantamine?
A. Confusion
B. Anorexia
C. Disorientation
D. A and C

10) Drug that increase effectiveness of Donepezil:


A. Carbamazepine
B. Erythromycin
C. Phenytoin
D. All of the above

11) What drugs is/are not a treatment for Alzheimer's?


A. Rivastigmine
B. Donepezil
C. Memantine
D. Gingko Biloba
E. Galantamine

12) Which herbal product is used to treat Alzheimer and dementia?


A. Gingko biloba
B. Vitamin D
C. St. John Wort
D. Omega 3
E. Licorice

13. What is incorrect statement?


A) donepazil is selective acetylcholinesterase inhibitor
B) Rivastigmine is a acetylcholinesterase and butyryl cholinesterase inhibitor
C) Galantamine is acetylcholinesterase inhibitor and also effect on acetyl choline by binding with
nicotinic receptors

Copyright © 2000-2014 TIPS Inc. Unauthorized reproduction of this manual is prohibited. This manual is being used during
review sessions conducted by PharmacyPrep. 95-2
PharmacyPrep,Com Dementia
D) Memantine is NMDA receptor antagonist
E) None of the above

Copyright © 2000-2014 TIPS Inc. Unauthorized reproduction of this manual is prohibited. This manual is being used during
review sessions conducted by PharmacyPrep. 95-3
PharmacyPrep,Com Dementia
CORRECT ANSWERS
DEMENTIA
1) Ans: C
Tips: deficiency of acetylcholine and damage of brain cells in older age causes dementia.
2) C
3) E
4) E
Tips: Lifestyle factors like alcohol intake have no evidence worsening Alzheimer's.
5) C
6) E
Tips: old age is a risk factor for Alzheimer's dementia
7) Ans: B
8) Ans: A
9) Ans: B
10) Ans: A
Tips: Drugs that increases Donepezil are inhibitors of CYP 2D6 or CYP 3A4. Drugs that decreases Donepezil
are inducers of CYP 2D6 or CYP 3A4.
11) Ans: C
12) Ans: A
13) E

Copyright © 2000-2014 TIPS Inc. Unauthorized reproduction of this manual is prohibited. This manual is being used during
review sessions conducted by PharmacyPrep. 95-4
PharmacyPrep.com Seizure and antiepileptic drugs

PHARMACY PREP
SEIZURE & ANTIEPILEPTICS
1) What type of seizure symptoms in-patient is NOT unconscious?
A) Simple partial
B) Complex partial
C) Generalized seizures
D) Petit mal seizures
E) Absence seizures
2) Phenytoin is indicated the treatment of all of the following types of seizure EXCEPT:
A-Grand mal
B-Complex partial
C-Simple partial
D-Tonic clonic
E) Absence seizure (petit mal)
3)Which of the following drug is NOT used in absence seizure patient?
A-Phenytoin
B-Valproic acid
C-Diazepam
D-Ethosuximide
E-All of the above
4) Which of the following drug least likely to effect on oral contraceptives:
A-Tetracycline
B-Phenytoin
C-Carbamazepine
D-Gabapentin
E-Phenobarbital
5) Which of the following is the drug of choice in status epilepticus?
A-phenytoin iv
B-Carbamazepine iv
C-Diazepam iv
D-Infliximab
E-Valproic acid
6) Which of the following anticonvulsive drug treatment most likely cause kidney stone formation.
A-Carbamazepine
B-Phenytoin

Copyright © 2000-2014 TIPS Inc. Unauthorized reproduction of this manual is prohibited. This manual is being used during
review sessions conducted by PharmacyPrep. 96-1
PharmacyPrep.com Seizure and antiepileptic drugs
C-Tiagabine
D-Topiramate
E-Phenobarbital
7)The most common side effects of anti seizure drugs:
I-Appetite and body weight changes
II-GI symptoms
III-Head ache and dizziness
A-I only B-III only C-I and II D-II and III E-I, II, III
8) Which of the following antiepileptic drugs has weight loss side effects:
A-Valproic acid
B-valproate
C-Phenytoin
D-Topiramate
E-Carbamazepine
9) A 25-year-old female patient taking gabapentin for neuropathic pain, which of the following statement
is/are true:
I-She should avoid taking oral contraceptive because, this drug decrease effectiveness of gabapentin
II-Gabapentin chemical structure is not similar to GABA
III-Gabapentin chemical structure contains GABA structure.
A-I only
B-III only
C-I and II only
D-II and III only
E-All of the above
10) Which of the antiseizure drug has weight loss side effects.
A-Topiramate
B-Carbamazepine
C-Phenytoin
D-Valproic acid
E-Gabapentin
11) A 25-year-old female patient taking gabapentin for neuropathic pain, which of the following statement is
not true:
A-She should avoid taking oral contraceptive because, this drug decrease effectiveness of gabapentin
B-Gabapentin have advantage of the least drug interactions
C-Gabapentin chemical structure contains GABA structure
D-Gabapentin can be used seizures and neuralgia
E-Gabapentin is GABA analog

Copyright © 2000-2014 TIPS Inc. Unauthorized reproduction of this manual is prohibited. This manual is being used during
review sessions conducted by PharmacyPrep. 96-2
PharmacyPrep.com Seizure and antiepileptic drugs
12) What is incorrect about phenytoin?
A-Phenytoin available as suspension, iv, chewable tablets and capsule.
B-Always recommend chlorhexidine to treat deficiency of folic acid
C-Phenytoin blood levels should be monitored to 10 to 20 mcg/ml
D-Phenytoin have saturated kinetics, thus overdose can give toxicity
E-Chronic use of phenytoin can cause megaloblastic anemia
13) Symptoms of Steven Johnson’s Syndrome, include all except
A-Rash
B-Sores on mucus membrane,
C-Blistering mucus membrane, typically mouth, eyes and vagina.
D- Patchy areas rashes.
E-Butterfly rash
14-What is drug of choice in petit-mal seizure?
A-Diazepam
B-Phenytoin
C-Carbamazepine
D-Valproic acid and ethosuximide
E-Gabapentin
15-Valproic acid is used in treatment of
a) Petit-mal
b) Grand-mal
c) Febrile seizure in children
d) All of the above
e) A and C only

16-Anticonvulsive agent that cause weight loss?


A-Topiramate B-Phenytoin C-Carbamazepine D-Valproic acid
E-Gabapentin
17-Phenytoin causes elevation of blood glucose levels by?
A-Increasing insulin degradation
B-Decreasing insulin release
C-Increasing glucose absorption
D-increasing glycogen release
18-Side effects of phenytoin are?
A-Gingival hyperplasia
B-Steven Johnson syndrome
C-Hirsutism
D-All of the above
E-A and C

Copyright © 2000-2014 TIPS Inc. Unauthorized reproduction of this manual is prohibited. This manual is being used during
review sessions conducted by PharmacyPrep. 96-3
PharmacyPrep.com Seizure and antiepileptic drugs
19-Epilepsy cause due to deficiency of?
A-Sodium ion B-GABA C-Serotonin D-Dopamine E-Acetylcholine
20-Early symptoms of epilepsy are?
A-Part of brain is involved
B-Sensory seizure or focal motor seizure
C-Patient is conscious
D-All of the above
21-Which one of the following CNS receptors is directly coupled to an ion channels, and this do not involve
secondary messenger system?
A-Alpha (NE)
B-dopamine D2 (DA)
C-Acetyl choline (N)
D- serotonin (5HT)
E- None of the above
22-What are antimuscarinic drugs that are used in treatment of antipsychotic induced Parkinson’s disease,
A-Benztropine
B-Trihexphenidyl
C-Bromocriptine
D-Diphenhydramine
E-Procyclidine
23-What type of local anesthetic is the most effective?
A-The most hydrophilic B-The most lipophylic C-The least hydrophilic
D-The least lipophylic
24-Metabolism of local anesthetic takes place in?
A-Skin B-Tissue C-Liver D-Gut E-Local area
25) All of the following are GABA analogs except?
A-Gabapentin B-Pregabalin C-Vigabatrin D-Beclofen E-Phenytoin
26) Tachycardia is the side effect of?
A-Anticholinergic
B-Sympathomimetics
C-Sympathetic blockers
D-Non dihydropyridine type of CCBs
E-All of the above

Copyright © 2000-2014 TIPS Inc. Unauthorized reproduction of this manual is prohibited. This manual is being used during
review sessions conducted by PharmacyPrep. 96-4
PharmacyPrep.com Seizure and antiepileptic drugs
27) Treatment of epilepsy for?
A-Inhibiting GABA neurotransmitter
B-Stimulating NMDA neurotransmitter (N-methyl D-Aspartate)
C-Decrease firing threshold impulse
D-Stimulating sodium ion channels
E-Stimulating potassium ion channels
28) Valproic acid is used to treat?
I-Petit-mal (absent of seizure) II-Grand-mal (general seizure) III-Febrile seizure in children
A-I only B-III only C-I and II only D-II and III only E-I, II,III
29. When changing from phenytoin syrup to phenytoin capsule, at same dose, change in effect is due to
I. Change in salt form
II. Change in dose
III. Change in dose elimination

A-I only B-III only C-I and II only D-II and III only E-I, II,III
30) What is drug of choice for petit mal (absence) seizure?
A-Diazepam iv
B-Diazepam oral
C-Ethosuccimide and valproic acid
D-Phenytoin
E-Carbamazepine
31) Side effects of phenytoin include?
I-Gingival hyperplasia, ataxia, nystagmus
II-Steven Johnsons syndrome
III-Hirsutism and acne
A-I only B-III only C-I and II only D-II and III only E-I, II,III
32) Epilepsy is resulted from the deficiency of?--> GABA action; Early symptoms of partial seizures are?
I-Part of the brain
II-Sensory seizure or focal motor seizure
III-Patient is conscious
A-I only B-III only C-I and II only D-II and III only E-I, II,III
33)What are the common SEs of antiepileptic drugs?
A) Stevens Johnson Syndrome
B) Kidney stone formation
C)Head ache, dizziness, GI, appetite, weight gain,
D) Weight loss, euphoria

Copyright © 2000-2014 TIPS Inc. Unauthorized reproduction of this manual is prohibited. This manual is being used during
review sessions conducted by PharmacyPrep. 96-5
PharmacyPrep.com Seizure and antiepileptic drugs
34) A 25 yo women with one 2 yr old child. She has been using antiepileptic drugs for the past 1 year. Which
drug least likely decrease efficacy of antiepileptic drugs?
a) Phenytoin B) Carbamazepine C) Valproic acid D) Lamotrigine
35) MP is a 23 year old women. In her medication profile she is valproic acid daily for seizure. She calls to
your pharmacy and ask if she should stop taking her medication because for the past few weeks she has no
seizures? What is the common reason for medication non compliance in patient using antiepileptic
medications?
A) because of drugs taste bad
B) because recently had no seizure
C) because expensive medications
D) because complicated storage conditions
36) Ethosuximide is drug of choice for the following seizures:
A. Absence seizures
B. Tonic clonic
C. status epilepticus
D. A&B
37) Oral contraceptives interact with:
A. Antibiotics
B. Anticonvulsant
C. Antifungals
D. All of the above

38) What drugs should be avoided in absence seizure?


A. Phenytoin and valproic acid
B. Valproic acid and carbamazepine
C. Gabapentin and Phenytoin
D. Topiramate and Ethosuximide
E. Phenytoin and Carbamazepine

39) Drugs that has least likely have drug interaction with oral contraceptives except:
A. Gabapentin
B. Ethosuximide
C. Valproic acid
D. Lamotrigine

40) One side effect of Phenytoin is gingival hyperplasia, the best way to prevent it:
A. Use chlorhexidine wash
B. Use dental floss
C. Take Vitamin C
D. Gargle with water

Copyright © 2000-2014 TIPS Inc. Unauthorized reproduction of this manual is prohibited. This manual is being used during
review sessions conducted by PharmacyPrep. 96-6
PharmacyPrep.com Seizure and antiepileptic drugs
41) Anti epileptic drug that doesn’t cause weight gain?
A. Valproic acid
B. Lamotrigine
C. Topiramate
D. Phenobarbital

42) Phenytoin available in which of the dosage forms?


A. Injection
B. Capsule
C. Chewable tablets
D. All of the above

43) Valproic acid is the drug of choice for


A. Absence seizure
B. Myoclonus
C. Tonic
D. all of the above

44) Drug use as “add on” for patients nearly seizure free:
A. Gabapentin
B. Clobazam
C. Valproic acid
D. Phenytoin

45) Phenytoin metabolism is not inhibited by:


A. Sulfonamide
B. Isoniazid
C. Carbamazepine
D. Chloramphenicol

46) Divalproex is a mixture of:


A. Gabapentin
B. Valproic acid
C. Sodium valproate
D. A and C

47) What drug inhibits glutamate and aspartate release and blocks sodium channels and prevents repetitive
firing:
A. Topiramate
B. Lamotrigine
C. Clobazam
D. Gabapentin

Copyright © 2000-2014 TIPS Inc. Unauthorized reproduction of this manual is prohibited. This manual is being used during
review sessions conducted by PharmacyPrep. 96-7
PharmacyPrep.com Seizure and antiepileptic drugs
48) What antiepileptic is found in hospital emergence carts?
A. Diazepam iv
B. Chlorpromazine
C. Carbamazepine
D. Lithium
E) Phenytoin IV
ANSWERS:
1) A
2. E
Tips: Grand malValproic acid, Complex partial Carbamazepine, Simple partial Carbamazepine,
Tonic-clonicValproic acid, Absence seizure Ethosuximide, and Status epilepticus Diazepam
3) A
4) D
5) C
6) D
Tips: topiramate is a derivative of fructose. Indicated as anti seizure drug—partial seizure, tonic clonic
seizures. Has high kidney stone formation side effect. Fibrates also causes gall bladder stone formation
7) E
Tips: valproic acid and valproate increase appetite and may results in weight gain, carbamazepine may cause
dizziness and confusion.
8) D
9) B
10) A
11) A
Tips: Gabapentin do not decrease efficacy of oral contraceptives and can be used along with oral
contraceptives.
12) B

Copyright © 2000-2014 TIPS Inc. Unauthorized reproduction of this manual is prohibited. This manual is being used during
review sessions conducted by PharmacyPrep. 96-8
PharmacyPrep.com Seizure and antiepileptic drugs
13) E
Tips: butterfly rash symptoms occurs in lupus erythromatus
14- D
15- D
16- A
17- B
18- D
19- B
20- D
21- C
Tips; Ach receptors, in the CNS are present on less than 5% of the neuronal population.
22- A and B
23- B
24- C
25) E
26) A
27) A
28) C
Tips: valproic acid is the drug of choice for generalized seizures and also can be used for petit mal seizures.
29. A
Tips: Phenytoin syrup 100mg- more potent 8% increase in drug content; Phenytoin caps 100mg – free acid;
Sodium salt of Phenytoin – Dilantin caps.
30) C
31) C

Copyright © 2000-2014 TIPS Inc. Unauthorized reproduction of this manual is prohibited. This manual is being used during
review sessions conducted by PharmacyPrep. 96-9
PharmacyPrep.com Seizure and antiepileptic drugs
32) A
33) C
Drugs that cause SJS: SASPAN: Sulphonamides, antiepileptic drugs (CBZ)
34) C
35. B
Tips: Evidence shows there are two reasons seizure medication have compliance problems. 1) because of
side effects patient may not take regularly. 2) patient recently had no seizure.
Reference: Patient medication related non compliance. CE TT 2010 June
36. A
37. D
38. E
Tips: Phenytoin and Carbamazepine are not use in absence seizure
39. B
40. A
41. C
42. D
43. D
44. B
45. C
Tips: Carbamazepine stimulates phenytoin metabolism
46. D
47. B
48. A

Copyright © 2000-2014 TIPS Inc. Unauthorized reproduction of this manual is prohibited. This manual is being used during
review sessions conducted by PharmacyPrep. 96-10
PharmacyPrep.Com Parkinson’s Disease

PHARMACY PREP
PARKINSON’S DISEASE
1) Parkinson patients are characterized by;
A. Increased ratio of dopaminergic/cholinergic activity in neostratum
B. most commonly occurs in elderly over 40 years of age patient
C. Decreased ratio of dopaminergic/cholinergic activity in neostratum.
D. All of the above
2-Involuntary movements typically involves mouth, face, limbs and trunk is characterized as:
A-Trigeminal neuralgia
B-Tardive dyskinesia
C-Neuroleptic malignant syndrome
D-Extrapyramidal symptoms
E-Akathisia
3-Which of the following Catecholamine Methyl Transferase (COMT) inhibitor has fatal liver toxicity:
A-Selegiline
B-Tolcapone
C-Entacapone
D-Pergolide
E-Amantadine
4-A 60-year-old adult who has been treated with haloperidol for 3 weeks. He presents with muscle stiffness,
tremors. This is the most likely what kind of side effect:
A-Acute dystonia
B-Akathisia
C-Tardive dyskinesia
D-Parkinsonism disease
E-Pseudo parkinsonism
5-Which of the following is NOT used in treatment of Extrapyramidal side effects (EPS)?
A-Benztropine
B-Diphenhydramine
C-Trihexyphenidyl
D-Donepezil
E-All of the above
6-Selegiline is:
A-COMT inhibitor
B-MAO-B selective inhibitor
C-Non-selective MAO inhibitor

Copyright © 2000-2014 TIPS Inc. Unauthorized reproduction of this manual is prohibited. This manual is being used during
review sessions conducted by PharmacyPrep. 97-1
PharmacyPrep.Com Parkinson’s Disease
D-Levodopa preparations
E-MAO-A selective inhibitor
7) Which of the following is a COMT inhibitor?
A. Amantadine,
B. Bromocriptine,
C. Levodopa,
D. Selegiline
E. Entacapone

8) Why COMT inhibitors (Tolcapone, entacapone) always should used


with levodopa?
A-COMT inhibitor decrease metabolism of levodopa to 3-O-methyldopa
B-COMT inhibitors have synergistic action with levodopa
C-COMT inhibitors have additive action with levodopa
D-COMT inhibitors should not combine with levodopa
E-None of the above
9)All of the following statement is correct, except?
A-Deficiency of dopamine gives Parkinson’s disease
B-Tardive dyskinesia is NOT a symptom of Parkinson’s disease
C-Selegiline is a selective MAO- type B inhibitor
D-Tolcapone and entacapone are COMT inhibitor
E-Ropinarole and pramipexole are non-selective dopamine agonist
10-Tardive Dyskinesia is?
A-No movement or difficulty in movement
B-Involuntary movement of limbs, trunk, mouth and face
C-Slow movement
D-Rigidity
E-Postural instability
11)Which of the following antiviral drug indicated in Parkinson disease treatment:
A-Pergolide
B-Amantadine
C-Rimantadine
D-Selegiline
E-Entacapone
12) A 75 yo Parkinson's patient is on levodopa/carbidopa (Sinement) therapy taking morning and night time.
However, recently patient experiencing rigidity and involuntary movement in the evening before taking night
time dose? It is characterized as?
A) fluctuation or on-off effect B) Dyskinesia C) Tardive dyskinesia D) Dystonia E) wearing off effect

Copyright © 2000-2014 TIPS Inc. Unauthorized reproduction of this manual is prohibited. This manual is being used during
review sessions conducted by PharmacyPrep. 97-2
PharmacyPrep.Com Parkinson’s Disease
13) A doctor has entacapone therapy to a Parkinson's patient. Already is using levodopa/carbidopa
(Sinement SR).
A) to treat wearing off effect B) To treat tardive dyskinesia C) to treat side effect of levodopa D) to potentiate
action of levodopa therapy E) none
14) A patient is on levodopa/carbidopa therapy for the past 3 yrs. Recently patient is getting periods of poor
movements i.e return of tremor, rigidity or slowness. This conditions is referred as?
A) Tardive dyskinesia B) Akithesia C) Dystonia D) on-off effect or fluctuations E) Freezing
15) Which of the following drug extends the L-dopa effects?
A) bromocriptine B) Selegiline C) Ropinirole D) Pramipexole E) Carbidopa
16) Which of the following is NOT a Parkinson's symptoms?
A) tremors
B) rigidity
C) jerks
D) gait
E) Tardive dyskinesia
17) Which of the following drugs have levodopa sparing effect?
A) Bromocriptine
B) amantadine
C) pramipexole
D) Ropinirole
E) All of the above
18) Which of the following is NOT side effect of levodopa?
A) hypotension B) Arrhythmias C) peptic ulcer D) sudden onset of sleep E) pulmonary disease
19) Levodopa is taken empty stomach, what is correct?
A) take 30 min before meals
B) take 8 hours after meals
C) take 2 hours after meals
D) take 4 hours after meals
20) Hypofunction of nigrostriatal pathway resulting in decrease of dopamine levels and increase of
acetylcholine levels. The deficiency in dopamine would cause:
A. Alzheimer’s disease
B. Parkinson’s disease
C. Schizophrenia
D. Serotoninergic syndrome

Copyright © 2000-2014 TIPS Inc. Unauthorized reproduction of this manual is prohibited. This manual is being used during
review sessions conducted by PharmacyPrep. 97-3
PharmacyPrep.Com Parkinson’s Disease
21) Which of the following is the COMT inhibitor;
A. Tolcapone
B. Entracapone
C. Pergolide
D. A&B
22) Treatment of Parkinson’s disease may be treated with all of the following agents, EXCEPT:
A. Amantadine
B. Selegiline
C. Pergolide
D. Donezepil
E. Pramipexol

23) Pramipexole and Ropinirole are categorized as?


A. Selective D1 agonist
B. Non selective D1 and D2 agonist
C. Non selective D1 and D2 antagonist
D. Selective D2 agonist

24) The following antiviral drug can be used in PD treatment:


A. Levodopa
B. levodopa/carbidopa
C. Pergolide
D. Amantadine

25) What is the most common side effect of levodopa?


A. Tremor
B. Dyskinesia
C. Hallucination
D. Hypotension

26) MP is a 75 year old patient diagnosed with Parkinson's disease. Doctor considering to prescribe
medication. What is the drug of choice for Parkinson disease is?
A. Levodopa
B. Carbidopa
C. A and B
D. Dopamine

27) Which of the following drug should be avoided in Parkinson's disease?


A. Oral contraceptive pills
B. Metochlopramide
C. B and D
D. Antipsychotic drug

Copyright © 2000-2014 TIPS Inc. Unauthorized reproduction of this manual is prohibited. This manual is being used during
review sessions conducted by PharmacyPrep. 97-4
PharmacyPrep.Com Parkinson’s Disease
28. What is not a Parkinson disease symptom?
A. Tardive dyskinesia
B. Akathisia
C. Tremor
D. Bradykinesia

29) In treating parkinsonism, there is an increase of dopamine but on the other hand it will increase also
what neurotransmitter?
A. GABA
B. Acetylcholine
C. Norepinephrine
D. Epinephrine

30) Parkinson's disease symptoms characterized by difficulty in initiating movements is:


A. Akinesia
B. Bradykinesia
C. Dyskinesia
D. Anaesthesia

31) Over treatment of antiparkinson's drugs can result to:


A. Alzheimer's
B. Psychosis
C. Seizure
D. Respiratory arrest

32) An antinauseating drug that should be avoided in Parkinson patients:


A. Metoclopramide
B. Diclectin
C. A and B
D. Ondansetron

33) Which isn’t true about Parkinson drug?


A. Levodopa does penetrate brain
B. Carbidopa does not penetrate brain
C. Dopamine does not penetrate the brain
D. All of the above

34) How do you manage “wearing off”?


A. Switch to a different dopamine agonist
B. Stop the drug right away
C. Decrease dose of dopamine agonist
D. None of the above

Copyright © 2000-2014 TIPS Inc. Unauthorized reproduction of this manual is prohibited. This manual is being used during
review sessions conducted by PharmacyPrep. 97-5
PharmacyPrep.Com Parkinson’s Disease
35) Nausea and vomiting are adverse effects of Levodopa, how do you manage this?
A. Take levodopa with food
B. Take levodopa without food
C. Take it on empty stomach
D. Do not crash or chew

36) Which of the following is the suitable combination of drugs are initiated after as drug of choice for
Parkinson disease is?
A. Levodopa and carbidopa
B. Carbidopa and Dopamine
C. Levodopa and Tolcapone
D. Dopamine and Entacapone

37) Which of the following antiviral drug indicated in Parkinson disease treatment:
A. Valacyclovir
B. Amantadine
C. Acyclovir
D. selegiline
E. Interferons

38) Parkinson’s disease is caused by deficiency of?


A-Dopamine in brain
B-Acetylcholine
C-GABA in CNS
D-Serotonin in brain
E-Histamines

Copyright © 2000-2014 TIPS Inc. Unauthorized reproduction of this manual is prohibited. This manual is being used during
review sessions conducted by PharmacyPrep. 97-6
PharmacyPrep.Com Parkinson’s Disease
CORRECT ANSWERS
Parkinson's Disease
1- A
Tips: That is why Dopamine agonist & anticholinergics are effective
2- B
Tips: dyskinesia = slow movement, tardive dyskinesia = involuntary movement of mouth, face, limbs and
trunk. Akinesia = difficulty in movement.
3- B
4- E
Tips: Pseudo parkinsonism (TRAP) may be side effects of antipsychotics. Pseudo parkinsonism may disappear
after discontinuation of medication.
5- D
Tips: anticholinergic agents such as benztropine, diphenhydramine, and trihexyphenidyl are use in treatment
of EPS. Donepezil is acetyl cholinesterase inhibitor is not indicated in EPS treatment.
6- B
Tips: MAO-B selective inhibitors; treatment for Parkinson’s disease is: BALSAC = Bromocriptine, Amantadine,
Levodopa, Selegiline, Antimuscarinic (benztropine), and COMT inhibitors
7) E
8) A
9) E
Tips: Ropinirole and pramipexole are selective D2 agonist
10- B
11) B
12) E
13) A
14) D

Copyright © 2000-2014 TIPS Inc. Unauthorized reproduction of this manual is prohibited. This manual is being used during
review sessions conducted by PharmacyPrep. 97-7
PharmacyPrep.Com Parkinson’s Disease
Tips: End-of-dose wearing off gives on-off effect. This phenomenon is related to the increasing loss neuronal
storage capability of dopamine as well as short half life of dopa.
15) B
16) E
Which of the following is NOT side effect of levodopa?
A) hypotension B) Arrhythmias C) peptic ulcer D) sudden onset of sleep E) pulmonary disease
Ans: D
17) E
Tips: levodopa is more effective than dopamine agonist, amantadine, and MAOI, thus it is spared for older
ages.
18) D
19 A
20 B
21 D
22 D
23 D
24 D
25 B
26 C
27 C
28 A
29 B
30 A
31 B
32 A
33 D
34 A
35 A
36 C
37 B
38 A

Copyright © 2000-2014 TIPS Inc. Unauthorized reproduction of this manual is prohibited. This manual is being used during
review sessions conducted by PharmacyPrep. 97-8
Pharmacyprep.com Antimicrobial agents

PHARMACY PREP
ANTI-INFECTIVE AGENTS – ANTIBIOTICS
1- Cephalosporin mechanism of action may include:
a) Inhibit protein synthesis
b) Inhibit cell wall synthesis
c) Inhibit synthesis of essential metabolites
d) Alter the function of cell membrane permeability
e) Inhibit translation of genetic information
2- The bacteriostatic effect of sulphametoxazole may be increased by:
a) Penicillin
b) Sulbactam
c) Clavulanic acid
d) Tarzobactam
e) Trimetroprim
3-Correct statements regarding Penicillin V may include which of the following?
I-β-lactamase sensitive antibiotic – acid stable
II- Has a short half-life because is rapidly excreted by glomerular filtration
III- Probenecid increase penicillin’s activity
a) I only
b) III only
c) I and II only
d) II and III only
e) All are correct
4- The only antibiotic available as chewable tablet include:
a) Penicillin
b) Amoxacillin
c) Ampicillin
d) Erytromycin
e) Claritromycin
5- Correct statements regarding Erythromycin ESTOLATE may include:
I- Primarily effective against gram-positive bacteria and also have activity against some gram-negative
bacteria
II- It is a macrolid antibiotic very useful as alternative to penicillin for those that have allergy to penicillin
antibiotics
III- Regular erythromycin is better absorbed from GIT that erythromycin estolate
a) I only
b) III only
c) I and II only

Copyright © 2000-2014 TIPS Inc. Unauthorized reproduction of this manual is prohibited. This manual is being used during
review sessions conducted by PharmacyPrep. 98-1
Pharmacyprep.com Antimicrobial agents
d) II and III only
e) All are correct
6- Penicillin should be administrated 1 hour before meals or 2 to 3 hours after meals in order to:
I- Improve the bioavailability
II- Maximize the dissolution rate
III- Avoid interaction with calcium ions
a) I only
b) III only
c) I and II only
d) II and III only
e) All are correct
7-Drugs used in the treatment of meningitis are all of the following, EXCEPT:
a) Penicillins
b) Cephalosporins
c) Gentamycin
d) Streptomycin
e) Sulfonamides
8- Examples of antibiotic that is an acid stable penicillinase resistant include:
a) Amoxacillin
b) Oxacillin
c) Ampicillin
d) Carbenicillin
e) Penicillin V
9- All of the following penicillins are acid labile penicillins, EXCEPT:
a) Penicillin V
b) Penicillin G
c) Methicillin
d) Nafcillin
e) Carbenicillin
10-Which of the following are/is correct regarding tetracycline?
I- Inhibit protein synthesis
II- May be used in dental works
III- Cannot be taken for children less than 8 years old
a) I only
b) III only
c) I and II only
d) II and III only
e) All are correct

Copyright © 2000-2014 TIPS Inc. Unauthorized reproduction of this manual is prohibited. This manual is being used during
review sessions conducted by PharmacyPrep. 98-2
Pharmacyprep.com Antimicrobial agents
11- Which of the following describes the mechanism of action of sulphametoxazole?
a) Inhibit protein synthesis
b) Inhibit cell growth by competitive antagonism with folic acid synthesis
c) Inhibit synthesis of essential metabolites
d) Alter the function of cell membrane permeability
e) Inhibit translation of genetic information
12- Mechanism of inhibit protein synthesis is characteristic of all the following antibiotics, EXCEPT:
a) Aminoglycosides
b) Tetracycline
c) Cephalexin
d) Chlorafenicol
e) Clindamycin
13- Which of the following agents may interfere with the regular effect of penicillins?
a) Antacids
b) Food
c) Probenecid
d) Aminoglycosides
e) All of the above
14- Which of the following drugs cannot be taken orally?
a) INH
b) PABA
c) ASA
d) Streptomycin
e) Oxacillin
15- Example of drug used in prophylaxis of malaria include:
a) Cloroquine
b) Quinine
c) Primaquin
d) Quinin
e) Quinidine
16- Which of the following is NOT an anti-fungal drug?
a) Amphotericin B
b) Metronidazole
c) Grisofulvin
d) Nystatin
e) Clotrimazole

Copyright © 2000-2014 TIPS Inc. Unauthorized reproduction of this manual is prohibited. This manual is being used during
review sessions conducted by PharmacyPrep. 98-3
Pharmacyprep.com Antimicrobial agents
17- Examples of drugs that may inhibit DNA gyrase may include:
a) Gentamycin
b) Ceftriaxone
c) Norfloxacine
d) Erythromycin
e) Penicillin
18- Which of the following agents may best treat Meningococcal encephalitis?
a) Sulfametoxazole
b) Erythromycin
c) Cephalexin
d) Tetracycline
e) Penicillin
19- Correct statements regarding aminoglycosides may include:
I- Has a half-life of 32hours
II- The effect continue remain after all drug is eliminated
III- The relation plasma concentration and half-life is not clear
a) I only
b) III only
c) I and II only
d) II and III only
e) All are correct
20- Which of the following statement is/are correct regarding Gray Babies syndrome?
I- Occurs in premature and new born infants when choraphenicol is administrated during the first few days
of life
II- Due to inability of the infant to metabolize the drug because of glucoronyl transferase deficiency
III- Glucoronyl transferase is required to detoxify cloraphenicol
a) I only
b) III only
c) I and II only
d) II and III only
e) All are correct
21- Correct statements regarding Pellagra include:
I- Caused due to deficiency of niacin
II- Caused by failure in converting tryptophan to niacin
III- Characterized by dermatitis, diarrhea and dementia
a) I only
b) III only
c) I and II only
d) II and III only

Copyright © 2000-2014 TIPS Inc. Unauthorized reproduction of this manual is prohibited. This manual is being used during
review sessions conducted by PharmacyPrep. 98-4
Pharmacyprep.com Antimicrobial agents
e) All are correct
22) Gonorrhea, a sexually transmitted disease may be treated by which of the
following agents?
I) Cefixime
II) Ceftriaxone
III) Ciprofloxacin
a) I only
b) III only
c) I and II only
d) II and III only
e) All are correct
23- Nosocomial infections may be best treated by which of the following agents?
rd
I- Cephalosporin 3 generation
II- Macrolides antibiotics
III- Penicillins
a) I only
b) III only
c) I and II only
d) II and III only
e) All are correct
24- Viral encephalitis, a CNS infection, may be best be treated by which of the following agents?
a) Benzyl penicillin
b) Ceftriaxone
c) Acyclovir
d) Streptomycin
e) Vancomycin
25) Example of drug(s) used to treat pneumonia include:
I) Ampicilin combined with aminoglycosides
II) Chlaritromycin
III) Levofloxacin
a) I only
b) III only
c) I and II only
d) II and III only
e) All are correct
26- Otite extern, an ear infection may be best is best treated by:
a) Ciprofloxacin eardrop
b) Gentamicin eardrop

Copyright © 2000-2014 TIPS Inc. Unauthorized reproduction of this manual is prohibited. This manual is being used during
review sessions conducted by PharmacyPrep. 98-5
Pharmacyprep.com Antimicrobial agents
c) Prednisone eardrop
d) Levobunolol eardrop
e) Saline solution
27) Otite media is an ear infection normally caused by S. pneumoniae and may be the best treated by:
a) Amoxacillin
b) Penicillin V
c) Ampicilin
d) Amoxacillin clavulanate
e) Oxacillin
28- First line treatment for Urinary Tract Infection (UTI) may include:
a) Sulphametoxazole + Trimetroprine
b) Amoxacillin clavulanated
c) Penicillin
d) Erythromycin
e) All are very effective in treating UTI
29- Which of the following agents would be used in the treatment of infections caused by Helicobacter pylori
as combination therapy?
a) Penicillin, amoxacillin, ampicillin and metronidazole
b) Chlaritromycin, amoxacillin, omeprazole and metronidazole
c) Chlaritromycin, azithromycin and lansoprazole
d) Ceftriaxone and metronidazole only
e) Combinations of antibiotic should not be used due to resistance.
30- Treatment of large intestine infection mainly caused by food intoxication may include:
I- Lots of fluids
II- Ciprofloxacin
III- Metronidazole
a) I only
b) III only
c) I and II only
d) II and III only
e) All are correct
31- Colitis associated with antibiotic use can best be treated by:
a) Cephalosporins
b) Macrolides
c) Aminoglycosides
d) Vancomycin
e) Penicillin

Copyright © 2000-2014 TIPS Inc. Unauthorized reproduction of this manual is prohibited. This manual is being used during
review sessions conducted by PharmacyPrep. 98-6
Pharmacyprep.com Antimicrobial agents
32- Antibiotics that often cause colitis-acute inflammation of colon as side effect include:
I- Broad-spectrum penicillins
II- Cephalosporins
III- Clindamycin
a) I only
b) III only
c) I and II only
d) II and III only
e) All are correct
33- Drug used together with chloroquine in the treatment of malaria with the purpose to kill persistants
parasites in the liver of a person infected with Plasmodium vivax is:
a) Quinine
b) Pyrimethamine
c) Sulfadoxine
d) Primaquine
e) Quinidine
34- Patient is resistant to chloroquine and need to be treated for malaria. Which of the following drugs or
combination should be more appropriate for this patient?
a) Atovaquone only
b) Chloroquine and primaquine
c) Quinine and doxacycline
d) Mefloquine
e) C and D are correct
35- Correct statements regarding antibiotics may include:
a) Drugs derivated from synthetic substances that can kill or inhibit bacterial growth
b) Drugs derivated from natural and synthetic source that can kill or inhibit bacteria growth
c) Drug with ability only to kill bacteria
d) Drug with ability only to inhibit bacterial growth
e) All are wrong regarding antibiotics
36- Drugs derivated from synthetic substances that can kill bacteria is normally called:
a) Bacteriostatic antibiotic
b) Bactericidal antibiotic
c) Bactericidal antimicrobial
d) Bacteriostatic antimicrobial
e) Bacteriostatic bactericidal antimicrobial
37- Antiviral drugs act in many different ways, they can be classified in:
a) RNA inhibitors only

Copyright © 2000-2014 TIPS Inc. Unauthorized reproduction of this manual is prohibited. This manual is being used during
review sessions conducted by PharmacyPrep. 98-7
Pharmacyprep.com Antimicrobial agents
b) DNA inhibitors only
c) RNA and DNA inhibitors
d) Protease inhibitors
e) All are correct
38- Examples of very narrow spectrum penicillins include all of the following, EXCEPT:
a) Amoxacillin
b) Nafcillin
c) Metcillin
d) Oxacillin
e) All are very narrow spectrum penicillins
39- Examples of extended spectrum penicillin may include:
a) Amoxacillin
b) Ampicillin
c) Cloxacillin
d) Carbenicillin
e) Dicloxacillin
40- Resistance is the major problem with penicillins due to ability of bacteria to breakdown the lactam ring of
penicillin and inactivates the drug. To overcome these problem penicillins can be administrated together
with β-lactamase inhibitors.
Examples of β-lactamase inhibitors is/are:
I- Clavulanic acid
II- Sulbactam
III- Tazobactam
a) I only
b) III only
c) I and II only
d) II and III only
e) All are correct
41- Correct statements regarding CEPHALOSPORINS include:
I- Bactericidal antimicrobial
II- Derivated from natural source: Cephalosporium acremonium
III- Broad-spectrum agent
a) I only
b) III only
c) I and II only
d) II and III only
e) All are correct
42- Example of fourth generation cephalosporin include:
a) Ceftriaxone

Copyright © 2000-2014 TIPS Inc. Unauthorized reproduction of this manual is prohibited. This manual is being used during
review sessions conducted by PharmacyPrep. 98-8
Pharmacyprep.com Antimicrobial agents
b) Cefipime
c) Cefixime
d) Cefotaxime
e) Cefoxitin
43- Correct statement regarding the mechanism of action of Vancomycin may include:
a) Inhibit release of tRNA bound to peptyl site after peptide bond formation
b) Inhibit peptide bound formation between acylaminoacids bound to the peptyl and acceptor site
c) Inhibit stage II of cell wall synthesis preventing polymerization of the linear peptideoglycan
d) Inhibit DNA gyrase in the bacteria
e) Bind to ergosterol from pores resulting in linkage of cellular contents
44- Which of the following is one of the most common side effects of vancomycin?
a) Ototoxicity
b) Dissulfiran like side effect
c) Diarrhea
d) Weight gain
e) Hallucinations
45- All of the following are narrow spectrum protein synthesis inhibitors, EXCEPT:
a) Erythromycin
b) Azithromycin
c) Chlarithromycin
d) Tetracycline
e) Clindamycin
46) Which of the following is NOT a side effect of tetracycline?
a) Alteration in gut flora
b) Dissulfiran like reactions
c) Teeth permanent discoloration
d) Fanconi like syndrome
e) Hyperchloremic metabolic acidosis
47- Which of the following agents can be classified as macrolide antibiotics?
I- Azithromycin
II- Chlarithromycin
III- Erythromycin
a) I only
b) III only
c) I and II only
d) II and III only
e) All are correct

Copyright © 2000-2014 TIPS Inc. Unauthorized reproduction of this manual is prohibited. This manual is being used during
review sessions conducted by PharmacyPrep. 98-9
Pharmacyprep.com Antimicrobial agents
48- Wrong statement regarding MACROLIDES ANTIBIOTICS include:
a) Used in both gram-positive and gram-negative infections
b) Used as an alternative to penicillin when patient has allergy to penicillins
c) Inhibit cell wall synthesis of microorganisms
d) Increase gastric motility causing cramps and diarrhea as major side effect
e) Very safe for children
49- All of the following are examples of aminoglycosides drugs, EXCEPT:
a) Nystatin
b) Gentamicin
c) Amikacin
d) Streptomycin
e) Garamycin
50- Which of the following is an anti-infective agent with lowest therapeutic index?
a) Macrolide
b) Cephalosporins
c) Clorafenicol
d) Aminoglycosides
e) Antifungal agents
51- Fluoroquinolone are anti-infective agents analogs of:
a) PABA
b) Folic acid
c) Vitamin K
d) Cyanide
e) Nalidixic acid
52- Which of the following best classify metronidazole?
a) Antiprotozoa
b) Antibiotic
c) Antifungal
d) Antiviral
e) Antimicrobial
53- Example of drug(s) used in treatment of tuberculosis include:
I- Isoniazide
II- Rifampin
III- Ethambutol
a) I only
b) III only
c) I and II only
d) II and III only
e) All are correct

Copyright © 2000-2014 TIPS Inc. Unauthorized reproduction of this manual is prohibited. This manual is being used during
review sessions conducted by PharmacyPrep. 98-10
Pharmacyprep.com Antimicrobial agents
54- Drug of choice in the treatment of percutaneous blood infection-schistosomyasis may include:
a) Niclosamide
b) Prazinquatel
c) Mebendazole
d) Dapsone
e) Thiabendazole
55- All are examples of antihelmintic drugs, EXCEPT:
a) Niclosamide
b) Prazinquatel
c) Mebendazole
d) Dapsone
e) Thiabendazole
56- Which of the following is considered a polyene antifungal agent?
a) Miconazole
b) Clotrimazole
c) Amphotericin
d) Itraconazole
e) Ketoconazole
57- Wrong statement regarding antifungal agents includes which of the following?
a) Polyene antifungal act binding to ergosterol from pores resulting in linkage of cellular contents
b) Azoles antifungal act blocking the synthesis of ergosterol.
c) Polyene antifungal has the widest spectrum compared to any other agent
d) Side effects of antifungals include fever, chills, anorexia, headaches and muscle pain
e) Side effects of antifungals cannot be treat for Tylenol due to high drug interaction.
58- Which of the following statement is RIGHT regarding influenza treatment?
I- Type A influenza is best treated by amantadine and rimantadine
II- Type B influenza is best treated by oseltamivir and zenamivir
III- Influenza should not be treated with antiviral agents
a) I only
b) III only
c) I and II only
d) II and III only
e) All are correct
59- All of the following are examples of quinolone antimicrobial agents, EXCEPT
a) Cephalexine
b) Ofloxacine
c) Norfloxacine

Copyright © 2000-2014 TIPS Inc. Unauthorized reproduction of this manual is prohibited. This manual is being used during
review sessions conducted by PharmacyPrep. 98-11
Pharmacyprep.com Antimicrobial agents
d) Ciprofloxacine
e) Levofloxacine
60- Which of the following conditions is quinolone antimicrobial drugs mainly used?
a) Respiratory tract infections
b) Urinary tract infections
c) Gastrointestinal tract infections
d) Endocarditis
e) Cerebral infections
61- Which of the following statements best describes the action of the antitubercular agent ISONIAZID?
a) Inhibit cell wall synthesis of mycobacterium
b) Inhibit the DNA duplication of mycobacterium
c) Inhibit arachidonic acids that feed mycobacterium
d) Inhibit mycolic acids that are unique for mycobacterium
e) Inhibit protein synthesis of mycobacterium
62) Which of the following complications is considered the gravest side effect associated with isoniazid
treatment?
a) Cardiac dysfunction
b) Mental dysfunction
c) Hepatitis
d) Renal complications
e) Discoloration of skin
63- The side effects of isoniazid are potentialized by deficiency of which vitamin?
a) Thiamine
b) Pyridoxine
c) Folic acid
d) Niacin
e) Cyanocobalamin
64- All of the following are examples of amino penicillin, EXCEPT:
a) Ampicillin
b) Cloxacillin
c) Amoxacillin
d) Bicampicilin
e) All are correct
65- Which of the following is an antiviral drug used in the treatment of AIDS-HIV?
I- Acyclovir
II- Zidovudine
III- Stavudine

Copyright © 2000-2014 TIPS Inc. Unauthorized reproduction of this manual is prohibited. This manual is being used during
review sessions conducted by PharmacyPrep. 98-12
Pharmacyprep.com Antimicrobial agents
a) I only
b) III only
c) I and II only
d) II and III only
e) All are correct
66- Bacterial resistance to an antibiotic may occurs due to:
I- Mutation
II- Adaptation
III- Gene transfer
a) I only
b) III only
c) I and II only
d) II and III only
e) All are correct
67- The effectiveness of VANCOMYCIN is seen ONLY in:
a) Virus
b) Gram-negative bacterias
c) Gram-positive bacterias
d) Both gram-positive and negative bacterias
e) Plasmodium
68- Protein synthesis inhibitors may bind to:
a) 30s ribosomal subunit only
b) 50s ribosomal subunit only
c) 70s ribosomal subunit only
d) 30s and 50s and 70s ribosomal subunits
e) 30s and 70s ribosomal subunits
69- Broad-spectrum antibiotic that may cause severe impairment of bone marrow include which of the
following agents?
a) Tetracycline
b) Aminoglycosides
c) Vancomycin
d) Clorafenicol
e) Clindamycin
70- Which of the following agent is classified as lincosamide antibiotic?
a) Tetracycline
b) Aminoglycosides
c) Vancomycin
d) Clorafenicol
e) Clindamycin

Copyright © 2000-2014 TIPS Inc. Unauthorized reproduction of this manual is prohibited. This manual is being used during
review sessions conducted by PharmacyPrep. 98-13
Pharmacyprep.com Antimicrobial agents
71-Sulfonamide antibiotics inhibit synthesis of?
A-Dihydropteroic acid
B-Dihydrofolic acid
C-Tetrahydrofolic acid
D-THF cofactors
E-Nucleic acid synthesis
Ans-A
72- Ciprofloxacin mechanism of action may include:
a) Inhibit protein synthesis
b) Inhibit cell wall synthesis
c) Inhibit synthesis of essential metabolites
d) Inhibit DNA gyrase
e) Inhibit translation of genetic information
Ans-D

Copyright © 2000-2014 TIPS Inc. Unauthorized reproduction of this manual is prohibited. This manual is being used during
review sessions conducted by PharmacyPrep. 98-14
Pharmacyprep.com Antimicrobial agents
CORRECT ANSWERS
ANTI-INFECTIVE AGENTS
1- B
Comments: Cephalosporin is antibiotics that act by inhibition of bacteria’s cell wall synthesis.
2- E
Comments: Trimetropin is used together with sulphametoxazole to avoid resistance and to increase the
action of sulphametoxazole.
3- E
Comments: Penicillin V is an acid-stable beta-lactamase sensitive antibiotic often used together with
probenecid, a beta-lactamase inhibitor, to avoid resistance of bacterias.
4- B
Comments: Amoxacillin is the only antibiotic available as chewable tablets.
5- C
Comments: Erythromycin estolate is a salt type of erythromycin; a macrolide antibiotic that is better
absorbed from GIT that erythromycin base.
6- A
Comments: Penicillins should be administrated in an empty stomach in order to improve its bioavailability.
7- D
Comments: Streptomycin is not used in the treatment of meningitis. The main treatment of bacterial
meningitis may include penicillins, cephalosporins and vancomycin while viral meningitis is mainly treated by
acyclovir IV.
8- B
Comments: Oxacillin and cloxacillin are examples of acid-stable penicillanase resistant.
9- B
Comments: Penicillin V is an acid-stable penicillanase sensitive.
10- E
Comments: Tetracycline is broad-spectrum protein synthesis inhibitors that must be avoided in children less
than 8 years old due to its irreversible side effects as deposition of the drug in teeth and bones causing
permanent discoloration of teeth and bad structural-formation in children’s teeth.
11- B

Copyright © 2000-2014 TIPS Inc. Unauthorized reproduction of this manual is prohibited. This manual is being used during
review sessions conducted by PharmacyPrep. 98-15
Pharmacyprep.com Antimicrobial agents
Comments: Sulphametoxazole act competing with PABA for incorporation into the pathway leading to the
synthesis of folic acid. Results in inhibition of folic acid synthesis that is required for synthesis of purines and
pyrimidines.
12- C
Comments: Cephalexin is a first generation cephalosporin that acts by inhibition of bacterial cell wall
synthesis.
13- E
Comments: Antacids and food interfere with the regular effect of penicillin by decreasing the bioavailability
of penicillin while probenecid and streptomycin increase the bioavailability and effectiveness of penicillin
when used currently.
14- D
Comments: Streptomycin is only available in injectable form because it is not properly absorbed by GIT.
15- A
Comments: Cloroquine is the main drug in prophylaxis and treatment of malaria.
16- B
Comments: Metronidazole is antiprotozoa drug acting by breakage of cell DNA.
17- C
Comments: Norfloxacine is a quinolone synthetic bactericidal antibiotic that acts by inhibition of DNA gyrase
in the bacteria.
18- E
Comments: Meningococcal encephalitis is a type of septic meningitis that normally is best treated by
penicillins, having Ceftriaxone and vancomycin as alternative treatment.
19- D
Comments: Aminoglycosides are inhibitors of protein synthesis characterized for having a very small
therapeutic window and for having the relation plasma concentration and half-life not clear, therefore its
therapeutic use should be closely monitored to avoid serious side effect.
20- E
Comments: Gray Babies syndrome occurs in premature and new born infants when choraphenicol is
administrated during the first few days of life. It happens due to inability of the infant to metabolize the drug
because of glucoronyl transferase deficiency that is essential to detoxify cloraphenicol.

Copyright © 2000-2014 TIPS Inc. Unauthorized reproduction of this manual is prohibited. This manual is being used during
review sessions conducted by PharmacyPrep. 98-16
Pharmacyprep.com Antimicrobial agents
21- E
Comments: Pellagra is a disease characterized by niacin deficiency and/or failure of body to convert
tryptophan to niacin. It is mainly characterized by dermatitis, diarrhea and dementia.
22- E
Comments: Cefixime 400mg orally as single dose, ceftriaxone 125mg IM as single dose and ciprofloxacin
500mg orally as single dose are the recommended regimens for the treatment of uncomplicated gonorrhea.
23- A
Comments: Nosocomial infections are normally hospital developed secondary diseases and may be best
treated by third generation cephalosporin or ciprofloxacin.
24- C
Comments: Viral encephalitis is mainly caused by HSV-1 and 2 viruses and may be treated by acyclovir IV.
25- E
Comments: The treatment of pneumonia is based on the agent causative. Bacteria pneumonia is mainly
treated by the currently used of amoxacillin with aminoglycosides or clarithromycin or levofloxacin.
26- B
Comments: Otite externa also known as swimmer’s ear is mainly caused by pseudomonas, staphylococcus or
enterobacterias and may be treated by garamycin or gentamycin eardrops.
27- D
Comments: Otite media may be caused by caused by S. pneumoniae, H. influenza and M. cataharlis, and may
be treated by amoxacillin clavulanated.
28- A
Comments: First line treatment of UTI associated with E.coli infected agent is mainly treated by
sulphametoxazole and trimetroprim.
29- B
Comments: Helicobacter pylori is a GI infection associated with peptic ulcer and gastric cancer that may be
treated by the “triple therapy” that involves amoxacillin, clarithromycin and omeprazole with or without
metronidazole.
30- E

Copyright © 2000-2014 TIPS Inc. Unauthorized reproduction of this manual is prohibited. This manual is being used during
review sessions conducted by PharmacyPrep. 98-17
Pharmacyprep.com Antimicrobial agents
Comments: Treatment of large intestine infection mainly caused by food intoxication include ciprofloxacin
and metronidazole as therapeutic agents and increase the volume of intake fluids as a non-pharmacologic
help.
31- D
Comments: Vancomycin is the drug of choice in the treatment of pseudomembranous colitis associated with
antibiotic use.
32- E
Comments: Antibiotics that often cause colitis-acute inflammation of colon as side effect include broad-
spectrum penicillins, cephalosporins and clindamycin.
33- D
Comments: Primaquine is normally used together with Cloroquine in the treatment of malaria with the
purpose to kill persistants parasites in the liver of a person infected with Plasmodium vivax.
34- E
Comments: Treatment of malaria in those patients resistant to Cloroquine is normally done by the currently
use of quinine and doxacycline.
35- B
Comments: Antibiotics are drugs derivated from natural sources that can kill or inhibit bacteria growth.
36- C
Comments: Bactericidal drugs have the capacity to completely kill the bacteria and antimicrobial are drugs
derivated from synthetic substances that can kill or inhibit bacterial growth.
37- E
Comments: Antiviral drugs are classified as RNA inhibitor, DNA inhibitors, RNA and DNA inhibitors, and
protease inhibitors.
38- A
Comments: Amoxacillin is a wide spectrum penicillin antibiotic.
39- D
Comments: Carbenicillin is extended spectrum penicillin often given IV for serious infections.
40- E
Comments: All, clavulanic acid, sulbactam and tazobactam are beta-lactamase inhibitors used together with
penicillins sensitive.

Copyright © 2000-2014 TIPS Inc. Unauthorized reproduction of this manual is prohibited. This manual is being used during
review sessions conducted by PharmacyPrep. 98-18
Pharmacyprep.com Antimicrobial agents
41- D
Comments: Cephalosporins are bactericidal antibiotics derivated from cephalosporium acremonium. They
are broad-spectrum agents and are resistant to many types of beta-lactamase. They may be classified by
first, second, third and fourth generation.
42- B
Comments: Cefipime is a fourth generation cephalosporin antibiotic.
43- C
Comments: Vancomycin is a bactericidal agent that acts inhibiting stage II of cell wall synthesis preventing
polymerization of the linear peptideoglycan.
44- A
Comments: Side effects of vancomycin may include ototoxicity, phlebitis, chills and fever.
45- D
Comments: Tetracyclines are broad-spectrum inhibitors of protein synthesis.
46- B
Comments: Dissulfiran side effects are characterized by skin rashes, headache, acne, drowsiness, tiredness,
impotence and metallic or garlic taste in the mouth therefore is not a side effect of tetracyclines.
47- E
Comments: Azithromycin, clarithromycin and erythromycin are examples of macrolides antibiotics.
48- C
Comments: Macrolide antibiotics are protein synthesis inhibitors and have no effect on cell wall synthesis
inhibition.
49- A
Comments: Nystatin is a polyene antifungal drug.
50- D
Comments: Aminoglycosides are protein synthesis inhibitor agents characterized by its low therapeutic
index.
51- E
Comments: Fluoroquinolones are quinolones analog of nalidixic acid.
52- A

Copyright © 2000-2014 TIPS Inc. Unauthorized reproduction of this manual is prohibited. This manual is being used during
review sessions conducted by PharmacyPrep. 98-19
Pharmacyprep.com Antimicrobial agents
Comments: Metronidazole is an antiprotozoa agent that acts by breakage of cell DNA.
53- E
Comments: Isoniazide is the single most important drug used in TB. Rifampin is normally used in combination
with Isoniazide. Ethambutol is only used in TB treatment given in combination with isoniazide.
54- B
Comments: Prazinquatel is an antihelmintic drug used in the treatment of schistosomiase.
55- D
Comments: Dapsone is an antimycobacterium drug of choice in the treatment of leprosy.
56- C
Comments: Amphotericin is the polyene antifungal prototype.
57- E
Comments: Tylenol and ibuprofen are considered the best drugs in the treatment of side effects caused by
antifungal drugs.
58- C
Comments: Influenza is a viral disease and must be treated by antiviral agents.
59- C
Comments: Cephalexine is a first generation cephalosporin.
60- B
Comments: The most common use of quinolone is in the treatment of urinary tract infections.
61- D
Comments: Isoniazide act by inhibiting mycolic acids that are unique for mycobacterium
62- C
Comments: Hepatitis may be a grave side effect associated with isoniazide therapy.
63- B
Comments: Isoniazide is a structural congener of pyridoxine-Vitamin B6, therefore deficiency of pyridoxine
potentate the effects of isoniazide.
64- B

Copyright © 2000-2014 TIPS Inc. Unauthorized reproduction of this manual is prohibited. This manual is being used during
review sessions conducted by PharmacyPrep. 98-20
Pharmacyprep.com Antimicrobial agents
Comments: Aminopenicillins are penicillins with amine group on its molecular structure. Cloxacillin does not
have an amine group on its molecular structure
65- D
Comments: Acyclovir is not used in HIV treatment.
66- E
Comments: Bacterial resistance to an antibiotic may occurs by mutation, adaptation and/or gene transfer.
67- C
Comments: Vancomycin is a narrow spectrum bactericidal agent with activity only against gram-positive
bacterias.
68- D
Comments: Protein synthesis inhibitors bind to 30s (aminoglycosides), 50s(Choraphenicol) and 70s
(macrolide) ribosomal subunits
69- D
Comments: Choraphenicol is a broad-spectrum antibiotic that may cause hematological disorders as side
effect including severe impairment of bone marrow.
70- E
Comments: Clindamycin is a lincosamide antibiotic that acts by inhibition of bacterial of protein synthesis.
71: A
72: D
BIBLIOGRAPHIC REFERENCE
1- COMPREHENSIVE PHARMACY REVIEW – Lippincott William & Wilkins – Fourth edition
2- CPS-COMPENDIUM OF PHARMACEUTICALS AND SPECIALITIES - Canadian Pharmacist Association – 2001
edition.
th
3- MEDICAL DICTIONARY – Dorland’s illustrated – 27 edition.
4- PHARMACY PREP – Lectures series & study guide for Evaluating Examination-TIPS - 2003/2004

Copyright © 2000-2014 TIPS Inc. Unauthorized reproduction of this manual is prohibited. This manual is being used during
review sessions conducted by PharmacyPrep. 98-21
Pharmacyprep.com Antimicrobial agents
5- THERAPUTIC CHOICES – Canadian Pharmacist Association -Third edition
th
6- USP DI – Drug Information for the Health Care Professional–15 edition – Volume I.

Copyright © 2000-2014 TIPS Inc. Unauthorized reproduction of this manual is prohibited. This manual is being used during
review sessions conducted by PharmacyPrep. 98-22
PharmacyPrep.Com Anticancer drugs and chemotherapy

PHARMACY PREP
ANTICANCER DRUGS AND CHEMOTHERAPY
1) A cancer patient experiencing severe nausea and vomiting, should be recommended, all except?
A-Avoid large meals
B-Avoid fat-oily meals
C-Avoid taking small and frequent meals
D-Take divided meals
E-Avoid heavy meals
2) Which of the following chemotherapeutic agent is categorized as low emitogenic?
A) Cisplatin B) Cyclophosphamide C) Iphosphomide D) Vincristine
3) Hemorrhagic cystitis results from irritation of the lining of the bladder by acrolein, a metabolite of
ifosfamide and cyclophosphamide. Acrolein can be inactivated by?
A-Mesna
B-Doxarazoxane
C-Chlropropamide
D-Erythropoeitins
E-None of the above
4) The drug of choice to treat delayed nausea and vomiting?
A-Ondansetron
B-Metoclopramide
C-Dexamethasone
D-Dimenhydrinate
E-Scopolamine
5) Cardio protective agent is?
A-Mesna B-Dexorasoxane C-Acroline D-None of the above
6) Stomatitis is characterized by all of the following signs and symptoms EXCEPT
A. headache B)erythema c)bleeding d)ulcerations e)dryness of mouth

7) What is incorrect about trastuzumab?


A) it is approved for breast cancer
B) It is recombinant MoAB selectively binds to HER-2 receptors
C) It is developed by pharmacogenetic methodologies
D) HER-2 is over expressed in breast cancer patients
E) none of the above

Copyright © 2000-2014 TIPS Inc. Unauthorized reproduction of this manual is prohibited. This manual is being used during
review sessions conducted by PharmacyPrep. 99-1
PharmacyPrep.Com Anticancer drugs and chemotherapy
8) Examples of microtubule targeting drugs?
A) vincristine
B) vinblastine
C) Peclitaxol
D) Estramustine
E) all of the above
9-All of the following chemotherapy agents are vesicants EXCEPT
A)doxorubicin b)mechlorethamine c)vincristine d)methotrexate e)idarubicin

Copyright © 2000-2014 TIPS Inc. Unauthorized reproduction of this manual is prohibited. This manual is being used during
review sessions conducted by PharmacyPrep. 99-2
PharmacyPrep.Com Anticancer drugs and chemotherapy
ANSWERS:
1) C
Tips: taking small and frequent meals is recommended to minimize chemotherapy induced nausea and
vomiting
2) D
Tips: Very low emetic risk drugs: Vinblastine, vincristine, bleomycin, Trastuzumab. Very highly emitogenic
drugs Cyclophosphamide, cisplatin, streptozocin, carmustine,
3) A
Tips: Mesna may be used to inactivate the acrolein, thus preventing hemorrhagic cystitis. Chemo drugs such
as cyclophosphamide and ifosphamide toxicity is prevented by uroprotective agent MESNA.
4) C
5) B
Tips: Cardiotoxic drugs daunorubicin, doxorubicin, idarubicin, mitoxantrone ( Dexrozoxane is cardio
protective agent used with doxorobucin to prevent cardiotoxicity)
6) A
Tips: Stomatitis, or mucositis, is an inflammation of the mucous membranes, particularly the oral mucosa.
Although the symptoms generally are limited to the mouth and throat, stomatitis may affect any part of the
gastrointestinal tract, potentially causing diarrhea and anal fissures.
7) E
8) E
9- D
Tips: Vesicant chemotherapy agents may cause local necrosis of extravasated outside the vein. Doxorubicin,
idarubicin, mechlorethamine, and vincristine are all classified as vesicants.
Chemotherapy that cause necrosis of tissue (vesicant drugs) gives immediate pain or burning: daunorubicin,
doxorubicin, dactinomycin, vinblastine, vincristine, idarobucin, mitomycin
Chemotherapy agent that have least myelosuppression (bone marrow suppression): bleomycin, vincristine,
aspariginase
Chemotherapy most likely cause alopecia: cyclophosphamide, donorubicin, peclitaxel, mechlorethamine.
Pulmonary toxicity: Bleomycin, busulfan, carmustine and mitomycin

Copyright © 2000-2014 TIPS Inc. Unauthorized reproduction of this manual is prohibited. This manual is being used during
review sessions conducted by PharmacyPrep. 99-3
Pharmacy Prep
Natural Products and Pharmacognosy
A customer of your pharmacy wants to buy ginkgo biloba to enhance memory. What to do?
A) give gingko biloba
B) do not recommend gingko
C) refer to doctor
D) refer to emergency
E) say there is no clinical evidence
Ans: A
Tips: As long as patient is not medication and/or have no medical conditions that do not interfere with natural
products if this helps to alleviate their problems. I will give it to patient.
2) A customer of your pharmacy wants to buy ginkgo biloba to enhance memory. Patient currently on warfarin
therapy . What to do?
A) give gingko biloba
B) do not recommend gingko because, has reports of drug interactions
C) refer to doctor
D) refer to emergency
E) say there is no clinical evidence
Ans: B
3) A customer of your pharmacy wants to buy garlic pills to treat borderline high cholesterol. What to do?
A) give garlic pills
B) do not recommend garlic pills
C) refer to doctor
D) refer to emergency
E) say there is no clinical evidence
Ans: C

Copyright © 2000-2014 TIPS Inc. Unauthorized reproduction of this manual is prohibited. This manual is being used during
review sessions conducted by PharmacyPrep. 100-1
ANSWERS:
1) A
Tips: As long as patient is not medication and/or have no medical conditions that do not interfere with natural
products if this helps to alleviate their problems. I will give it to patient.
2) B
3) C

Copyright © 2000-2014 TIPS Inc. Unauthorized reproduction of this manual is prohibited. This manual is being used during
review sessions conducted by PharmacyPrep. 100-2

You might also like